Sunteți pe pagina 1din 148

ACP IMKSAP®

Medical Knowledge
Self-Assessment
Program s

Endocrinology and
Metabolism

lc
14AMAPRA
Category 1 Credits™
A pAme rican College of Physicians® available until
Leading Internal Medicine, Improving Lives December 31,2018.
Endocrinology and Metabolism

Committee Jennifer Sipos, MD2


Associate Professor of Medici ne
Cynthia A. Burns, MD, FACP, Section Editor 1
Director. Benign Thyroid Program
Director, Internal Medicine Clerkship & Acting Internships
Division of Endocrinology, Diabetes, and Metabolism
Associate Professor. Department of Internal Medicine
The Ohio State University School of Medicine
Section on Endocri nology & Metabolism
Columbus, Ohio
Wake Forest University School of Medicine
Winston-Salem, North Carolina
Editor-in-Chief
Howard H. Weitz, MD, MACP, Associate Editor1
Director, Jefferson Heart Institute Philip A. Masters, MD, FACP1
Director, Division of Card iology Senior Physician Educator
Sidney Kimmel Medical College at Thomas Jefferson Director, Clinical Conten t Development
University American College of Physicians
Philadelphia, Pennsylvania Philadelphia. Pennsylvania

Jessicah Collins, MD 1
Clinical Endocrinologist
Director, Clinical Program Development
Diabetes a nd Endocrinology Clinic Cynthia D. Smith, MD, FACP2
Augusta Health American College of Physicians
Fishersville, Virginia Philadelphia, Pennsylvania

Kristen G. Hairston, MD, MPH 1


Medical Director, Joslin Diabetes Center Endocrinology and Metabolism
Associate Professor Reviewers
Depat1ment of Internal Medicine Amindra Singh Arora, MD 1
Section on Endocrinology & Metabolism · Stewart F. Babbott, MD, FACP 1
Wake Forest University School of Medicine John K. Chamberlain, MD, MACP 1
Winston-Salem. North Carolina Terence Chan, MD 2
Pieter A. Cohen, MD 2
Erika B. Johnston- MacAnanny, MD 1
Benjamin P. Geisler. MD 2
Assistant Professor
Richard Hoffman. MD. MPH 1
Medical Director. Center for Reproductive Medici ne
Sowmya Kanikkannan, MD, FACP 1
Associate Faculty, Women's Health Center of Excellence
Marl< D. Siegel. MD, FACP2
Wake Forest Center for Reproductive Medici ne
Wake Forest School of Medicine
Winston-Salem, North Carolina Endocrinology and Metabolism
Reviewer Representing the American
Wanda C. Lakey, MD, MHS2
Assista nt Professor of Medicine
Society for Clinical Pharmacology &
Division of Endocrinology Therapeutics
Du ke University Medical Center Linda A. Hershey, MD. PhD2
Durham VA Medical Center
Durham, North Carolina Endocrinology and Metabolism
Sarah Mayson, MD2 ACP Editorial Staff
Assistant Professor Randy Hendrickson 1• Production Administrator/ Editor
Department of Medicine Katie ldelP, Manager, Clinical Skills and Digital Programs
Division of Endocrinology, Metabolism and Diabetes Margaret Wells1, Director, Self-Assessment and Educational
University of Colorado School of Medicine Programs
Au rora, Colorado Becky Krumm 1• Managing Editor

iii
ACP Principal Staff Terence Chan, MD
Honoraria
Patrick C. Alguire, MD, FACP2
AstraZeneca (Ca nada)
Senior Vice President, Medical Education
Sean McKinney• Pie.ter A. Cohen, MD
Vice Preside nt, Medical Education Stock Options/Holdings (spouse)
Bio Reference Labs, ldexx Laboratories, Johnson &
Margaret Wells 1
Johnson, Mettler Toledo International Inc., Stryker
Director, Self-Assessm ent and Educational Programs
Corp., Biota Pharmaceuticals, Pfizer, ResMed Inc., Vertex
Becky Krumm1 Pharmaceuticals
Managing Editor Honoraria
Katie Idell 1 Consumer Union, Wall Street Journal
Manager, Clin ical Skills Program a nd Digital Products Benjamin P. Geisler, MD
Valerie A. Dangovetsky 1 Royalties
Administrator Up To Date
Consultantship
Ellen McDonald, PhD 1
Wing Tech Inc., Ardian Medtronic LLC, Med tronic Vascular
Senior Staff Editor
Inc., Amgen Inc.
Megan ZborowskP
Linda A. Hershey, MD, PhD
Senior Staff Editor
Research Grants/Co ntracts
Randy Hendrickson • Baxter, Novartis, Forum Pharmaceuticals (fo rmerly
Production Admin istrator/ Ed itor En Vivo)
Linnea Donnarumma1 Honoraria
Staff Editor Med Link Neurology
Susan Galeone 1 Wanda C. Lakey, MD, MHS
Staff Editor Research Gra nts! Contracts
Jackie Twomey1 Janssen, Regeneron, Amarin, Sanofl/ Aventis
Staj]'Editor Sarah Mayson, MD 2
Julia Nawrocki1 Other (Sub-Investigato r for Clinical Studies)
Staj]'Editor Novartis, Cortendo

Kimberly Kerns• Mark D. Siegel, MD, FACP


Administrative Coordinator Consultantship
Siemens
Rosemarie Houton 1
Administrative Representative Jennifer Sipos, MD
Consultantship
1. Has no relatio nships with any e ntity producing, marketing, re se lling, o r d istributi ng Genzyme
hea lth ca re goods o r services consumed by, o r use d o n, patients.

2 . Has disclosed relat io nship{s) with any e ntity producing, marketing, resel ling , o r Cynthia D. Smith, MD, FACP
distrib uting health care goods o r services consumed by, or used on, p Clti ent s.
Stock Options/ Holdings
Merck a nd Co.; spousal employment at Merck
Disclosure of Relationships with any entity producing,
marketing, reselling, or distributing health care goods or Acknowledgments
services consumed by, or used on, patients.
The American College of Physicians (ACP) gratefully
Patrick C. Alguire, MD, FACP acknow ledges the special contributions to the develop-
Consu ltantship ment and production of the 17th edition of the Medical
National Board of Medical Exam iners Know ledge Self-Assessment Program ' (M KSAP ' 17) made
Royalties by the followi ng people:
UpToDate
Graphic Design: Michael Ripca (Gra phics Tech nical
Stock Options/Holdings
Administrator) a nd WFGD Studio (Graphic Designers) .
Amgen Inc, Bristol-Myers Squibb, GlaxoSmithKiine, Stryker
Corporation, Zimmer Orthopedics, Teva Pharmaceuticals, Production/ Systems: Dan Hoffmann (Director, Web Services
Medtronic, Covidien, Inc., Express Scripts & Systems Development), Neil Koh l (Senior Architect),

iv
Chris Patterson (Senior Architect), and Scott Hurd (Manager, For information on how to apply MKSAP 17 Continuing
Web Projects & CMS Services). Medical Education (CME) credits to the Royal College
MOC Program, visit the MKSAP Resource Site at
MKSAP 17 Digital: Under the direction of Steven Spadt,
mksap.acponline.org.
Vice President, Digital Products & Services, t he digital ver-
sion ofMKSAP 17 was developed within the ACP's Digital
Product Development Department, led by Brian Sweigard
The Royal Australasian College
(Director). Other m embers of the team included Dan
Barron (Senior Web Application Developer/ Architect) ,
of Physicians CPO Program
Ch ris Forrest (Senior Software Developer/ Design Lead), In Australia, MKSAP 17 is a Category 3 program t ha t may
Kara Kronen wetter (Senior Web Developer), Brad Lord be used by Fellows of The Royal Aus tralasia n College
(Senior Web Applicati on Developer), John McKnight of Physicians (RACP) to meet mandatory Continuing
(Se nior Web Developer) , and Nate Pershall (Senior Web Professional Development (CPD) points. Two CPD cred-
Developer) . its are awarded for each of t he 200 AMA PRA Category 1
Creditsn' available in MKSAP 17. More informa tion about
The College also wishes to acknowledge that many other using MKSAP 17 for this purpose is available at the MKSAP
persons, too numerous to mention, h ave contributed to Resource Site at mksap.acponline.org and at www.racp.
the production of this program. Without their dedicated edu.au. CPO credits earned through MKSAP 17 should be
efforts, t his program would not have been possible. reported at the MyCPD site at WW\>V.racp.edu.au/ mycpd.

MKSAP Resource Site Continuing Medical Education


(mksap.acponline.org) The American College of Physicia ns (ACP) is accredited
The MKSAP Resource Site (m ksap.acponline.org) is a by the Accreditation Council for Con tinuing Medical
contin ually updated site t hat p rovides links to MKS AP Education (ACCME) to provide continuing medical educa-
17 online answer sheets for p rint subscribers; the lat- tion for physicians.
est d etails on Continuing Medica l Edu cation (CME)
and Maintenance of Certification (MOC) in the United The AC P designates this enduring material, MKSAP 17,
States, Canada, an d Australia; errata; and ot her new for a maximum of 20 0 AMA PRA Ca tegory 1 Credits 1M .
information . · Physicians should claim only the credit commensurate
with the extent of their participation in the activity.
Up to 14 AMA PRA Category 1 CreditsTMare available frQm
ABIM Maintenance of Certification December 31, 2015 , to December 31, 2018, for th e MKSAP
Check the MKSAP Resource Site (mksap.acponline.org) 17 Endocrinology and Metabolism section.
for the latest information on how MKSAP tests can be
used to apply to the American Board of Internal Medicine
for Maintenance of Certifica tion (MOC) points. Learning Objectives
The learning objectives of MKSAP 17 are to:
o Close gaps between actual care in your practice and
Royal College Maintenance
of Certification preferred standards of care, based on best evidence
o Diagnose d isease states that are less common ami
In Canada, MKSAP 17 is an Accredited Self-Assessment sometimes overlooked or confusing
Program (Section 3) as defined by the Maintenance of o Improve management of comorbid conditions that can
Certification (MOC) Program ofThe Royal College of complicate patient care
Physicians a nd Surgeons of Canada and approved by o Determine.w hen to refer patients for surgery or care by
the Canadian Society of Internal Medicine on December subspecialists
9, 2014. Approval extends from July 31, 2015 until July o Pass the ABIM Certification Examination
31, 2018 for the Part A sections. Approval extends from o Pass the ABIM Maintenance of Certification Examination
December 31, 2015 to December 31, 2018 for the Part B
sections.
Fellows of the Royal College may earn three credits per Target Audience
hour for participating in MKSAP 17 under Section 3. o General intern ists and primary care physicians
MKSAP 17 also meets m ultiple CanMEDS Roles, includ- o Subspecialists who need to remain up-to-date in internal
ing that of Medical Expert, Communicator, Collaborator, medicine and in areas outside of their own subspecialty
Manager, Health Advocate, Scholar, and Professional. area

v
• Residents preparing for the certification examination in Accreditation Council for Continuing Medical Education
internal medicine (ACCME), contributors to all ACP continuing medical
• Physicians preparing for maintenance of certification in education activities are required to disclose all relevant
in ternal medicine (recertification) tlnancial relationships with any entity producing, mar-
keting, re-selling, or distributing health care goods or
services consumed by, or used on, patients. Cont ributors
Earn "Instantaneous" CME are required to use generic names in the discussion of
Credits Online therapeutic options and are required to identify any unap-
Print subscribers can enter t heir answers online to earn proved , off-label, or investigative use of commercial prod-
instantaneous Contin uing Medical Education (CME) cred- ucts or devices. Where a trade name is used, all available
its. You can submit your answers using online a nswer t rade names for the same product type are also included .
sheets that are provided at m ksap.acponline.org, where a lftrade-name products manufactured by companies with
record of your MKSAP 17 credits will be available. To earn whom contributors have relationsh ips are discussed, con-
CME credits, you need to answer all of the questions in tributors are asked to provide evidence- based citations
a test and earn a score of at least SO% correct (number of in support of the discussion. The information is reviewed
correct answers divided by the total n umber of qu estions). by the committee responsible for producing this text. If
Take any of t he following approaches: necessary, adjustments to topics or contributors' roles in
content development are made to balance the discussion.
1. Use the printed answer sheet a t t he back of this book to
Further , all readers of this text are asked to eval uate t he
record your answers. Go to mksap.acponline.org, access
content for evidence of commercial bias and send ai1Y rel-
t he a ppropriate online a nswer sheet, transcribe your
evant comments to mksap_editors@acponline.org so that
answers, and submit your test for instantaneous CME
fu ture decisions about content and cont ributors can be
credits. There is no additional fee for this service.
made in light of this information.
2. Go to mksap.acponline.org, access the a ppropriate
online answer sheet, directly enter your answers, and
submit your test for instantaneous CME credits. There is Resolution of Conflicts
no additional fee for this service. To resolve all conflicts of interest and influences of vested
3. Pay a $15 processing fee per a nswer sheet a nd sub mit interests, the American College of Physicians (AC P) pre-
t he pri nted answer sheet at the back of this book by cluded members of the con tent-creation committee from
mail or fax, as instructed on the answer sheet. Make decidi ng on any content issues tha t involved generic or
sure you calculate your score and fax the answer trade-name products associated with proprietary entities
sheet to 215- 351-2799 or mail t he sheet to with which these committee members had relationships.
Member and Customer Service, American College In addition, content was based on best evidence and
of Physicia ns, 190 N. Independence Mall West, updated clinical care guidelines, when such evidence a nd
Philadelphia, PA 19106-1572, using the courtesy enve- guidelines were available. Contributors' disclosure infor-
lope provided in your MKSA P 17 slipcase. You w ill mation can be found w ith the list of contributors' names
need your 10-digit order num ber and 8-digit A;CP and those of ACP principal staff listed in the begi nning of
lD n umber, which are printed on your packing slip . this book.
Please allow 4 to 6 weeks for your score report to be
emailed back to you. Be sure to include your email
address for a response. . Hospital-Based Medicine
If you do n ot have a 10-digit order number and 8-digit For t he conven ie nce of subscribers who provide care in
ACP lD number or if you need help creating a user hospital settings, conten t that is specitlc to the h ospital
name and password to access the MKSAP 17 online setting has been highlighted in blue. Hospital icons (C])
answer sheets, go to mksap.acponline.org or email h ighligh t where the hospital-based content begins,
custserv@acponline.org. contin ues over more t han one page, a nd ends.

Disclosure Policy High Value Care Key Points


It is the policy of the American College of Physicia ns Key Points in t he text that relate to High Value Care con-
(ACP) to ensure balance, independence, objectivity, and cepts (that is, concepts that discuss balancing clinical
scientific rigor in all ofits educational activities. To this benefit with costs and harms) are designated by the HVC
end, and consistent with the policies of the ACP and t he icon (HVC).

vi
Educational Disclaimer tronic or mechanical, including photocopy, without the
The editors and publisher ofMKSAP 17 recognize that the express consent of the American College of Physicians.
MKSA P 17 is for individual use only. On ly one account
development of new material offers many opportunities for
error. Despite our best efforts. some errors may persist in per subscription will be permitted tor the purpose of
print. Drug dosage schedules are, we believe, accurate and earning Con tinuing Medical Education (CME) credits and
Maintenance of Certification (MOC) poin ts/credits and for
in accordance with current standards. Readers are advised,
other authorized uses ofMKSAP 17.
however. to ensure that the recommended dosages in
MKSAP 17 concur with the information provided in the
product in formation material. This is especially important
in cases of new. infrequently used, or highly toxic drugs. Unauthorized Use of This Book
Application of the information in MKSAP 17 remains the Is Against the Law
professional responsibility of the practitioner. Unauthorized reproduction of this publication is unlaw-
ful. The American College of Physicians (ACP) prohibits
The primary purpose of MKSAP 17 is educational.
reproduction of this publication or any of its parts in any
Information presented, as well as publications. technol-
form either for individual use or for distribution.
ogies, products, and/or services discussed, is intended
to inform subscribers about the knowledge. techniques. The ACP \•V iii consider granting an individual permission
and experiences of the contributors. A diversity of pro- to reproduce only limited portions of this publication fo r
fessional opinion exists, and the views of the contribu- his or her own exclusive use. Send requests in writing to
tors are their own and not those of the America n College MKSAP Permissions, American College of Physic;ans. 190 N.
of Physicians (ACP). Inclusion of any material in t he Independence Mall West, Philadelphia, PA 19106-1572. or
program does not co nstitute endorsement or recommen- email your request to mksap_editors@acponline.org.
da tion by the ACP. The ACP does not warrant the safety.
MKSAP 17 ISBN: 978-1-938245-18-3
reliability, accuracy, completeness, or usefulness of and
(Endocrinology and Metabolism) ISBN: 978-1- 938245-25-1
disclaims any and all liability for damages and claims
that may result from the usc of information, publica- Printed in the United States of America.
tions, tec hnologies. p roducts, and/or services discussed
For order information in the United States o r Canada call
in t his program.
800-523-1546. extension 2600. All ot her count ries call
215-351-2600, (M-F, 9 5 PM ET). Fax inquiries to
215-351-2799 or email to custserv@acponline.org.
Publisher's Information
Copyright c 2015 American College of Physicians.
All rights reserved. Errata
This publication is protected by copyright. No part of Errata for MKSAP 17 will be available through the MKSAP
this publication may be reproduced, stored in a retrieval Resource Site at mksap.acponli ne.org as new informat ion
system, or transmitted in any form or by any means. elec becomes known to the editors.

vi i
Table of Contents

Disorders of Glucose Metabolism Gonadotropin-Producing Adenomas ........... 28


IJiabetes Mellitus ................................. 1 Thyroid-Stimulating Hormone- Secreting
Tumors ...................... ..... ... . .. . .. 28
Screening for Diabetes Mellitus .......... ...... . 1
Excess Antidiuretic Hormone Secretion . . .. . .. . . 28
Diagnostic Criteria for Diabetes Mellitus ....... . . 1
Cushing Disease ...................... . ... .. 28
Classiflcation of Diabetes Mellitus .. . .. . .... . .. . . 1
Management of Diabetes Mellitus .......... . .... 5 Disorders of the Adrenal Glands
Inpatient Management of Hyperglycemia ........... 11
Adrenal Anatomy and Physiology ............ . ..... 29
Hospitalized Patients with Diabetes Mellitus ..... 11
Adrenal Hormone Excess .. ......... . . . . .... . .. .. . 30
Hospitalized Patients Without Diabetes Mellitus .... 12
Cushing Syndrome . ......................... 30
Management of Hypoglycemia ........ . .. . . ....... 12
Pheochromocytomas and Paragangliomas....... 32
Hypoglycemia in Patients with Diabetes
Primary Hyperaldosteronism .. ............ ... 34
Mellitus ... . .. .. ...... .......... ............ 12
Androgen-Producing Adrenal Tumors .... .. .... 36
Hypoglycemia in Patients Without Diabetes
Mellitus . .. . ........ ................. ....... 13 Adrenal Insufficiency .... .............. .......... 36
Acute Complications of Diabetes Mellitus ........ .. .. 14 Primary Adrenal Failure .... . .. . .............. 36
Diabetic Ketoacidosis and Hyperglycemic Adrenal Function During Critical Illness ..... . .. 38
1-lyperosmolar Syndrome ... . .. . .. . . .... .. . ... 14 Adrenal Masses ............................. . .. . 39
Chronic Complications of Diabetes Mellitus ......... 16 Incidentally Noted Adrenal Masses .... .. ....... 39
Cardiovascular Morbidity . .. . .. . . . . . ....... ... 16 Adrenocortical Ca rci noma ...... .. . .... ....... 40
Diabetic R_elinopathy ................. . .. . ... 17
Disorders of the Thyroid Gland
Diabetic Nephropathy ............... . . . ...... 17
Thyroid Anatomy and Physiology . .. .... ........... 40
Diabetic Neuropathy .................. . ...... 17
Evaluation ofThyroid Function .. ..... . ....... . ... . _
Diabetic Foot Ulcers ...... .... . .. ; .... . ...... 18
Functional Thyroid Disorders . .. ..... . ...... . . .... 42
llypoglycemic Unawareness ... . ... . ....... ... 18
Thyrotoxicosis .......... . .. ........ . ... . .... 42
Thyroid Hormone Deficiency ............ . . ... 45
Disorders of the Pituitary Gland Drug-Induced Thyroid Dysfunction ............ 46
Hypothala mic and Pituitary Anatomy and Thyroid Function and Disease in Pregnancy ....... . . 46
Physiology .................. . .. . .. . ..... : . . . ... 18 Euthyroid Sick Syndrome......................... 48
Pituitary Tumors ...... ..... .. . .. . .. . . .. .. . ...... 20 Thyroid Emergencies ...... ..... . ....... ....... . . 48
Approach to a Sellar Mass . . ... . .............. 20 Thyroid Storm ..... . ........... . ....... . .. . . 48
Mass Effects of Pituitary Tumors . .............. 21 Myxedema Coma ...... ....... ....... . .. . .. . . 49
Treatment of Clinically Nonfunctioning Structural Disorders of the Thyroid Gland . .. . ... . . . . SO
Pituitary Tumors ........................... . 21 Thyroid Nodules . ... . ................... .... SO
Hypopituitarism ............. . ............ .. . ... 21 Goiters .......... .. .. . ... . .... .. ... . . . .. ... 51
Adrenocorticotropic Hormone Deficiency Thyroid Cancer ........ . .. .............. . ....... 52
(Secondary Cortisol Deficiency) ............... 22
Thyroid-Stimulating Hormone Deficiency ....... 23 Reproductive Disorders
Gonadotropin Deficiency ............ . ........ 24 Physiology of Female Reproduction ........ . ....... 53
Growth Hormone Deficiency ... .. ... . ......... 24 Amenorrhea ................ .. . . .. .. . . . ......... 54
Central Diabetes Insipidus ...... . . . . . .. . .. .. .. 24 Clinical Features . . .............. .... . ..... .. 54
Panhypopituitarism ........... ... . . ..... .. . . 25 Evaluation of Amenorrhea .. ... .. ............. 55
Pituitary Hormone Excess ....................... . 25 Hyperandrogenism Syndromes .. .................. 55
llyperprolactinemia and Prolactinoma .... . .... 25 Hirsutism and Polycystic Ovary Syndrome . . .. .. 55
Acromegaly .. .. .. . ......................... 27 Androgen Abuse in Women .. .... .. . ...... . . .. 56

ix
Female In fertility ................ .. . . .. . . . . . . .. . . 56 Diagnosis and Causes of Hypercalcem ia .... .. . . 62
Physiology of Male Reproduction . . . . . ..... . ... .. . . 57 Treatment of Hypercalcemia ... .. . ............ 65
Hypogonadi sm . .. . .. . .... . .... . . . .... ... . .. . ... 57 Hypocalcemia ....... . ... ... .. . ....... . ....... . . 66
Primary Hypogonadism .. ... . . .. . . .. . . .. . . .. . 57 Clinical Features of Hypocalcemia . ... . . . ... . .. 66
Secondary Hypogonadism . . ...... . ..... . . .... 57 Diagnosis and Causes of Hypocalcem ia ... . .. . .. 66
Androgen Deficiency in t he Aging Male . . .. . .. . . 57 Treatment of Hypocalcem ia ...... . ...... . .. . . . 66
Evaluation of Male Hypogon adism .. . .... . . . ... 58 Metabolic Bone Disease . ... . .. . .... . . .. . . .... .. .. 66
Testosterone Replacement Therapy . .. . . . . ... . ...... 58 Osteopenia and Osteoporosis . . . . . .. . . . . .. .... 66
Anab olic Steroid Abuse in Men . ... . .. . .. . .. . . . .. .. 60 Vitamin D Deficien cy . . . . . . . ... . . .. . . . . .. .. . . 69
Male Infe rtility . .. .. . .. . .... . .. . . . . .. .. .. . . . . . . .. 60 Paget Disease of Bone .. .. . . . .. .. . . . . .. . .. .. . . 70
Gynecomastia . .. . .. . . . . . .. . .. . .. . . . .. .. . . .. . . . . 60
Bibliography .. . . . . ...... . .. . ....... . .. . ... . . . . 71
Calcium and Bone Disorders
Calcium Homeostasis and Bone Physiology . . .. . . .. . . 61 Self-Assessment Test . . .. . . . .. ............ . .. . .. . 73
Hypercalcemia ..... . ..... . . . .. .. ...... .......... 62
Clinical Features of Hypercalcemia . .. . . .. . ..... 62 Index .. . ... . .. . ...... .. . . . . . .. . . . . ... . ..... . . 137

X
Endocrinology and Metabolism High
Value Care Recommendations

The American College of Physicia ns, in collaboration with • Microprolactinomas in asymptomatic patients do not
multiple other organizations, is engaged in a worldwide require treatment; however, surveillance is recom-
initiative to promote t h e practice of High Value Care mended (see Item 65) .
(HVC) . The goals of the HVC initiative are to improve • Ifthe thyroid-stimulating hormone level is fra nkly
health care outcomes by providing care of proven b enefit abnormal, additional evaluation of thyroid function
and reduci ng costs by avoiding lmnecessary and even should be considered to determine the extent of the
harmful interventions. The initiative comprises several dysfunction; measure thyroxine (T) when t he t hyroid-
programs that integrate the important concept of health stimulating hormone is elevated and measure both
care value (balancing clinical beneflt w it h costs and thyroxine (T4 ) and triiodothyronine (T) when the
harms) for a given intervention into a broad range of edu- thyroid-stimulating hormone is suppressed.
cational materials to address the needs of t rainees, prac- • There is no clinical indication for serial measurement of
ticing physicians, and patients. thyroid antibody titers to determine the need for or to
guide therapy except to monitor for residual disease in
HVC content has been integrated into MKSAP 17 in several
patients treated fo r thyroid cancer
important ways. MKSAP 17 now includes HVC-identified
• Radioactive iodine uptake is a measure of iodine uptake
key points in the text, HVC-focused multiple choice
by t he thyroid over 24 hours; it is used to evaluate the
questions, and, for subscribers to MKSAP Digital, an HVC
cause of hyperthyroidism and is not indicated in patients
custom quiz. From the text and questions, we have gen -
with normal or elevated thyroid-stimulating hormone
erated the following list of HVC recommendations that
levels.
meet t he detlnition below of h igh value care and bring us
• In patients with subclinical hypert hyroidism, repeat
closer to our goal of improving patient outcomes while
assessment of thyroid function should be performed
conserving fin.ite resources.
6 to 12 weeks after the initial tests, as the values will
High Value Care Recommendation: A recommendation to normalize in up to 30'Yo of patients.
choose diagnostic and management strategies for patients • An elevated serum thyroid-stimulating hormone level_
in specific clinical sit uations that balance clin ical benefit indicates the diagnosis of primary hypothyroidism ;
with cost and harms with the goal of improving patient thyroid imaging is not indicated unless t here is concern
outcomes. for a nodule on physical examination.
• The typical pattern of euthyroid sick syndrome, nonthy-
Below are t he High Valu e Care Recommendations for t he
roidal ill ness syndrome, or low triiodothyron ine (T) syn-
Endocrinology and Metabolism section ofMKSAP 17.
drome is a mildly elevated thyroid-stimulating hormone
• Lifestyle moditlcations are a cost-effective intervention level and slightly low thyroxine (T4) and t riiodothyronine
that h as been proven to decrease the risk of patients with (T) levels; this pattern should not p rompt further testing
prediabetes developing type 2 diabetes by 41% to 58'Yo. in t he hospital.
• The data for the role and cost-effectiveness of self-mon- • After patients with euthyroid sick syndrome a re dis-
itoring of blood glucose levels are less clear for regimens charged from the hospital, t hyroid function abnormal-
w ithout multiple dai ly insulin injections and noninsulin ities may persist for several weeks so follow-up thyroid
regimens; generally this should be avoided. function should not be repeated until 6 weeks after
• For noncritically ill patients who a re eating, the use of discharge.
basal and prandial subcutaneous insulin is the preferred • A serum t hyroid -stim ulating hormone measurement
and safest choice for achieving inpatient glycemic con- is t he initial laboratory test in a patient with a thy-
trol; oral agents and noninsulin injectable agents do not roid nodule; if the thyroid-stimulating hormone is
have proven safety or efficacy data in the hospital setting. suppressed, then measuremen t of thyroxine (T4 ) and
• Isolated adult-onset growth hormone deficiency is triiodothyronine (T) should be performed , and a radio-
extremely rare, and its clinical significance is debated; nuclide scan should be considered to identifY "ho t" or
evaluation for growth hormone deficiency should be functioning nodules, which have a very low likelihood
reserved for adults with at least one known pituitaty hor- of malignancy and typically do not require fine-needle
mone deficiency. aspiration.

xi
• If the thyroid-sti mulating hormone level is high or vitamin D status. and is the recommended test fo r
normal, the radion uclide scan is u nnecessary as it is vitamin D deficiency.
unlikely to reveal a hot nodule and ultrasonography is • A 25-hydrm.y vitamin D level between 30 and 40 ng/mL
an inexpensive and highly effective method for strati- (75-100 nmol!L) is deemed sufficient for bone health;
fica tion of mal ignancy r isk for nonfunctioning thyroid most expert groups recommend screening all groups at
nodules. least once for evidence of deficiency sinee U.S. incidence
• Measurement of testosterone levels is not recommended is 30'li, to 60'Yo of t he population, however, it should not
if a patient is having regular morning erections, does not be a serial, recurring screening test.
have true gynecomastia on examination, and has a nor- • Treatment tor low bone mass in postmenopausal women
mal testicular examination, as it is highly unlikely t hat he involves lifestyle modification (maximizing weight-
has testosterone deficiency. bearing exercise and avoidance of tobacco or excessive
• Mild , chronic, asymptomatic gynecomastia in the male alcohol) and vitamin D and calcium supplementation; t he
patient does not warrant evaluation. need for pharmacologic therapy is based on the 10-year
• 25-Hydroxyvitamin D has a relatively long half-life of estimated fractu re risk for a major osteoporotic
several weeks, is the best indicator of whole body fracture or for hip fracture) (see ltem 12).

xii
Endocrinology and Metabolism

does not warrant repeat measurement. The diabetes screening


Disorders of Glucose tests have several advantages and disadvantages lo consider. FPC
Metabolism is cheaper and more readily available in most countries com-
pared with hemoglobin A1c, but lhe requirement for overnight
Diabetes Mellitus fast ing can be problematic. OGTI best reflects the pathophysiol-
Diabetes mellitus is a chronic metabolic disease characterized ogy of diabetes by identifying postprandial hyperglycemia sec-
by elevated plasma glucose levels as a consequence of insulin ondary to pancreatic beta-cell deficiency; however. the test is
deficiency, impaired action of insulin secondary to insu lin time intensive. Hemoglobin A1e testing is more convenient w ith
resistance, or a combination of both abnormalities. Prediabetes no fasting requirement, is unaffected by acute stress or illness,
is defined as elevated plasma glucose levels below the diagnos and provides an accurate reflection of the average plasma glu-
tic criteria for diabetes. but above the normal range. cose over the previous 3 months. By contrast, hemoglobin A1e
testing can miss early glucose abnormalities. such as postpran-
Screening for Diabetes Mellitus dial hyperglycemia. Another disadvantage is its decreased reli-
Patients with diabetes mellitus may exhibit classic symptoms abili ty in the setting of anemia. hemoglobinopathies, or kidney
(polyuria, polyd ipsia, polyphagia), or more commonly. they can or liver disease. Furthermore. conditions that affect the turno-
be asymptomatic. Diabetes screening may detect an ea rly asymp- ver of erythrocytes, such as anemias/hemoglobinopathies and
tomatic phase. Current guidelines do not recommend routine pregnancy, can affect the reliability of hemoglobin A1c.
screening for type 1 diabetes as there is no consistent evidence
that early treatment during the asymptomatic stage prevents Classification of Diabetes Me llitus
progression of the disease. Similarly. it has not been firmly estab- Categories for class ification of diabetes encompass a range of
lished that screening improves clinical outcomes in type 2 diabe- insulin abnormalities. including absolute or relative insulin
tes. However. microvascular and macrovascula r disease can be deficiency. insulin resistance, or a combination of these abnor-
present at the time of diagnosis of type 2 diabetes. which is maliti es (Table 3).
indicative of ongoing organ damage durin,g the asymptomatic
phase. Furthermore. there is evidence that the microvascular and Insulin Deficiency
macrovascular disease associated with type 2 diabetes may be Type 1 Diabetes Mellitus
reduced with improved glucose control ea rly in the disease Type 1 diabetes occurs in the setting of insulin deficiency. It
course and that treatment of prediabetes may delay the onset of accounts for 5'Yo of diagnosed diabetes cases. The underlying
type 2 diabetes. In 2008, the U.S. Preventive Services Task Force mechanism is destruction of insulin-produci ng pancreatic
(USPSTF) rec:ommended screening for type 2 d iabetes only in beta cells. which can be autoimmune-mediated, idiopathic, or
asymptomatic adults with a sustai ned blood pressure level acquired .
(treated or untreated) greater than 135/ 80 mm J-Ig. Updated Autoimmune-mediated type 1 diabetes mellitus can result
USPSTF draft guidelines from 2014 have expanded screening from a combination of genetic, environmental, and autoimmune
recomm endations to all adults in primary care settings w ith risk factors. There is a strong association between type 1 diabetes and
factors for the development of diabetes (Table 1). In contrast. the specific HLA antigens. One or more precipitating events, such as
American Diabetes Association (ADA) recommends screening for viral infections. can trigger the autoinunune process of beta-cell
type 2 diabetes based on BMT(2>:25) with additional risk factors, destruction in genetically susceptible persons. Autoantibodies
includi ng a history of gesta tional diabetes. or age (2':45 years). (one or more) can be present at the time of diagnosis. including
antibodies to the follow ing: islet cells, glutamic acid decarboxy-
Diagnostic Criteria for Diabetes Mellitus lase (GAD65), tyrosine phosphatases IA-2 and IA-2P, insulin,
Prediabetes and diabetes can be diagnosed based on the ele- and zinc transporter autoantibodies. Measurements of autoanti-
vated results from one of the following screening tests repeated bodies to GAD65 and IA-2 are recommended for initial confir-
on t\>vo separate occasions: tasti ng plasma glucose (FPC), 2-hour mation as both of these assays are highly automated. In addition.
postprandial gl ucose during an oral glucose tolerance test autoantibodies to GAD65 persist longer than those to islet cells
(OGTI) , or hemoglobin A1c (Table 2). A random plasma glucose after the development of diabetes.
level greater than or equal to 200 mg/dL (11.1 mmoi!L) with Autoinmm ne-mediated type 1 diabetes has classically been
classic hyperglycemic symptoms is d iagnostic of diabetes and considered a disease of children and young. thin adults,

1
Disorders of Glucos e M eta bolism

ADA USPSTF

Screening criteria BMI >25• and at least one additional risk factor: 2008 g uidelines:
Physical inactivity Sustained BP > 135/ 80 mm Hg treated o r
untreated
First-degree relative with diabetes
20 14 updated draft:
High-risk race/ethn icity (black, Latino, Native Ame rican,
Asian America n, Pacific Islander) Screening of adult s in pri ma ry care settings with at
least one of the following risk factors for IFG, IGT,
Delivery of a baby weighing >4.1 kg (91b )
o r type 2 d iabetes mellitus:
History of GDM
Age <!45 years
Hypertension (<! 140/ 90 mm Hg or on antihypertensive Overweight or obese
medication)
First-degree relative w ith d iabetes
HDL cholesterol <35 mg/dl (0 .9 0 mmoi/ L) and/or
trig lyceride level >250 mg/ dl (2.82 mmoi/L) History of GDM
Polycystic ovary syndrome History of polycystic ovary syndrome
Hemoglobin A 1 c <!5.7%, IGT, or IFG on previo us testing High-risk race/ethnicity (black, American Ind ian/
A laska Native, Asian American, Hispanic/ Latino,
Other conditions associated w ith insul in resistance and Native Hawaiian/Pacific Islander)
(severe obesity, acanthosis nigricans)
History of CVD
Additional screening A ll p atient s age 45 years or older Pat ient s age 45 years or younger with any of the
criteria other risk factors in the screen ing crite ria
Additional screening Use of glucocorticoids or anti psychotics
considerations
Screening intervals 3-year interva ls if results are no rmal. Yearly testing if 3-year intervals if low-risk and normal plasma
pred iabetes (hemoglob in A 1c between 5.7% and 6.5%, g lucose values. In high-risk adult s or tho se with
IGT, IFG) is diagnosed . near abnormal test values, yearly testing may be
beneficial.

a At-risk BM I may be some e thnic groups.

AD A ""' American Diabetes Association; BP = blood pressu re; C V D = cardiovascular disease; GDM = g estational d iabetes me llitus; IFG = impaired fasting glucose;
IGT ,.. impa ired glucose tolera nce; USPSTF - U.S. Preventive Se rvice s Task Force.

Dolo from Ame ricon Diabetes Association. Classification ond d iagnosis of d iabetes. Sec. 2./n Standa rds of Medico/ Ca re in Dta be tes-2015. Diabetes Ca re 20 15;38 Supp/ I :S8· 16.
[PMID: 25537714]

TABLE 2. Diagnostic Criteria for Diabetes Mellitus•


Test Normal Range Increased Risk for Diabetes
Diabetes (Prediabetes)

Random p lasma g lucose Classic hyp erg lycemic symptoms plus a ra ndom
p lasma g lucose <!200 mg/ dl(11. 1 mmoi/L)
Fasting p lasma g lucoseb <1 00 mg/ d l 1 00-125 mg/dl <! 126 mg /dl
(5.6 mmo i/L) (5.6-6.9 mmoi/L) (7 .0 mmoi/L)
Plasma glucose during a <1 40 mg/ dl 140-199 mg/ dl <!200 mg/dL
2-hour 7 5-g OGTT
(7.8 mmoi/L) (7.8-11 .0 mmoi/L) (11.1 mmoi/ L)

Hemog lobin A 1c <5.7% 5.7%-6.4% <!6.5%

OGTI = oral glucose to lerance test.

the absence o f hype rglyce mic sympto ms, an ab normal fasti ng plasma glucose, O GTI, or he moglobin A 1< should b e confirme d by re pe at testing. The sa me test should be
used whe n repeati ng the measurement for confirmation. If tvvo different tests are pe rforme d and on ly on e has ab norm al resu lts, the A merican Dia bete s Association reco mmends
re peatin g the test with the abnormal results.

bFastin g fo r at least 8 hours.

Data from American Dtabe tes Association. (2 ) Class ificotion a nd diagnosts of dia bete s. Diabete s Care 20 I S Jan;38 Sup pi:S8-S 16.IPM/D: 2553771 4 ]

2
Disorders of Glucose Metabolism

TABLE 3. Classification of Diabetes Mellitus duce some insulin during a "honeymoon" phase lasting several
Insulin Deficiency•
weeks Lo many months, although this is not an adequate or
susta ined effect. Insulin therapy is therefore recommended dur-
Immune-mediated ing the honeymoon phase to reduce the metabolic stress on the
Type 1 diabetes functioning beta cells to preserve any residual fu nction for as
LADA long as possible. Late autoimmune diabetes in adults (LADA)
presents in patients with autoantibodies to pa ncreatic beta-cell
Rare forms: "stiff man" syndrome, anti-insulin receptor antibodies
antigens and beta-cell destruction who did not require insulin
Idiopathic (sero negative)
ini tially but eventually progressed to an insulin requirement.
Acq ui red Patients with autoimmu ne- mediated type 1 diabetes are at an
Diseases of t he exocri ne pancreas: pancreatitis, trau ma/ increased risk to develop other autoimmune diseases, such as
pancreatectomy, neoplasia, cystic fi brosis, hemochromatosis,
thyroiditis and celiac disease most commonly Thus. screening
fibrocalcu lo us, pancreatopathy
for associated autoimmune diseases should be considered at the
Infections: congenital rubella
time of diagnosis and/or the development of signs and symp-
Drug-related: Vacor (rat poison), intravenous pentamidine toms. Consensus on the frequency and effectiveness of repeat
Insulin Resistance screening for associated autoimmune diseases is lacking.
Idiopathic type 1 diabetes can present with relative insu-
Type 2 Diabetesb
lin deficiency and episodic DKA without evidence for autoim-
Ketosis prone< munity. There is a strong genetic history of diabetes, and Asian
Other or Rare Types and African ancestry appears to be a predisposing fac tor.
Acquired type 1 diabetes can be caused by diseases affect-
Genetic defects in beta-cell function (including six distinct
MODY syndromes) ing the exocrine pancreas, infections, or drugs. Diffuse dam-
age to the pancreas and beta cells or impaired insulin secretion
Genetic defects in insulin actio n
with subsequen t insulin deficiency occurs in these scenarios.
Endocrinopath ies:
(acromegaly, Cushing syndrome, glucagonoma,
, KEY POINTS
pheochromocytoma, hyperthyroidism)d • Prediabetes and diabetes mellitus can be diagnosed
(somatostatinoma, aldosteronoma)• based on the elevated resu lts from one of the following
Drug-related: • screening tests repeated on two separate occasions: fast-
(gl ucocorticoids, thiazides, diazoxide, tacrolimus,
ing plasma glucose, 2- hour postprandial glucose during
cyclosporine, niacin, HIV protease inhibitors, atypical an oral glucose tolerance test, or hemoglobin A1c.
anti psychotics [clozapine, olanzapine])1
• Measurements of autoantibodies to GAD65 and IA-2 are
Genetic syndromes: recommended for initial confirmation of autoimmune-
Down syndrome9 mediated type 1 diabetes mellitus.
Wolfram syndrome (DIDMOAD)h
(Klinefelter, Turner, and Prader-Willi syndromes; myotonic Insulin Resistance
dystrophy)d
Insulin resistance is characterized by the inability of the
DIDMOAD = d iabetes insipidus, diabetes mellitus, optic atrophy, and deafness; LADA = peripheral cells to utilize insulin effectively. with a compensa-
late autoimmune diabetes in adults; MODY =maturity-onset diabetes of the young.
11
Beta-ce ll destruction usually leading t o absolut e insulin deficiency.
tory increase in the amount of insu lin secreted by the pancre-
bfnsulin resistance with pro gressive relative insulin deficiency. atic beta cells in response to hyperglycemia. The pancreas
' More common in nonwhite patients who present with diabet1c ketoacidosis but exhibits a relative insulin deficiency when it ca nnot produce
become non-insulin dependent over time.
enough insulin to overcome the hyperglycemia. Obesity pre-
dJmpaired insulin action.
disposes to the development of insulin resistance.
'"Impaired insulin secretion.
flmpaired insulin secretion, impaired insulin action or altered hepatic glucose metabolism.
Metabo lic Sy ndrome
9fnsulin deficiency, immune-med1ated.
Metabolic syndrome is the coexistence of a group of risk factors
that increases a person's probability for the development of
typ e 2 diabetes mellitus and cardiovascular disease (CVD). In
although it can occur at any age or BMl range. The initial pres- addition to impaired glucose metabolism, these risk factors
entation can range from modest elevations in plasma glucose include central body obesity. hypertension, and hyperlipidemia
levels to diabetic ketoacidosis (DKA). The time course for beta- (Table 4). Metabolic syndrome increases the relative risk of
cell destruction is also variable, although it is frequently more developing CVD by twofold and diabetes by fivefold. although it
rapid in children compared with adults. Upon initiation of is not clear whether the combination of factors associated with
insulin therapy for type 1 diabetes, the remaini ng functioning metabolic syndrome impans a greater risk than that attributa-
pancreatic beta cells may temporarily regain the ability to pro- ble to each individual risk factor present. In the presence of one

3
Disorders of Glucose Metab olism

TABLE 4 . AHAINHLBI Criteria for the Definition of the Insulin resistance from obesity in the setting of relative
Metabolic Syndrome insulin deficiency contributes to the development of type 2
Clinical Criteria Qualifying Measurements diabetes in the majority of patients. The degree of hyperglyce-
(Must meet at least mia depends on the extent of beta-cell functi on, which ca n
3 ofthe 5 criteria) decl ine over time.
Type 2 diabetes generally has an insidious onset of pro-
Waist circumference• Men 40 in ( 102 cm)b
longed asymptomatic hyperglycemia. Most patients do not
Women 35 in (89 em)<
present with the classic symptoms of polydipsia, polyphagia, or
FastingTG <:150mg/d l (1.7mmoi/L)or polyuria. Patients with type 2 diabetes may present first w ith
Drug therapy targeting increased TG macrovascular or microvascular changes. Although type 2 dia-
HDL cholesterol Men <40 mg/dl (1.0 mmoi/L) betes is often considered an adult disease, the incidence is
Women <50 mg/dl (1.3 mmoi/L) or increasing along w ith obesity rates in children and adolescents.
Residual insulin production from the beta cells is insu f-
Drug t herapy targeting decreased
HDL ficient to control glucose adequately, but it is able to suppress
Blood pressure Systolic 2:130 mm Hg or
lipolysis for most persons with type 2 diabetes. Under extreme
metabolic stress, such as an illness, some patients with type 2
Diastolic <:85 mm Hg or
diabetes cannot suppress lipolysis and present w ith DKA.
Fasting glucose Drug therapy for hypertension Ketosis-prone patients w it h type 2 diabe tes are more likely to
Blood glucose <: 100 mg/dl be overvveight or obese, middle-aged, male, and of black or
(5.6 mmoi/L) or
Latino ethn icity. Insulin use during the time of metabolic
Drug therapy for increased glucose stress can often restore the beta cells from the glucose toxicity
AHA = American Heart Association; HDL - high-d ensity lipoprotein cholesterol; with a return to diet, exercise, and oral hypoglycemic agents
NHLBI - National Heart, Lung, and Blood Institute; TG - trig lycerides.
for glucose control. Prior to switching from insulin to oral
"So m e individuals with minima lly elevated waist circumference measure m ents [e .g.,
(men 37-39 in or 94-99 em) or (women 31 -34 in or 79-86 em)] may still be at nsk for
therapy. pancreatic beta-cell function should be assessed w ith
type 2 d iabetes or cardiovascu lar disease and will benefit from lifestyle interventions. fas ti ng C-peptide and glucose measurements.
bA lower waist circu mference o f 3 5 in (89 em } should be used for Asian American m en . Lifestyle modifications alone or combined with therapeu-
cA lower waist circu m fere nce of 3 1 in {79 em) should be used for As1an American tic interventions can preven t or delay the development of type
wo men .
2 diabetes in high-risk persons. Lifestyle modifications arc a
Data from Gru ndy SM, J J, Dan iels SR, et al. Diagnosis and manage ment
of the m etabolic syndrome: An American Heart Association/ Nat ional Heart, Lung,
cost-effective intervention. Several randomized controlled tri-
and Blood In stitute Scientific Statement: Executi ve Summary Circulation. 2005 O ct als provide evidence that diet changes, increased daily exer-
25;112(17) 2735-52.(PMID: 16157765 ]
cise, and weight loss targets of S'Yo to 7% can significantly
decrease the risk of developing type 2 diabetes in persons wi th
or more risk factors for metabolic syndrome, the Endocrine prediabetes by 41 % to .58%. Additionally, metformin has been
Society recommends a 3-year screening interval for the meta- shown to reduce the risk of diabetes in patients with prediabe-
bolic syndrome components including waist circumference, tes, although this effect was not as robust as lifestyle interven-
fasting lipid profile, fasting plasma glucose, and blood pressure. tions. In the setting of impaired glucose tolerance, impaired
Calculation ofthe 10-year CVD risk is recommended persons fasting glucose values, or hemoglobin Ale values between .5.7%
with metabolic syndrome to determine the need for lifestyle and 6.4%, the ADA recommends considering metformin for
modifications and therapeutic interventions to prevent or delay prevention of type 2 diabetes.
progression to type 2 diabetes or CVD. The Framingham Risk Additional therapeutic agents, such as lipase in hibitors,
Score and the new Pooled Cohort Equation from the American a -glucosidase inhibitors, and thiazolidinediones, have been
College of Cardiology/American Heart Association are fre- evaluated to delay or prevent type 2 diabetes; however, the
quently used w ithin the United States to assess CVD risk. effectiveness, cost, and potential side effects must be consid-
ered before implementation (TableS).
Type 2 Diabetes Mellitus
KEY POINTS
Type 2 diabetes mellitus accounts for most (90%-95%) diag-
nosed diabetes cases. It affects 10 .9 million (26. 9%) of adults in • Lifestyle modifications are a cost effective intervention HVC
the United States aged 6S years or older. Asian Americans, that has been proven to decrease the risk of patients with
American Indians, Alaska Natives, Hispanics. and non- prediabetes developing type 2 diabetes by 41% to 58%.
Hispanic black persons are at an increased risk for developing • In high- risk persons, the American Diabetes
diabetes compared w ith non-Hispanic white persons. The Association recommends considering mettormin for
etiology of type 2 diabetes is likely multifactorial. There is often prevention of type 2 diabetes, particularly in patients
a strong fa mily history of type 2 diabetes among first-degree w ho are younger than 60 years of age, have a BM!
relatives, although the specific genes responsible for the glu- greater than 35 , or have a history of gestational diabetes.
cose abnormalities remain unidentified.

4
Disorders of Glucose Metabolism

TABLE 5 . Strategies to Prevent or Delay Onset of Type 2 t imes daily initially, to include fasting and 1-hour or 2-hour
Diabetes Mellitus postprandial values. Postprandial hyperglycemia in pregnancy
Intervention Effectiveness may predict worse fetal outcomes and complica tions.
Li restyle interventions and/or pha rmacologic agents
Diet and exercise Sustained weig ht loss of 7%,
·should be implemented when glucose goals for gestational
with at least 150 minutes of
moderate exercise per week, diabetes are not met. Nutrition requirements for gestational
shown to delay onset of diabetes should allow for appropriate maternal weight gain
diabetes by up to 3 years fo r normal fetal growth while obtain ing goal glucose values. A
Smoking cessation Modestly effective as long as moderate exercise program is recommended for glycemic
it does not cause weight gain,
but is always recom mended
control.
Insulin has traditionally been the mainstay of therapy for
Bariatric surgery Effective if used in morbid ly
obese persons (BMI >40) gestational diabetes when glycemic goals are not met with diet
and exercise. Off-label use of metformin and glyburide in
Metformin• Shown to delay onset of
d iabetes by up to 3 years pregnancy has been studied and there appears to be equiva-
lence in efficacy with insulin; however. long-term safety data
Lipase inhibitors (orlistat) Shown to d elay onset of
d iabetes by up to 3 years are lacking.
a-Glucosidase inhibitors Shown to delay onset o f Gestational diabetes resolves after pregnancy for most
(acarbose, voglibose) diabetes by up to 3 years women ; however, the risk of developing type 2 diabetes isS%
Thiazolidinediones Show n to delay onset of to 10% after delivery and 35% to 60% in the subsequent 10 to
(troglitazone, rosiglitazone, d iabetes by up to 3 years 20 years. The ADA recommends diabetes screening fo r women
pioglitazone) with a history of gestational diabetes usi ng standard criteria at
Insulin and insulin Ineffective 6 to 12 weeks postpa1tum and every 3 years Lhereafter.
secretagogues (sulfonylureas,
megl itinid es) KEY POINTS
ACE inh ibitors and Ineffective • Lifestyle interventions should be implemented to meet
angiotensin receptor b lockers
glycemic goals in women with gestational diabetes;
Estrogen-progestin Modest effect only however, when these are not met, insulin should be
"Preferred. initiated.
• For women with a history of gestational diabetes, diabe-
tes screening using standard criteria should occur at 6
Gestational Diabetes Mellitus to 12 weeks postpartum and every 3 years thereafter.
Relative insulin deficiency during the increased insulin resist-
ance associated with pregnancy can result in the development Uncommon Types of Diabetes Mellitus
of gestational diabetes mellitus. Pregnant women at high risk Genetic defects in beta-cell fu nction and insulin action cause
for developing gestational diabetes include those from certain some uncommon forms of diabetes (see Table 3). Maturity-
racial or ethnic groups (Hispanic/ Latino Americans, blacks, onset diabetes of the young (MODY) is an autosomal dominant
and American Indians), overweight or obese women, women monogenetic defect that affects beta-cell functio n but not
older than 25 years of age, and women with a strong family insulin action. MODY should be suspected in non-obese
history of type 2 diabetes. An estimated 2% to 10% of pregnant patients with a strong family history for diabetes when the
women have gestational diabetes. Complications related to onset of diabetes occurs before 25 years of age in the absence
gestationa l diabetes include miscarriage, fe tal deformities, of autoantibodies. Genetic defects in insulin action cause insu-
large for gestational age infants, macrosomia, preeclampsia, · lin resistance with varying degrees of hyperglycemia, as seen
complications during labor and delivery, increased perinatal with congenital lipodystrophy.
compli cations, and mortality. Complication risk is on a con- Several endocrinopathies can impair insulin action or
tinuum wilh increasing hyperglycemia. secretion as a consequence of excess hormone production.
Disagreement exists among consensus groups regarding Conditions such as Cushing synd rome and pheochromocy-
the definition, screening methods, and diagnostic criteria for toma decrease the action of insulin seconda ry to excess
gestational diabetes. High- risk pregnant women should be cortisol and epinephrine, respect ively. The hypokalemia
screened for overt diabetes at the initial prenatal visit using induced by hyperaldosteronism ca n inhibit t he secretion of
criteria for nonpregnant women, according to the International insulin.
Association of Diabetes and Pregnancy Study Group (IADPSG)
and the ADA. In the absence of overt diabetes at the initial Management of Diabetes Mellitus
office visit, diabetes screening should occur between 24 and The most effective management of diabetes mellitus includes a
28 weeks' gestalion. Once a diagnosis of gestational diabetes is multidisciplinary approach, including patient education and
made, glucose monitoring should be performed at least four support, engaging patients in their care and decision making,

5
Disorders of Glucose Metabolism

lifestyle modifications with diet and exercise, reduced caloric Patient Education
intake for overweight and obese patients, and pharmacologic Diabetes self- management education (DSME) and diabetes
therapies when necessary to meet individualized glycemic self-management support (DSMS) are recommended at the
goals (Table 6). time of diagnosis of prediabetes or diabetes and throughout

Postprandial Capillary Glucose


A1c• (1 -2 hours after meal)•

Healthy Early in disease course <7.0% wit hout 70-130 mg/d l < 180 mg/dL(1 0 .0 mmo i/L)
severe recurre nt (3.9-7.2 mmoi/L)
Few comorbidities
hypoglycemia
Preconception
(<6.5%for
Patient preference select patient s)b

Life expectancy> 10 years


Complex health Significant comorbidities, including <8.0% without
issues advanced atherosclerosis or severe recurrent
m icrovascular compl ications hypoglycemia
Longer duration of diabetes
Frequent hypoglycemia
Hypoglycemia unawareness
Life expectancy <1 0 years
I Older adults Few comorbidities <7.0%-7.5% 90-130 mg/d L
without severe (5.0-7.2 mmoi/L)
Extended life expectancy
recurrent
No impairment of cognition or hypoglycemia
function
M ultiple comorbidities <8.0% without 90-1 50 mg/d i.,
severe recu rre nt (5.0-8.3 mmoi/L)
"Hypoglycemic risk
hypoglycemia
Fall risk
Mild impa irments in cognition and
function
Poor hea lth <8.5% without 100-180 mg/dl
severe recu rrent (5.6-1 0.0 mmoi/L)
Chronic comorbidities with end-stage
hypoglycemia
disease
Long-term care placement
Moderate-to-severe in
cognition and fu nction
Limited life expectancy
Pregnant Preexisting type 1 o r type 2 diabetes <6.0% without 60-99 mg/ d l 100-129 mg/ d l (5.6-7.1 mmoi/L)
women severe recurrent (3.3-5.5 mmoi/L)
hypoglycemia
Gestational diabetes s;95 mg/ dl 1-hour after meal: s: 140 mg/dl
(5.3 mmoi/ L) (7 8 mmoi/L)
2-hours after meal: s: 120 mg/dl
(6.7 mmoi/L)

.)Recommend e d if goa l can b e met without seve re rec urre nt hypoglyce mia. If se vere re curre nt hypoglycemia is pre sent, there is no reco mmended hemoglobin A 1c go al, as
modification of the patient's diabetes mellitus re gimen to resolve severe recurrent hypoglycemia should take p recedence. When severe recurrent hypoglycemia is resolved, an
hemoglobin A 1c goal can be chosen, and treatment decisions can again be made based on that ind ividualized hem oglobin A 1, goal without frequent hypoglycemia.

bThls can be considered for patients with an early diag nosis of diabetes mellitus, no signi ficant cardiovascular disease, or managed with lifestyle modifi cati ons or metformin.

Data from Amen can D iabetes AssoCiatio n. Glyce mic targets. Sec. 6.1n Standards of M ed tcal Care 1n D iabetes-2015 Diabetes Care. 20 15;3 8(Suppl 1) S33-S40 [PMID: 25537705].
(Modification o f Table 6 .2 (p. 537).

Data from American D iabetes Associatio n. O lder adults. Sec. 10.1n Standards of M edical Care in D tabetes-201 5. Diab ete s Care. 20 15;38(Suppl 1):S6 7·S69.jPM ID: 25537711 ].
(Modification of Table 10.1 (p. 568).

Data from American D iabetes Associatio n. Man agement of d iabetes in pregnancy. Sec. 12.1n Standard s of Medical Cam in D iabetes-201 5. Diabetes Care. 2015;38(Suppl 1):
S77-S79. (PMID: 2553771 3]

6
Disorders of Glucose Metabolism

the lifetime of the patient. DSMS is an individualized plan that TABLE 7. Comparison of Hemoglobin A 1 c Value and
provides opportunit ies for ed ucational and motivational Estimated Plasma Glucose Level
support fo r diabetes self-management. DSME and DSMS Hemoglobin A 1c (%) Estimated Average Plasma
jointly provide an opportunity for collaboration between the Glucose Level
patient and health care providers to assess educational needs mg/dl(mmoi/L)
and abilities, develop personal t reatment goals, learn self-
management skills, and provide ongoing psychosocial and 6 126(7.0)
clinical support. Improved outcomes and reduced costs have 7 154(8.6)
been associated w ith DSME and DSMS. 8 183 (10.2)
9 212 (11 .8)
Self-Monitoring of Blood Glucose
10 240(13.4)
Blood glucose monitoring can involve a variety of modalities,
including self-monito ring of blood glucose (SMBG), hemo- 11 2 69(14.9)
globin A1c, or continuous glucose monitoring (CGM). 12 2 98(16.5)
SMBG is recommended for patients on multiple daily Ad apted with perm ission of Ame rica n Diabete s Association, from Translati ng the
injection (MDI) insulin therapy or continuous subcutaneous A 1C assay into e stim ated average g lucose va lues. N athan D M , Kuene n J, Borg R,
Zheng H, Shoenfeld D, Heine RJ , A 1C-d erived average g lucose study g roup.
insulin infus ion (CSII) therapy. SMBG should be performed [erratum 1n Diabetes Care. 2009;32( 1):207 J. Diabetes Care 2008; 31(8): 14 76.
[ PMID: 185400 461
freque ntly during several critical time periods: preprandial ,
bedtime, before and after exercise, periods of symptomatic
hypoglycemia or hyperglycemia , and before important activi- CGM combined with intensive insulin t herapy in adults
ties such as operating dangerous machi nery. Monitoring (::::25 years of age) with type 1 diabetes as a successful modality
blood glucose levels 1 to 2 hours after food consumption to lower hemoglobin A1e levels. The greatest improvement s in
(postprandial) can be useful to assess prandial insulin cover- glycemic control are associated w ith longer periods of CGM
age in patients with at-goal preprand ial readings but with use. In patients using a CGM system, it is important to note
hemoglobin A1c not at goal. Overnight blood glucose monitor- that it does not replace SMBG. Calibration with SMBG is
ing can help detect hypoglycemia or dawn phenomenon. requ ired at least twice daily with CGM systems. All CGM glu-
Success with SMBG requires the physician and patient to act cose values that warrant an immediate intervention should be
upon the information that it provides. This can include insu- confirmed w ith SMBG prior to action due to a lag time ranging
lin dose adjustii.1ents, changes in meal content, or changes in from 5 to 21 minutes for several CGM brands between capillary
activity level to reach individualized glycemic goals. The data blood glucose and interstitial glucose. Rapid glucose fluctua-
for the role and cost-effect iveness of SMBG are less clear for tions further increase t he lag time.
regimens without multiple daily insulin tnj ections and non- KEY POINTS
insulin regimens.
• Blood glucose monitoring, including self- monitoring of
It is often necessa ry to combine both SMBG and hemo-
blood glucose levels, hemoglobin A1e levels, or continu-
globin A1e to determine if adequate control of glucose has been
ous glucose monitoring, is recommended for patients
achieved. There is a strong correlation betvveen hemoglobin
w ith diabetes mellitus requiring multiple daily insulin
A1e and the average 3-month plasma glucose value. Therefore,
injections or continuous intravenou s insulin injection
the ADA and the American Association for Clinical Chemistry
therapy.
advocate reporting both the hemoglobin A1e and the estimated
plasma glucose levels (Table 7). Hemoglobin A1e monitoring • The data for the role and cost-effectiveness of self- HVC
should be measured at the time of diagnosis and every monitoring of blood glucose levels are less clear for
3 months while making changes to achieve glycemic goals: regimens without multiple daily insulin injections and
Testing intervals can be decreased to twice yearly after glyce- non insulin regimens; generally this should be avoided.
mic goals have been met.
CGM technology measures real-time glucose values from Nonpharmacologic Approaches
the interstitial flu id every few seconds through the temporary Nonpharmaco logic approaches to diabetes management
placement of a sensor subcutaneously for 3 to 7 days. The sen- should be implemented throughout the lifespan of the
sor is connected to a transmitter that sends the data through patient. These approaches can be used alone or as adjunct
wireless radiofrequency to a display device. CGM glucose val- t herapy in type 2 diabetes to improve t he success rate of
ues average +/ - l S% from a laboratory glucose measurement. pharmacologic agents. Medical nutriti.on therapy and exer-
CGM may be useful in persons with frequent hypoglycemia, cise can be used in conj unct ion w ith insulin t herapy for
hypoglycemic unawareness, or extreme fluctuations in glu- patients with type 1 diabetes.
cose levels. CGM systems can rapidly identify hypo- or hyper- Medical nutrition therapy is an essential component of
glycemia th at is not always detected with SMBG or hemoglobin any successful management plan for patients w ith prediabetes
A1e measurements. Additionally, the ADA endorses the use of or diabetes. Modest weight loss (2.0-8.0 kg [4.4-17.6lb] or 7%)

7
Disorders of Glucose Metabolism

through caloric reduction can benefit some ovenveight or 42% to 57%. Intensive insulin therapy has risks, including
obese adults w ith type 2 diabetes. significant increases in hypoglycem ia and weight gain.
Consistent exercise provides beneficial effects on glucose Therapy should therefore be individually ta ilored for each
control, weight, and cardiovascular status. For persons with patient's preferences, li festyle. educa tion level, fi nancial
diabetes in whom no contraindications exist, aerobic exercise resources, and comorbidities.
should consist of at least 150 minutes/week at a moderate Available insulin preparations and their activity profiles
intensity level. 75 minutes/week at a vigorous activity level, or are ind icated in Table 8. Most persons with type 1 diabetes are
a combination of these two. Resistance training should be sensitive to the effects of exogenous insulin Lherapy, w ith in i-
incorporated in to the exercise routine at least 2 days per week. tial total daily doses of insulin typically ranging from 0.3 to
Hypoglycemia and extreme hyperglycemia can worsen if pre- 1 U/kg/d. A basal insulin dose should account for half of the
sent at the time of exe rcise and should be corrected before total daily dose of insulin, while the remaining insulin should
proceeding with increased physical activity. be divided to cover the number of meals consumed during the
Bariatric surgical procedures (restrictive and bypass) ca n day Basal insulin coverage can be p rovided with one to two
be considered in obese patients with type 2 diabetes. Weight daily injections of insulin detemi r, glargine, or neutral prola-
loss and diabetes remission rates are significa nt with these mine Hagedorn (N PH) insulin. CSll can also provide basal
procedures, but the long-term benefits require additional coverage with a nalogue insulin. For prandial coverage. ana-
studies. See MKSAP 17 Gastroenterology and Hepatology and logue or regular insulin is injected prior to meal consumption
MKSAP 17 General In ternal Medicine fo r more information. or analogue insulin is bolused with CSll prior to meals. Insulin
Depression, anxiety, and diabetes-related stress are com- dosing immediately after a meal is appropriate in certain situ-
mon among patients with diabetes and may impair their abil- ations, particularly when food intake is unpredictable.
ity to achieve success with a diabetes management plan. Postprandial insulin dosing allows for a reduction in the insu-
Screening should occur continuously during the course of lin dose that is co mmensu rate with the amount of food
diabetes treatment. ingested to avoid hypoglycemia that could have resulted from
the full insulin dose. For example, the postprandial insulin
Pharmacologic Therapy dose is reduced by SO 'Yo if on ly half of the meal is consumed.
An ind ivid ualized treatment goal will help guide the selection
of the optimal treatment regimen. For many persons with
diabetes, a reasonable goal for hemoglobin A1c is less than 7.0% TABLE 8. Pharmacokinetic Properties of Insulin Products•
(or Jess than 6.5%, if this can be achieved without sign ificant Insulin Type Onset Peak Duration
hypoglycemia). If severe recurrent hypoglycemia is present.
Rapid-acting or 5-15 min 45-90 min 2-4 h
there is no recommended hemoglobin A1c goal, as modifica- analogue (l ispro,
tion of the patient's diabetes regimen to resolve severe recur- aspart, glulisine) 1- -
rent hypoglycemia should take precedence. The increased Short-acting (regular) 0.5-1h 2-4 h 4-8 h
risks of hypoglycemia outweigh the risks of diabetes complica- NPH insulin 1-3 h 4-10 h 10-18 h
tions in older patients with longer disease duration, which
Detemir 1-2 h No neb 12-24 he
necessitates consideration of a less-stringent glycemic goal.
The recommended goals from the ADA for blood glucose and Glarg ine 2-3 h No neb 20-24+ h
hemoglobin A1c levels are located in Table 6. Pre-mixed insu lins
70% NPH/30% 0.5-1 h 2-10 h 10-18 h
Therapy for Type 1 Diabetes Mellitus regular
Lifelong insulin t herapy is the first-line treatment fo r type 1 50% NPH/50% 0.5-1 h 2-10 hd 10-18 h
diabetes. Physiologic insulin therapy, also known as intensive regu lar
insulin therapy, is the ideal insulin regimen as it attempts to 75% NPU 2S% lispro 10-20 min 1-6 h 10-18 h
mimic the actions of normal pancreatic beta cells. Intensive 50% NPU50% lispro 10-20 min 1-6 hd 10-18 h
insulin therapy includes multiple daily injections (MDI) 70% NPA/30% aspart 10-20 min 1-6 h 10-18 h
(<::3 per d ay) w ith an intermediate or long-acting insulin for
basal coverage and multiple preprandial injections through - NPA = neutral protamine aspart; NPH • neutral protam•ne Hagedorn; NPL - neutral
protamine lispro.
out the day w ith analogue or regular insulin. Intensive insu- "The time course of each insulin varies significantly between persons and with in
lin therapy can also include continuous subcuta neous insu- the same person on different days. ThNefore, the time periods listed should be
considered general guidelines only.
lin infus ion (CSII) and meal-time boluses with an insulin
bBoth insulin detemir and insulin glargine car' produce a peak effeCt in some per-
pump. Data support targeting normal glycemic levels with a sons, especially at higher doses.
goal hemoglobin A,c of less than 7% for most persons with Clhe durati on of action for insulin detemir varies depending on the dose g iven.
type 1 diabetes to reduce long-term complications. Long- 0 Premixed insulins con ta ining a larger proport1on of rap1d - or short-acting insulin
term physiologic insulin t herapy reduces early microvascular tend to have larg er peaks occurring at an earlier time than m ixtures contai ning
smaller proportions of rapid and short-acting insulin.
disease by 34% to 76% and reduces cardiovascular events by

8
Disorders of Glucose Metabolism

Regular insulin requires a longer time interval between KEY POINTS


prandial injection and food consumption compared with ana-
• Lifelong insulin therapy is the first-line treatment for
logue insulin due to its longer onset of action. Classic carbohy-
type 1 diabetes; physiologic insulin therapy reduces
drate counting with prandial analogue insulin allows flexibility
early microvascular disease by 34% to 76% in patients
and variety in the types and sizes of meals consumed by
with type 1 diabetes mellitus compared with nonphysi-
adjusting the dose based on the number of carbohydrates
ologic regimens.
ingested. Typically, 1 U of analogue insulin is used to cover
every 10 to 20 g of carbohydrate in the meal. Modified carbo- • Continuous subcutaneous insulin infusion is a cost-
hyd rate counting !or patients who are unwil ling or cannot effective treatment modality and should be considered
count carbohydrates includes tlxed prandial doses of regu lar for select patients w ith type 1 d iabetes mellitus if ade-
or analogue insulin that can be adjusted by SO% based upon quate glycemic control is n ot achieved with adherence
the size of the meal: regular (100% dose) . small (SO% dose), or to multiple daily injection therapy.
large (lSO'Yo dose) . In the setting of pre-meal hyperglycemia.
the prandial dose of insulin determined by the classic or Therapy fo r Type 2 Dia betes Mellitus
modified carbohydrate counting methods can be combined Lifestyle modifications must often be combined with oral
with a supplemental insulin dose to correct the hyperglyce- pharmacologic agents for optimal glycemic control, particu-
mia. There are a variety of methods to determine the supple- larly as type 2 diabetes progresses w ith continued loss of pan-
mental insulin dose needed for correction; however. an creatic beta-cell function and insulin production. Multiple oral
addi tionall U of analogue or regular insulin for every SO mg/dL agents may be required or used in conjunction with noninsu-
(2.8 mmol!L) above the target blood glucose at the pre- meal lin injectable agents or insulin as glycemic control worsens.
measurement is a reasonable starting point. For example, an There are many options tor oral agents, with major differences
additional 2 U of regular or analogue insulin would be admin- in cost, timing of administration, mechanism of action, and
istered for a patient with a blood glucose level of 210 mg/dL side-effect profiles (Table 9).
(11.7 mmoi!L) if the target blood glucose is 1SO mg/dL Metformin is the recommended fi rst-line therapy to be
(8. 3 mmol!L) or less. Wh en administering any prandial or sup- initiated either in conjunction with lifestyle modifications at
plemental insulin doses, t he duration of action of previous the time of diagnosis or within 6 weeks of failing to obtain
analogue or regular insulin injections must be considered, as glycemic control with lifestyle changes alone. Metformin has a
the risk of insulin-slacking and subsequent hypoglycem ia lower incidence of hypoglycemia and weight gain compared
increases if the. dosing is too frequent. Allowing at least 3 to with some of the other oral agents and insulin. Gastrointestinal
4 hours between injections can decrease this risk. PremLxed side effects (such as abdominal cramping or diarrhea) are
insulins containing a tlxed percentage of a long-acting and common with metformin ; initial low doses with gradual
regular or analogue insulin are given twi<;e daily. particularly increases and administration of the tablet following a substan-
in patients who are unable to comply with more frequent daily tial meal can improve tolerance to the medication. Due to the
injections, although greater glycemic variability and hypogly- potential risk of lactic acidosis, contraindications to met-
cemia are concerns when utilizing a nonphysiologic regimen. formi n therapy include semm creatini ne greater than l.S mg/
CSII should be considered for select patients with type 1 dL (133 Jlmoi/L) in men and 1.4 mg/dL (124 Jlmoi/L) in women,
diabetes if adequate glycemic control is not achieved with symptomatic heart failure or liver disease, and illness with
adherence to MDI therapy. CSII may be beneficial in several hemodynamic instability. Metformi n must be withheld for
scenarios, including significant early morning hyperglycemia 48 hours in the setting of intravenous contrast dye. In a non-
("dawn phenomenon"), labile plasma glucose values and fre- hospitalized patient, metformin should be withheld with any
quent DKA frequent severe hypoglycem ia or hypoglycemic illness that may cause dehydration.
unawareness, preconception and pregnancy. or active life- If lifestyle modifications and maximally tolerated doses
styles/patient preference. If a patient is not adherent with of metformin fail to adequately control glucose, additional
insulin injections and blood glucose monitoring. adherence is agents should be added every 3 months until glycemic goals
unli kely to increase because a pump is prescribed; therefore, have been m et. Without strong compa rative-effectiveness
pump therapy is not recommended in the nonadherent patient. data to identify the best class of second line dmgs to be
Cost of the insulin regimen chosen should be weighed implemented. several factors must be considered. Patient
against potential benefits. MDI regimens require more insulin preferences and financial resources are key components to
supplies and glucose monitoring. Insulin analogues demon- developing an individualized treatment plan. Another
strate fewer hypoglycemic events. but cost more than regular important determining factor in selection of the second-line
human insulin. Insuli n pens increase both convenience and drug class is the patient's weight. Weight-neutral dmg classes
cost when compared with insu lin in vials. Insulin pump sup- include a -glucosidase inhibitors and dipeptidyl peptidase-4
plies are expensive compared with other insulin therapies; (DPP-4) in hibitors. If weight loss is a desired effect, gluca-
however, data from several analyses indi cate that overall CSII gon-like peptide 1 (G LP-1) mimetics, pramlin tide, a nd
is a cost-efleclive treatment modality. sodium -glucose transporter-2 (SG LT2) inhibitors are

9
Disorders of Glucose Metabolism

TABLE 9 . Pharmacologic Agents Used to Lower Blood Glucose Levels in Type 2 Diabetes Mellitus
Class Mechanism of Action Effect on Risks and Concerns Long-Term Studies on
Weight Definitive Outcomes

Insu lin• Decreases hepatic Increase Hypoglycemia ; insu lin Decrease in both
glucose production, · allergy (rare) microvascular and
increases peripheral macrovascular event s
glucose uptake
Sulfonylureas Stimulate insulin Increase Hypoglycemia (especially in Decrease in microvascular
(tolbutamide, secretion drugs w ith long ha lf-lives or events but possible increase
chlorpropa mide, in older population s); in macrovascular eve nts
gl ipizide , g lybu ride, weight gain with tolbutamide,
gl iclazide, g limepiride)b chlopropamide, g lyburide,
and gl ipizide; not seen with
gl icl azide o r glimepiride
I Biguanides (metformin)b Decrease hepatic Neutral Diarrhea and abdominal Decrease in both
g lucose production, discomfort; lactic acidosis microvascular and
increase insulin- (rare); contraindicated in macrovascular events
mediated uptake of p resence of progressive
glucose in muscles liver, kidney or cardiac
failure
a-Glucosidase inhibitors Inhibit polysaccharide Neutral Flatulence; abdominal May reduce CVD events
(acarbose, mig litol, absorption discomfort
vog libose)b
Thiazolidined io nes Increase peripheral Increase Fluid retention; heart Unclear whether
(rosig litazone, uptake of glucose, failure; macular edema; p ioglitazone causes net
p iogl itazone)b decrease hepatic osteoporosis (possible harm o r good
glucose prod uction increased risk o f bladder
cancer with pioglitazone)
, Meglitinides Sti mulate insulin release Increase Hypoglycemia None
(repag linide,
nateglinide)b
Amylinomimetics Slow gast ric em ptyi ng, Decrease Nausea; vomiting; ' None
(pramlintide)• suppress glucagon increased hypog lycemic
secretion, increase risk with insulin
sat iety
GLP- 1 mimetics Slow gastric emptying, Decrease Hypoglycemia when used None
(exenatide and suppress glucagon in combination with
liraglutide)" secretion, increase sulfonyl ureas; nausea and
sat iety vomiting; possible
increased risk of pancreatitis
and chronic kidney disease
DPP-4 inh ibitors Slow gastric emptying, Neutral Hypoglycemia when used No increase in ischemic
(sitagliptin, saxagl iptin, suppress glucagon in combi nation with ca rd iovascu lar events;
vildagliptin, linagliptin, secretion su lfonyl ureas; nausea; increased rate of
alogl iptin)b increased risk of infections; hospitalization for hea rt
possible increased risk of failure w ith saxagliptin
pancreatit is
SGLT2 inhibitors Increases kid ney Decrease Hypog lycemia w ith insulin None
(dapagliflozin and excretion of g lucose secretagogues and insu lin;
canagliflozin)b hypotension; kid ney
impairment; hypersensitivity
reactions; increased
candida! genital infections
and urinary tract infections

CVD =cardiovascular disease; DPP-4 = dipeptidyl peptidase-4; GLP-1 =glucagon-like peptide-1; SGLT2 = sodium-glucose co-transporter 2.

"Injection.

"'ral.

10
Disorders of Glucose Metabolism

candidates to consider. Weight gain is like ly wit h the use of and severe hyperglycemia that can lead to volume depletion
insulin, sulfonylureas, thiazolidinediones, and meglitinides. and electrolyte abnormalities.
The risk of hypoglycemia must be considered with the selec-
tion of any therapeutic agent, particularly when it is com- Hospitalized Patients with Diabetes M ellit us
bined with insulin secretagogues or insulin. Gastrointestinal Critically ill patients with type 1 diabetes mellitus will require
side effe cts fro m GLP-1 mimetics a nd pramlintide may insulin therapy upon admission to the hospital. For critically
decrease tolerability for some patient s and should not be ill patients with type 2 diabetes, intravenous insulin infusion
used in patients with gastroparesis. Patients with freq uent therapy should be initiated when plasma glucose levels exceed
candida! genital infections would not be ideal candidates for 180 to 200 mg/dL (10-11.1 mmol/L). Glucose goals on intrave-
SGLT2 inhibitor therapy. nous insulin are 140 to 200 mg/dL (7.8-11.1 mmol/L) with
In sulin therapy should be strongly considered in the set- frequent bedside point-of-care (POC) monitoring every 1 to
ting of symptomatic hyperglycemia or markedly elevated 2 hours for insulin adjustments.
hemoglobin A1c (>8.5% to 9%) at the time of diagnosis or In noncritically ill patients, the ADA and AACE advoca te a
when lifestyle modifications and/or non insuli n therapies fai l premeal glucose goal of less than 140 mg/dL (7.8 mmol!L)
to achieve glycemic goals. The American Association of and random plasma glucose values less than 180 mg/dL
Clinical Endocrinologists (AACE) recommends weight-based (10 mmol/L). Therapy adjustments should be considered when
initiation of basal insulin at initial doses of 0.1 to 0.3 U/ kg. plasma glucose levels are less than 100 mg/dL (5 .6 mmol!L)
The dose should be increased several units every 2 to 3 days and are necessary when glucose values tall below 70 mg/dL
to reach fasting plasma glucose goals, based on the patient's (3.9 mmol/L) to avoid continued hypoglycemia. In contrast, the
SMBG readings. Reductions of insulin doses by 10% to 40% American College of Physicians (ACP) recommends avoiding
should be made in the setting of hypoglycemia wi th insulin glucose levels less than 140 mg/dL (7.8 mmol/ L) owing to the
titrations. If glycemic goals are not met with basal insulin, increased risk of hypoglycemia with tighter glycemic control.
then prandial insulin should be added to the regi men with Insulin is the prefe rred therapy and li kely the safest choice
frequent titration of doses for optimal glucose control. When for achieving inpatient glycemic control. Use of sliding scale
premeal glucose values are not at a patient-specific goal, the insulin alone is not recommended, as it is not physiologic and
p receding prandial insuli n dose should be increased or frequently causes large glucose fluctuations owing to the
decreased by 10% to 20% in the setting of hyper- or hypogly- inherent reactive nature of its dosing, coupled with the near
cemia, respectively. (Also see section on Therapy for Type 1 universal Jag time petween measurement of glucose and injec-
Diabetes Mellitus.) tion of insulin that occurs in most hospitals. The recom-
mended insulin regimen should incorporate both basal and
KEY POINTS
prandial coverage. In the setting of preprandial hyperglycemia,
• For patients with type 2 diabetes mellitus, mettormin is prandial coverage can be supplemented w ith additional insu-
recommended first-line therapy and should be initiated lin (correction factor insu lin). Prandial coverage should
in conjunction with lifestyle modifications; it has a account for the carbohydrates consumed at each meal and be
lower incidence of hypoglycemia and weight gain com- adjusted accordingly. POC glucose monitoring should coincide
pared with some of the other oral agents and insulin. with insulin administration before meals and at bedtime, w ith
• In older patients with type 2 diabetes mellitus of longer overnight measurements to monitor for hypoglycemia only if
disease duration, treatment of severe recurrent hypo- fasting readings are elevated or the patient is symptomatic.
glycemia should take precedence over controlling This glucose monitoring regimen w ill simulate the patient's
hemoglobin A1c values; the increased risks of hypogly- home routine after discharge. POC monitoring should occur
cemia outweigh t he risks of diabetes complications. every 6 hours when a patient is on insulin therapy and receives
nothing by mouth.
Outpatient CSII therapy can be continued if the patient is
physically and mentally able to safely administer this therapy
c:Jinpatient Management under proper. supervision from health care providers wit h CSII
of Hyperglycemia expertise. POC glucose monitoring, basal rates of insulin, and
Inpatient hyperglycemia. defined as consistently elevated patient-initiated bolus amounts of insulin should be docu-
plasma glucose values above 140 mg/dL (7.8 mmol/L), is asso- mented in the medical record.
ciated with poor outcomes. Attempts to decrease morbidity Oral agents and noninsulin injectable agents do not have
and mortality with tight glycemic control (80-110 mg/dL [4.4- safety or efficacy data in the hospital setting. The safest recom-
6.1 mmol/L]) have not consistently demonstrated improve- mendation is to discontinue these agents upon admission to the
ments in adverse outcomes and. in some settings, have shown hospital, although continuation can be considered in a stable
increased rates of severe hypoglycemic events and mortality. patient with glycemic control at goal and no anticipated changes
As a result, revised inpatient glycemic targets are less stringent in nutrition or hemodynamic status. These agents can be par-
than outpatient glucose targets to avoid both hypoglycemia ticularly dangerous in fasting states or when organ perfusion or

11
Di sord e rs of Glucose M et abolism

CJ function
I
is compromised. Resumption of these medications
may be considered once a patient is stable with regular activities
h . f"h . I d. h C]
corrected . cognitive function begins to decline a nd can be
rapidly followed by loss of consciousness, seizures, a nd
CONT. am nu t ntmn
. . or at t e time o osp1ta ISC arge. death. Relat ive hypoglycemia occurs when a patient has
KEY POINTS
symptoms of hypoglycemia but the plasma glucose level is
gre?ter than 70 mg/dL (3.9 mmol!L) . This can occur w it h
• For critically ill patients with type 2 diabetes mellitus, rapid decreases in glucose or w ith correction of glucose to
intravenous insulin infusion therapy should be init iated near- normal glycemic levels in a patient w it h a history of
when plasma glucose levels exceed 180 to 200 mg/dL pro longed hyperglycemia (plasma glucose >200 mg/dL
(10- 11. 1 mmol/L); glucose goals on int ravenous insu -
[11. 1 mmo!IL]) . Relative hypoglycemia ca n be di minished if
lin are 140 to 200 mg/dL (7.8 -11.1 mmoi/L) with fre- glucose levels are mai ntai ned closer to normal ranges and if
quent bed side point-of-care monitoring every 1 to t reatment to goal glucose level is achieved over a longer
2 hours. period of time in patients with a history of prolonged uncon-
HVC • For noncritically ill patients, basal and prandial subcu- trolled diabetes.
taneous insulin is the preferred and safest choice tor The etiology of hypoglycem ia can be quite variable.
ach ieving in patient glycemic; oral agents and noninsu- Exercise can lead to hypoglycemia if app ropriate measures
lin injectable agents do not have proven safety or effi- are not taken to avoid it. Prior to exercise, consumption of a
cacy data in the hospital setting. snack w ith 15 to 30 g of carbohyd rates can hel p reduce t he
ri sk of hypoglycemia. or a patient can reduce the dose of
CJ Hospitalized Patients Without Diabetes Mellitus prandial insuli n given at the meal prior to the planned exer-
cise, if on an MDI regimen. A snack with complex carbohy-
Hyperglycemia as a result of acute stress related to illness.
drates is often required after prolonged exercise to replenish
concomitant medicat ions. or enteraLparenteral nut rition ca n
glycogen stores si nce glucose u tilization can be prolonged in
occur in patients without a previous history of glucose abnor
mu scles and th e liver. In overweight/obese pat ients.
malities. The glycemic goals and glucose-management strate-
decreasing t he insulin dose instead of ingesting snacks
gies in th is population should follow those for hospitalized
befo re exercise can avoid addition al weight gain. Poor tim-
patients with diabetes. l lyperglycemia in hospital ized patients
ing or skipping of meals or consu mption of smaller a mounts
may also ind icate the presence of previously und iagnosed
of food wit hout an adjustment to insu lin doses or oral hypo
diabetes. Measurement of hemoglobin A1c in hyperglycemic
glycemi c agents can cause hypoglycemia. Use of a non-
non hospitalized patien ts, if feasible. can provide insight into
physiologic sliding scale insul in regimen or use of a n
the length of the h yperglycemia. A hemoglobin A1c level
aggressive supplemental insu li n correct ion factor regime n
greater than 6 .5'i'o suggests long-standing hyperglycemia.
is often the etiology of hypoglycemic events. A reduction in
Follow up diabetes screening and e1re should be implemented
kidney function. particularly in elderly pa tients, can ·
arter discharge from the hospital.
decrease clea rance of insulin or insulin secretagogues and
lead to prolonged hypoglycem ia. Alcohol consumption can
cause delayed hypoglycemia.
Management of Hypoglycemia Treatment of hypoglycemia is twofold: immediate cor-
Hypoglycemia in Patients with Diabetes Mellitus rection of hypoglycem ia and prevention of future events. If a
l lypoglycemia is a com mon complication of intensi\re thera- patient is conscious, IS to 20 g of a carbohydrate with glucose
peutic regimens in patients with diabetes mellitus, often li mit- should be consumed. Glucose tablets or glucose gel are ideal
ing the ability to safely reach glycemic goals for many patients. treatment regimens. The blood glucose level should be
Avoidance of hypoglycemia prior to focusing on a patient's checked again after 15 minutes, and consumption of IS to
hemoglobin Ale goal is of utmost importance because of the . 20 g of glucose should occur again if the hypoglycem ia does
sign ificant morbidity and mortality associated with low not improve to greater t han 70 mg 'dL (3. 9 mmoll L). Since the
plasma glucose levels. effects of the insulin or oral hypoglycemic agents are likely
Hypoglycemia is defined as a plasma glucose level Jess still present. a meal or snack should be consumed after the
than 70 mg/dL (3 .9 mmol / L). Insuli n secretion ceases when glucose has been corrected to avoid continued hypoglycemia.
the glucose level falls below 80 mg/dL (4 .4 mmoL L). Every patient with diabetes on medications associated wit h
Hyperadrenergic sympto ms begi n to alert the patien t to hypoglycemia should receive a prescription tor a glucagon kit,
hypoglycemia through an increase in heart rate. sweating, which should be used when oral consumption of glucose is
tremors, hunger. and anxiety when glucose levels decline. not possible or safe.
Typically, the body responds to hypoglycemia by secreting Relaxing the glycemic targets and hemoglobin A1c goals
counterregulatory hormones. such as glucagon. epineph ri ne, and reducing doses of therapeutic agents will decrease the risk
norepinephrine. cortisol. and growth hormone. in succes of future hypoglycemia. A review of a patien t's diabetes self-
sion based on the escalating degree of hypoglycemia. If the management skills ca n also help ide ntify recu rring risk fac tors
counterrcgu latory measures fail or the hypoglycemia is not fo r hypoglycemia. Cl

12
Disord e rs of G l ucose Metabolism

KEY POINTS Fasting Hypoglycemia


• Hypoglycemia is defined as a plasma glucose level less Suspected hypoglycemia that is either spontaneous or begins
than 70 mg/dL (3 .9 mmol!L) and is associated with sig- after a fast should be evaluated by the following simultaneous
nificant morbidity and mortality. laboratory measurements: glucose. insulin. C peptide, proin
. sulin. hydroxybutyrate, and insulin secret<Jgoguc screen.
• Relaxing the glycemic targets and hemoglobin A10 goals
C-peptide and proinsulin are measures of the endogenous
and reducing doses of t herapeutic agents w ill decrease production of insulin. is suppressed by
the risk of future hypoglycemia. endogenous and exogenous insulin, but wou ld be unsup
pressed in a normal physiologic state of hypoglycem ia or in a

CJ Hypoglycemia in Patients Without


Diabetes Mellitus
CONT.
non-insulin-mediated condition. If hypoglycemia is not pre
sent at the time of evaluat ion, a 72 hour tast is ind icated,
Hypoglycemia in patients w it hout diabetes is rare. thus evalu- which is typically performed in consultation with an endocri
ation for pathologic hypoglycemia shou ld only occur when nologist. This test involves measurement of the above
Whipple triad is present: symptomatic hypoglycemia. docu mentioned laboratory values ever) 6 hours unti l the plasma
mentccl hypoglycemia at 55 mg 'dL (3. 1 mmol L) or lower, and glucose level reaches 60 mg dL (3.3 mmol L) and subse
prompt symptomatic relief w ith correction of hypoglycemia. quently every I to 2 hours un til specific plasma glucose,
Hypoglycemia should not be con fi rmed with POC glucose symptom. or time criteria arc mel. This test also involves
monitors, but instead with a more accurate established labora- measuring the response to admin istration.
tory method. I Iypoglycemia in patients without d iabetes is Evaluation for anti insulin antibodies can detect the rare con
usually related to drugs, illness. hormonal deficiency. non dition of insulin autoimmune hypoglycemia as the underly
b let cell tumor, endoge nous hyperinsu linism noninsulinoma. ing etiology fo r the hypoglycemia.
pancreatogenous hypoglycemia. depletion of hepatic glycogen
stores. or alcohol ingestion. Diagnostic studies should be Postprandial Hypoglycemia
obtained during a spontaneous hypoglycemic episode or dur Postprandial hypoglycemia without a history of a prior bariat
ing an attempt to recreate a scenario known to cause hypogl) ric procedure is rare. It typical!) occu rs w ith in S hours of tiJOd
cemia, such as prolonged tasting or after a mi xed meal. which consumption. A mixed meal tolerance test is USU<llly per
consists of the type of food that induces the hypoglycem ia. fo rmed in consultation with an endocrinologist and measures
typically a simple carbohydrate-rich meaL such as orange the glucose level as symptoms occur. Glucose. insulin. proin
juice, pancakes, and syrup. Hypoglycemia has classically been sulin, and C peptide levels arc measured prior to the meal and
categorized as occurri ng in the fasting versus postprandial repeated at 30-minute intervals or at the time of symptomatic
state. although the etiologies of each of these classifications of hypoglycemia (blood glucose level <60 mg dL [:3.:3 mmol L])
hypoglycemia are not mutually exclusiye. The differential w ith in the 5 hours after meal consumption. If symptomatic
diagnoses based on the laboratory test results are found in hypoglycemia occurs. insulin aritibodies are measured and an
Table 10 . Imaging studies should not occur unless biochemical oral hypoglycemic agent screening test is obtained . Treat men!.
evidence of endogenous hyperinsulinism is confirmed and is vvith or without the detection of pathologic hypoglycemia on
related to a tumor or pancreatic abnormalil). the mixed meal test. often involves small, frequent complex

TAB LE 10. Differential Diagnosis of Spontaneous Fasting Hypoglycemia• in a Patient Without Diabetes
Diagnosis Serum Plasma Plasma Serum Serum Insulin Urine or Blood Metabolites
Insulin C-Peptide Proinsulin Antibodies of Sulfonylureas or
Meglitinides

lnsul inoma i i i l Negative Neg at ive


Su rreptitious use of i i i l Negative Positive
sulfonylureas
of meg liti nides
Surreptitious use of i l J. l Negative Negative
insulin
Insulin autoimmune i i i l Positive Negat ive
hypoglycemia

'Symptom atic hypoglycem ia, fasting plasma glucose 55 mg/dl (3.1 m moi/L) or lower, and prompt sympto mati c reli ef with co rrection of hypo glycemia (Whip ple triad).

Data fro m Cryer PE, Axelrod L, G rossman AB, e t a L Evaluation and manageme nt o f adult hypogl ycemic d isord ers: an Endocrine Society Clinical Practice G uideline . J Clin
Endocrino l Metab. 2009 Mar;94(3):709-28.1PMID: 19088 155 ]

13
Di sord e rs of Glucose M etabolism

Cl sensation
meals composed of protein, fa t, and carbohydrate to avoid the
CONl
of hypoglycemia.
ketone laboratory measurements often use the nitroprusside
reaction, w hich only estimates acetoacetate and acetone lev-
els t hat may not be elevated initially. Although hyperglyce-
mia is th e typical finding at presentation w ith DKA, patients
Acute Complications of can present with a range of plasma glucose values, including
those in the normal ra nge (Figure 1) . The anion gap is ele-
Diabetes Mellitus
vated. Stress- related mild leukocytosis is often present.
Diabetic Ketoacidosis and Hype rglycemic lligher levels of leukocytosis may indicate an infectious pro-
Hyperosmolar Syndrome cess as the etiology of the hyperglycemia. Serum sodium
Diabetic ketoacidosis (DKA) and hyperglycemic hyperosmo- levels can be low due to osmotic shifts of water from the
lar syndrome (! Il lS) are acute complications of uncontrolled intracellula r to extracellular spaces. Normal or elevated
hyperglycemia with life-t hreatening consequences if not serum sodium levels are indicative of severe volume deple-
recogn ized and treated early. DKA typically occurs in the tion. Serum potassium levels may be elevated due to shifts
setting of hyperglycemia with re lative or absolute insulin fro m the intracellular to ext racellular spaces due to ketoaci-
deficiency and a n increase in counterregulatory hormones. dosis and the absence of suftlcien t insulin . Normal or low
Sufficient amounts of insulin a re not present t o suppress potassium levels on presentation indicate low potassium
lipolysis and oxidation of free fatty acids. which results in stores in the body w ith need for correction prior to initiation
ketone body product ion a nd subsequent metabolic acidosis. of insulin therapy to avoid cardiac arrhythmias. Seru m amyl-
DKA occurs more frequently with type 1 diabetes. although ase a nd lipase levels also can be elevated in the absence of
10% to 30'Yu of cases occur in patients w ith type 2 diabetes. pancreatitis.
HHS occurs in the setting of partial insulin deficiency that is HHS typically presen ts with normal or small amounts
more typical of type 2 diabetes. There is sufficient insulin in of urine or serum ketones. Plasma glucose values in HHS are
patients with HHS to suppress lipolysis and production of typically greater t han in DKA and ca n exceed 1200 mg/dL
keto ne bodies, but inadequate amoun ts to prevent t he (66.6 m mol!L) . The serum osmolality is elevated greater
hyperglycemia, dehydration, and hyperosmolali!y charac- than 320 m Osm / kg Hp . The serum bica rbonate level is
teristic of HHS. greater t han 18 mEq / L (18 mmol!L), and the pH remains
Several risk factors can precipitate the development of greater than 7.3.
extreme hyperglycemia: infection, intentional or inadvertent Treatment ofDKA.and HHS requires correct ion of hyper-
insulin therapy no11adherence, myocardial infarction. stress, glycemia with intravenous insulin infusions, frequent mon i-
trauma, and confou nding medications. such as glucocorti- toring and replacement of electrolytes, correction of hypov-
coids or atypical antipsychotic agents. In addition. DKA may olemia with intravenous fl uids, and possible correct ion of
be the initial clinical presentation in some patients with previ- acidosis (Table 11) . The ICU is the best place fo r management ·
ously undiagnosed type 1 or type 2 diabetes. An'illness or event of severe hyperglycem ia because of the frequent monitoring
that leads to dehydration will often precipitate the hyperglyce- required with intravenou s insulin therapy, the need for mon-
mia associated with HHS. itoring fo r potential electrolyte-induced a rrhythmias, and the
Symptoms of extreme hyperglycemia in DKA and HHS potential fo r rapid decompensation. Plasma glucose levels
include polyuria, polydipsia, unintentional weight loss, vomit- should be monitored initially every hour wh ile on insulin
ing, weakness, and mentation changes. De hydra lion and met- infusion therapy. Electrolytes should be monitored every 2 to
abolic abnormalities worsen as hyperglycemia progresses, 4 hours. depending on the initial electrolyte deficits and level
which can lead to respiratory fail ure. lethargy, obtundation, of acidosis. Cl
coma, and death. DKA can occur within several hours of the
KEY POINTS
inciting event. The development of HHS is less acute than DKA
and may take days to weeks to develop. HHS typically presents • The development of hyperglycemic hyperosmolar syn-
with more extreme hyperglycemia and mental status changes drome is less acute than that of diabetic ketoacidosis
compared with DKA. and may take pays to weeks to develop; however, hyper-
The init ia l evaluation of severe hyperglycemia incl udes glycemic hyperosmolar syndrome typically presents
serologic studies (plasma glucose, serum ketones. blood with more extreme hyperglycemia and mental status
urea n itrogen. creatinine, electrolytes, calculated anion changes compared with diabetic ketoacidosis.
gap, arterial blood gases, osmola lity. complete blood cou nt • Treatment of diabetic ketoacidosis an d hyperglycemic
w ith d ifferential, blood cultures), urine studies (ketones, hyperosmolar syndrome requires correction of hyper-
urinalysis, urine culture), chest radiograph. and a n elec glycemia with intravenous insulin infusions, frequent
trocardiogram. monitoring and replacement of electrolytes, correction
Urine and serum ketones are elevated in DKA; however, of hypovolemia with intravenous fluids, and possible
a negative measurement initially does not exclude DKA. correction of acidosis.
is t he major ketone body in DKA, but

14
Disorders of Glucose Metabolism

28
Hyperosmolar
26 Normal Nonketotic
Range Hyperglycemia
24

22
,
20

18

"'0<: 16
-e
"'v
:.0 14
E
::l
; 12
II>

10

8
Classic

6 DKA
Euglycemic
DKA
4

0
100 200 300 400 500 600 700 800 900 1000
5.6 11.1 16 .7 22.2 27.8 33.3 38.9 44.4 50.0 55.5
Plasma glucose (mg /dl [mmoi/ L])

FIGURE 1 . Spectrum of metabolic decompensation that occurs in diabetic ketoacidosis. DKA =diabetic ketoacidosis.

TABLE 11. Management of Hyperglycemic Crisis (DKA and HHS)


Fluids Insulin (Regular) Potassium Correction of Acidosis

Assess for volume status, then Give regular insulin, 0.1 U/kg, as Assess f or adequate kidney If pH is <6.9, g ive sodium
g ive 0.9% sal ine at 1 Uh initial ly an intravenous bolus followed function, wit h adeq uate urine bicarbonate, 100 mmo l in
in all patie nts, and continue if by 0.1 U/kg/h as an intravenous output (approximately 50 4 00 ml o f water, and
patient is severely hypovolemic. infusion; ifthe p lasma g lucose mUh). If serum potassiu m is pot assium chloride, 20 m Eq ,
Switch to 0.45% normal saline level does not decrease by < 3.3 mEq/L (3 .3 m moi/L}, do infused over 2 hours. If pH is
at 2 50-500 mUh if corrected 10% in the first hour, g ive an not start insul in but inst ead 6.9 or g reater, do not give
serum sod iu m level becomes additional bolus of 0.14 U/ kg give intravenous potassium sodium b icarbonate.
norma l o r hig h. When the and resu me previous infusion ch lo ride, 20-30 mEq/ h, th rough
plasma g lucose level reaches rat e; when the plasma g lucose ' a central line catheter until the
200 mg/dl ( 11 .1 mmoi/L) in level reaches 200 mg/dl(11 .1 serum potassium level is
patients with D KA or 300 mg/ mmoi/L} in DKA and 300 mg/dl >3.3 mEq/L (3 .3 m moi/ L); then
dl (16.7 mmo i/L) in HHS, switch (16.7 mmoi/L) in HHS, reduce to add 20-30 mEq.of potassium
to 5% dextrose with 0.45% 0 .02-0.05 U/kg/h, and maintain ch loride to each liter of
normal saline at 150-2 50 mUh. the plasma gl ucose level intravenous fluids to keep the
between 150-200 mg/d L serum potassium level in the
(8.3-11 .1 mmoi/L) until anion 4.0-5.0 mEq/L(4 .0-5.0 mmoi/ L)
gap acidosis is resolved in DKA. range.lf the serum potassium
level is >5.2 mEq/L(5.2 mmoi/L),
do not give potassium chloride
but instead start insu lin and
intraveno us fluids and check
th e serum pot assiu m level
every 2 hours.

DKA = d iabeti c ketoacidosis; HHS = hyperglycemic hyperosmolar syndrome .

15
Disorders of Glucose Metabolism

Chronic Complications The most recent American College of Cardiology/


American Heart Association guideli nes base treatment recom-
of Diabetes Mellitus mendations for patients with diabetes on age, the presence of
Cardiovascular Morbidity atherosclerotic cardiovascular disease (ASCVD), or estimated
Cardiovascular disease (CVD) is a major contributor to mor- 10-year ASCVD risk using the Pooled Cohort Equations; a spe-
bidity and mortality among patients with diabetes mellitus. cific goal LDL cholesterol level is no longer used in these guide-
Diabetes alone is an independent risk factor for CVD and is lines. Treat patients w ith diabetes and known cardiovascular
considered a CVD equivalent. Concomitant risk factors in or other vascular disease with h igh-intensity stalin therapy. In
patients with diabetes, such as hypertension, obesity. and dys- the absence of known cardiovascular or vascular disease, pro-
lipidemia, also con tribu te to the development of CVD and vide high intensity statin therapy to patients with diabetes
should be identified early through screening (Table 12). if the LDL cholesterol level is greater tha n 190 mg/dL
The 8 111 Joi nt National Commi ttee (JNC-8) recently (4. 9 mmo!IL) or the 10-year ASCVD risk is equal to or greater
revised its recom mended blood pressure goals for patients than 7.5%. Provide moderate-intensity statin therapy for
with diabetes to 140/90 mm Hg or less, citing a lack of data patients w ith diabetes and a 10-year ASCVD risk less than
to support lower targets. In contrast. the American Diabetes 7.S'Yo. Consider withholding statin therapy in patients w ith
Association (ADA) recommends a blood pressure goal of less diabetes younger than 40 years without additional cardiovas-
than 140/80 mm Hg. The ADA advocates for a lower systolic cular risk factors. In contrast, ADA guidelines continue to
blood pressure (<130 mm Hg) in select patients (young, long recommend an LDL cholesterol goal in patients with diabetes
life expectancy, increased risk of stroke), ifthis can be accom- of less than 100 mg/dL (2.6 mmol/L). with the option of LDL
plished safely. Although JNC-8 does not specify use of an ACE cholesterol less t han 70 mg/dL (1.8 mmol / L) in patients with
inhibitor or an angiotensin receptor blocker (ARB) as initial clinical ASCVD. Therefore, it is recommended that statin ther-
therapy fo r patients w ith diabetes and hypertension in the apy be added to lifestyle modifications in patients with diabe-
absence of chro nic kidney disease, the ADA recommends tes who have clinical ASCVD. are older than 40 years of age
preferential use of these agents in treati ng hypertension in w ith CVD risk fac tors, or are younger t han 40 years of age with
these patients. LDL cholesterol not at goal.

TABLE 12. Screening Recommendations for Chronic Complications of Diabetes Mellitus


Chronic Clinical Situation When to Start Screening Frequency Preferred Screening Test
Complication Screening

Retinopathy Type 1 diabetes • At 5 years after Annually" Dilated and comprehensive


diagnosis eye examination
Type 2 diabetes At diagnosis Annually" Dilated and comprehensive
eye examination
In p regnant women with First trimester Every trimester and then Dilated and comprehensive
either type of diabetes closely for 1 year postpartum eye examination
In women with either During preconception Same as recommendations Dilated and comprehensive
type of diabetes planning for pregnant women o nce eye examination
planning to conceive conception occu rs
Nephropathy Type 1 diabetes At 5 years after Annua llyb A lbumin-creatinine ratio o n
diagnosis random spot urine
Type 2 diabetes At diagnosis Annua llyb Albumin-creatinine ratio o n
random spot urine
Neuropathy Type 1 diabetes At 5 years after Annually 1 0-g monofi lament, 128-Hz
(distal symmetric diagnosis tuning fork, ankle reflexes
polyneuropathy)
Type 2 diabetes At d iagnosis Annually 1 0-g monofilament, 128-Hz
tuning fork, ankle reflexes
Cardiovascular Hypertension At d iagnosis Every visit Blood pressure
disease measurement
Dyslipidemia At d iagnosis Fasting lipid profile

•It IS reasonable to screen every 2 years if no diabetic retinopathy is present and to screen more often than annually if d iabetic retinopathy is advanced or progressing rap idly.

hThe A merican D1abetes Assoctat1on guidelines state that it is re asona b le to assess p rogressio n of disease and response to therapeutic interve ntions with continue d monitoring o f
uri ne albumin excretion.

cit is reasonable to screen every 2 years if lipid parameters arc at goa l.

16
Disorde rs of Glucose Metabolism

KEY POINTS dom spot urine collection or a 24-hour urine collection.


Persistently elevated levels of urine albumin excretion are
• High-intensity stat in therapy is indicated for patients
defined as greater than or equal to 30 mg/g in a spot urine
w ith diabetes and known cardiovascula r or vascular
measurement or 30 to 299 mg/ 24 h and greater than or equal
disease; LDH cholesterol greater than 190 mg/dL
to 300 mg/24 h. Urine albumin levels should be elevated on
(4. 9 mmol/ L). or atherosclerotic cardiovascular disease
multiple samples over 3 to 6 months to diagnose albuminuria,
10-year risk of equal to or greater than 7.5%.
as false-positive elevations can occur in the setting of illness,
• Moderate intensity statin therapy is indicated for patients menstruation, recent exercise, extreme hyperglycemia or
with diabetes 40 years of age and older and an atheroscle- hypertension, and heart failure. Screening timelines for urine
rotic cardiovascular disease 10-year risk less than 7.5%. album in excretion are found in Table 12. An nual measure-
ments of serum creatinine and an estimated glomerular filtra-
Diabetic Retinopathy tion rate (GFR) can be utilized in conjunction with the urine
Among adults aged 20 to 74 years, diabetic retinopathy is the albumin measurement to determine the stage of chronic kidney
leading preventable cause of blindness. Changes associated disease. When the estimated GFR is less than 30 mL/min/
with nonproliferative retinopathy include retinal thickening 1.73 m 2• a referral to a nephrologist is recommended.
from macular edema, infarcts (resulting in "cotton wool" spots Diabetic nephropathy can be preven ted or delayed with
or soft exudates) . hard exudates, and hemorrhages. With pro- optimal plasma glucose and blood pressure controL In non-
liferative retinopathy, neovascula rization occu rs secondary to pregna nt normotensive patients with persistently elevated
chronic retinal ischemia. These new vessels may rupture. urine albumin excre tion. an ACE inhibitor or angiotensin
causing intraocular hemorrhage and subsequent fibrosis and receptor blocker (ARB) is recommended to decrease progres-
retinal detachment. sion of nephropathy. In nonpregnant hypertensive patients
Risk factors for diabetic retinopathy include long-term with persistently elevated urine albumin excretion and hyper-
diabetes, poorly controlled diabetes. hypertension. a nd tension, the ACE inhibitor or ARB should be titrated to achieve
nephropathy. Retinopathy can be accelerated in pregnant a blood pressure goal of less than 130/80 mm Hg. Measurement
women with type 1 diabetes. Rapid improvements in glycem ic of u rine albumin annually after initiation of therapy with an
levels for pregnant women and nonpregnant patients can tem- ACE inhibitor or ARB is reasonable to assess disease progres-
porarily worsen preexisting retinopathy. sion and therapeutic response as evidenced by stabilization or
Screening guideli nes vary depending on the type of dia- reduction of urine albumin excretion. Data are conflicting
betes. time of dfagnosis. and pregnancy status (see Table 12). regarding the ability of low-protein diets to slow the progres-
Optimal blood glucose and blood pressure control can sion of kidney disease, but these diets may be considered if
prevent or delay the progression of diabetic retinopathy. Laser nephropathy progresses while using an ACE inhibitor or A_RB
photocoagulation is used to treat retinopathy as severity pro- or after ach ieving target plasma glucose and blood pressure
gresses. Focal laser photocoagulation of the reti na can restore goals. ACE inhibitor/ARB combination treatment is n ot rec-
some vision and reduce the risk of further vision loss with ommended.
macula r edema. Pa nretinal laser photocoagulation reduces
KEY POINTS
continued vision loss in proliferative diabetic retinopathy and
severe nonproliferativc diabetic retinopathy. Laser photocoag- • Elevated urinary albumin excretion is defmed as greater
ulation can also reduce the risk of retinopathy progression than or equal to 30 mg/g in a spot urine measurement:
associated with pregnancy. lnlravitreal injections ofanliangio- annual measurements of serum creatinine and an esti-
genic agents, such as vascular endothelial grow th factor inhib- mated glomerular filtration rate can be utilized in con-
itors, may also be included in the management of proliferative junction with the urine albumin measurement to deter-
retinopathy and macula r edema. mine the presence of diabetic nephropathy and. if
present. the stage of chronic kidney disease.
KEY POINT
• In nonpregnant normotensive patients with persistently
• Optimal blood glucose and blood pressure control can increased urine albumin excretion, an ACE inhibitor or
prevent or delay the progression of diabetic retinopathy: angiotensin receptor blocker is recommended to
however, laser photocoagulation is used to treat diabetic decrease progression of diabetic nephropathy.
retinopathy as the severity progresses.

Diabetic Nephropathy Diabetic Neuropathy


Diabetic nephropathy not only increases the risk of progres- There are several categories of diabetic neuropathy, which may
sion to end-stage kidney disease. but is also a risk factor for present separately or in combination. Symptoms of diabetic
CVD. neuropathy depend on the nerve(s) or nerve root that is
Measurement of' increased protein excretion can be per- affected and may present as focal or diffuse disease. Achieving
tbn11ed by two methods: albumin-creatinine ratio on a ran- optimal glycemic control early in t he course of diabetes can

17
Disorders of the Pituitary Gland

prevent the development of neuropathy, and sustained optimal tions, pain, ankle reflexes, and foot deformities. Patients
glucose levels can delay the progression of neuropathy. should inspect their feet daily tor early detection of any abnor-
Distal symmetric polyneuropathy (DPN) is the most mality and wear appropriate foo lwear. Allhough patients with
common form of diabetic neuropathy. It is characterized by a diabetes have different footwear needs, the selection of shoes
"stocking-glove" distribution that ascends proximally. DPN should take into account several important fac tors: intended
frequently presents as a sensation of numbness, tingling, use, plantar protection, shape and fit on the foot, and stability
burning, heaviness, pain, or sensitivity to light touch. The issues (see MKSAP 17lnfectious Disease).
pain may worsen at night and w ith walking. Muscle weakness
may occur in severe cases. DPN is a risk factor for muscle and Hypoglycemic Unawareness
joint deformities, such as Charcot foot, and foot ulcers. DPN Frequent severe hypoglycemia can diminish the abilily to
evaluation includes assessment of ankle reflexes, vibration detect life-threatening hypoglycemia. This u nawareness is
sensation with a 128-Hz tuning fork, and touch w ith a 10-g caused by fa ilure of the release of counterregulatory hormones
monofilament and pinprick. Screening intervals are found in to trigger an autonomic response to decreased glucose levels.
Table 12. Management of DPN symptoms may require one or Continual avoidance of hypoglycemia for several weeks or
more classes of drugs, incl uding antidepressants (amitripty- longer may help restore the body's ability to detect hypoglyce-
line, venlafaxine, duloxetine, paroxetine), anticonvulsants mia. Plasma glucose levels should be kept greater than 150 mgl
(pregabalin, gabapentin, valproate) , or capsaicin cream. dL (8.3 mmol!L) during the time period when restoration of
Autonomic neuropathy can affect a single organ or multi- hypoglycemic symptoms is the goal to avoid unintended and
ple organs. Symptoms may include gastroparesis, diarrhea, unexpected hypoglycemia. Continuous glucose monitoring
constipation , neurogenic bladder, abnormal hidrosis, and systems can be useful for hypoglycemia management by alert-
erectile dysfunction . Cardiac symptoms include resting sinus ing the patient to rapid decreases in glucose levels to allow
tachycardia, orthostatic or postprand ial hypotension, exercise prompt correction and avoidance of hypoglycemia (see Self-
intolerance, and silent myocardial infarction. Cardiovascular Monitoring of Blood Glucose) .
autonomic neuropathy is an independent risk factor for mor-
KEY POINT
tality, which underscores the need to reduce other cardiovas-
cular risk factors in these patients. • Distal symmetric polyneuropathy (DPN) is the most com-
Diabetic amyotrophy occurs in older patients or those mon form of diabetic neuropathy, and it presents as a sen-
with type 2 diabetes, and may be due to infarcts in the major sation of numbness,or burning pain in a stocking-glove
nerve trunks of the leg. It can present acutely w ith severe pain distribution; management may require one or more
and asymmetric proximal weakness or pain in the leg, weight classes of drugs, including antidepressants (amitriptyline,
loss, and autonomic neuropathy. Partial remission may occur venlafaxine, duloxetine, paroxetine), anticonvulsants
over many months. Without any approved treatments for dia- (pregabaJin, gabapentin, or capsaicin cream
betic amyotrophy, management consists of symptomatic ther-
apy tor neuropathic pain and ambulatory aids, if necessary.
Mononeuropathies can occur acutely with a cranial or Disorders of the
peripheral distribution. There are no specific treatments for
these mononeuropathies, as the symptoms usua) resolve
Pituitary Gland
within a few months. Nerve compression syndromes, such as Hypothalamic and Pituitary
carpal tunnel syndrome or peroneal palsy, occur frequently in
patients with diabeles. See MKSAP 17 Neurology for more
Anatomy and Physiology
information regarding diabetic neuropathy. Referral to a neurolo- The anterior pituitary is made up of glandular tissue t hat
gist tor electrodiagnostic testing or evaluation for nondiabetic- , receives its blood supply fro m the hypothalamus through the
related etiologies should occur w ith severe, rapidly progressive, hypothala mic-pituitary portal plexus, whereas the posterior
or atypical neuropathies. pituitary consists of direct extension of neurons from the
hypothalamus. Both the portal blood system and the hypo-
Diabetic Foot Ulcers thalamic neurons transverse from the hypothalamus to the
Diabetic foot ulcers increase the risk for amputation and subse- pituitary by way of the pituitary stalk. The hypothalamus
quent morbidity and disability. The etiology is often multifacto- regulates anterior pitu itary gland function by synthesizing
rial. Loss of peripheral sensation can result in significant injuries specific stimulating and inhibiting hormones, which are
that may be undetected by the patient. Peripheral at1erial disease released in the portal blood. Posterior pituitary hormones are
predisposes to the development of lower extremity ischemic synthesized in the hypothalamus and travel through hypotha-
ulcers and impairs healing. Altered leukocyte function from lamic neurons to be secreted by the posterior pituitary gland.
hyperglycemia can impede wound healing of injuries. The anterior and posterior lobes are joined by the Rathke
Clinicians should evaluate t he feet at least annually to pouch. Table 13 lists the pituitary hormones and initial testing
assess fo r pedal pulses, sensation, ulcers, skin or nail infec- fo r suspected pituitary hormone excess or deficiency.

18
Disorders o f the Pituitary Gland

Pituitary Hormone Peripheral Hormone

ACTH Cortisol 24 hour urine free cortisol (x2) OR nocturnal salivary


cortisol (x 2) OR overnight low dose dexamethasone test
ADH ADH Simu ltaneous serum, urine sod ium, and urine osmolality
GH IGF-1 IGF-1
TSH Thyroxine, TSH, free (or total) thyroxine
triiodothyronine

Pituitary Hormone Deficiency


Pituitary Hormone Peripheral Hormone Initial Test(s) Confirmatory Test•

ACTH Cortisol Simultaneo us ACTH , cortisol ACTH stimulation test


ADH ADH Simultaneous serum sod ium, urine and serum osmolality Water deprivation test
LH and FSHb Sex hormones Simultaneo us LH, FSH, testosterone (male). est riol (female)
TSH Thyroxine, Simultaneo us TSH, free (or tota l) thyroxine
tri iodothyronine

ACTH • adrenocorticotropic hormone; ADH = antidiuretic hormone; FSH • follicle -stimulating hormone; GH - growth hormone; IG F-1 • insulin-like growth factor 1;
LH - luteiniz1ng hormone; TSH - thyroid -stimulating hormo ne.

•See Table 15 for additio nal informatio n on co nfirmatory testing for pituitary dysfunctio n.

bRoutine testing for deficiency is not recommended without specific signs of deficiency such as amenorrhea. gynecomastia, or impotence.

The anterior pituita ry gland secretes and releases six hor- gonadotropin-releasing hormone (GnRH)_ Ll-1 and FS!-1 regu-
mones: adrenocorticotropic hormone (ACTH), thyroid-stimu- late normal male and female reproductive function. GI-l pro-
lating hormone (TSH), the gonadotropins- luteinizing duction is regulated by somatostatin . Prolacti n controls
hormone (LH ) an d follicle-stimulating hormone (FSH), lactation and is inhibited by dopamine.
growth hormone (GH) , and prolactin. ACT!-1 is released in The posterior pituitary gland secretes oxytocin, which is
response to corticotrophin-releasing hormone (CRH) and acts necessary fo r parturition. and anridiuretic hormone (ADH_
on the adrenal gla nds to promote the synthesis and secretion also called vasopressin)_ wh ich regulates water balance_ .
or cortisol. TSH is released in response to t hyrotropin-releas- The pituitary gland is posterior and superior to the sphe-
ing hormone (TR!-1) and acts on the thyroid to stimulate thy- noid sinus, which provides surgical access to the gland, and is
roid hormone production_ LH and FS!-1 are differentially adjacent to the optic chiasm, the carotid arteries, and the cav-
released from the pituitary gland in response to pulses of ernous sinuses (Figure 2).

FIGURE 2 . Acorona l MRI{left) and sag ittal MRI(right) showing the pitu itary gland (open arrow), pituitary stalk(th in arrow), optic chiasm(a rrowhead), sphenoid sinus
(star), and carotid artery (curved arrow).

19
Disorders of th e Pituitary Gland

The pituitary gland is best imaged using MRI w ith gado- Pituitary tumors are almost always nonmalignanl. Two
linium. Because the normal pituitary is relatively small, a exceptions arc metastatic disease and the very rare pituitary
dedicated pituitary protocol that obtains thin MRl slices carcinoma. Additional kinds of noncancerous pituitary lesions
t hrough the sella is used. include craniopharyngiomas, meningiomas, and Rathke cleft
cysts. Inflammatory and infiltrative disorders, including lym-
phocytic hypophysitis, sarcoidosis, hemochromatosis, amyloi-
Pituitary Tumors dosis, Langerhans cell histiocy tosis, lymphoma, and
Pituitary adenomas, which are benign, are the most common tuberculosis, can affect the pituitary gland.
tumor of the pituitary gland. A tumor less t han 1 em is defi ned Lymphocytic hypophysitis is an infla mmatory pituitary
as a microadenoma, and a tumor 1 em or larger is termed a lymphocytic infiltration that most commonly occurs in preg-
macroadenoma (Figure 3). Pituitary adenomas are common. nant and postpartum women. It may cause transient or per-
Autopsy studies document t hat 10% of the general population manent pituitary insufficie ncy. Lymphocytic hypophysitis is
had undiagnosed pituitary adenomas. Frequently, pituitary treated with glucocorticoids.
adenomas are incidental findings on imaging studies completed A sellar mass can compress normal surrounding tissue
for other reasons. When patients undergo brai n MRI, 10% to and impair normal neu rologic and pituitary function. Pituitary
38% are found to have incidental pituitary m.icroadenomas and adenomas may also be functional and secrete excess hormone.
0 .2% have incidental pituitary macroadenomas. Certain genetic
mutations increase the chance of developing a pituitary tumor. Incidentally Noted Pituitary Masses CJ
When a pituitary tumor is incidentally noted. investigation
Approach to a Sellar Mass must determine (1) whether it is causing a mass effect, (2)
When a sellar mass is noted, pituitary adenomas are most whether it is secreting excess hormones, and (3) whether it has
likely; however, they need to be distinguished from ot her a propensity to grow and cause problems in the future. After a
pituitary lesions and nonpathologic pituitary enlargement. thorough history and physical examination, biochemical test-
The pituitary gland is enlarged diffusely in untreated pri- ing can be undertaken in a targeted fashion based on the
mary hypothyroidism and during pregnancy. When possible, patient's clinical signs and symptoms. Initial tests could include
imaging of the pituitary gland should be avoided or delayed in measurement of 8 A\1 cortisol, TSH. free (or total} thyroxine
pregnancy and in untreated primary hypothyroidism becau se (T 1) . prolactin. and insulin-like growth factor 1 (IGF 1).
gland enlargement on imaging may prompt an expensive and If the tumor is nc)t causing mass effect and there is no
unnecessary evaluation for pituitary hormone abnormality e\'iclence of hormone excess. a pituitary MRI should be
and tumor. repeated in 6 months for a macroadenoma and 12 months for

FIGURE 3. Acoronal MRI (le ft) and sagittal MRI (right) showing a large pituitary macroadenoma. The norma l pituitary gland and the optic chiasm cannot be seen because
of co mpression from the tumor. The tumor is invasive into the left cavernous sinus. Likely, the tumor appears heterogeneous because of internal necrosis.

20
Disorders of th e Pituitary Gland

Cl aMRJs
microadenoma to assess for growth. If no grovvth occurs,
should be repeated every to 2 years for the next 3 years
J
tumor may cause minimal peripheral vision loss, bitemporal
hemianopsia, or complete blindness. Visual field testing is a
CONT. and t h en mtermlttent
. . ly t l1ereaf'ter. 1n a pattent
. at n.s k f'or can sensitive measure of optic nerve damage and should be evalu-
cer or with a history of cancer, metastatic disease must be ated by an ophthalmologist in patients who report a change in
excluded . Cl · vision . who have a pituita ry tumor t hat abuts or compresses
the chiasm on MRl, or who have any evidence of gross periph-
Empty Sella eral vision loss on physical examination. Change in vision due
Empty sella is diagnosed when the normal pituitary gland is to optic chiasm compression is an ind ication for treating a
not visualized or is excessively small on MRI; it is a radiologic pituitary tu mor.
fi nding and not a distinct cli nical condition. The pituitary sella Pituitary tumors can also invade surrounding brain tissue
is said to be "empty" because normal tissue is not seen. The leading to seizures and neurologic manifestatio ns. Pituitary
fi nding may be primarily due to increased cerebrospinal fluid tumors can invade the cavernous sinus, causing damage to
entering and enlarging the sella, or it may be secondary to a cranial nerves Ill, IV, and VI that pass through the sinus caus-
tumor, previous pituitary surgery, radiation, or infarction. ing diplopia and extraocular muscle palsies/paralysis. Cl
Empty sella can also occur as a congenital abnormality when
KEY POINT
the sella is normal size, but the pituitary is small. When empty
sella is found incidentally on imaging, an evaluation should be • Pituitary masses can compress the normal pituitary
completed to determine if there is a known cause fo r second- gland, causing hormone deficiencies; a large pituitary
ary empty sella and if the patient has signs or symptoms of mass may cause panhypopituitarism in which there is
pituitary hormone deficiency. A patient without signs or impaired secretion of all pituitary hormones.
symptoms should be screened fo r cortisol deficiency and
hypothyroidism with 8 AM cortisol, TSH, and free (or total) Tr Treatment of Clinically Nonfunctioning
A patient with signs of pituitary hormone deficiency should Pituitary Tumors
receive a more complete biochemical evaluation of the pitui- Nonfunction ing pituitary tumors that are growing or causing
tary axes, based on the signs and symptoms found. mass effect are treated with neurosurgery. The most common
Repeat imaging is not necessary un less indicated as sur- surgical approach is transsphenoidal through the nares or the
veillance for the underlying pathology that resulted in the mouth. A very large or invasive tumor may require craniotomy
empty sella. for decompressiori. Indications for surgery include mass effect,
KEY POINTS .
particularly a visual field defect; tumor that abuts the optic
chiasm; tumor growth; or an invasive tumor (invading the
• Incidentally noted pituitary tumors are common, and
brain or cavernous sinus) . Surgery should also be considered
biochemical testing is informed by fiqdings on history
in a patient with a tumor close to the optic chiasm who pians
and physical examination.
to become pregnant (due to the physiologic enlargement of the
• Ini tial tests tor pituitary incidentally noted masses pituitary associated with pregnancy) .
include measurement of 8 AM cot1isol, thyroid-stimulat- Functional pituitary tumors will be discussed later in this
ing hormone, free (or total) thyroxine (T), prolactin, chapter, based on the hormone in excess (see Pituita ry
and insulin-like growth factor L Hormone Excess) .
• Empty sella is diagnosed when the normal pituitary KEY POINT
gland is not visualized or is excessively small on MRl;
• Nonfunctioning pituitary tumors that are growing or
it is a radiologic finding and not a distinct clinical
causing mass effect are treated with neurosurgery.
condition.

CJ Mass Effects of Pituitary Tumors Hypopituitarism


Pi tuitary tumors may cause headaches in some but not all Hypopituitar'ism is caused by one or more pituitary hormone
patients: the size of the tumor does not always correlate with deficiencies, usually resulting from damage to the normal
the presence and severity of headache. pituitary gland by a tumor. Hypopituitarism can also occur as
Pitu itary masses can compress the normal pi tu itary a complication from surgery if the normal gland or the pitu i-
gland. causing hormone deficiencies. A large pituitary mass tary stalk is damaged during tumor resection or radiation
may cause panhypopituitarism in which there is impaired therapy. Additional causes of hypopituitarism are listed in
secretion of all pituitary hormones. Table 14.
Because the optic chiasm is located superior to the pitui- Pituitary apoplexy is acute hemorrhage into the pituitary CJ
tary gland. a large pituitary mass may compress the optic gland often at the site of a preexisting pituitary adenoma
chiasm resulting in vision changes. Depending on the size of (typically a macroadenoma) . Pituitary apoplexy can cause
the tumor ami severity of optic nerve damage. a pituitary acute pituitary hormone deficiency or mass effect from rapid

21
Disord e rs of th e Pitu itary G land

Adrenocorticotropic Hormone Deficiency


Pitu itary adenoma
(Secondary Cortisol Deficiency) Cl
Alt hough secondary cortisol deficiency may result from dam-
Pitu itary surg ery
age to the pituitary gland or pituitary stalk that impairs ACTH
Pituitary rad iation production, it is most commonly iatrogenic due to exogenous
Pituitary apop lexy glucocorticoid use that suppresses pituitary ACfH secretion .
Pituitary infarction Patients with secondary cortisol deficiency have only gluco-
Craniopharyngioma corticoid deficiency. The remainder of the adrenal gland func-
tions normally and the renin-angiotensin system is intact, so
Metastatic tumor
these patients do not have mineralocorticoid deficiency (see
Meni ngio ma
Disorders of the Adrenal Glands for a discussion of primary
Lymphocytic hypophysitis adrenal failure) . Although patients with secondary cortisol
Sarcoidosis deficiency do require stress-dose glucocorticoids, they are at
Langerhans cell histiocytosis Jess risk for hypotension, hyponatremia, and adrenal crisis
Lymphoma than those with primary cortisol deficiency (failure of the
adrenal glands) because the production of mineralocorticoid is
Hemochromatosis
retained. Also, unlike patients w ith primary cortisol defi-
Congenital deficiencies
ciency, patients with secondary cortisol deficiency do not
Hypothal amic disease develop hyperpigmentation or bronzing of the skin because
ACfH and its pro hormone responsible for these changes, pro
opiomelanocortin (POMC), are not hypersecreted.
Cl expansion of the sellar contents due to bleeding. It is an endo-
crine and neurosurgical emergency. Acute ACfH deficiency is
Oral, injectable (including joint injections), and even top-
ical glucocorticoids are able to suppress ACTH secretion.
CONT. common and can be life-threatening. If suspected, stress-dose
Glucocorticoids prescribed at doses above physiologic replace-
glucocorticoid replacement should be initialed emergently.
ment for longer than 3 weeks should be tapered when discon-
Patients with vision changes or loss associated with apoplexy
tinued to allow recovery of the pituitary-adrenal axis; if ther-
require urgent surgical decompression. C]
apy has lasted less 3 weeks, no taper is required for
Hypopituitarism can occur due to postpartum pituitary
pituitary-adrenal axis recovery.
infarct ion (Sheehan-syndrome) because of excessive postpar-
When tapering glucocorticoids, the patient can be tran-
tum hemorrhage causing hypotension and hypoperfusion.
sitioned to a hydrocortisone dose that is 10% to 20% lower
Patients who may have Sheehan syndrome should be emer-
than the equivalent, current glucocorticoid dose. The dose _
gently tested and treated for seconda ry cortisol deficiency A
can then be decreased by 2.5 to 5 mg of hydrocortisone every
patient with Sheehan syndrome will not lactate because of
1 to 2 weeks. When tapering with p rednisone, taper large
prolactin deficiency; no treatment is available to induce lacta-
doses by 25% to SO% weekly until the patient is on a dose of
tion. Other hormone deficiencies can be evaluated 6 weeks
after delivery.
s mg daily and then taper by 1 mg every 1 to 2 weeks. It is dif-
ficult to taper dexamethasone due to the limited mg tablets
GH and gonadotropin deficiencies often occur early when
available. The taper can be slower if symptoms such as light-
the pituitary gland is damaged by tumor, radiation, su'rgery, or
headedness persist.
hemorrhage because the cell lines that synthesize GH (soma-
After prolonged glucocorticoid use. recovery ofthe pituitary-
totrophs) and LH and FSH (gonadotrophs) are most sensitive
adrenal axis shou ld be tested prior to discontinuing glucocor-
to inj ury. Secondary hypothyroidism (TSH deficiency) and
ticoid replacement. Specifically, morning serum cortisol
secondary cortisol deficiency (ACTH deficiency) often occur
' should normalize to greater than 11 j.lg/dL (303.6 nmol / L)
later in the disease process. Pituitary adenomas can cause
when glucocorticoids are withheld for 36 to 48 hours follow-
elevation in prolactin due to stalk compression, leading to a
ing the taper. Even after endogenous ACTH production has
decrease in dopaminergic inhibition of prolactin secretion.
returned, patients may require more time to mount an ade-
KEY POINTS . quate ACfH response to stress. After discontinuing the gluco-
• Pituitary tumors and surgery for pituitary tumors are corticoid taper, the patient can undergo an ACTH stimulation
the most common causes of hypopituitarism. test (Table 15) to document adequate glucocorticoid response
to stress. The diagnosis relies on demonstrating a low basal
• Stress-dose glucocorticoid replacement should be initi-
serum cortisol level that does not increase appropriately after
ated emergently in patients with pituitary apoplexy or
stimulation with the ACfH analogue cosyntropin. This is done
infarction, as well as emergent neurosurgical interven-
by measuring early morning (8 A'vt) serum cortisol. A serum
tion ; patients with vision loss associated with apoplexy
cortisol level less than 3 J.LgldL (82.8 nmol!L) is consistent with
require urgent surgical decompression.
cortisol deficiency. A normal response is a peak serum cortisol

22
Disorders of the Pituitary Gland

TABLE 15. Dynamic Testing for Pituitary Dysfunction


Indication Test Technique Interpretation

ACTH (cortisol) ACTH stimulation Measure baseline serum cortisol level. Serum cortisol level > 18Jlg/dl (496.8 nmoi/ L)
deficiency test Administer 250 Jlg of synthetic ACTH. indicates a normal response.
Measure cortisol levels at 30 and 60 m inutes.
ADH deficiency Water dep rivation Patie nt empties b ladder, and baseline Water deprivation test interpretation:
(DI) test, followed by weight is measured . Measu re uri ne volu me
desmopressin and osmolal ity hou r ly. Measure serum Urine osmolality >600 mOsm/ kg H20 is a
challeng e if sodium , osmolality, and weight every norma l response t o water deprivation,
indicated 2 hours. indicating ADH production and peripheral
effect are intact.
The t est is st o pped when one of t he Urine o smola lity <600 mOsm/ kg H20 , serum
fo llowing occurs: osmolality >295 mOsm/ kg H20 and/or seru m
sod ium > 145 m Eq/L ( 145 mmoi/ L) are
Urine osmolality exceeds 600 mOsm/ kg
d iagnost ic of Dl.
H20
Patient has lost 5% of body weight
Desmopressin challenge interpretation:
Urine osmolality is stable for 2-3 h whi le
serum osmolalit y rises > 100% increase in urine osmolality is
d iagno stic of complete central Dl.
- Plasma osmolality >295 mOsm/ kg H 20
0% increase in urine osmolality is diagnostic of
- Serum sodium> 145 mEq/ L (145 mmoi/L)
complete nephrogen ic Dl.
Desmopressin challenge if final urine
> 50% increase in urine osmolality is
osmolal ity <600 mOsm/ kg H2 0 , serum
d iagnostic of partial centra l Dl.
osmolality >295 mOsm/ kg H2 0 , o r serum
sodium >1 45 mEq/ L (145 mm o i/L): <50% increase in urine osmo lality is
diagnostic of partial nephrogenic Dl.
Give desmopressin 1 Jlg subcutaneously.
Measure urine osmo lality every 30 m inutes
for 2 hours.
Growth Gl ucose 75 g oral glucose t o lerance test. Measure GH <0.2 ng/ml (0.2 Jlg/L) is a normal response.
hormone excess tolera nce test g lucose and GH at 0, 30, 60, 90, 120, and GH <!1 .0 ng/ml(1.0 Jlg/L)(or <!0 .3 ng/m l
(acromegaly) , 150 minutes. [0.3 Jlg/L] on an ultrasensitive assay) is
diagnostic of acromega ly.

ACTH ... adrenocorticotropic hormone; ADH = antid iuretic hormone; D l = diabetes insipidus; G H "" growth hormone.

Cl greater than 20 Jlg/dL (552 nmolr L) . When the test result is


normal, patients no longer require daily cortisol replacement.
physiologic stress (major surgery, trauma. critical illness, or
childbirth). hydrocortisone (150-200 mg/d intravenously in
CONT. but should follow '·sick day rules" (increasing cortisol replace- 3 - 4 divided doses; 100 mg/d the next day; taper to baseline in
ment dose during illness) for up to a year after cessation of 3 - 5 days) may be used. An alternative would be dexametha-
da ily cortisol replacement. sone (6-8 mg/d intravenously in 2- 3 divided doses). If the
Symptoms of secondary cortisol deficie ncy include patient has pituitary apoplexy and urgent/emergent neuro-
weight Joss. nausea. vomiting. lightheadedness, hypoglyce- surgery is planned with no time for ACTH-stimulation test-
mia. hypotension. and hyponatremia. Secondary cortisol deti ing. the patient should empi rically be treated with glucocor-
ciency is also diagnosed using an ACTH stimulation test. ticoids and then receive an ACTH stimulation test 4 to 8 weeks
Secondary cortisol deficiency can be life threatening and must · after surgery. 1:J
be treated with glucocorticoid replacement. often with hydro-
cortisone. although prednisone or dexamethasone may also be Thyroid-Stimulating Hormone Deficiency
used. Hydrocortisone (15-30 mg,d) should be administered in Thyroid-stimulating hormone (ISH) deficiency leads to sec-
2 to 3 d ivided doses. or hydrocortisone should be dosed 10 to ondary or central hypothyroidism. Secondary hypothyroidism
20 mg in the morning and 5 to 10 mg in the early afternoon. is clinically identical to primary hypothyroidism (see Disorders
Patients require stress doses of glucocorticoids when of the Thyroid Gland).
acutely ill, hospitalized. or undergoing the stress of surgery. Secondary hypothyroidism is diagnosed by demonstrating
For moderate physiologic stress (minor or moderate surgery a simultaneously inappropriately normal or low ISH and low
with general anesthesia), hydrocortisone should used (45- T., (free or total). Patients are treated with levothyroxine
75 mg/d orally or intravenously in 3 - 4 divided doses for replacement in the same manner as primary hypothyroidism;
2-3 days) . Prednisone (10- 20 mg or dexamethasone 2-3 mg 'd however, the serum ISH cannot be used to monitor and assess
in 1-2 divided doses) may be used alternatively. For major for adequacy of thyroid hormone replacement dosing. Instead,

23
Di sord e rs of t he Pituitary Gl and

the levothyroxine dose is adjusted based on free T4 levels with isolated adult-onset GI-l deficiency is extremely rare, and its
the goal of obtaining a value w ithin the normal reference range. clinical significance in adults is debated. Therefore, evaluation
KEY POINTS
for GH deficiency is recommended in patients with at least one
known pituitary hormone deficiency. Unfortu nately, GI-l ther-
• Pat ients with secondary cortisol deficiency have iso- apy has been used inappropriately as an alternative medica-
lated glucocort icoid deficiency without mineralocorti- tion. GI-l naturally declines w ith age and does not require
coid deficiency; in addition, they do not develop hyper- replacement. The use of GH does not promote longevity and
pigmentation or bronzing of the skin because when used inappropriately can be harmful. Specifically, GH
adrenocorticotropic hormone and pro-opiomelanocortin therapy can encourage cancer growth, worsening the disease
are not hypersecreted. in a patient with cancer, or promoting growth of an occult,
• Secondary or central hypothyroidism is diagnosed by undiagnosed cancer.
demonstrating a simultaneously inappropriately normal Because GH secretion is pulsatile, testing random levels is
or low thyroid-stimulating hormone and low t hyroxine not diagnostically useful. Therefore, GH deficiency is diag-
(T4) (free or total) level. nosed by measurement of!GF-1. A GH detlciency is contlrmed
by measuring the response of serum GH on a stimulatory test,
such as the insulin tolerance test. An insulin tolerance test car-
Gonadotropin Deficiency
ries a high risk of severe hypoglycemia, so referral to an endo-
The pituitary gland normally secretes LH and FSI-! in response
crinologist for testing is appropriate.
to GnRH from the hypothalamus. LH and FSH stimulate the
A decision regarding replacement therapy should be
secretion of normal male and female sex hormones; LH and
made based on that patient's symptoms, goals, and risks in
FSH deficiency causes hypogonadotropic hypogonadism (see
consultation with the patient's endocrinologist. When clini-
Reproductive Disorders) .
cally indicated, GH deficiency is treated with daily subcutane-
Hypogonadotropic hypogonadism may be caused by
ous GI-l injections. In an otherwise hea lthy adult, treatment of
GnRI-! deficiency. The most common cause of GnRI-! defi-
GH deficiency can improve quality of life and increase the
ciency in women is hypothalamic a menorrhea, which is
percentage of lean muscle mass. Also, it can reduce the risk of
associated w ith excess exercise, illn ess, or anorexia.
osteoporosis. However, the risks and benefits of therapy must
Additional causes of Gn RI-! deficiency include congenital
be carefully considered. Replacement ofG H is cost prohi bitive
GnRH deficiency and Kallmann syndrome, a condition in
for some patients. It is contraindicated in patients with cancer
wh ich hypothalamic neurons responsible for releasing GnRH
and should not be used in patients with an untreated pituitary
fail to migrate into the hypothalamus during embryonic
tumor due to potential stimu lation of tumor growth.
development.
Treatment of hypogonadotropic hypogoqadism depends KEY POINT
on the goals of therapy and whether the patient desires fertil- • Isolated adult-onset growth hormone deficiency is HVC
ity. Fertility treatment requires replacement of the gonadotro- extremely rare, and its clinical significance is debated;
pins in men and women. Premenopausal women who do not evaluation tor growth hormone deficiency should be
desire fertility may be treated with estrogen- and progester- reserved for adults with at least one known pituitary
one-containing oral contraceptives (after assessment of risk of hormone deficiency.
thromboembolic d isease). Treatment of premenopausal hypo-
gonadotropic hypogonadism is recommended to avoid loss of
estrogen-dependent bone at a young age, which could lead to
Central Diabetes Insipidus
Central diabetes insipidus (Dl) results from inadequate pro- Cl
osteoporosis. Treatment of postmenopausal hypogonadotropic duction of antidiuretic hormone (ADH) by the posterior pitui-
hypogonadism is not indicated. Men who do not desire fertility tary gland. In the presence of ADH, aquaporin water channels
may be lrealed wilh lesloslerone replacement therapy (see are inserted in the collecting tubules and allow water to be
Reproductive Disorders). reabsorbed. In the absence of AD!-!, excessive water is excreted
by the kidneys. Excretion of more than 3liters of urine per day
Growth Hormone Deficiency is considered polyuric.
Growth hormone (GH) is vital for normal linear gro-vvth, and The severity of Dl varies w ith the completeness of the
deficiency prior to puberty will lead to short stature. At pubetiy, deficiency. Patients describe mild to extreme polyuria and cor-
the epip hyses close, halti ng linear growth. In adulthood, GI-l responding thirst; partial OJ is common.
production is necessary for normal physiology but is not as Frank hypernatremia is u nusual because patients develop
important fo r gro-vvt h as during childhood. In adults, GH defi- extreme thirst and polydipsia, and with free access to water,
ciency causes fatigue, loss of muscle mass, an increased ratio of can maintain serum sodium in the high normal range. When
fatty tissue to lean tissue, and increased risk for osteoporosis. patients do not drink enough to replace the water lost in the
GH deficiency is often the first hormone deficiency to urine, due to poor or absent thirst drive or lack of free access
occur when a pat ient is developing pituitary insufficiency, but to water, they develop hypernatremia.

24
Disorders of the Pituitary Gland

In the patient with polyuria, DI is diagnosed with simul


Cl taneous
CONl
laboratory evidence of inability to concentrate urine
Pituitary Hormone Excess
Pituitary tumors are called fu nctional when they secrete
in the face of elevated serum sodium and osmolality, with
excessive amounts of hormone. The most common fu nctional
inappropriately low urine osmolality. If necessary, a water
pituitary tumor is a prolactinoma . GH and ACT!-! overproduc-
deprivation test can confirm the diagnosis (see Table lS).
tion by pituita ry tumors is imp011ant to recognize because the
Palienls with mild partial DI with an adequate thirst drive
clinical consequences of oversecretion are potentially severe.
and access to water may choose to compensate without hor
ISH-secreting tu mors cause hyperthy roidism but are
mone replacement therapy, but highly symptomatic polyuria
extremely rare.
and nocturia that interferes with restful sleep and daily func
Occasionally, a pituitary tumor can oversecretc more than
tion necessitate treatment. In those requiring treatment, hor
one hormone, most commonly GI-l and prolactin, or less com-
mone replacement is with desmopressin (1 -desamino s-O-
monly, TSH and GI-l or prolactin .
arginine vasopressin, or dDAVP) either intranasally.
subcutaneously, or orally. Desmopressin is not absorbed well in
Hyperprolactinemia and Prolactinoma
the gastrointestinal tract, so oral doses are much higher than
intranasal or subcutaneous doses. Most patients with Dl require Causes
either evening dosing to aid in sleep or twice daily dosing of Prolactinomas are pituitary tumors that secrete excessive
desmopressin . If ADH is overreplaced, patients will develop amounts of prolactin; however, they a re not the only cause of
water intoxication, volume overload, and hyponatremia. CJ hyperprolactinemia (Table 16).
The most comm on cause of hyperprolactinemia is phys-
KEY POINTS iologic: prolactin is released during pregnancy and postpar-
• In the patient with polyu ria, diabetes insipidus is diag- tum to cause lactation. Nipple stimulation such as during sex
nosed by clinical symptoms with simultaneous labora- can cause m ild hyperprolactinemia (serum prolac tin <40 ng/
tory evidence of inability to concentrate urine mL [40 !J.g/L]). Physiologic stress, coitus, and exercise ca n
with elevated serum sodium and osmolality, and inap- also increase prolactin leve ls up to 40 ng/ mL (40
propriately low urine osmolality; a water deprivation Nipple pie rcing can raise prolactin levels above 200 ng/mL
test can confl rm the diagnosis. (200 !J.g/ L). Cli nical breast exam ination should not raise pro-
• Treatment of central diabetes insipidus is once or tv.rice lact in levels above the reference range, u nless evaluation for
daily hormone replacement with desmopressin. m ilk production is performed. but if desired. palpation of the
breast can be deferred until after a serum prolactin level is
measured .
Cl Panhypopituitarism
Panhypopituitarism occurs when patients lack all anterior and
Medications are a common cause of hyperprolactinem ia
(see Table 16) . Antipsychotic agents cause hyperprolactinemia
posterior pituitary hormone production. F'anhypopituitarism
due to their antidopaminergic effect that interrupts the inhibi-
may be caused by a large or aggressive pituitary tumor or as a
tion of prolactin by dopamine. Specific agents, such as risperi-
complication of surgery. If the pituitary stalk is transected dur-
done or metoclopramide, may ra ise the prolactin level above
ing surgery or as the result of trauma. panhypopituitarism will
200 ng/mL (200 !J.g!L). Evaluation tor pituitary hypersecretion
result acutely. Patients with panhypopituitarism require life-
when a patient is taking a medication known to ra ise the pro-
long replacement ofT 1, cortisol, and ADH because these deft
lactin level is difficult. Wh en the prolactin level is only mildly
ciencies can be life threatening. GH and sex hormones are
elevated (<SO ng/mL [SO !J.g!LJ), it may be reasonable to assume
replaced dependent on each patient's preference, coupled
that hyperprolactinemia is a medication side effect. When
with a discussion of the risks and benefits of therapy. In addi significantly elevated (>100 ng/mL [100 !J.giLJ), either the
tion to requiring exogenous gonadotropins to conceive, a
reproductive-aged woman with panhypopituitarism will not'
go into spontaneous labor and will not lactate. These pregnan TABLE 16. Causes of Hyperprolactinemia
cies are classified as high risk. and obstetric care should be Physiologic Me dications Other
provided by a maternal-fetal specialist.
Patients with panhypopituitarism shou ld wear medical Pregnancy A ntipsychotic agents• Prolactinoma

alert identification documenting their panhypopituitarism. Lactation Metoclopramide Pituitary tumor-stal k


compression
specifically noting the need for stress-dose glucocorticoid Nipple Cimetidine
therapy and desmopressin dosing in emergent situations. CJ stimulation
Verapami l
Hypothyroidism
Cirrhosis
KEY POINT Met hyldopa
Chronic kidney
• Patients with panhypopituitarism require lifelong replace- Opiates disease
ment of thyroxine (T), cortisol, and antidiuretic hormone Cocai ne
because these deficiencies can be life-threatening. "Includ ing risperidone. olanzapin e, haloperidol. chlorpromazine, and clomipramine.

25
Disorders of the Pituitary Gland

medication needs to be withheld to further assess or a pitui- ceptive pills (if fertility is not desired) or dopamine agonists.
tary MRI obtained to evaluate for prolactinoma. Caution is Postmenopausal women with microadenomas do not require
warranted when discontinuation of an antipsychotic agent is treatment. Patients w ith hypogonadism from medication-
being considered, and consultation with a psychiatrist is rec- induced hyperprolactinemia may be treated with hormone
ommended prior to discontinuation. replacement.
Another common cause of hyperprolactinemia is primary Unlike other pituitary tumors. medication rather than
hypothyroidism. Hypothyroidism can cause diffuse swelling of surgery is fi rst- line therapy for prolactinomas. Even patients
the pituitary gland that may resemble enlargement due to a with severe mass effect such as vision loss are treated with
pituitary adenoma on imaging. Therefore. a patient with pri- medical therapy initially. Rarely, very large tumors or more
mary hyp othyroidism and hyperprolactinemia should be invasive prolactinomas do not shrink w ith medical therapy
treated v.rith thyroid hormone replacement with retesting of and. also rarely, continue to grow. In these patients, surgery
the prolactin level once the TSH has normalized. Further evalu- should be considered, fol lowed by radiotherapy if growth
ation is indicated if the hyperprolactinemia does not correct recurs or continues. After being debulked, the prolactinoma
when hypothyroidism is treated. If pituitary imaging has noted may respond better to medical therapy.
pituitary enlargement prior to treatment of hypothyroidism. Prolactinomas are treated w ith dopamine agonists (DA) .
repeat MRI should be obtained when the TSH is normal. The two FDA-approved dopamine agonists are bromocriptine
Nonfunctioning p ituitary adenomas ca n also cause and cabergoline. Dopamine agonists typically decrease the size
hyperprolactinemia by compressing the pituitary stalk and and hormone production of prolactinomas rapidly. Response
decreasing dopamine inhibition of prolactin secretion. It is to therapy can be monitored by checking sentm prolactin
important to distinguish between prolactinomas and non- levels 1 month after initiating therapy and then every 3 to
functioning pituitary adenomas as the cause of hyperpro- -1 months. Decreasing serum prolactin usually correlates with
lactinemia because of different treatment approaches. decreasing size of the tumor. MRl should be repeated in 1 year
for microprolactinomas if the prolactin level normalizes on
Clinical Features and Diagnosis dopamine agonists. After tumor shrinkage is confirmed, addi-
Physiologically, prolactin induces and regulates lactation. tional MR!s are not necessary unless the serum prolactin level
Hence, elevated levels of prolactin cause galactorrhea. Women rises. An MRI should be repeated after 3 months or medical
are more likely to develop galactorrhea than men. therapy for macroprolactinomas, or if prolactin levels are ris-
Hyperprolactinemia also causes hypogonadotropic hypo- ing on therapy with good medication adherence. MRTshould
gonadism because of negative feedback on GnRH. LH. and FSH be repeated every 6 to 12 months until the macroprolactinoma
by high levels of prolactin. Both men and women present with is stable on serial studies and the prolactin level is not rising.·
hypogonadism. Women of reproductive age often present ear- Bromocriptine is dosed 1 to 3 times daily, so adherence
lier than men because of amenorrhea. They may also have early can be challenging. When in itiated, it is assoc iated with
menopausal symptoms. Symptoms in men are insidious and orthostasis and lightheadedness, and patients can have dizzi-
may go unrecognized for years. Both men and women with ness, nau sea. and headache during treatment. Cabergoline is
hyperprolactinemia are likely to be infertile and are at risk for much better tolerated and more effective at normalizing prol-
osteoporosis. Postmenopausal women are already hypogonadal actin and tumor so it is typically the initial therapy
because of ovarian failure; therefore. hyperprolactinemia may chosen. It is dosed once or twice a week, but typically costs
have minimal clinical implications in this population. However. more than bromocriptine.
the cause of postmenopausal hyperprolactinemia still requires Therapy may be tapered after the prolactin level has been
diagnosis because it may be due to a pituitary tumor. normal for 2 yea rs. and there is no longer a visible tumor on
Diagnostic imaging is indicated in situations in w hich pituitary MRI. After discontinuing the dopamine agon ist, pro-
there is unexplained hyperprolactinemia. lactin levels should be followed once a month for 3 months,
The degree of hyperprolactinemia is useful in differenti- then every 3 months fo r the fi rst year. and then annually
ating prolacti nomas from non fu nctioning macroadenomas. In thereafter; a pituitary MRI should be repeated if the prolactin
general, large nonfunction ing tumors cause mild serum pro- level rises above normal.
lactin elevations (<100 ng/m L [100 ).lg L]) from stalk compres-
sion. Macroprolactinomas raise serum prolactin levels to Prolactinomas and Pregnancy
greater than 250 ng/ mL (250 ).lg/ L). Very large macroprolacti- Hyperprolactinemia is a frequent cause of infertility because
nomas may raise prolactin levels greater than 10,000 ng tmL of the effect on gonadotropin release. DA therapy lowers pro-
(10,000 ).lg/L) . lactin. normalizing gonadotropin regulation and allowing
normal ovulation. DA therapy should be discontinued when
Therapy the pregnancy is diagnosed. The pituitary increases in size
Patients w ith microprolactinomas without symptoms of during normal pregnancy, and prolactinomas can increase in
hypogonadism do not require treatment. Symptomatic women size as well. The risk for significant tumor expansion is negli-
w ith microadenomas may be treated w ith either oral contra- gible in patients with microprolactinomas.

26
Disorders of the Pituitary Gland

Women with macroprolactinomas are at risk tor clinically Clinical Features and Diagnosis in the Adult
significant tumor growth or vision compromise during preg- Patient with Acromegaly
nancy. If the tu mor is very large or abuts the optic chiasm, Patients have changes in facial structure such as a prominent
patients should be counseled on risk of tumor growth during brow and jawline, an enlarged skull, a large nose, facial edema,
pregnancy, as well as the risks and benetits of surgical resec- excessive spacing between teeth, and macroglossia. The hands
tion of the tumor before pregnancy. DA therapy is sometimes and fee t may be disproportionately large. Other manifestations
continued during pregnancy if the patient has a history of may include arthritis, skin tags, diabetes mellitus, hyperten-
visual field detect. sion, colon polyps, thickened skin, and excessive perspiration.
Pregnant women with macroprolactinomas should be Acromegaly can cause severe obstructive sleep apnea because
assessed clinically at least once per trimester and have visual of soft-tissue swelling and macroglossia. Additionally, it can
fields tested every trimester or more freq uently for vision result in heart disease, including left ventricular hypertrophy,
change. Changes in visual fields or severe headache are ind ica- cardiomyopathy, valvular heart disease, arrhythmia and dias
tions to proceed with pituitary MRI. If the macroprolactinoma tolic heart failure. Increased rates of cancer are observed in
causes mass effect during pregnancy, bromocriptine may be acromegaly, including colon, esophageal, and gastric adeno-
started. If the bromocriptine does not decrease tumor size and carci nomas; thyroid cancer; and melanoma. Acromegaly
reduce symptoms of mass effect, surgical debulking may be increases mortality, likely due to cardiovascular disease, diabe-
necessary. tes, sleep apnea. and cancer. Age-appropriate testing for these
Normal pregnancy causes hyperprolactinemia, so hyper- conditions should occur for the lifetime of t he patient with
prolactinemia from prolactinoma does not require treatment acromegaly.
during pregnancy. Prolactin levels should not be measured Acromegaly is diagnosed biochemically. Because GH is
duri ng pregnancy. Postpartum, prolactin levels return to pulsatile throughout the day, it is not usefu l for diagnosis, so
normal within a few months, and lactation becomes non- measurement of serum IGF-1 is used instead . Excess GH is
prolactin media ted. confi rmed with an oral glucose tolerance test (see Table 15)
because glucose normally suppresses GH levels to less than
KEY POINTS
1 ng/mL (1 GH levels greater than 1 ng/mL (1 are
• Prolactinomas, pregnancy and lactation, or medications diagnostic of GH excess. A pituitary MRI should be obtained
such as antipsychotic agents are frequent causes of once GH excess is confirmed biochemically. Consultation with
hyperprolactinemia. an endocrinologist is recommended if IGF-1 is elevated.
• A patient with primary hypothyroidism and hyper-
prolactinemia should be treated with t hyroid hor- Treatment
mone replacement with retesting of the prolactin Treatment of acromegaly is transsphenoidal tumor resection;
level once the thyroid-stimulating hormone level has surgery is the only treatment that is potentially curative. ·tn
normalized. many instances, cure with surgery is not possible and addi-
tional therapy is necessary to treat the residual GH excess and
• Dopamine agonists (bromocriptine and cabergoline) are
fi rst-line therapy for symptomatic patients with hyper- tumor.
Remission is achieved when IGF-1 levels are w ithi n t he
prolactinemia and prolactinomas.
normal reference range for age and the response of GH to a
glucose tolerance test is normal. Patients not achieving
Acromegaly remission require medication to decrease GH levels and the
Acromegaly is a rare diagnosis that is often missed for years long-term effects of GH excess. The initial therapy of choice
because of the insidious onset and rare presentation in pri- is injectable somatostatin analogues to inhibit GH secret ion.
mary care; however, it has very serious implications for 9 If a patient fails to benefi t from somatostatin a nalogue treat-
patient's health and longevity and must be diagnosed and ment. high-dose dopamine agon ist therapy is marginally
treated in as timely a manner as possible. effective when t he tumor co-secretes prolactin. If IGF- 1
rema ins elevated , pegvisomant, a GH receptor blocker, is
Causes used. Pegvisomant effectively lowe rs IGF-1 levels, but
Acromegaly is the clinical syndrome that occurs when a pitui- patients on pegvisoma nt have risk of tumor growth because
tary tumor secretes excessive amounts of GH in an adult the medication works in the peripheral tissues as an antag-
patient. Prior to puberty, patients with a GH-secreting tumor onist to GH and does not decrease GH production by
develop excessive longitudinal growth and gigantism. a term the tumor.
used to indicate excessive growth and height above normal for Stereotactic radiosurgery (ga m ma knife) may be offered
age. Because epiphyseal grovvth plates require sex hormones to to increase the cha nce of rem ission or cure. External beam
close, patients with large pituitary tumors causing hypog- radiation carri es a high risk of causing pituitary insuffi-
onadism will n ot have closure of their growth plates and w ill ciency, but the risk is decreased when stereotactic radiosur-
continue growth into adulthood. gery is used.

27
Disorders of the Pituita ry Gland

When acromegaly is in remission, MRI and hormone test- Cushing syndrome refers to hypercortisolism from any cause,
ing should be completed annually. When the pituitary tumor exogenous or endogenous. ACTH-dependent or not. The most
is stable but the IGF-l level is elevated, MRI should be repeated common cause of endogenous Cushing syndrome is Cushing
an nually and treatment should be altered until the IGF-1 disease. When undiagnosed, Cushing disease is associated
declines. \\ ith devastating long term morbidity such as diabetes. mor
KEY POINTS
bid obesit). hypertension. infertil ity. and osteoporosis.
The initial step in evaluation fo r Cushing disease is to seek
• Acromegaly occurs when a pituitary tumor secretes biochemical evidence of hypcrcortisolism (see Disorders of the
excessive amounts of growth hormone in an adu lt Adrenal Glands).
patient resulting in changes in facial stmcture, an Once ACTH dependent Cushing syndrome is cont1rmed
enla rged skull, a large nose, facial edema, excessive biochemically, a pituitary MRl should be obtained. If no pitui
spacing between teeth. macroglossia, and dispropor- tary tumor or a tumor less than 6 mm is visualized on MRI. an
tionately large hands and feet. 8 mg dexamethasone suppression test is used to differentiate
• Treatment of acromegaly is transsphenoidal tumor Cushing disease from an ectopic source of ACTH. Ectopic
resection; however. in some patients, adjuvant radiation ACfH production from a nonpituirary tumor (most often lung.
therapy or medical therapy, such as injectable somato- pancreas, or thymus carcinomas) is \'ery uncommon.
statin analogues, is needed fo r residual disease. Dexamethasone is adm inistered at 11 P\1, and cortisol is
tested at 8 A\1. A pituitary source of ACTH will respond to
negative feedback from high doses of dexamethasone. sup
Gonadotropin-Producing Adenomas
pressing cortisol to less than 5 11g elL (138 nmol L), while an
Gonadotropin-producing pituitary adenomas are typically
ectopic source of ACTH will not have suppressible cortisol.
asymptomatic and are treated similarly to nonfunctioning
However, this test has low sensitivity (88".,) and specificity
adenomas because they either do not secrete functional gon-
(57" ) for Cushing disease, so intrapetrosal sinus sampling
adotropins or do not secrete enough FSH or LH to produce a
(IPSS) is often recommended before exploratory pituitary sur-
clinical syndrome. Often. the diagnosis is made postopera-
gery. ln lPSS, a catheter is threaded through the petrosal sinus.
tively. based on h istopathologic staining of surgical pathology
and AC'TH levels in the sinus are compared with those in the
specimens.
periphery after the administration of corticotropin releasing
hormone (CRH). A central to peripheral gradient greater than
Thyroid-Stimulatjng Hormone-Secreting Tumors
2.0 before CRH or greater than 3.0 after CRH is diagnostic of
ISH-secreting rumors are extremely rare. These tumors may
Cushing disease (95";, sensitivity. 93% specificity) . Imaging of
co-secrete TSH and prolactin or GH. ISH-secreting tumors
the chest and abdomen is indicated in patients vvith a sus-
cause hyperthyroidism. Patients with ISH-secreting tumors
pected ectopic source of ACTH.
have e ither an inappropriately normal or a h'igh ISH leve l
with a simultaneous e levation and T3 levels. They pre- Treatment
sent with identical symptoms associated with non -ISH- Cushi ng disease is treated by transsphenoidal pituita ry
mediated thyrotoxicosis (see Disorders of the Thyroid Gland). tumor resection. wh ich ma) be cu rative. Endogenous :\CTH
After biochemical proof of TSH excess is obtained. pituitary production in the remain ing normal pitu ita ry gland wi ll be
imaging is recommended to confirm a pituitary mass. suppressed after removal of the tumor due to long standi ng
Neu rosurgery is first- line therapy, but patients often require h) percortisolism, so patients with successful surgical treat
additional medical therapy w ith eit her somatostati n ana - mcnt will have acute ACTH deficiency and require gluco
logues or dopamine agon ists. corticoid replacement. It may take up to 1 year for
endogenous ACTH production to return to normal. and
Excess Antidiuretic Hormone Secretion sometimes the hypotha lamic pituitary adrena l axis does
The syndrome ofinappropriate ADH secretion (SlADH) causes not recover. After successful resection. Cushing disease can
water retention and hyponatremia. Central nervous system recur. and patients must be monitored annually f(Jr several
pathology such as stroke, hemorrhage, trauma. or infection years. and then less frequent!). or sooner if symptoms of
can cause SlADH because of the excessive release of hypotha- hypercort isolism recur.
lamic and pituitary ADH. Also, transient SlADH is a common If surgical cure is not achieved. patients may be offered
complica tion of pituitary surgery, occurring in about one third pituitary radiation or medical therapy. Medical options include
of patients approximately 3 to 10 days after surgery (see inhibitors of adrenal enzyme synthesis of cortisol. kctocona
MKSAP17 Nephrology). zole, or metyrapone: the dopam ine agonist, cabergolinc: or the
somatostatin analogue. pasireotide. Medical cu re of Cushing
1:) Cushing Disease disease has a relatively low success rate. but hypercortisolism
Cushing disease is the term used to indicate excess cortisol symptom control is an achievable goal in all patients with
product ion due to an ACTH-secreting pituitary adenoma. endogenous Cushing syndrome.

28
Disorders of the Adrenal Glands

Cl and Inwhopatients
CONI
who do not benefit from surgical treatment
have an inadequate response to medical treat-
Synthesis of adrenal androgens, dehydroepiandrosterone
(DHEA) and its sulfate (DI-lEAS). and androstenedione, occurs
mmt. bilateral to remove the target of ACT II primarily in the zona reticularis and is regulated by 1\CTH.
stimulation is an option. However. these patients will Although the adrenal androgens themselves have minima.!
require litelong glucocorticoid and mineralocorticoid intrinsic androgenic activity, they are converted peripherally
replacement. Cl to testosterone and dihydrotestosterone. Unlike glucocorti-
coids and mineralocorticoids. deficiencies of adrenal andro-
KEY POINTS
gens are not typically recognized due to parallel production of
• Cushing disease refers to excess cortisol production due gonadal androgens.
to a n adrenocorticotropic hormone (ACTH)-secreting The adrenal medulla and extra-ad renal sites of the sym-
pituitary adenoma; Cushing syndrome refers to hyper- pathetic nervous system consist of chromaffin cells, which
cortisolism from any cause, exogenous or endogenous. synthesize catecholamine hormones from the am ino acid
ACrH-dependent or not. tyrosine (Figure 4). Catecholamines are stored within chro
• Cushing disease is treated by transsphenoidal pituitary maffin granules. which release their contents in response to
tumor resection; after surgery glucocorticoid replace- stress. Although catecholamine excess produces disease,
ment therapy will be required at least transiently while hypofunction of the adrenal medulla does not because of
the hypothalamic-pituitary-adrenal axis recovers redundancy of ca techolamine production throughout the
endogenous function. sympathetic nervous system. Norepinephrine is synthesized in
the adrenal medulla and the extra-adrenal sites of the sympa-
thetic nervous system. It causes vasoconstriction ctue to pref-
Disorders of the erential binding to a -receptors. Epinephrine is a lmost
exclusively produced in the adrenal medulla. It bi nds pre-
Adrenal Glands dominantly to P- receptors. and thus has positive effects on
cardiac inotropy and chronotropy, produces peripheral vaso-
Adrenal Anatomy and Physiology dilation. and increases plasma glucose levels in response to
Located just superior to each kidney. the paired adrenal glands hypoglycemia.
consist of an outer cortex and an inner medulla that are dis-
tinct in embryologic origin and endocrine function. The adre
nal cortex is composed of three zones: the zona (outer) glo-
merulosa. zona (middle) fasciculata. and zona (inner)
reticularis. Within these zones corticosteroid hormones are
synthesized from cholesterol by cytochrqme P450 enzymes.
Aldosterone, the principal mineralocorticoid hormone, is pro-
duced in the zona glomerulosa. Aldosterone production is
triggered by an increase in the extracellular potassium con-
centration and by activation of aldosterone synthase through
the renin-angiotensin-aldosterone pathway. Upon binding to
type l mineralocorticoid receptors (MR) in the kidney. aldos-
terone promotes potassium wasting and sodium retention.
which leads to an increase in intravascular volume and conse-
quent ly blood pressure.
The zona fasciculata is the main site of glucocorticoid
synthesis. Production of cortisol. the principal glucocorti-
coid. is stimulated by adrenocorticotropic hormone (ACTH)
secretion from the anterior pituitary. Cortisol secretion
varies according to the circadian rhythm with relatively
Dihydroxymandelic acid I"" Vanillylmandetic acid
L . __ __ _ __ J

little secretion overnight, peak levels in the early morning,


FIGURE 4. Catecholamine hormones are produced in the adrenal medulla
a nd smaller oscillations throughout the day. Cortisol atten- and sympathetic ganglia. The pathways of synthesis and degradation are shown.
uates inflammatory responses and contributes to glucose Excessive catecholamine secretion can occur with pheochromocytomas and
homeostasis by promoting lipolysis, hepatic gluconeogen- paraganglionomas.
esis. and insulin resistance. Physical stress (for example. 1 =Tyrosine hydroxylase
2 = DOPA decarboxylase
critical illness) stimulates increased co rtisol secretion,
3 = Dopamine
which enhances vascular smooth muscle tone and respon- 4 = Phenylethanolamine N·methyltransferase (PNMT)
siveness to endogenous vasoconstrictors, thereby augment- 5 = Monoamine oxidase (MAO)
ing blood pressure. 6 = Catechoi·O·methyltransferase (COMT)

29
Disorders of the Adrenal Glands

Adrenal Hormone Excess Clinical manifestations ofCS are listed in Table 18. Clinical
fmdings that are highly specific for CS include centripetal obe-
Cushing Syndrome
sity, facial plethora, abnormal fat deposition in the supraclav-
Cushing syndrome (CS) is a rare disorder affecting two to three icular or dorsocervical (.. buflalo hump") areas, and wide (>1
persons per miJiion per year that results fro m elevated levels of em) violaceous striae (Figure 5). It is important to initiate
cortisol. Poor suppressibil ity of cortisol with dexamethasone evaluation for CS in patients who have specific signs and
and loss of normal diurnal variation in cortisol secretion are symptoms of CS. rather than in patients who are diffusely
seen. Without treatment. it is associated with high morbidity obese, have nonpathologic striae. and are having trouble losing
and mortality. weight because endogenous CS is such a rare condition w ith a
However, iatrogenic hypercortisolism from the admin- costly evaluation algorithm.
istration of exogenous oral. inhaled. intra-articular. or topi- Biochemical testing is used to establish the diagnosis of CS.
cal glucocorticoids is often seen in cli nical practice and is It is critical that the biochemical diagnosis is firmly established
the most common cause ofCS overall. The pharmacokinetics prior to any imaging studies due to the relatively high prevalence
and relative potencies of synthetic oral glucocorticoids are of clinically insignificant pituitary and adrenal nodules. At least
shown in Table 17. The sustained administra tion of any syn- two first-line tests should be diagnostically abnormal before the
the tic glucocorticoid above the normal physiologic cortisol diagnosis is confirmed. Initial tests include the overnight low-
requirement can result in iatrogenic CS and hypothalamic- dose dexamethasone suppression test (LOST), 24-hour urine
pituitary- adrenal (HPA) axis suppression. but is more likely free cortisol (UFC). and late-night (L ) salivary cortisol. All three
to occur the longer the half-life of the drug. Doses equivalent tests have similar diagnostic utility, but the LOST or LN salivary
to prednisone 5 mg/d or less are unlikely to cause clinically cortisol tests are more convenient. The 24-hour UFC ;.1 11d LN
significant HPA axis suppression. while those in excess of 10 salivary cortisol tests should be performed at least twice to
to 20 mg/d commonly do after 3 weeks or more of consecu- ensure reproducibility of results. Because the secretion of corti-
tive use. sol is pulsatile. measurement of random serum cortisol is neither
Endogenous CS can result from ACIH-dependent and sensitive nor specific fo r the diagnosis of CS. An algorithm to
ACfH-i ndependent causes. Cushing disease, which results establish the diagnosis ofCS is shown in Figure 6. Referral to an
from the autonomous secretion of ACTH by a corticotroph endocrinologist is indicated ifrn'o initial tests are abnormal.
adenoma of the pituitary gland. is the cause ofCS in more than
two thirds of patients (see Disorders of the Pituitary Gland).
Ectopic ACIH secretion by carcinomas and carcinoid tumors
(usually bronchial origin) is less common. accounting for 10%
to 15% of cases, wh ile ectopic corticotropin-releasing hormone
(CRH) production is rare. The most common ACTH-
independent etiologies of CS are adrenal adenomas and carci-
nomas. which collectively account ,for approximately 20% of
CS cases.
CS must be differentiatedlrom other disorders and clini-
cal states that are associated with physiologic hypercortisolism
(pseudo-Cushing syndrome). Causes of pseudo-Cushing syn -
drome include severe obesity, polycystic ovary syndrome. FIGURE 5. Wide violaceous striae are seen on the abdomen of a patient with
pregnancy, ano rexia nervosa. depression, alcoholism. and Cush ing syndrome. Striae larger than 1 em in width are highly specific for hyper-
extreme physical stress, as in the setting of infection. cortisolism.

TABLE 17. Dose Equivalence and Relative Potencies of Common Synthetic Oral Glucocorticoids
Synthetic Glucocorticoid Equivalent Biologic Relative Anti- Relative
Replacement Half-Life Inflammatory Mineralocorticoid
Dose (mg)• (hours) Potencyb Potency c

Hydrocortisone 20 8-12 1/125


Prednisolone/prednisone 5 18-36 4 1/156
Methylprednisolone 4 18-36 5 0
Dexamethasone 0.75 36-54 25-50 0
comm on glucocort icoid dosmg for primary adrenal fa •lure equivalent to hydrocortisone, 20 mg.

hAnt •·•nrlammatory pote ncy relative to hydrocort isone.

c;Mineralocorticoid potency relat1ve to fludrocort•sone. _j


30
Disorders of the Adrenal Glands

TABLE 1 8 . Clinical Features of Cushing Syndrome other proteins. Therefore the LOST should not be performed
when CBG is likely to be abnormal, such as with malnutrition,
Specific Findings Less Specific Associated
Findings Conditions• cirrhosis. the nephrotic syndrome, and hyperestrogenemia
(oral contraceptive pills or pregnancy) . There is no clear asso-
Centripetal obesity Easy bruising Osteoporosis
ciation between dexamethasone responses and BMI or weight,
Facial plethora Excessive skin Hypertension and therefore the LOST may be used similarly in the obese
fragility
Supraclavicular fat Diabetes mellitus population. The LOST is best avoided in patients taking medi-
pads Proximal muscle cations that could accelerate dexamethasone metabolism,
Obesity
weakness
Dorsocervical fat
Depression
such as antiepileptic drugs (phenytoin, phenobarbital, and
pads Impaired memory carbamazepine), rifampin, or pioglitazone. Concomitant
Hypokalemia
Wide violaceous Temporal baldingb measurement of serum dexamethasone can confirm altered
striae Nephrolithiasis
Hirsutism (in dexamethasone metabolism and patient adherence.
women)b VTE/ PE Measuring 24-hour UFC circumve nts problems related to
Menstrual cortisol pulsat ility and binding protein abnormalities. The test
abnormalitiesb should be performed at least twice to ensure accuracy. To con-
PE • pulmonary embolism; VTE = venous thro mboembo li sm. firm adequate collection. 24-hour urine creatinine is also
'Medic al d isorders that may be seen in association w rth but are not s pecefic for measured (normal range, 20-25 mg/kg/24 h [177- 221 mmol/
Cushing syndrome.
kg/ 24 h) in men; 15-20 mg/ kg/ 24 h [133-177 mmol/U 24 h) in
bFcatures of a ndroge n excess see n with pituitary corticotroph adenoma or adreno-
conical carcenoma.
women). A test is considered abnormal when UFC exceeds the
upper limit of the normal range of the assay (45 h [124
nmol/24 h)), while values greater than 3 times normal are
diagnostic ofCS. Less marked elevations are seen with pseudo-
In the standard LOST, dexamethasone (0.5 mg) is admin- Cushing synd rome and polyuria. A falsely low UFC can occur
istered every 6 hours for 48 hours and serum cortisol is meas- in chronic kidney disease and when CS is subclinical or mild.
ured at 9 AM. ln the overnight LOST, 1 mg of dexamethasone is The LN salivary cortisol test is performed between 11 PM
admin istered at 11 PM or midnight, and serum cortisol is meas- and midn ight. The normal evening nadir in cortisol secretion
ured the next morning at 8 AM. With either test, serum cortisol is lost in patients with CS, while it is preserved in patients with
will typically be suppressed to less than 2 {.lg/dL pseudo-Cushing syndrome. Both emotional and physical
(55 nmol!L). Sfandard assays measure total serum cortisol. or stress (for example, exercise) can cause a physiologic increase
that which is bound to cortisol-binding globulin (CBG) and of salivary cortisol. False-positive results are seen with

Perform initial testing:


Exclude use of
· 24 hr UFC•
exogenous
• 1 mg DST
glucocorticoids
· LN salivary cortisol•

Abnormal:
Exclude physiologic
hypercortisolism

Physiologic Physiologic
hypercortisolism hypercortisolism CS unlikely
excluded suspected 1· 1·

Normal:
Abnormal :
Repeat initial abnormal
CS confirmed
test; do addit ional tests

FIGURE 6 . Algorithm to confi rm or rule out the diag nosis of Cushing syndrome. CS = Cushing syndrome; DST=dexamethasone suppression test; LNsaliva ry
cortisol = late-night salivary cortisol; UFC =urine free cortisol.
•Must be perlormed at least twice.

31
Disorders of the Adrenal Glands

cigarette smoking or use of chewing tobacco. LN salivary cor- suppression and contralateral adrenal atrophy from long-stand-
tisol testing should not be performed in patients with erratic ing elevated col1isol levels. All patients should therefore be treated
sleep schedules (for example. shift-workers). with stress-dose glucocorticoids during the petioperative period
After CS has been confirmed biochemically. fur1her testing and continued on physiologic replacement until HPA a-xis recov-
is required to distinguish ACfH-dependent or -independent ery has been confirmed. Following successful surgery. the physi-
causes, and consultation with an endocrinologist is recom- cal changes associated w ith CS can take up to 1 year to resolve.
mended. The first step is to measure plasma ACfl-1 on two
KEY POINTS
separate occasions. With ad renal (ACTH- independent) CS.
plasma ACTH is usually less than 5 pg/mL (1.1 pmol/L), • Cushing syndrome results from endogenous hypercorti-
whereas values greater than 20 pg/ mL (4.4 pmoi/L) are typi- solism or exogenous exposure to glucocorticoids; it is
cally seen with ACTH dependent causes. Plasma ACTH values associated w ith poor suppressibility of endogenous cor-
of 5 to 20 pg/mL (1.1-LI.4 pmol /L) are nondiagnostic but are tisol production with oral dexamethasone.
more likely to be seen with ACfH-dependen t disorders. For a • The most common cause of Cushing syndrome is the
discussion of the evalua tion and management of ACTH- administration of exogenous glucocorticoid therapy for
dependent CS. see Disorders of the Pituitary Gland. another medical condition.
The next step in the evaluation of ACfH-independent CS is • Initial tests for Cushing syndrome include the overnight
with imaging of the ad renal glands, such as dedicated adrenal low-dose dexamethasone suppression test, 24-hour
imaging with thin-section CT or MRI. Both studies have equal urine free cortisol, and late-night salivary cortisol.
sensitivity; however. MRI is more costly. Adrenal adenomas and
ca rcinomas can usually be distinguished from one another Pheochromocytomas and Paragangliomas
radiographically (Table 19). Surgery is considered first-line Paragangliomas are tumors composed of chromaffin cells.
treatment for adrenal adenomas and non metastatic adrenocor- Approx imately 80'Yo are intra-adrenal (pheochromocytomas):
tical ca rcinomas (ACCs). When surgery is delayed for patients the rest originate from extra-adrenal sympathetic or parasym-
w ith overt CS. adrenal enzyme inhibitors (metyrapone, keto- pathetic paraganglia. The most common location for extra-
conazole, and etomidate) can be used to reduce cortisol levels adrenal sympathetic paragangliomas is the abdomen, whereas
and decrease the risk of complications. such as oppol1unistic pa rasympathetic paragangliomas are usually found in the head
infections and cardiovascular events. The management of ACC and neck. Pheochromocytomas and extra-ad renal sympathetic
is discussed elsewhe re (see Adrenocol1ical Carcinoma). paragangliomas almost always secrete catecholami.nes (norepi-
Following adrermlectomy. patients with adrenal CS will nephrine. epinephrine. dopa mine): however. head and neck
often develop acute adrenal insufficiency because of HPA axis parasympathetic paragangliomas almost never do.

TABLE 19. Typical Imaging Characteristics of Adrenal Masses


Adrenal Mass Overall CT MRI Signal Intensity•

Adrenal adenoma Diameter <4 em Density < 10 HU ,lsointense on T2-weighted images


Homogeneous enhancementb Contrast washout >50% (10 min)
Round, clea r margins
Ad renocortical Usually > 4 em Density> 10 HU Hyperintense on T2-weig hted images
ca rcinoma
Heterogeneous enhancementb Contrast washout <50% ( 10 min )
Irreg ular margins
Calcifications, necrosis
Pheochromocytoma Variable size Density> 10 HU Hyperi ntense on T2-weighted images
Heterogeneous enhancementb, Contrast washout <50% (1 0 min)
cystic areas
Round, clea r margins
Can be bilateral
Metastases Variable margins Density > 1 0 HU Hyperintense on T2-weighted images
Can be bilateral Contrast washout <50% ( 10 min)

HU = Hounsfie ld units (me asure of radiodensity compared w ith water).


•S•gnal intensity as compared with liver.

bEnhancement follow ing intrave no us contrast adminis tratio n.

32
Disorders of the Adre nal Glands

Although catecholamine-secreting tumors are rare over- low (adrenal mass without typical radiographic appearance),
all, they are fou nd in 0.5% of patients with hypertension, and while measurement of plasma free metanephrines is preferred
pheochromocytomas account for 5% of adrenal incidentalo- when clinical suspicion is higher (known hereditary syn-
mas (see Incidentally Noted Adrenal Masses). Most pheochro- d rome). Referral to an endocri nologist is recommended when
mocytomas secrete norepinephri ne, resulting in episodic or biochemical testing is abnormal.
sustained hypertension. Orthostatic hypotension can also be Many medica tions and other substances cause falsely
seen and likely reflects low plasma volume. In addition to t he high levels of plasma and urine catechola mines or metane-
classic triad of diaphoresis, headache, and tachycardia, com- phri nes (Table 21): therefore, discontinuation of t hese agents
mon symptoms include palpitations, tremor, pallor. and anxi- before testing is recommended. If a catecholamine-secreting
ety. Less common features are papilledema, diabetes mellitus, tumor is strongly suspected in a critically ill hospitalized
and cardiomyopathy. Approximately 10%of pheochromocyto- patient, Cf or MRl of the abdomen is the preferred initial test
mas and 20% to SO% of paragangliomas are malignant. because biochemical testing cannot be interpreted reliably in
One third of pheochromocytomas and paragangliomas this setting.
occur in the context of a genetic disorder. Pheochromocytomas Following the biochemical diagnosis of pheochromocy-
are seen with multiple endocrine neoplasia (MEN) syndromes toma or catecholamine-secreting paraganglioma, radiographic
type 2A and 2B (Table 20). neurofibromatosis type 1, and von localization is needed. Because most catecholamine-secreting
Hippel-Lindau syndrome (VHL) . Paragangliomas a nd. less tumors are located in the abdomen, cr or MR! of the abdomen
frequently, pheochromocytomas can occur with familial para- and pelvis is the best initial study. lf negative, iod ine 123
ganglioma syndrome mutations, some ofvvhich are associated ( 121 !)-metaiodobenzylguanidine (MlBG) scanning can be per-

with high rates of malignancy. formed. Adjunctive diagnostic tests are Cf or MRl of the chest
The diagnosis of pheochromocytoma and paraganglioma or head and neck region.
is based on confirmation of the excessive secretion of catecho- When multiple tumors or malignant pheochromocyto-
lamines or their metabolites, as measured in the plasma or mas or paragangliomas are suspected, M!BG scanning should
urine. Eva luation is recommended. if clinically suspected. in be performed preoperatively. However, for identification of
the evaluation of an incidentally noted adrenal mass or in the metastatic disease. fluori ne 18 (18 F)-f1uorodeoxyglucose (FDG)
sett ing of hereditary pheochromocytoma or pa raganglioma PET scanning is superior to other diagnostic tests.
syndromes. The sensitivity of plasma free metanephrines is Preoperative pharmacologic treatment is mandatory tor
the highest of any screening test (96%-100%): however. its pheochromocytomas and paragangliomas to prevent life-
specificity is relatively low (85%-89%). Therefore, plasma free threatening cardiovascular complications related to the mas-
metanephrines will reliably exclude a pheochromocytoma sive release of catecholamines during surgery. Preoperative
when negative, but further testing is needed to confirm the blockade of a -adrenoceptors, usually with pheno)..'Y'?e.n-
diagnosis unless the result is markedlY' abnormal (above 4 zamine, is first-line medical therapy. The dosage is titrated to
limes the upper limit of normal). The sensitivity and specific- achieve a blood pressure below 130/80 111111 Hg seated and
ity of24-hour urine fractionated metanephrines and catecho- grea ter than 90 mm Hg (systolic) standing. Commonly used
lamines are to 98%. Due to the lower frequency of bul non-FDA approved alternatives include calcium cha nnel
false-positive results. 24-hour urine measurements are recom- blockers and selective a 1-blockers (terazosin or doxazosin).
mended when the pre-test probability of disease is relatively blockers (metoprolol or propranolol) are

Feature

MEN1 (inheritance of one mutated Parathyroid adenoma Pancreatic islet cell and ent eric tumors (gastrinoma, insuli no m a
allele with somatic mutation in (often m ultiple) most com m on)
other al lele leads to neoplasia)
Pituitary adenoma
Other (ca rcinoid tumors, adrenoco rtica l adenoma)
2A RET Medu llary thyroid Pheochromocytoma (often mult ifocal)
carcinoma
(exon 11, codon 634•) Parathyroid hyperplasia
28 RET M edu llary t hyroid Pheochromocytoma (ofte n multifocal)
carcinoma
(exo n 16, codon 918•) Mucosal neuroma
Gastrointest inal gangl ioneuroma
Marfanoid b ody habitus

"' Most co mmon mutation obse rved .

33
Disorders of the Adrenal Glands

TABLE 21. Substances Associated with False-Positive K EY P 0 I NT S (continued)


Biochemical Testing for Pheochromocytoma
• When clinical suspicion of pheochromocytoma or para-
Drug Class Medication/Substance ganglioma is low, measurement of 24-hour urine frac-
Analgesics Acetam inophen tionated metanephrines and catecholamines are the
tests of choice because of their high specificity (low
Antiemetics Prochlorperazine
false-positive rates) ; when clinical suspicion is high,
Antihypertensives Phenoxybenzamine•
measuring plasma free metanephrines is preferred due
Psychiatric Anti psychotics to its greater sensitivity.
medications
Buspirone • Preoperative a -adrenergic blockade is first-line medical
Monoamine oxidase inhibitors therapy for pheochromocytomas and paragangliomas;
Tricyclic antidepressants• blockers (metoprolol or propranolol)
Stim ulants Amphetamines are added after a -blockade to treat reflex tachycardia.
Cocai ne • Pheochromocytomas and paragangliomas require life-
Caffeine
long surveillance fo r recurrence with annual plasma
free metanephrine measurement.
Other agents Levodopa
Decongestants (pseudoephedrine) Primary Hyperaldosteronism CJ
Reserpine Primal) hyperaldosteronism (PA) results from the autono-
Withdrawal Clonid ine mous secretion of excessive aldosterone. PA is relatively com
Ethanol mon. occurring in approximate!) 10'% , of patients wit h
hypertension. Additional signs of PA include hypokalemia and
Illicit drugs
metabolic alkalosis. Without treatment. excess cardiovascular
aMost likely to cause false-positive results.
morbidil) and mortal it) are seen.
Aldosterone-producing adrenocortical adenomas (APA ;
aldosteronomas) cause approximately of PA. whereas
added later to treat reflex tachycardia. but should never be nearly all other cases are due to bilateral adrenal hyperplasia.
started before adequate a -blockade has been achieved due to L:nilateral adrenal hyperplasia and aldosterone secreting
the risk of hypertensive crisis from unopposed a -receptor adrenocortical carcinomas (ACCs) are rare. Fami lial hyperal
stimulation. A heart rate of 60 to 70/min seated and 70 to 80/ dosteron ism is also uncommon.
min standing can be targeted in most patients. Testing for PA should be considered in all patients with
Because it is associated with fev,rer diff1cult to control h) pcrtension. It should also be performed
and shorter postoperative hospital stays. laparoscopic adrenal- in patients with hypertension and an incidental!) noted adre
ectomy is preferred fo r pheochromocytoma except in the case nal mass or spontaneous or diuretic induced hypokalemia. A
of large or malignant tumors, when open adrenalectomy is number of disorders can mi mic PA clinically; however. the
required. Following the surgical removal of a catechola mine- resu lts of biochemical testing will differ. Examples include
secreting tumor, large-volume intravenous crystalloid is secunda!) hyperaldosteronism (renal artel)· stenosis or
administered to counter hypotension. Vasopressors (for exam- secreting tumors). autosomal dominant pseudoaldosteronism
ple, norepinephrine) are sometimes required. Long-term (Liddle syndrome). CS. certa in forms of congenital adrenal
follow-up is needed for pheochromocytomas and paraganglio- hyperplasia (CAH). and licorice induced hypermineralocorti
mas due to difficulty distinguishing benign from malignant coidism.
tumors. Metastases have been reported up to 20 years after Initial screening tor PAis with the simultaneous measure-
diagnosis. In addition to routine clinical surveillance, annual ment of midmorning ambulatol)· plasma ren in activit) (PRA)
measurement of plasma or urine metanephrines is indicated and plasma aldosterone concentration (PAC). in a volume
to assess for recurrent or metastatic disease. Metastatic disease replete normokalemic patient. Testing is positive if PAC is
is man aged with additional surgery, iodine 131 (1311) -labeled frank)) elevated (>15 ng dL [414 pmol L]}, PRA is suppressed,
MIBG therapy, chemotherapy, and/or radiotherapy. Cure is not and PAC PRA ratio is greater than 20. Many medications,
possible unless all disease can be surgically resected. including common antihypertensiYe agents. can affect meas
urements of PAC. PRA. or both (Table 22). Ho\veYer, because
KEY POINTS
patients undergoing screening often have drug-resistant hyper
• Pheochromocytomas are seen w ith mu ltiple endocrine tension. discontinuing all potentially offending medications
neoplasia (MEN) syndromes type 2A and 2B, neurofi- can be unsafe. Stopping m ineralocorticoid receptor antagonists
bromatosis type 1. and von Hippel-Lindau syndrome. (spironolactone and eplcrenone) for 4 to 6 weeks prior to
ing is recommended. Diuretics shou ld also be discontinued
(Contin ued)
prior to testing to assure euvolemia. \ lost other medications

34
Disorders of the Adre nal Glands

TABLE 22. The Effect of Commonly Prescribed lem ia w ith undetectable PRA and PAC greater than 30 ngldL
Medications on Measurements of Plasma Renin Activity and (828 pmol L). Confirmatory tests include oral and intravenous
Plasma Aldosterone Concentration salt loading and the fludroco rtisone suppression and captopril
Effect on Test Medication Class PRA PAC PAC/ challenge tests (Table 23) .
Results PRA Once the diagnosis of l'A h<lS been confirmed biochemi -
False-Positive a -Ad renoceptor H l i . cally. radiographic localization with abdomi nal CT is indi-
ago nist cated . CT is recommended over MRI in most cases due to
H l i similar efficacy and lower cost. .-\drenal hyperplas ia and
b locker adenomas can often be visualized and ad renocortical carci
Dire ct renin l l i noma can be ruled out. Adrenal vein sampling (AVS) is
inhib it or needed in most patients to determine the source of aldoster-
NSAID H l i one secretion when imaging is unrevealing and to confirm
False-Negative ACE inhibito r/ ARB n l l lateralization v\'hen imaging demonstrates an ad renal ade-
noma. AVS is especially important in older patients (40 years
Dihydropyrid ine i l l
CCB and older) because of a higher frequency of nonfunctioning
adrenal incidentalom<.ls. AVS should be performed at experi-
Diuretic• n i l
enced centers only.
M inera locorticoid n i l
The goals of treatment include improvement in blood
rece ptor
antagonist pressure (resolution of hypertension is un likely) . normali-
SSRI i l zation of serum potassi u m (this is very likely). reduc
tion in plasma aldosterone because hyperaldosteronem ia is
ARB - angioten sin recepto r antagonist; CCB = calcium channel blocker; PAC -
plas ma aldoste ro ne concentration; PRA - plasma renin act ivity; SSRI = selective associated with a blood pressure independent increase in
serotonin reuptake inhibitor.
cardiovascular events. The treatment of choice for PA due
potassium-spa ri ng (amiloride ) a nd potassium -wasting (hydrochlorothiazid e )
d iuretics .
to AP/\ or uni lateral <ldrenal hyperplasia is laparoscopic
ad rcnalcctomy.
For patients with bilateral adrenal hyperplasia or those
can be continued, but results must be interpreted in context. with unilateral causes or PA who are not surgical ca ndidates.
Cl For example, if PRA is suppressed despite treatment with an medical therapy \\'ilh a mineralocorticoid a ntagonist is indi
CONT ACE inhibitor o"r angiotensin receptor blocker. PA is likely If cated. Spironolactone is the most commonl:y used medication
results are difficult to interpret. repeat testing after eliminating due to its proven efficacy and cost-effectiveness. Eplerenone
potential interfe ri ng medications is adYised. Verapamil. is less likely to cause side effects (gynecomastia in men
hydralazine, and a -blockers (doxazosin) can be substituted for menstrual irregularities in women) because of greater mirl.er-
blood pressure control if necessary. Referral to an cndocrinolo alocotticoid receptor sclectiYity. Amiloridc is a potassium
gist is recommended when screening tests are abnormal. sparing diuretic that blocks the aldosterone sensitiYe sodium
Confirmatory testing is performed except when initial channel. Use of amiloride in PAis second-line therapy because
testi ng is diagnostic for PA, as in cases of spontaneous hypoka of lower efficacy. [::J

Captopril Adm inister: Captopril 25-50 mg ora lly after the patient has been PAC remains elevated and PRA suppressed
ch allenge test seated fo r 1 hour
(Normal response is suppressio n of PAC by
Measure: PAC, PRA, and cortisol at 0 and 1 or 2 hours while seated >30%)
Flud rocortisone Admi nister: Fludrocortisone 0.5 mg orally every 6 hours for 4 d ays · PAC >6 ng/ d l ( 165.6 pmoi/ L)
suppression along w ith sodium and potassium supplementation
PRA <1 ng/ mUh (1
test
Measure: Serum cortisol at 7 and 10 AM, and PAC and PRA at
(Cortisol at 10 AM lower than 7 AM)
1 0AM on day 4
Oral salt Administer: Sodi um chlorid e 6 g orally daily (in divided doses) for 24-hour urine aldost erone> 12
loading test 3 days
(Urine Na > 200 m Eq [220 mm oi/L])
Measure : 24-hou r urine aldosterone and urine Na on the third day
Intravenous salt Administ er: 2 L 0.9% saline intravenously over4 hours wh ile su pine PAC > 10 ng/dL (27 6.0 pm o i/L)
load ing t est
Measure: PAC, PRA, cortisol, and serum K at 0 and 4 hours

IM =intra m uscular; IV = intraven ous; K "" potassium ; Na .,. sodium; PAC - p lasm a a ldosterone concentration; PRA = plasma renin activity.

35
Disorders of the Adrena l Gl ands

KEY POINTS have at least one other autoimmune endocrine disorder, and
• Testing for primary hyperaldosteron ism is with the more than 80% have adrenal autoantibodies (21-hydroxylase
simultaneous measurement of midmorning ambulatory antibodies). Infiltralion of the adrenal glands by tuberculosis
plasma renin activity and plasma aldosterone levels; (Add ison disease) was formerly the most common etiology of
testing is positive if plasma aldosterone concentration is pri ma1y adrenal failure; now it is responsible for only 7"/o to
frankly elevated (>15 ng/dL [414 pmoi!LJ) , plasma renin 20%. Replacement of the adrenal glands can also occur with
activity is suppressed, and a ratio of the former over the metastatic cancer. Genetic causes include autoimmune poly-
latter is greater than 20. glandular syndromes (APS) type 1 and 2, congenital adrenal
hyperplasia, and X-linked adrenoleukodystrophy. Adrenal cri-
• The treatment of choice for primary hyperaldosteronism sis resulting from bilateral adrenal hemorrhage can occur with
due to an aldosteronoma or unilateral adrenal hyperpla-
the antiphospholipid syndrome, disseminated intravascular
sia is laparoscopic adrenalectomy; for patients with
coagulation, or systemic anticoagulation .
bilateral adrenal hyperplasia or those with unilateral The clinical presentation of pri mary ad renal failure
causes of primary hyperaldosteronism who are not can-
depends on disease chroni city and the presence of physical
didates for surgery, medical therapy with a mineralocor-
stressors. In autoimmune adrenalitis, the zona glomerulosa is
ticoid antagonist such as spironolactone is indicated. usually affected first, which is manifest by an increase in PRA.
With involvement of the zona fascic ulata, a diminished cort i-
Androgen-Producing Adrenal Tumors sol response to ACTH is seen, followed by an increase in basal
Pure androgen-secreting adrenal neoplasms are very rare. plasma ACTH, and lastly a decrease in serum cortisol. Patients
These tumors usually secrete DHEA and DHEAS and /or typically do not have symptoms until hypocortisolemia occurs.
androstenedione, which are converted peripherally to testos- Table 24 shows the clinical and laboratory manifestations of
terone. Approximately half of androgen-producing tumors are primary adrenal failure. Hyperpigmentation is a clinical hall-
benign and half are malignant. Manifestations of androgen- mark of this disorder that is not seen with secondary cmiisol
producing adrenal tumors are usually absent in adult men, deficiency (see Disorders of the Pituitary Gland tor discussion
although decreased testicular volume can occur. In women, of secondary cortisol deficiency).
rapid onset of hi rsutism, menstru al irregularities, and viriliza- Adrenal crisis may occur when onset of adrenal failure is
tion can be seen and, if present, should raise suspicion for abrupt (bilateral adrenal hemorrhage) or when increased
tumoral hyperandrogenism. Signs of virilization are deepen- stress occurs in the ' setting of chronic adrenal failure.
ing of t he voice, clltoromegaly, and temporal hair loss. The Manifestations of adrenal crisis include shock, hypotension,
diagnosis of an androgen-producing adrenal tumor is based fever, nausea, vomiting, abdominal pain, tachycardia, and
on demonstrating elevated levels ofDHEA and its sulfate (usu- even death . Aldosterone is critical to the maintenance of intra-
ally greater than 800 )lg/dL [21.6 )lmol!L]) amillor androsten- vascular volume and blood pressure, while cortisol contrib-
edione. Although adrenal androgen excess can be seen in 30% utes to augmentation of blood pressure mostly during times of
to 40% of women with polycystic ovary syndrome, mild eleva- increased physical stress (see Adrenal Anatomy and
tion ofDHEAS (approximately 300 )lg/dL [8.1 f.imol!L) is typi- Physiology). Aldosterone deficiency is the major impetus for
cal. Adrenal imaging with CT or MRI is indicated following the development of hypotension and shock in patients with
biochemical diagnosis of disease to locate the tumor. r:eatment untreated primary adrenal failure. Adrenal crisis is rare in the
is surgical removal of the tumor. setting of secondary cortisol deficiency because the renin-
angiotensin-aldosterone pathway is intact.

Adrenal Insufficiency Diagnosis


Adrenal insufficiency may be due to fail ure of the adrenal · The diagnosis of primary adrenal failure is based on demon -
glands (primary adrenal failure) , or there may be inadequate strating inappropriately low serum cortisol levels. Because
secretion of cortisol from the adrenals due to other causes, most assays measure total cort isol, abnormalities in cortisol-
including critical illness and pituitary ACTH deficiency (sec- binding protein or albumin can trigger spurious results. An
ondary cortisol deficiency) . For a discussion of secondary early morning (8 AM) serum cortisol of less than 3 )lg/dL
cortisol deficiency, see Disorders of the Pituitary Gland. (82 .8 nmol/L) is consistent with cortisol deficiency, whereas
values greater than 15 to 18 )lg/dL (414.0-496.8 nmol! L)
Primary Adrenal Failure exclude t he diagnosis when binding protein abnormalities and
Causes and Clinical Features synthetic glucocorticoid exposure are excluded. For patients
Primary adrenal failure is a rare disorder resulting from a fail- with nondiagnostic basal cortisol values (5-12 f.ig/dL [138-331.2
ure in production of all the hormones of the adrenal cortex. nmol!L), stimulation testing w ith synthetic ACTH (cosyntro-
The overall prevalence is 10 to 15 per 100,000 persons. pin) is indicated (see Disorders of the Pituitary Gland). A nor-
Autoimmune adrenalitis is the most common etiology mal response is a peak serum cortisol level greater than 20 f.ig /
accounting for 70% to 80%of cases. Up to two thirds of patients elL (552 nmol/ L). ACTH stimulation testing should not be used

36
;

Disord e rs of the Adrenal Glands

TABLE 24. Clinical and Laboratory Manifestations of Primary Adrenal Failure


Hormone Deficiency Symptoms Signs Laboratory
Findings

Cortisol Fatigue Hyperpigmentationb (palmar creases, l Seru m cortisol


exten·sor surfaces, b uccal mucosa)
Weakness I Plasma ACTH
Decrease in BP
Low-grade fever -1- Serum sodiumc
Weight loss -1- Plasma glucosed
Anorexia
Nausea/ vom iting
A bdomi nal pain
Arthralgia
Myalgia
Aldosterone Salt craving Orthostasis i PRA
Dizziness Hypotension -1- Serum sod ium

I Serum potassium
DHEAS Reduced libido• Decreased axil lary or pubic hai r• t Serum DHEAS
ACTH - adrenocotticotropic hormon e; BP - blo od pressure; DHEAS - d ehydroeptandrosterone sulfate; PRA - plasma re nm activity.

only

bOccurs excl usively in primary ad ren al failure.

ccortiso l inhibits th e se cretion of antid iuretic ho rm one (ADH), so hypoco rtisole mia will lead to increased secretion of AD H and hypo natre m ia .
1
dRare in ad ults.

for diagnosis in the critical care setting (see Adrenal Function longer-acting glucocorticoids (prednisone or dexamethasone)
During CriticaU!lness) . is acceptable. All patients with cortisol deficie ncy need to
Once the diagnosis of cortisol deficiency has been estab- receive instructions for increasing their cortisol replacement
lished, measurement of 8 A M plasma ACTH will differentiate dose during illness ("sick day rules"). Patients should always
primary and secondary causes. In adrenal failure, wear a medical alert identification.
ACTH is typically greater than 200 pg /mL (44 pmol /L) , In contrast to patients with secondary cortisol deficiency
whereas it will be low or inappropriately norma! in secondary (see Disorders of the Pituitary Gland) . those with primary
cortisol deficiency. Alt hough not specific for the diagnosis, adrenal failure also require mineralocorticoid replacement.
hyponatremia and hyperkalemia are characteristic of primary Usual doses are O.OS to 0.2 mg per day of tludrocortisone.
adrenal failure and principally result from aldosterone defi- Measurements of serum sodium and potassium help guide
ciency. dosing. Replacement of DHEA is controversial. It is not indi-
cated for men but can be considered for some women with

Cl Without appropriate treatment, primary adrenal failure is


Treatment primary adrenal failure. However, the objective benefit is min
imal, and there arc concerns regarding the quality and safety
uniformly fatal. Even when treated. the mortality of patients is. of U.S. preparations where DHEA is considered a supplement
twice that of the general population. Normal adrenal physiol- rather than a pharmaceutical.
ogy cannot be reproduced exactly by the administration of Patients who present emergently with suspected adrenal
exogenous glucocorticoids and mineralocorticoids. Moreover. crisis should· he treated empirically prior to confirmation of
the administration of doses of glucocorticoid in excess of the diagnosis. A blood sample should be drawn fo r serum
physiologic replacement can be associated with decreased cortisol, plasma ACTH, and routine chemistries. The patient
hone mineral density and features of CS, with increased risl< of should receive immediate treatment with 100 mg of hydro-
metabolic syndrome, type 2 diabetes mellitus, hypertension, cortisone intravenously and aggressive fluid resusc itation.
hyperlipidem ia, obesity, and cardiovascular disease. Avoidance Hydrocortisone is continued at 100 to 200 mg pe r day in
of chronic overreplacement is paramount. divided doses (every 6 8 hours) and then tapered to physio
Table 25 shows the medica l t reatment for primary adrenal logic replacement if cortisol deficiency is confirmed with the
failure. Most patients require glucocorticoid doses equivalent above testing. Other synthetic glucocorticoids can also be
to 12 ..5 to 2.5 mg of hydrocortisone daily. Hydrocortisone is used for the treatment of adrenal crisis; however. only hydro-
administered 2 to 3 t imes daily. while once daily dosing of cortisone in supraphysiologic doses has clinically relevant

37
Disorders of the Adrenal Glands

TABLE 25. Chronic Medical Treatment of Primary Adrenal Failure


Medication Basal Dose Considerations

Glucocorticoid• Hydrocortisone "Sick Day Rules": patient fol lows at home


Hydrocortisone Usually 12.5-25 mg/d, divided into For minor physiologic stress (upper respiratory infection, fever, minor
2-3 doses over the day surgery under local anesthesia)
Prednisone
Alternatives to hydrocortisone: 2-3 times basal dose for 2-3 days
Prednisolone
Prednisone 2.5-5 mg once daily Stress Dosing: health care providers follow w hile patient is in the hospita l
Dexamethasone
Dexamethasone 0.25-0.75 mg once For moderate physiolog ic stress (m inor or moderate surgery with genera l
da ily anesthesia)
How to dose: Hydrocortisone 45-75 mg/d orally or IV in 3-4 d ivided doses for 2-3 days
Titrate to clinical response with goal Alternatives: Prednisone 10-20 mg or dexamethasone 2-3 mg/d in
of no signs or symptoms of cortisol 1-2 divided doses
deficiency or excess (increase dose if
For major physiologic stress (major surgery, trauma, critical illness, or
symptoms of cortisol deficiency
childbirth)
remain; decrease if CS signs and
symptoms are present) Hydrocortisone 150-200 mg/day IV in 3-4 divided doses; 100 mg/day the
next day; taper to baseline in 3-5 days
Alternative : Dexamethasone 6-8 mg/d IV in 2-3 d ivided doses
Mineralocorticoid 0.05-0.2 mg once daily in the morning Fludrocortisone is not req uired if hydrocortisone dose >50 mg/d
Fludrocortisone How to dose:
T1trate to
1. Normal BP
2. Normal serum Na, K
Adrenal 25-50 mg once da ily Consider DHEA f or women w ith impaired mood or sense of well-being
and rogen when glucocorticoid replacement has been optimized.
DHEA

BP • blood pressure; CS • ..Cushing syndrome; OHEA - dehydroepiandrosterone; IV intravenous; Na = sodium; K = potassium

.,Shorter acting glucocorticoids may be preferred over longer actmg agents due w lower nsk of glucocorticoid excess_ Longer-actmg preparations have the advantage of once
daily dosing (see Table 17).

Cl ties
mineralocorticoid activity. lf present. electrolyte abnormali
and hypoglycemia should be treated. and precipitants of
replacements for which there has not been a demonstrated
biochemical need.
CONT. adrenal crisis (for example. infection) should be sought and
treated. Adrenal Function During Critical Illnes s
It is critical that patients with suspected primary or sec- Glucocorticoid deficiency related to critical illness is an entity
ondary cortisol deficiency and concomitant hypothyroidism that has not been well chamcterized. It has been postulated that
be treated with glucocorticoids first because correcting thy critical illness may lead to transient primar) or secondary cor
roid hormone deficiency will accelerate the clearance of corti I isol deficiency {ACfH deficiency) or an increase in tissue resist
sol and can precipitate acute adrenal crisis. ance to cortisol. The American College of Critical Care Medicine
In the nonmedical literature. the term .. adrenal fatigue" recommends considering this diagnosis in patients with hypo
has been used to describe a constellation of symptoms. tension who have responded insufficiently to fluids and \·aso
including difficulty sleeping. fatigue. and salt and sugar pressor therapy. A maximum increase in serum cortisol of
craving. hypothetically from long- term emotional or physi 9 jlg dL (248.4 nmol 1.) or less following the administration of
cal stress haYing a deleterious effect on the adrenal glands. synthetic ACfH has been associated with increased mortality
resulting in a simultaneous excess and deficiency of cortisol. from septic shock: howe\er. results of testing do predict benefit
llowever. there is no scientific evidence to support this from glucocorticoid therapy. In the setting of critical ill ness,
claim. and the term ··adrenal fatigue" should not be used. both CBG and albumin concentrations decrease resulting in
Proponents of adrenal fatigue prescribe synthetic glucocor- lower total cortisol. Free cortisol levels. either directly measured
ticoids and supplements containing adrenal. pituitary. or or calculated based on total cortisol and CRG. may be more reli
hypothalamic extracts that can cause iatrogenic CS. as well able in critically ill patients '' ith hypoalbuminemia. lt is not
as mineralocorticoid supplements that can lead to hyperten- known if free cortisollc,·els prO\ ide useful prognostic infi:>rma-
sion. Patients should receiYe appropriate e\·a]uation for their tion. The administration ofglucocorticoids has not been sho\\11
symptoms and be educated to aYoid taking hormonal to benefit critically ill patients who do not ha,·e shock. and the

38
Dis orders of the Adrenal Glands

, . , results of placebo controlled randomized trials in patients with be undertaken promptly. An algorithm for the evaluation and
1.1.1 septic shock arc conflicting. Further research is needed to dar follow up of an adrenal mass is shO\\ n in Figure 7.
CONT. if) iftherc is a population ofcritically ill patients who can objec The risk of malignancy \'aries according to size. Only 2°1.,
ti\·ely benefit from glucocorticoid therapy Cl of adrenal masses smaller than 4 em are cancerous: however.
25°1o of those larger than 6 em are malignant. An adrenal
mass's risk of malignancy can be clarified based on its appear-
Adrenal Masses ance on CT or MR! (see Table 19 tor the typical radiographic

Cl i\drenal
Incidentally Noted Adrenal Masses
masses are often discovered incidentally when
features of adrenal masses).
Adrenal metastases accoun t fo r about half of adrenal
abdominal imaging is performed fo r another reason. These masses in patients with known nonadrenal malignancies.
adrenal incidentalomas are seen on 4'Yo of all CT scans and 7"\, Cancers that metastasize to the adrena l glands include lym
of those performed in patients 70 years of age and older. The phomas. carcinomas. and melanomas. Percutaneous biopsy is
differential diagnosis includes benign and malignant neoplasia indicated to confirm the diagnosis of adrenal metastasis; how
of adrenal cortex or medulla. adrenal cysts. adrenal hyperpla- ever. this should never be performed prior to ruling out pheo-
sia. metastatic tumors of nonadrenal origin, and infections chromocytoma biochemically. Biopsy is not recommended
and infiltrative disorders. The most common cause of an adre when adrenocortical carcinoma (ACC) is suspected because it
nal mass is an adrenal adenoma. and adrenal metastasis is the cannot reliably distinguish benign from malignant adrenocor
next most common. The two main goals in the evaluation of an tical neoplasia. The evaluation and management of ACC are
incidentally noted adrenal mass are to identify adrenal masses covered in the Adrenocortical Carcinoma section.
that arc to be malignant and those that are associated One quarter of incidentally noted adrenal autono-
\\ith hormonal h) persecretion so that targeted treatment can mously secrete hormones (cott isol 6o;., 10\ ,: catecholamines

Incidentally Noted
Adrenal Mass
- Imag ing Phenotypeb -

Favors Benign: Suspicious:


o Size <4 em o Size >4 em

Imaging Phenotype o Density <10 HU' o Density >10 HU'


. o Contrast washout o Contrast washout 1-- Indications for Adrenalectomy
>50% (10 min)' <50% (10 min)'
Suspicious imaging
Hormonal Evaluation Growth >1 em/yea r
Functioning tumors:
Tests for Hormone Excessd -
• Pheochromocytoma

Follow Up• Cortisol: Aldosterone: o Aldosterone-producing adrenal tumor

o Who: All patients • Who: HTN or -.1-K+ o Subclinical CS with complicati on(s)9
• Repeat CT or MRI
o Test: LOST o Test: PRAIPAC
at 3- 6 months then
annually for 1-2
years
o Repeat hormonal Catecholamines: Androgens:
evaluation annually o W ho: All patients o Who: If suspectedf
for 4 years
o Test: Plasma or o Test: DHEAS,
urine metanephrines, testosterone,
urine catecholamines• androstenedione

FIGURE 7. Algorithm for the initial diagnostic evaluation and follow up of an incidentally noted adrenal mass. CS = Cushing syndrome; DH EAS = dehydroepiand roster·
one sulfate; HTN =hypertension; HU = Hou nsfield units; K =potassium; LDST = low·dose {1 ·mg) dexamethasone suppression test; PAC= plasma aldosterone concentra·
tion; PRA = plasma renin activity.
•Repeat imag1ng and hormone testmg are ind1cated for adrenal masses not meetlng wtena for surgery at Jnltlal diagnos;s.

'=Refer to Table 19 for more 0 and MRI fmdings.lf imaging is suspicious in a pafent with known ma lignancy, b1opsy should be considered to<onfirm ad renal metastasis afterscreening for pheoc hromocytoma is completed.
•CT scan fondings.

rtPositive screening tests usually require further bio<:hemical evaluation to conf1rm the d1agnos1s (see text)

'Measure plasma metanephrines if ra diographicappearance is typical for a pheo<hromocytoma; othetwJse measure 24·hour urine metanephrines and catecholamines.
1
Hormonal evaluat1on for an androgen-producing adrenal tumor IS ind1cated only1f clmicallysuspected based on the presence of h1rsut1Sm, v1nlizati on, or menstrual irreg ulan tJeS 1n women.
gAdrenalectomyis considered for confirmed cases of subclinical CS associated with recent onset of di abetes, hypertension, obesity, or low bone mass.

39
Disorders of the Thyroid Gland

Cl S'Yo; aldosterone 1')(,). Excess cortisol secretion is most com


mon; however, the majority of patients have subclinical
Adrenocortical Carcinoma
ACC is a rare malignancy aflecting 0.5 to 2 persons per million
CONT. ease wtt · h out c lasstc
. st tgmata
' ot· CS
" . D espt'te tlus.
. . .tmport ant
per year that is often associated with the excessive production
complications may be seen, including osteoporosis. hyperten of adrenal hormones. Patients with ACC most frequently pre-
sion, diabetes mellitus, and cardiovascular events. The LDST is sent w ith signs and symptoms related to hormonal excess.
the initial screening test of choice. A serum cortisol value They may also experience symptoms relat,ecl to local tumor
greater than S (138 nmol!L) is considered positive; how grovvth (abdom inal fullness, nausea , or back pain) or metasta-
ever. some advocate using a cut off of 1.8 (49.7 nmol/ L) sis. ACC is sometimes detected incidentally when abdominal
to increase diagnostic sensitivity if CS is suggested by history imaging is performed for another reason (see Incidentally
or physical examination. Because the specificity of the LOST is Noted Adrenal Masses).
only approximately 90%, the diagnosis of subclinical CS Autonomous secretion of adrenal hormones or their bio-
should be confirmed with additional testing. For a review, see logically inactive precursors is seen in more than 80% of
the Cushing Syndrome section. patients with ACC (cortisol SO%; multiple hormones 20%;
Aldosteronomas are usually smaller than 2 em . Case androgens 5% to 10%; aldosterone rarely). The pa thologic
detection fo r PAis performed in all patients with hypertension diagnosis of ACC is challenging, such that with low-risk
or those on antihypertensive medications. Testing for pathology but large tumor size or concerning imaging find-
mous secretion of adrenal androgens is performed if clinically ings (see Table 19) close interval radiographic follow up is
suspected following careful history and physical examination. needed after surgery.
All patients with an incidental adrenal mass should be tested The prognosis of ACC is very poor with an overall
for pheochromocytoma. Measurement of urine ity rate of 67% to 94%. Management depends on the extent of
metanephrines and catecholamines is the preferred first test in disease a't presentation. Open surgical resection is fi rst-line
most asymptomatic patients, clue to the lower incidence of treatment tor early disease. Adj uvant radiotherapy to the
false positive test resu Its. However, if the radiographic tumor bed is used when resection is incomplete. Adjuvant
ance of the adrenal mass is suspicious tor a pheochromocy medical therapy w ith mitotane, an adrenolytic drug, is
toma (see Table 19) or the patient is symptomatic, then plasma mended for patients with known or suspected residual or
free metanephrines should be measured (see Primary metastatic disease. Cytotoxic chemotherapy has poor efficacy.
Hyperaldosteronism. Adrenal Tumors. In addition to mitotane, inhibitors of adrenal steroidogenesis
and Pheochromocytomas and Paragangliomas) . (metyrapone, ketoconazole, and etomidate) are used to treat
Adrenal that arc larger than 4 em, those with CS, if present. Surgery for metastatic ACC is indicated if
risome rad iographic t1 ndings. and pheochromocytomas should toms related to hormonal hypersecretion cannot be controlled
be removed surgically. Surgery is also indicated tor unilateral w ith medical therapy alone. Percutaneous radiofrequency .
alclosteronomas and is considered tor patients with subclinical ablation may also be used to treat unresectable primary
CS associated with the recent onset of diabetes. hypertension, tumors or metastases when needed.
obesity. or low hone mass. For nonfunctioning adrenal masses.
KEY POINT
if imaging favo rs a benign lesion, repeat radiographic evalua
tion is recommended in 3 to 6 months. and then annually for) • Adrenocortical carcinoma is marked by signs and
to 2 years. Adenomas usually will not grow more em over symptoms related to hormonal excess as well as
12 months. More rapid growth should prompt adrenalectomy. toms related to local tumor growth (abdominal fullness,
Screening for hormonal hypersecretion is repeated annually nausea, or back pain) or metastasis.
for 4 years, as in the rare instance that the mass becomes
tiona]. it is likely to occur in the first 4 years following its dis
covery. A recent study documented subclinical CS on follow up
testing in approximately 8% of patients who were thought to
Disorders of the
have nonfunctioning adenomas at initial screening. Cl Thyroid Gland
KEY POINTS Thyroid Anatomy and Physiology
• The two main goal s of evaluation of ad renal inciden - In healthy adults in the Un ited States, each thyroid lobe nor-
talomas are to identify adrenal masses that are likely to mally measures up to 5 em in length, 2 em in wid th, and 2 em
be malignant and those that are associated w ith in depth; the entire gland weighs 10 to 20 grams. The isth mus,_,. ·
nal hypersecretion so that targeted treatment can be a thin band of thyroid tissue that connects the tvvo lobes, is 1
undertaken promptly. to 4 mm in thickness and is typically not palpable. Diffuse
• Adrenal masses that are larger than 4 em, those with thyroid disorders, such as lymphocytic thyroiditis, may result
worrisome radiographic findings, and in enlargement of the isthmus to 5 111111 or more, which may be
tomas should be removed surgically. palpable and give the clinician the false sense that the entire
thy roid is enlarged.

40

I
I
Disorders of the Thyroid Gland

There are two forms of active thyroid hormone: thyrox- range. During pregnancy, the range shifts lower and varies by
ine (T4 ) and triiodothyronine (T). Iodine is necessary for the trimester (see Thyroid Function and Disease in Pregnancy).
for mation of thyroid hormone. Deficiency may result in The second important exception is in the elderly. With aging,
hypothyroidism. The hypothalamic-pituitary- thyroid axis the reference range shifts higher; the upper limit of normal
responds to the subsequent hormone deflciency by increasing . extends to 7 11U/mL (7 mUlL) in patients older than 70 years.
thyroid-stimulating hormone (TS!-!) secretion, resulting in The third exception is in patients with known pituitary dys-
development of a goiter. Iodine is typically obtained through function or a risk of pituitary dysfunction (history of cranial
diet; it is present in seafood, dairy products, and iodized sal t. irradiation, pituitary surgery, or massive head trauma).
Although iodine defkiency is a worldwide health problem, it If the serum TSI-I level is frankly abnormal, additional
is relatively rare in the United States with the incorporation of evaluation of thyroid function should be considered to deter-
iodine into salt. mine the extent of the dysfunction. This is accomplished by
Thyroid hormone production is controlled by two main measuring T 1 when the TSH is elevated and by measuring T4
fo rces: secretion of TSH (thyrotropin) and regulation of and T:l when the TSH is suppressed.
peripheral conversion ofT4 toT 3 . TSH release from the anterior When indicated by an abnormal TSH, the circulating level
pituitary is stimulated by decreases in concentrations of serum of thyroid hormone should be assessed using Lola! or free T4
T,1 and T3 and secretion of thyrotropin-releasing hormone levels. Total T4 is a reflection of the bound and unbound frac-
(TRH) from the hypothalamus. The T3 and T4 , in turn, decrease tions of the hormone and is a reasonable method of assessing
secretion of TSH from the anterior p ituitary as parl of a nega- overall thyroid hormone levels in most patients. However, in
tive feedback loop. Additionally, T3 inhibits furthe r secretion of patients with disorders of protein metabolism, such as kidney
TRH from the hypothalamus. or liver disease, measurement of free T4 ensures a J11ore accu-
Although the thyroid gland produces both T3 and T4 , the rate representation of the hormone concentrations.
ratio of T4 to T3 secretion is nearly 20:1, with most T3 (80%) Additionally, conditions that raise serum total protein level,
resulting from 5' -deiodination ofT4 in peripheral tissues. such as in patients taking estrogen or during pregnancy, may
The vast majority of both hormones are bound to circulat- result in a higher tot al T.1 concentration, and measuring the
ing proteins, including thyroxine-binding globulin, trans- free T4 is indicated. The same rules apply to T3 as to T4 regard-
thyretin, albumin, and lipoproteins. The function of these ing protein levels, but free T:; has a very short half- life and
proteins is to increase the circulating pool of hormone by levels fluctuate more and are less reliable; therefore, it is con-
delaying clearance and maintaining a reservoir of hormone troversial whether total or free T3 should be measured in
available for use. Only a small percentage of total circulating patients at risk for protein abnormalities.
thyroid hormone is free (unbound); th is frac tion is readily Measurement of serum T3 is necessary if the patient has a
available for cellular uptake and determines the biologic activ- suppressed TSH level because, in some patients w ith thyro-
ity of the hormone. toxicosis, T 1 may be preferentially secreted over T 1 (T3 toxico-
sis) . In patients w ith an elevated TSH level, indicating
KEY POINT
hypothyroidism, measurement of serum T 1 is not helpful
• Thyroid hormone production is controlled by two main because it will be maintained in the normal range even in
forces: secretion of thyroid-stimulating hormone (thy- those with significant disease.
rotropin) and regulation of peripheral conversion of Thyroid autoant ibody measurement may be helpful
thyroxine (T4 ) to triiodothyronine (T) . under certain clinical circumstances. In patients with a
personal history of autoimmune disease (such as type 1
diabetes mellitus, systemic lupus erythematosus, or celiac
Evaluation of Thyroid Function disease) or a strong family history of t hyroid dysfunction,
In patients with an intact hypothalamic-pituitary axis, the measuring thyroid autoantibodies may indicate the cause of
initial laboratory test of thyroid activity is TSH measurement, the thyroid dysfunction or whether a patient is at risk for
which is exquisitely sensitive for detection of disorders of thy- developing thyroid autoimmune disease if the TSH is nor-
roid dysfunction. In patients for whom there is a high suspi- mal. There is no clinical indication for serial measurement
cion of thyroid or pituitary dysfunction, a concomitant thyroid of thyroid antibody titers to determine the need for or to
hormone (1) level should be assessed with the TSH level to guide therapy. There are three forms of thyroid autoanti-
evaluate fo r central hypothyroidism. The TS!-! level may reflect bodies: thyroid peroxidase (TPO), thyrotropin receptor
hypofunction (h igh TSH), hyperfunction (low TSH), or a nor- (TRAb) , and thyroglobulin (TgAb). Elevated titers of TPO
mal range. antibodies are associated with autoimmune hypothyroid-
The normal reference range tor TSH is variable among ism, or Hashimoto disease. Patien ls wilh TPO antibodies
laboratories but is generally between 0.5 and 5 (0 .5- and normal thyroid fu nction tests are at an increased risk of
5 m U/L), and determinations of normal TSH levels should be developing overt thyroid failure (2 %- 4% per yea r).
made based on the reference range of the laboratory being Thyrotropin receptor antibodies (TRAb) are divided into
used. There are t hree very important exceptions to this general three types: blocking (also called thyrotropin-binding

41
Disorders of the Thyroid Gland

inhibitory immunoglobulins), stimulating (also called KEY P 0 IN T S (continued)


thyroid-stimulating immunoglobulins or TSI). and neutral.
• If central hypothyroidism is suspected, a concomitant
The presence of TSI autoantibodies is responsible for the
thyroid hormone (thyroxine [T)) level should be
development of Graves disease. TSI autoantibodies should
assessed in conjunction with the thyroid-stimulating
be measured if autoimmune hyperthyroidism is suspected.
hormone level.
Thyroglobulin (Tg) is a glycoprotein located within the col-
loid on which thyroid hormones are synthesized and stored. • If the thyroid stimulating hormone level Js frankly HVC
Seru m Tg a nd TgAb measu rements are used to monitor abnormal, additional eva! uation of thyroid function
patients with thyroid cancer; serum Tg is a highly sensitive should be considered to determine the extent of the
and specific marker of residual thyroid tissue. After total dysfunction; measure thyroxine ('I) when the thyroid-
thyroidectomy and radioactive iodine ablation, the persis- stimulating hormone is elevated and measure both
tence of a detectable serum Tg is a possible indicator of thyroxine (T) and triiodothyronine (T) when the
residual or recurrent disease. Thyroglobuli n antibodies are thyroid stimulating hormone is suppressed.
present in up to 30% of patients. Their presen ce in the • There is no clinical indication tor serial measurement of HVC
serum is only significant in patients with thyroid cancer, as thyroid antibody titers to determine the need for or to
they can falsely lower the serum Tg. Therefore, TgAb titers guide therapy except to monitor for residual disease in
should always be assessed simultaneously with the Tg: if patients treated for thyroid cancer.
ant ibodies a re present, the Tg level may not be reliable. • Radioactive iodine uptake is a measure of iodine uptake HVC
Calcitonin, secreted by the C cells of the thyroid. is most by the thyroid over 24 hours; it is used to evaluate the
frequently used as a tumor marker in patients with a history cause of hyperthyroidism and is not indicated in
of medullary thyroid carcinoma. Serum calcitonin levels can patients with normal or elevated thyroid stimulating
help increase the sensitivity of detection of medullary thyroid hormone levels.
carcinoma when used in conjunction with fine-needle aspira-
tion (FNA). However. it is not recommended as a screening test
in all patients with t hyroid nodules because it lacks the requ i-
site specificity. Instead, measurement of calcitonin should be Functional Thyroid Disorders
considered if a patient with thyroid nodular disease has a his- Thyrotoxicosis
tory of hyperparathyroidism or a fami ly history of medullary Evaluation
thyroid carcinoma ar multiple endocrine neoplasia type 2. or Thyrotoxicosis is a term used to describe thyroid hormone
ifthere is clinical suspicion for these disorders. excess from all sources. whereas hyperthyroidism is the more
Radioactive iodine uptake (RAIU) is a measure of iodine specific term to describe thyroid gland overactivi ty.
uptake by the thyroid over a pre-specified time fra me. typi- Thyrotoxicosis may result from endogenous thyroid disorders, -
cally 24 hours. RAIU is used to evaluate the cause of hyperthy- pituita ry tumors. and exogenous levothyroxine. The most
roidism: it is not indicated in patients \•vith normal or elevated common causes of hyperthyroidism are Graves disease and
TSH levels. The degree of uptake is useful for distinguishing toxic adenoma(s).
the various causes of hyperthyroidism. RAIU percentage is The symptoms of thyrotoxicosis include heat intolerance,
typically very high in patients with Graves disease (diffusely palpitations, dyspnea, tremulousness, menstrual irregulari-
increased uptake) and only moderately elevated in t hose with ties, hyperdefecation, weight loss, increased appetite, proxi-
toxic mullinodular goiter (patchy uptake in areas of nodules mal muscle weakness, fatigue, insomnia, and mood
with relative suppressio n of normal tissue) . In contrast, the disturbances. The severity of symptoms may not correlate with
RAIU is very low in those with thyroidit is or exposure to exog- the level of thyroid hormone derange ment. In older patients,
enous thyroid h ormone. The presence of a "cold" nodule on many of the classic symptoms ofthyroid hormone excess may
isotope scanning is an indication for ultrasonography to help be absent, and the only presenting symptom may be atrial
determine if FNA is indicated. RAIU is contraindicated during fibrillation or heart failure: this is known as apathetic hyper-
pregnancy and while breastfeeding. thyroidism.
The initial evaluation based on clinical signs and/or
KEY POINTS .
symptoms of thyrotoxicosis should be measuremen t of
• The initial laboratory test of thyroid activity is thyroid- serum TSH alone. followed by measurement of T 1 and T3
stimulating hormone measurement; in patients with an levels if TSH is suppressed. The typical pattern of hyperthy-
intact anterior pituitary, measurement of thyroid- roidism is TSH suppression with an elevated T.1 and/or T3 . A
stimulating hormone is exquisitely sensitive for normal serum TSH in the setting of an elevated T 1 and/or T3
detection of disorders of thyroid hypofunction (high concentration suggests the presence of a TSH-secreting pitu-
thyroid-stimulating hormone) and hyperfunction (low itary adenoma; these tumors are extremely rare and a re
thyroid-stimulating hormone). ma naged different ly from other causes of t hyrotoxicosis (see
(Co ntinued)
later discussion).

42
Disorders of the Thyroid Gland

KEY POINT serious illness. they should be assessed promptly tor an


adverse reaction to the medication.
• The typical pattern of laboratory studies in hyperthy-
The goal of radioactive iodine ablation is to render the
roidism is thyroid-stimulating hormone suppression
patient hypothyroid, which can typically be accomplished in
with an elevated thyroxine (T) and/or triiodothyronine
. 90'l'o of patients w ith the fi rst treatment. Although a minority
(T) level.
_of patients may develop acute anterior neck pain from radia-
tion thyroid itis, radioactive iodine ablation is typically well
Management tolerated. It may take several months for the development of
Although the specific intervention used is usually determined hypothyroidism, so it is important to monitor thyroid fu nction
by the u nderlying cause and patient and physician preference. tests monthly after therapy. ln a patient w ith severe thyrotoxi
control of the thyrotoxic state may be achieved by one of three cosis, radioactive iodine may provide additional substrate to
treatment modalities: thionamides. radioactive iodine abla- the hyperfunctioning gland , resulting in exacerbation of the
tion, or surgery. hyperthyroid state. Consequently. it may be reasonable to in iti
Rapid control of adrenergic symptoms with a is ate a thionamide prior to ablation to lower t he thyroid hor-
indicated in most patients with thyrotoxicosis. Although pro mone levels.
pranolol is frequently used for its added effect of inhibition of Surgery is rarely fi rst-line therapy. given the in herent
peripheral conversion ofT 1 to T l. cardioselective risks with any surgery. Patients in whom control cannot be
such as atenolol, are preferred owing to the additional benefits achieved with thionamides and those who are not comfo rtable
of decreased central nervous system side effects and improved with radioiodine therapy are typically referred for surgery.
adherence w ith once daily dosing. Because of the increased intrathyroidal vascularity, t he proce-
Methimazole and propylthiouracil (PTL') are the two dure can be technically more difficult than a typical thyroid-
thionamidcs available in the Un ited States. Methimazole is the ectomy. Additionally. restoration of the euthyroid state befo re
preferred agent because it has a higher intrathyroidal retention surgery with thionamides is important to improve hemody-
(potency) . a preferable dosing regimen (typically once daily), namics during general anesthesia and decrease the patient's
and a reduced side effect profile. Antithyroid medications risk of thyroid storm. Cl
reduce T l and T, levels within a few days of initiation, but the
KEY POINT
fu ll effect may take several weeks. :-l"ormalization of a previ-
ously suppressed TSH level may take several months. It is • Control ofhyr,erthyroidism may be achieved by one of
critical, therefore, to monitor T4 and T 3 levels during treatment three treatment modalities: thfonamides, radioactive
of hyperthyroidism because the TSH may not be an accurate iodine ablation, or surgery; modality choice depends on
reflection of the thyroidal status. the underlying cause and patient preference.
Thionamides may be used to prepa.re patients for thy-
roidectomy or radioiodine treatment. or they may be used as Graves Disease
t he primary t herapy. Thionamides may be used for 1 to 2 years Graves disease is a multiorgan system autoimmune disorder
in patients with Graves disease in the hope of achieving that can affect the thyroid, eyes, and skin. It is frequently seen
remission; more det1nitive therapy with radioactive iodine or in women between the ages of 20 and 50 years and is the most
surgery may then be sought a fter that timeframe if hyperthy common cause of hyperthyroid ism in the United States.
roidism persists. Although thionamides are generally well Antibodies against the TSH receptor (TSI or TRAb) stimulate
tolerated, it is important to be famil ia r with their side-effect autonomous production of T and T3 . Patients frequently
profile. The most common reaction to antithyroid medica report a family history of Graves disease, Hashimoto thyroidi-
tions is a rash. seen in up to 10'1o of patients. Additionally. PTLi t is, or other autoimmune conditions.
may cause elevations of aminotransferase levels. Rare cases of On physical examination, patients have elevated sys-
fatal hepatotoxicity have been described w ith PTU. Therefore . tolic blood pressure with a widened pulse pressure, tachy-
its use is reserved for patients who cannot tolerate methima - ca rdia, and a diffuse ly enlarged thyroid. Further inspection
zole and during the first tri mester of pregnancy, when methi- of the thyroid may reveal a bruit. Careful examination of the
mazole has a possible teratogenic effect. A cholestatic pattern skin may reveal pretibial myxedema, an infiltrative process
of liver test abnormalities may also be seen w it h methimazole. that is typically patchy w ith a peau d'orange appearance
but it is typically temporary and milder than that seen w ith to the skin.
PTU. Both drugs may be associated w ith reversible agranulo Diagnosis of Graves disease is made clinically in most
cytosis in approximately 1 in 500 patients. Baseline liver instances, and measurement of TSl antibodies is reserved tor
chemistry studies and complete blood count w ith d iffe rential patients who are not markedly thyrotoxic on examination and
are recommended before initiation of antithyroid medica- do not have a classic smooth, rubbery, diffuse goiter. ln those
tions, with serial monitoring of the complete blood cou nt patients, TSI antibodies may help determine the cause of the
during therapy. If patients taking a th ionamide develop a hyperthyroidism. RAIU a nd scan w ill show markedly
tever. rash. severe sore throat, jaundice, or other symptoms of increased uptake with diffuse activity on t he scan.

43
Disorders of the Thyroid Gland

If ophthalmopathy is present, the patient may exhibit lid should be withheld for 5 to 7 days before the administration of
retraction (lid lag), whereby contraction of the levator palpe- radioactive iodine therapy. If a patienl has a particularly large
brae muscles of the eyelids results in immobility of the upper goiter with compressive symptoms or if there is concern lor
eyelid with downward rotation of the eye. Additionally, malignancy, surgery is recommended as flrst-line therapy.
patients may have proptosis, scleral injection, and periorbital
KEY POINT
edema.
Because thionamide drugs also have an immunomodula- • Radioactive iodine ablation or surgery is the most com-
tory effect that reduces au toantibody liters, antithyroid drugs mon treatment for toxic thyroid nodu les; indications fo r
are often first-line treatment for Graves disease. Up to SO% of surgery include a large goiter with compression symp-
patients may go into remission within 24 mon ths, and some toms or concern for malignancy.
may maintain a euthyroid state without further therapy atter an
Destructive Thyroiditis
initial treatment with thionamides. If the patient does not go
Thyroiditis is a self- limited inflammatory condition of the thyroid
into remission or if disease recurs, definitive therapy with radi-
resulting in the release of preformed thyroid hom1one into the
oactive iodine ablation or surgery is recommended. However, in
circulation. The duration of the thyrotoxic phase is typically 2 to
patients with Graves ophthalmopathy, there is an acute escala-
6 weeks, during which patients may exhibit classic symptoms of
tion of thyroid autoantibody t iters following radioiodine ther-
thyrotoxicosis. Following the release of preformed hormone, the
apy that may exacerbate ocular symptoms. Such patients may
damaged thyroid ceases production ofTJ and T4 during the recov-
be better treated with thionamides and/or surgery.
ery phase; consequently, administration of thionamides V\ri.ll not
be effective in treating elevated hormone levels. The patient may
Toxic Adenoma and Multinodular Goiter
then become clinically hypothyroid, a condition that may require
Activating mutations in the TSH receptor gene are responsible
temporary levothyroxine therapy. The length of the hypothyroid
for the autonomous production of thyroid hormone in a toxic
phase can vary but classically is 6 to 12 weeks.
nodule (adenoma) or in multiple hyperfunctioning nodules in
There are two categories of thyroiditis: painful and painless.
a toxic multinodular goiter. Because of this loss of normal
The causes of painful thyroiditis are inflammatory (de Quervain
regulation of thyroid hormone production, patients are at risk
or subacute granulomatous thyroiditis), infectious (suppurative),
for developing acute t hyrotoxicosis when exposed to iodine
and radiation-i.nduced. The pain, typically only present during
excess, particularly after a contrast load for medical testing
the thyrotoxic phase, can be quite intense. Treatment is aimed at
(Jod-Basedow phenomenon), such as in cardiac catheteriza-
controlling inflammatio·n with NSA!Ds or systemic glucocorti-
tion and contrast-enhanced CT scans. Although patients w ith
coids if severe. Subacute thyroiditis is the most c01mnon form
a toxic adenoma or multinodular goiter may exhibit the typi-
and is presumably caused by a postviral inflammatory process;
cal symptoms of thyrotoxicosis, they can be relatively asymp-
tomatic. On physical examination, a nodule npy be palpable many patients report a recent history of upper respiratory illness
or there may be a diffusely enlarged goiter w ith a nodular preceding the thyroiditis. Radiation thyroiditis may occur s to
10 days after treatment with radioactive iodine. This may be
contour but no discrete palpable nodules.
If a patient is suspected of having a toxic nodule, thyroid associated with transient exacerbation of the hyperthyroidism.
The accompanying pain is usually mild and lasts tor up to 1 week.
scintigraphy should be performed to determine if the nodule
is autonomous. The thyroid uptake scan will reveal increased Infectious thyroiditis is rare but may be seen in an immunocom-
promised patient; the most common causative organisms are
activity in the "hot" nodule with relative suppression of the
remaining thyroid tissue. These results should then be corre- Staphylococcus and Streptococcus species.
Painless thyroiditis is more commonly seen than painful
lated with the ultrasonographic tlndings to determine if any
thyroid itis and has several causes, including postpartum thy-
additional nodules exist, which will require further investiga-
roiditis, silent thyroiditis, and drug-induced thyroiditis.
tion with FNA.
' Postpartum thyroiditis may occur up to 1 year after delivery;
Radioactive iodine ablation or surgery is the most com-
t he frequency is variably reported but may occur in up to 10%
mon treatment f<>r toxic nodules. Thionamides can be used to
of pregnancies. Tl!e presence of TPO antibodies is nearly uni-
decrease hormone production in the short term, but unlike
versal, and the likelihood of subsequent permanent hypothy-
Graves disease, this condition has no chance of spontaneous
roidism is very high. Thyroiditis is also likely to recur in later
remission and would require lifelong medical therapy, which
pregnancies.
is not recommended. Radioiodine therapy will ideally ablate
only t he autonomous areas. In elderly patients, those with KEY POINT
coronary disease, those who are highly symptomatic, and • Thyroiditis is a self-limited inflammatory condition of
t hose with severe thyrotoxicosis, th ionamides are recom - the thyroid resulting in the release of preformed thyroid
mended to normalize thyroid hormone levels prior to radioac- hormone into the circulation; the duration of the thyro-
tive iodine; this is done to avoid exacerbation of the toxic phase is typically 2 to 6 weeks, which is followed
thyrotoxicosis due to release of preformed hormone from the by a hypothyroid phase typically lasting 6 to 12 weeks.
gland acutely after radioactive iodine ablation. Thionamides

44
Disorde rs of the Thyroid Gland

Central Hyperthyroidism trauma, pituitary surgery, or cranial radiation can cause cen-
TSH-secreting pituitary adenomas are extremely rare. In this tral hypothyroidism.
condition. serum TSH is detectable or normal in the setting of The clinical manifestat ions of hypothyroidism include
an elevated T., and/or T:l concen tration. A dedicated pituitary fatigue, cold intolerance, constipation, heavy menses, weight
MRI will demonstrate an adenoma. Treatment should tocus on gain , impaired concentration, dry skin, edema, depression,
removal of the pituitary tumor; thyroid-targeted therapy is mood changes, muscle cramps, myalgia, and reduced fe rtility
ineffective (see Disorders of t he Pituitary Gland). The physical examination fi ndings may include reduction in
basal temperature, diastolic hypertension, bradycardia, dry
Subclinical Hyperthyroidism skin, brittle hair, hoarseness, delayed relaxation phase of the
Su bclinical hype rthyroid ism is a laboratory-based diagnosis, deep tendon reflexes, and an enlarged thyroid.
defined as the presence of a suppressed TSH level with no rmal An elevated serum TSH level indicates the diagnosis of
T3 and levels. Repeat assessment of thyroid function should primary hypothyroidism. In patients with an elevated TSH
be performed 6 to 12 weeks after the initial tests, as the values that is less than 10 (10 mUlL), a low serum T4 measure-
will normalize in up to 30% of patients. Symptoms of thyro- ment is helpful, as a frankly low value indicates that thyroid
toxicosis are typically mild; most patients are asymptomatic. hormone replacement is necessary. The presence ofTPO anti-
Which patients will benefit most from normalization of bodies suggests that Hashimoto thyroiditis is the underlying
the TSH level is not universally agreed on. but consensus opin- cause. Thyroid imaging is not indicated unless there is concern
ion recommends treatment for patients with a TSH level below for a nodule on physical examination.
O.l llU/m L (0.1 mU/ L). The benefits oftreatment for asympto- KEY POINTS
matic patients w ith a TSH level between 0.1 )lU/mL (0.1 mU/L)
• An ele\·ated serum th) roid stimulating hormone le\ el HVC
and the lower limit of the normal reference range are less clear.
indicates the diagnosis of primar) h) pot h) roid ism:
Emerging data suggest that chronic subclinical hyperthyroid-
roid imaging is not ind icated unless there is concern for
ism has a negative effect on cardiac fu nction, the central nerv-
a nod ule on physical examination .
ous system, and bone mass. The risk of atrial fibrilla tion is
signitkantly increased when the TSH level is below 0.3 1-!U/mL • The most frequent cause of primary hypothyroidism is
(0.3 mU /L), so patients over the age of 65 years and those with Hashim oto thyroiditis (chronic lymphocytic thyroidi-
a history of coronary artery disease or tachyarrhythmias, as tis); the presence ofTPO antibodies is suggestive of
well as patients with osteoporosis, may benefit from normali- Hashimoto thyroid itis.
zation of the TSH level. Radioiodine is the preferred treatment,
but often the gland does not have sufficient iodine avidity and Management
methimazole must be used. In patients with a TSH level greater than 10 !J.U/mL (10 mUlL),
daily thyroid hormone replacement is recommended.
KEY POINTS
Thyroid hormone replacement with levothyroxine alone
HVC • In patients\\ ith subclinical hyperthyroidism. repeat is recommended. The goal of thyroid hormone replacement
assessment of th) roid function should be performed 6 therapy is normalization of the TSH. The starting dose can be
to 12 weeks after the initial tests. as the \alucs \\ill nor weight-based at 1.67 !lg/kg/d, using ideal body weight. In
mal ize in up to 30\, of patients. patients with prevalent cardiac disease, tachyarrhythmias, or
• Treatment for subclinical hyperthyroidism is recom- multiple comorbidities, or in those who are older t han
mended when the thyroid-stimula ting hormone level is 65 years, the dose should not be based on weight but rather
less than 0 .1 Jl.U/mL (0.1 mU/L) . shou ld be 25 to SO The dose should be titrated based on
TSH levels measured 6 to 8 weeks after a ny dose change. To
improve gastrointestinal absorption, levothyroxine should be
Thyroid Hormone Deficiency taken on an empty stomach, 1 hour be tore or 2 to 3 hours after
Hypothyroidism ingestion of tood or medications that would interfere with
Evaluation absorption, such as calcium- or iron-containing supplements.
Hypothyroidism refers to low circulating thyroid hormone Patients with celiac disease may require higher levothyroxine
levels. Hypothyroidism is more prevalent in women than men doses because of impaired absorption.
(2% versus 0.2%) and in those with other autoimmune dis- There has been significa nt debate regarding the need for
eases. The most f'req uent cause is Hashimoto thyroiditis, also supplementation of T3 (liothyronine) in patients with hypo-
known as chronic lymphocytic t hyroiditis. Iatrogenic causes thyroidism. The short half-life of Ta triggers acute spikes in
include surgery, radioiodine therapy, and external beam radia- serum T1 levels, which are of sign ificant concern for elderly
tion therapy to the neck. Hypothyroidism may also be medica- patients or those with preexisting cardiac issues. Additionally,
tion induced; the most common causative agents include numerous studies have failed to show a clear benefit of a T,/ T3
lithium, amiodarone, interferons, interleukin-2, and tyrosine combination over T4 alone; therefore, t his is not generally
kinase inhibitors. Ra rely, pituitary tumors, severe head recommended.

45
Disorders of the Thyroid Gland

KEY POINT effect). The resu lt is a temporary reduction in circulating T3


• In patients w ith a thyroid-stimulating hormone level and T4 levels with a minor rise in the TSH level; these cha nges
greater than 10 J.LUimL (10 mUlL), daily thyroid hor- typically reverse within the first 3 months of treatment and
require no intervention.
mone replacement is recommen ded and should be
Facultative effects, seen in up to 15"/o of patients, may
taken on an empty stomach ; the dose should be titrated
cause either hypo- or hyperthyroidism. In areas of iodine suf-
based on thyroid-stimulating hormone levels measures
ficiency, hypothyroidism is t he more common toxicity. Those
6 to 8 weeks after any dose change.
at highest risk are women with preexisting TPO antibody
positivity. Amiodarone-induced t hyrotoxicosis (AIT) is more
Subclinical Hypothyroidism commonly seen in males and in those living in iodine-defi-
Subclinical hypothyroidism is defined as an elevated serum cient areas. Type 1 AIT is t he result of exposure to excess iodine
TSH level with a normal T4 level. The potential causes are the and occurs in those with preexisting thyroid conditions, such
same as for overt hypothyroidism. Repeat measurement of as latent Graves disease or nodular goiter, in which the iodine
the TSH level is recommended, particularly in an asympto- increases unregulated thyroid hormone production. Type 2
matic patient, as it will normalize in up to 30% of patients by AIT is the result of the cytotoxic effects of amiodarone on thy-
6 weeks. roid tissue, producing a clinical picture of pai n less thyroiditis,
Patients typically have mild or no symptoms of hypothy- with abnormal release of t hyroid hormone. The treatments
roidism . Subclin ical hypothyroidism may be associated with diffe r with each type, but distinguishing between the two
several laboratory abnormalities including elevated total cho- forms of AIT often can be challenging and may require the aid
lesterol, LDL cholesterol, and C-reactive protein levels. Large of an endocrinologist. The time to recovery of normal thyroid
studies suggest that these laboratory abnormalities translate function may be several months, even with prompt diagnosis
into an increased risk of atherosclerosis and cardiac events. and treatment. Discontinuation of amiodarone is typically
However, supplementation with levothyroxine has not been necessary, particularly in those patients with type 1 AlT.
shown to mitigate this risk. Treatment is generally recom-
mended for those with a TSH level greater than 10 J.L U/mL KEY POINTS
(10 mUlL), but levothyroxine treatment should be considered • In the majority of patients. amiodarone causes a tempo-
in patients who have positive TPO antibodies or a large goiter. rary reduction in circulating triiodothyronine (T) and
as these patients are at risk for progression to overt hypothy- thyroxine (T) and levels with a minor rise in the thy-
roidism at rates of 3% to 8"/o per year. A goal TSH level less than roid-stimulating hormone that reverses within first
or equal to 2.5 11U/ mL (2.5 mUlL) is recommended for women 3 months of treatment and requires no intervention.
with subclinical hypothyroidism and positive TPO ant ibody • In 15% of patients, amiodarone may cause either
status who are planning to conceive. hypo- or hyperthyroidism; those at highest risk for
KEY POINT amiodarone-induced hypothyroidism are women with
preexisting thyroid peroxidase antibody positivity.
HVC • ln patients\\ ith suspected subclinical h) pothyroidism,
repeat measurement of the thyroid stimulating hor
mone level is recommended. as it \\ill normalize in up
to 30' " of patients b) 6 \\eeks. Thyroid Function and Disease
in Pregnancy
Drug-Induced Thyroid Dysfunction Significa nt changes in thyroid function occur during preg-
Various medications can affect thyroid function and are listed nancy; understanding the normal physiology during gestation
in Table 26 based on their mechanism of action. is critical for a correct interpretation of thyroid laboratory
Amiodarone may have a potentially toxic effect on the studies. Abnormali ties of thyroid function can have a dramatic
thyroid. The iodine content of amiodarone is 37"/o by weight. It effect on the health of the mother and the fetus. A diagra m of
is stored in fat, myocardium, liver, lung. and thyroid tissues, the physiologic changes in thyroid function during each tri-
with a half-life exceeding SO days. This long half-life. coupled mester is shown in Figure 8.
with the high iodine content, renders it a potentially toxic Increased estrogen levels cause a rise in thyroxine-bind-
compound to the thyroid. The two types of amiodarone thy- ing globulin. To maintain a stable free T4 and T3 , thyroid hor-
roid toxicity are changes in thyroid function studies seen in all mone production is increased and TSH remains within th e
patients (obligatory effects), and those seen in only a subset of normal reference range tor the patient's trimester (see later
patients (facultative effects). discussion for trimester-specific ranges). Routine screening of
The obligatory effects result from the increased circulat- TSH is not indicated for every pregnant woman. TSH screening
ing iodine after initiation of the drug. Adaptation to the acute is indicated in women w ith a risk of thyroid gland dysfunc-
iodine excess causes a reduction in organ itlcation of iodine tion, including those already on thyroid hormone replacement
and reduced production of thyroid hormone (Wolff-Chaikoff therapy; those with autoimmune disorders, goiter, previous

46
D i sorders of the Thyroid Gland

TABLE 26. MedicationsthatAffectThyroid Function


Mechanism of Action Drugs Comments

Decreased absorption or Calcium It is recommended that levothyro xine ingestion be separated from
e nterohepat ic ci rculat ion t hese medicat io ns by several hours
Proton pump inhibit ors
Iron
C holestyramine
Aluminum hydroxide
Soybean oil
Sucralfate
Psyllium
Increased metabolism of Phenytoin Hig her levot hyroxine doses m ay b e req uired to mainta in
levothyroxine levothyroxine in t he normal range
Carbamazep ine
Rifampin
Phenobarb ita l
Sertraline
Thyroid it is Amiodarone May cause hypo- or hyperthyroidism
Lithium
Inte rferon alfa
lnt e rleukin-2
Tyrosine kinase inhibit ors
De novo development of Interferon alfa May develop Hashimoto thyro iditis, G raves disease, or painless
antithyroid ant ibodies thyroiditis
Inh ibit ion ofTSH synthesis or Glucocorticoids
release
Dopam ine
Dobutamine
Octreot ide
Increased thyro xine-b inding Estrogen False e levation oftot al T 3, t otal T4 1evels; free T3, T4 may be more
g lob ulin accurate reflectio n of hormone levels .
Tamoxifen
Methadone
Decreased thyroxine-bi ndi ng Androgen therapy False lowering oftota l T3, tota l T4 1evels; free T3 , T 4 may be more
globulin accurate reflection of hormone leve ls
Glucocorticoids
Niacin
1 T3 = triiodothyronine; T 4 = thyroxine; TSH = thyroid-stimulating hormone.

head/neck irradiation, previous thyroid surgery, known posi-, pregnancy, with adjustments in thyroid hormone replacement
t ive TPO antibodies or positive TSI antibodies, or a strong fam- dosing as needed to maintain the TSH within the trimester-
ily history of thy roid dysfunction; those who live in specific normal range. The largest dose escalations typically
iodine-deficient areas; or those older t han 30 years. In patients occur in the first trimester, with more dose stability later in
on levothyroxine replacement, the dose of t he medication may pregnancy.
need to be increased, on average by 30% to SO%, and patients Diagnosing possible hyperthyroidism during pregnancy
should have their TSH level checked as soon as a pregnancy may be challenging because some physiologic changes during
test is positive. gestation may overlap w ith symptoms of thyrotoxicosis, such
Fetal thyroid tissue is not functional untillO to 12 weeks' as tachycardia, fatigue, and heat intolerance . Serum TSH and
gestation, necessitating maternal thyroid hormone transfer human chorionic gonadotropin have a common a -subun it,
through t he placenta. Thyroid hormone deficiency can nega- allowing cross- reactivity at the TSH receptor. Consequently,
tively affect fetal neurocognitive development. It is critical to TSH declines during the fi rst trimester; the reference range
maintain a euthyroid state during pregnancy in these patients. shifts down to 0.03 to 2.5 )l U!mL (0 .03- 2.5 mUlL). During the
TSH testing should be performed every 6 weeks throughout second and third trimesters t he upper limit of TSH rises to

47
Disorde rs of t he Thyroid Gland

perturbations oft he hypothalam ic pituitary- thyroid (llPT)


that occur in up to 75% of hospitalized patients. The underlying
cause of the critical illness may influence the pattern of thyroid
function abnormalities. Drugs that are frequently used in criti-
TBG cally ill patients can have a significa nt effect on the HPT axis (see
Table 26). The typical pattern is initially a low T 1 level. followed
by a decline in the T 1 level. As the patient becomes more criti
cally ill, the TSH level may also decline, creating a clinical pic-
ture tha t is difficult to discern from central hypothyroidism.
Rarely, TSH can be elevated in ESS.
Two general guidelines are important in evaluating a criti-
cally ill patient. First, measurement of TSH alone should be
obtained only if there is a high clinical suspicion of thyroid dys-
function. If TSH is abnormal, the previously described recom-
mendations for additional laboratory studies should be followed.
If the TSH is greater than 20 !-! u,m L (20 m U L) or is undetectable,
ESS is less likely to be the cause and overt thyroid dysfunction
should be strongly considered. If the TSH falls between U1csc two
\",!lues. historical clues and examination findings are very impor-
Week 10 20 30 40
tant for identifYing patients with true thyroid dysfunctioP.
FIGURE 8 . Thyroid function in pregnancy. HCG = human chorionic gonadotro· After discharge from the hospital, thyroid function abnor-
pin; T4 =thyroxine; TBG =thyroid-binding globulin; TSH = thyroid·stimulating malities may persist for several weeks. The typical pattern is a
hormone.
mildly elevated TSH level and slightly low T 1 and T3 levels. In a
clinically euthyroid patient, thyroid function tests should be
3.0 ).lU/ mL (3.0 m U/L). An addit ional complicating factor is repeated 6 weeks after hospitalization to conflrm oven thyroid
that radioiodine sca nning is contra indicated during preg- dysfunction with persistent TSH abnormality or confirm ESS
nancy because of the risk for fetal thyroid exposure to radia- \-\'ith normalization ofTSH. Cl
tion . Instead, several clin ical clues may be used to help
KEY POINTS
determine if the patient has thyrotoxicosis. including the pres-
ence of a goiter. ophthalmopathy, or TSI antibodies. all of • Critical illness can cause changes in thyroid functio n
which are suggestive of Graves disease. tests in up to 75% of hospitalized patients, known as
The use of thionamides is considered sa(e during preg- euthyroid sick syndrome; measurement ofTSH should
nancy, bu t PTU is preferred during t he first trimester because only be obtained in the hospital V.rhen there is a high
of potential teratogen ic effects from methimazole during clinical suspicion of thyroid dysfunction.
organogenesis. Although rarely indicated. surgery may be per- • The typical pattern of euthyroid sick syndrome, nonthy- HVC
formed during the second trimester. It should be reserved for roidal illness syndrome. or low triiodothyronine (T)
those who are unable to tolerate thionamides or who have syndrome is a mildly elevated thyroid stimulating hor-
inadequate control on medical therapy. Radioiodine therapy is mone level and slightly low thyroxine (T 1) and triiodo
contraindicated during pregnancy and while breastfeeding. thyronine (T) levels.
Treatment goals wilh lhionamides a re a detectable TSH in the • After patients with euthyroid sick syndrome are discharged HVC
lower end of the pregnancy reference range and a free T 1 in the from the hospital, thyroid function abnormalities may per-
upper end of the reference range. sist for several weeks so follow up thyroid function tests
KEY POINT should not be repeated until6 weeks after discharge.
• In pregnant patients on Jevothyroxine replacement. the
dose may need to be increased, on average by 30'Yo to 50%:
patients should have their thyroid-stimulating hormone Thyroid Emergencies C)
level checked as soon as a pregnancy test is positive. Although most thyroid conditions are not urgent, thyroid
storm and myxedema coma represent true medical emergen-
cies requiring critical care. Failure to make a timely diagnosis
C) Euthyroid Sick Syndrome and institute treatment is associated vvith a high mortality rate.
Euthyroid sickS) ndromc (ESS). nonthyroidal illness syndrome.
or low Tl syndrome are various names that have been assigned Thyroid Storm
to the changes seen in thyroid function test results during criti- Thyroid storm is a severe manifestation of thyrotoxicosis with
cal illness. Although not a true syndrome. there are significa nt lite threatening secondary systemic decompensation (shock).

48
Disord e rs of the Thyroid Gland

Cl The cardinal featu res tor diagnosis include elevated tempera-


ture. significant tachycardia. heart failure. gastrointestinal dys
bleeding, or metabolic derangements. Cold exposure appears
to be a risk factor. as this condition is more commonly seen in
CONT. function (nausea, vomiting. diarrhea, and/or jaundice), and the winter months.
neurologic disturbances. The range of central nervous system Mental status changes and hypothermia are the most
manifestations includes increasing agitation, emotional lability, . common clinical manifestations. The spect rum of' mental sta
confusion, paranoia, psychosis, or coma. Although thyroid tus changes includes lethargy, stupor, coma. depression. or
storm has been reported with many causes of thyrotoxicosis. it even psychosis. Hypothermia (temperature less than 34.4 oc
occurs most commonly with Graves disease. Thyroid storm [94.0 oF]) is present in nearly all patients; lower temperatures
may be precipitated by another event such as infection, surgery, are associa ted with a worse prognosis. Ventilatory drive is
myoca rdial infarction. trauma, or parturition. Admi nistration decreased, resu lting in hypoxemia and hypercapnia. Add itional
of rJdioactive iodine therapy to a p<ltient with untreated or signs include bradycardia, hypoglycemia. hyponatre mia, and /
uncontrolled hyperthyroidism can trigger thyroid storm. or hypotension. A significant percentage of patients experi-
The diagnosis is based on clinical presentation but can ence seizu res, which may he related to the coexisting meta-
generally be ruled out if T4 and T 3 levels are within normal bolic derangements.
limits. If myxedema coma is suspected , the serum TSH and T 1
Treatment of thyroid storm should be directed toward levels should be tested immediately. Diagnosis is made based
reduction of thyroid hormone production, decreasing peripheral on the clinical presentation and the coexisting metabolic
conversion of T 1 to T3 • addressing adrenergic symptoms and abnormalities. The serum cortisol level should be checked as
thermoregulatory changes, searchi ng for and treati ng precipitat- soon as possible to evaluate for concomitJnt adrenJI insufti
ing factors, and reversing systemic decompensation. ciency prior to initiation of thyroid hormone replacement.
Thionamides and are the mainstay of treatment to While awaiting the results of the serum cortisol measurement,
reduce thyroid hormone production and control adrenergic it is generally advisable to empirically initiate high-dose gluco-
symptoms. PTU and propranolol are the preferred Jgents corticoid therapy. Th is therapy may he d iscon tinued if the
because they have the added benefit of blocking peripheral con- serum cortisol level is found to be normal or high.
version of T1 to T3 . Additionally. high-dose glucocorticoids The treatment of myxedema coma is aimed at restoration
reduce T.1 conversion to bioactive T1. At least 1 hour after the first of the euthyroid state with thyroid hormone therapy, support-
dose of a tltionamidc, iodine drops should be administered to ive care (mechanical ventilation, vasopressors. and glucocorti
inhibit further release of thyroid hormone from the gland . coids), warmed in travenous fl uids, warm ing blankets, and
Acetam inophen·and cooling blankets may be used to control the management of the underlying precipitating event. The exact
hyperthermia. However, even with ag_gressive therapy and sup dose and preparation of thyroid hormone to admin ister is
portive measures, mortality rates are as high as 15'Yo to 20%. CJ controversial; minimal clinical trial information is available to
ascertain the optimal treatment regimen. It is importa nt to
KEY POINTS
balance the need for rapid reinstatement of a euthyroid state
• Thyroid storm is a severe manifestation of thyrotoxicosis with the risk of precipitating a fatal cardiac event due to
with life-threatening secondary systemic decompensation increased cardiac work with administration of thyroid hor-
(shock); it occurs most commonly with u nderlying mone. Generally. intravenous levothyroxine therapy is admin-
Graves disease coupled with a precipitating factor such as istered. initially as an intravenous bolus of 200 to 500 lJ.g,
infection, surgery, myocardial infarction, or parturition followed by daily doses of SO to 100 lJ.g intravenously unt il
and mortality is 15% to 20%. transition to an oral fo rm ulat ion is feasible. Treatment with T,
• In addition to supportive care and treating the partici- is not recom mended.
pating cause, thionamides and are the ma in- Even with aggressive therapy. the motiality rate for myx-
stay of treatment to reduce thyroid hormone production edema coma is 20% to 25%. CJ
and are often combined with iodine drops and high-
KEY POINTS
dose glucocorticoids to treat thyroid storm.
• Myxedema coma is an extreme but rare manifestation

Cl Myxedema coma is an extreme but rare manifestat ion of hypo-


Myxedema Coma of hypothyroidism, resulting in life- threa tening second-
ary systemic decompensation and a mortality rate of
thyroidism. resulting in life-threatening secondary systemic 20% to 25%.
decompensation. Without a frankly low T 1 level, myxedema • The treatment of myxedema coma is aimed at restoration
coma is unlikely. regardless of the degree of TSH elevation. It of the euthyroid state with thyroid hormone therapy,
has a very high mortality rate if there is a delay in treatment. supportive care (mechanical ventilation, vasopressors,
Myxedema coma is more common in elderly women; it may and glucocorticoids), warmed intravenous fluids,
occur in those with a history of hypothyroidism or no ante warming blankets, and management of t he underlying
cedent illness. Precipitating events are frequent. such as myo- precipitating event.
card ial infarction, infection, stroke, trauma, gastrointestinal

49
Disord ers of the Thyroid Gland

Structural Disorders a neck ultrasound that includes evaluation of the thyroid and
cervical lymph nodes.
of the Thyroid Gland Nodules are frequently detected incidentally on imaging
Thyroid Nodules studies performed for other reasons. The diagnostic evaluat ion
Nodularity of the thyroid is extremely common; large popula- of incidentally discovered thyroid nodules is identical to those
tion studies suggest that up to 5% of women and 1% of men that are clinically detected, w ith the same rate of malignancy.
have a clinically evident nodule. The prevalence increases with Nodules incidentally identified on fluorodeoxyglucose-PET
age. In autopsy series and screening ultrasound studies, nod- (FOG-PET) scan ning, however, have a malignancy rate of 30%
ules may be seen in up to 60% . to 50%. Consequently, FOG-avid nodules found on PET scans
The differential diagnosis for a nodule in the thyroid is require heightened suspicion and a lower threshold for inter-
varied and includes both primary thyroid disorders and meta- vention or diagnostic evaluation.
static spread from other primary malignancies (Table 27). Most A ca reful history should be performed in patients with a
thyroid nodules are benign, w ith only approximately lO'Yo thyroid nodule. Increased risk of malignancy is found in
harboring a malignancy. Ultrasonography is an inexpensive patients w ith h istory of radiation exposure to the head or
and highly effec tive method for stratification of mal ignancy neck, a family history of thyroid cancer, or a personal history
risk. All patients with a suspected thyroid nodule should have of thyroid cancer. Add itional factors that increase the risk for
malignancy in a nodule include male sex, extremes of age (<20
or >60 years), rapid nodule growth, and hoarseness. On phys-
TABLE 27 . Types of Thyroid Nodules ical examination, the nodule should be assessed for texture,
Benign Malignant mobility, and associated lymphadenopathy. If the nodule is
hard, fixed to surrounding tissue (nonmobile w ith swallow-
M ultinodular goiter Papillary thyroid cancer
(colloid ade noma)
ing), and/or there is associated cervical lymphadenopathy, the
risk of malignancy is greater. Pain is an uncommon fi nding
Hashimoto (chronic Follicu lar thyroid cancer
lymphocytic) thyroiditis w ith thyroid nodules, but when present it is usually associated
with benign conditions.
Co llo id cyst Medullary t hyroid cancer
A serum ISH measurement is the initial laboratory test in
Hemorrhagic cyst Anaplastic thyroid cancer
a patient w ith a t hyroid nodule. If the TSH is suppressed,
Foll icu lar ade noma Primary thyroid lymphoma measurement ofT1 and T3 should be performed, and a radio-
Hurthle cell adenoma Metastatic ca ncer nuclide scan should be considered (Figure 9). The objective of
Breast the scan is to identify "hot" or functioning nodules, which
Melanoma have a very low likelihood of malignancy and typically do not
require FNA. In contrast, if the TSH is high or normal, the ·
Renal cell
radio nuclide scan is unnecessary as it is unlikely to reveal a hot

Histo"ry, physical, TSH

Low TSH

Ultrasound Thyroid scan, FT4, FT 3

>1cm <1 em
Evaluate for US-guided FNA Repeat US in 6-24 months

Treat hyperthyroidism

FIGURE 9 . Initial evaluation of a thyroid nodule. There are size thresholds for FNA based on US appearance. Aless suspicious lesion may not need FNA until it is larger
than 2 em, suspicious nodu les if larger than 1 em. FNA = fine·needle aspiration; FT3 =free triiodothyronine; FT4 = free thyroxine; TSH = thyroid-stimulating hormone; US=
ultrasound.

50
Disorders of the Thyroid Gland

nodule; the evaluation should proceed with an ultrasound and Nodules that are benign by FNA should be followed with
possible FNA. As t hyroid nodular disease can be altered by repeat ultrasound examination in 6 to 18 months to assess fo r
normalization of the TSH, ultrasound and FNAshould be post- significant changes. If the nodule is stable on repeat imaging
poned in patients with elevated TSH until TSH is normal, and lacks suspicious features, clinical examination and repeat
un less there is marked concern for malignancy. One-time . ultrasound can be extended to longer intervals, such as 3 to
measurement of t hyroid antibodies may be appropriate if 5 years. Greater than SO% change in nodule volume or interval
autoimmune thyroid disease is suspected or if multinodular development of concerning ultrasound characteristics should
goiter is identified by u ltrasound to stratify the patient's future prompt a repeat FNA to evaluate for a false-negative initial
risk of developing overt t hyroid failure. Semm thyroglobulin biopsy.
measu rement is not useful and is not recommended. Malignant nodules and those that are suspicious for
FNA, perform ed under ultrasound guidance, is the opti- malignancy require prompt excision; this is typically done
mal test to determine whether a nodule is malignant. When with total thyroidectomy, but hem ithyroidectomy may be
performed by an experienced clinician, FNA is safe and rela- preferable for patients younger than 45 years of age with a
tively simple to perform. The sensitivity of FNA cytology is 90% tumor smaller than 4 em. A nond iagnostic FNA warrants a
to 95%, and the false-negative rate is 3% to 5%. FNA of a nodule repeat attempt. In a solid nodule with two unsatisfactory biop-
is generally recommended for those nodules larger than 1 em sies, diagnostic hemithyroidectomy is indicated. Surgical
that are solid and hypoechoic. The threshold for FNA of nod- complications include hypoparathyroidism and recurrent
ules that are partially cystic and lacking suspicious ultrasound laryngeal nerve paresis; although typically temporary, either
features is 2 em in size or greater. Aspirating a nodule of 5 mm complication may be permanent in up to 3% of patients.
or more may be considered if a patient has risk fac tors such as
KEY POINTS '
a personal or family history of thyroid cancer or prior radia-
tion exposure. • Thyroid nodules are fou nd in 1% to S'Yo of the popula-
The sonographic appearance of a nodule may be used to tion; most thyroid nodules are benign, with only
assess the risk of malignancy and thereby guide the decision of approximately 10% harboring a malignancy
which nodules require biopsy. Features concerning for malig- • A serum thyroid slimulaling hormone measurement is HVC
nancy include microcalcifications. marked hypoechogenicity, the initial laboratory test in a patient with a thyroid
irregula r borders, and taller-than-wide shape. Such findings nodule: if the thyroid stimulating hormone is sup-
are nearly 70% specific for cancer, but their poor sensitivity pressed. then measurement of thyroxine (T) and triio
cannot exclude t he presence of malignancy dothyronine (T) should be performed. and a radionu-
The various diagnoses obtained on FNA and the associ- clide scan should be considered to identif)r "hot" or
ated risks of malignancy are listed in Table 28. functioning nodules, which have a very low likelihood
of malignancy and typically do not require fine-needle·
aspiration.
TABLE 28. Diagnoses Obtained by Fine-Needle
• If the thyroid -stimulating hormone level is high or nor- HVC
Aspiration of Thyroid Nodules and Risk for Malignancy
mal, the radionuclide scan is unnecessary as it is
FNA Diagnosis Risk for Management
unlikely to reveal a hot nodule and ultrasonography is
Malignancy
an inexpensive and highly effective method for stratifi-
Benign <3% Serial u ltrasound cation ofmaligmmcy risk for nonfunctioni ng thyroid
examinations for nodules.
growth
• Fine-needle aspiration of a nodule is generally recom-
Atypia o f uncerta in 5%- 10% Repeat FNA
significance/fo llicular mended for those nodules larger t han 1 em (0.4 in) that
lesion of uncertain are solid and hypoechoic and is the optimal test to
significance
determine whether a nodule is malignant.
Suspicious for 20%-30% Hemithyroidectomy
foll icula r lesion
Goiters
Suspicious for 50%-75% Hem ithyroidectomy
ma lignancy or total thyroidectomy Multinodular Goiter
Ma lignant 97%-100% Total thyroidectomy Multinodular goiters occur more frequently w ith advancing
age, low iodine intake, or Hashimoto disease. The risk for
Nond iagnostic 0%-50% Repeat FNA; if two
nond iagnostic FNAs, malignancy is the same for multiple nodules as it is for a soli-
surgery tary nodule; therefore, the evaluation and management are
Mod ified from: C•bas ES, Ali SZ; NCI Thyro id FNA State of t he Sci ence Conference. identical. Biopsy should be performed on the three or four
The Bethesda System For Reporting Thyro id Cytopathology. Am J Clin Pathol. nodules (larger than 1 em) with the most suspicious ultra-
2009 Nov; 132(5):658·6 5. 1PMID: 198468051
sound features. In t he absence of suspicious features, the larg-
FNA = fi ne-needle aspira tion.
est nodules should be chosen fo r aspiration.

51
Disorders of the Thyroid Gland

A large multinodular goiter may be associated with com- the trend, there is evidence that larger tumors are increasingly
pressive symptoms such as dysphagia, hoarseness, or posi- being discovered.
tional dyspnea. To assess the extent of mass effect, add itional The vast majority of patients w ith thyroid cancer have
testing and imaging, including noncontrast cr of the neck/ well-differentiated thyroid ca ncer, with excellent long-term
chest, barium swallow, direct laryngoscopy, and/or spirometry survival. The major forms and their relative frequency are
with flow-volume loops, may be indicated. Levothyroxine listed in Figure 10. The most common well-differentiated thy-
therapy to suppress TSH secretion and reduce goiter size is roid cancers are papillary. papillary-follicular variant, and
generally not helpful, poses a risk of thyrotoxicosis, and is not follicular. There are rare. less well-differentiated variants of
recommended. Rad ioactive iodine ablation is not an option fo r papillary thyroid cancer (columnar, tall cell, insular, oxyphilic,
euthyroid and hypothyroid patients. Surgical removal is t he clear cell, diffuse sclerosing) that are more aggressive and
treatment of choice if the compressive sym ptoms are signifi- carry a worse prognosis. Anaplastic thyroid cancer is undif-
cant, if maligna ncy is suspected, or if the patient desires cos- feren tiated and is the most aggressive form of thyroid cancer;
metic intervention. 1-year survival rates range from 20% to 30%.
Staging and prognosis of well-differentiated thyroid can-
KEY POINTS
cers (papillary and follicular) are based on the American Joint
• In patien ts w ith a multinodular goiter, the risk for Com mittee on Cancer criteria, wh ich include age (<45 or
malignancy is the same for multiple nodules as it is for years), primary tumor size. local and distant metastases, and
a solita ry nodule; therefore, the evaluation and manage- capsular and lymphovascular invasion. However. because the
ment are identical. majority of patients have excellent survival. T (tumor) N (node)
• Surgical removal of a large multinodular goiter is the M (metastasis) staging plays a minimal role in the management
treatment of choice if the compressive symptoms are of thyroid cancers. Instead, decisions regarding treatment are
sign ificant, if malignancy is suspected, or if the patient aimed at lowering the likelihood of recurrent disease.
desires cosmetic intervention. Treatment of well-differentiated thyroid cancer includes
a combination of su rgery, radioactive iodine, and levothyrox-
Simple Goiter ine suppression. The extent of surgery is largely based on the
A simple goiter is defined as an enla rgement of t he thyroid tumo r size; solitary tumors smaller than 1 em may be suffi-
gland without the presence of nodules. It may be seen in cien tly managed with lobectomy alone. Patients with larger
condit ions of dyshormonogenesis. autoimmune thyroid dis- tumors, multifocal disease, nodal metastases, or a history of
ease, or p ri mary thyroid lymphoma. Primary thyroid lym- irradiation are best treated with total or near- total thyroidec-
phoma is a rare condition that typically occurs in elderly tomy. For patients younger than 45 years of age with a tumor
women with a history of Hashimoto thyroiditis. The clinical smaller than 4 em and no evidence of nodal or distant metas-
presentation is a symptomatic, rapidly enlarging goiter w ith tases, hemithyroidectomy may be a reasonable alternative to ·
a very firm texture. Patients may also have systemic lym- total thyroidectomy. The decision to administer radioactive
phoma symptoms and lateral cervical lymphadenopathy. The iodine is based on two factors: improvement in mortality rates
diagnosis ca n be made by FNA. Treatment typically involves and/or reduction in recurrence risk. Patients with distant
chemotherapy and/or radiation therapy. Surgery generally is metastases have improved survival with successful radioiodine
not indicated, but it can be used to aid in diagnosis when
FNA is not informative.

Thyroid Cancer
The incidence of thyroid cancer is rising at a faster rate than
any other type of malignancy; the incidence has more than
doubl ed in the last 30 years. This increase is due solely to pap- • Papillary
illary cancers, with the highest rate of rise occurring in tumors
• Follicular
measuring less than 2 em. Meanwhile, the survival rate for
Medullary
thyroid cancer has remained stable or slightly improved.
• Anaplastic
Autopsy series reveal that occult thyroid cancers measuring
less than 1 em may be identified in as many as 20'Yo of dissected Other (thyroid lymphoma
and metastases from ot her
specimens. This fi nding, coupled with the improving survival cancers)
rate, has led some investigators to conclude that t he change in
incidence of thyroid cancer is due solely to increased inciden-
tal detection of indolent tu mors because of greater use of FIGURE 1 0 . Relative frequency of the types of thyroid cancer.
imaging modalities. Although there is little doubt that esca-
Data from Hun dahl SA, Fleming ID, Fremgen AM, Menck HR. ANational C.n<er Data Baso report on 53,856 cases
lated detection of othervvise occult tumors has contributed to of thyroid carcinoma treated in the U.S., 1985·1995. Cancer. 1998 Dec 15;83(1 2):2638-48.1PMID: 9874472)

52
Reproductive Disorders

therapy, whereas administra tion of radioactive iodine may


decrease the likelihood of recurrent disease in those patients
with nodal metastases. Suppression ofTSI-I w ith levothyroxine
therapy may also be used to improve morbidity and reduce
mortality, particularly in patients with persistent disease or
distant metastases. The necessary degree of TSH suppression
varies according to the risk of cancer progression and comor-
bidities of the patient Patients w ith persistent disease typi-
cally require lowering of their TSI-I level to less tha n 0.1 11U/mL
(0.1 mUlL), whereas those who are free of disease but have a
high risk fo r recurrence should have a target TS!-I level of0.1 to
0.5 !-!Uim L (0 .1-0.5 mUlL) for 5 to 10 years. Those patients
who are disease-free with a low risk of recurrence should FIGURE 11 . Female reproductive axis. Pulses of GnRH drive LH and FSH pro-
maintain a TSI-I level of0 .3 to 2.0 11U/mL (0.3-2.0 m UlL) . duction. LH acts on theca cells to stimulate androgen (principally androstenedione)
prod uction .Androstened ione is metabolized to estradiol in granulosa cells. FSH
Medullary thyroid cancer rep resents less than 10% of all
acts on granu losa cells to enhance follicle matu ration. Granulosa cells prod uce
thyroid cancers. Approximately 2S'Yo of medullary thyroid can- inh ibin Bas a feed back regulator of FSH production. FSH ; foll icle-stimulating
cers are hereditary; all patients w ith medullary thyroid cancer hormone; GnRH ; gonadotropin-releasi ng hormone; LH ; luteinizing hormone;
should be screened with RET proto-oncogene sequencing. -(circled); negative feedback.
Medullary t hyroid cancer may be associated with several syn-
dromes. including multiple endocrine neoplasia type 2A
FSH, under control of pulsatile GnRH secretion, rises in the
(MEN2A) (wh ich may include pheochromocytoma and hyper-
early menstrual cycle to promote recruitment and growth of a
parathyroidism), MEN2B (marfanoid habitus and mucosal
follicle containing a microscopic oocyte (follicular phase).
ganglioneuromas), or fa milial medullary thyroid cancer (med-
Granulosa cells lining the follicle secrete estradiol, which contrib-
ullary thyroid cancer alone). Biochemical screening tor pheo-
utes to negative feedback inhibition of FSI-I secretion and result-
chromocytoma with measurement of plasma fraclionaled
ant monofollicular development in the majority of women.
metanephrine levels should be done in all patients w ith an
Estradiol also stimulates endometrial proliferation. Further into
RET mutation prior to thyroidectomy.
the follicular phase, estradiol levels peak and exert acute positive
KEY POINTS feedback on the pituitary gland, which elicits an LH surge. This
• The vast majority of patients with well-differentiated LH surge results in ovulation and initiates the luteal phase of the
thyroid cancer have excellent long-term survivaL menstrual cycle. LH stimulates androgen production by the theca
• Treatment of well-differentiated thyrrud cancer includes cells, which also li ne the follicle; androgen is subsequently aro-
a combination of surgery, radioactive iodine, and leva- matized to estrogen in the granulosa cells via aromatase enzyme.
thyroxine suppression of thyroid-stimulating hormone After the LH surge and ovulation, the follicle develops into the
for patients with persistent disease or high risk of corpus luteum, which secretes both estradiol and progesterone
recurrence. and causes the secretory phase of the endometrium in prepara-
tion for implantation of a fertilized oocyte. With implantation, the
early embryo produces human chorionic gonadotropin, which
maintains the corpus luteum. However, when a fertilized embryo
Reproductive Disorders is not present, progesterone levels decline, leading to the men-
strual phase of the endometri um and menstrual bleeding.
Physiology of Female An average menstrual cycle ranges from 25 to 35 days in
Reproduction length . The follicular phase may vary in each woman but is
A regular, predictable menstrual cycle requ ires coordination typically from 14 to 21 days. Variability in a menstrual cycle is
of inhibition and stimulation betvveen the hypothalamus ty pically the result of a shortened or lengthened follicular phase,
(secre ting gonadotropin- releasing hormone [GnRH)), the more commonly seen duri ng the first 5 years of menstruation.
pituitary (secreting follicle-stimulating hormone [FSH] and A decrease in follicular phase length occurs conunonly in peri-
luteinizing hormone [LI-1]), and the ovaries (secreting estra- menopause. The luteal phase is usually 14 days and is constant.
diol and progesterone) . The coordination of these signals is In women younger than 40 years of age, menstrual cycles less
referred to as the hypothalamic-pituitary-ovarian axis. The than 25 days or greater than 35 days are likely anovulatory.
GnRH pulse frequency varies throughout the menstrual KEY POINT
cycle to promote fo llicular development and ovulati on
• In women younger than 40 years of age, menstrual
(Figure 11). The phases of the menstrual cycle are referred to
cycles less than 25 days or greater than 35 days are li kely
in reference to the activity of the ovary (follicular and luteal
anovulatory
phases).

53
Reproductive Disorders

Amenorrhea small fu nctional pockets of active endometrium with out-


flow obstruction by synechia closer to the cervix, resulting
Clinical Features
in cyclic pain and hematometrium identifiable on ultra-
Primary Amenorrhea sound.
Primary amenorrhea is the absence of menses by age 16 years . After structural causes and pregna ncy a re excluded , the
accompanied by normal sexual hair pattern and normal breast hormonal status should be assessed. Low estradiol and inap-
development. Primary amenorrhea with absence of thelarche propriately normal FSH and LH levels indicate hypogonado-
(breast development at the beginning of puberty) and/or tropic hypogonadism and point to a central cause
adrenarche (androgen production increase that typically (hypothalamic-pituitary). Low estradiol in the setting of
occurs at age 8 or 9 years) prior to 14 years of age should be elevated FSH and LH levels indicates hypergonadotropic
evaluated. Pregnancy must be ruled out in any patient with hypogonadism and points to ovarian insufficiency.
primary amenorrhea before additional evaluation occurs. The most common hormonal cause of secondary amen-
Causes of primary amenorrhea may be genetic, hormonal, or orrhea is polycystic ovary syndrome (PCOS) (see Hirsutism
structural. Fifty percent of patients with primary amenorrhea and Polycystic Ovary Syndrome) , which accounts for 40"/o of
have a chromosomal abnormality, such as Turner syndrome cases. Additional hormonal causes of secondary amenorrhea
(4S,XO) (gonadal dysgenesis), although some patients with include hypothalamic amenorrhea (hypogonadotropic hypog-
Turner mosaicism may have secondary amenorrhea (see onadism), hyperprolactinemia, thyroid disease, and prema-
Secondary Amenorrhea). ture ovarian insufficiency (POI) (hypergonadotropic
Turner syndrome is commonly characterized by other hypogonadism).
clinical manifestations such as short stature, neck webbing, Hypogonadotropic hypogonadism caused by hypotha-
recurrent otitis media with hearing loss, aortic coarctation, lamic amenorrhea (HA) or functional hypothalamic amenor-
and bicuspid aortic valve. The diagnosis of Turner syndrome rhea (FHA) affects 3% of women between the ages of 18 and
may be made w ith a karyotype. 40 years. Risk factors include low BMI and low body fat per-
Approximately lS% of patients presenting w ith primary centage, rapid and substantial weight loss or weight gain,
amenorrhea may have an anatomic abnormality of the uterus, eating disorders, excessive exercise, severe emotional stress,
cervix, or vagina such as mollerian agenesis, transverse vaginal or acute and chronic illness. FS H and LH levels are inappro-
septum, or imperforate hymen. Digital vaginal examination, priately low in HA but may be inappropriately normal in
transvaginal ultrasound, or MRI may help to ident ify outflow FHA. Estradiol levels are typically low, and patients may
tract anomalies. experience vasomotor symptoms and sleep disturbance. If
KEY POINT left untreated, patients are at increased risk tor osteoporosis
• Fifty percent of patients with primary amenorrhea have owing to this low-estrogen state. Recovery of menses may
a chromosomal abnormality, such as Turner syndrome occur if BMI returns to normal. Cognitive-behavioral therapy ·
(4S,XO) (gonadal dysgenesis); IS% of patients presenting fo r cases caused by emotional stress has been shown to be
w ith primary amenorrhea may have an anatomic effective.
abnormality of the uterus, cervix, or vagina. Hyperprolactinemia causes secondary amenorrhea
through direct inhibition of GnRH secretion. Treatment of the
cause of hyperprolactinemia typically results in restoration of
Secondary Amenorrhea menses. Hypothyroidism may cause secondary amenorrhea
Secondary amenorrhea is the absence of a menstrual cycle through increased thyrotropin -releasing hormon e levels,
fo r three cycles or 6 months in previously menstruat ing which causes stimulation of prolactin secretion.
women. The most common cause of seco ndary amenorrhea Hypergonadotropic hypogonadism as a resu lt of POl is
is p regnancy. A potential structural cause of second ary defined as amenorrhea before age 40 years in the setting of
amenorrhea, such as Asherman syndrome, should be con- two elevated FSH levels (>40 mU!mL [40 U!L]) more than
sidered. Asherman syndrome is an uncommon complica- 1 month apart. Possible causes include Turner mosaicism (in
tion of dilat ion and curettage. intrauterine device place- which secondary amenorrhea may occur due to POI), fragile X
ment, or surgical procedures such as hysteroscopic premutation, chemotherapy or radiation, and autoimmune
myomectomy; it is caused by lack of basal endometrium oophoritis. In patients in whom an autoimmune cause is diag-
proliferation and formation of adhesions (synechiae). nosed, evaluation of other endocrine organs (thyroid, parathy-
Diagnosis should be considered in any woman w ith amen - roid, pancreas, and adrenal) is recommended at the time of
orrhea and past exposure to uterine instrumentation. The diagnosis and annually thereafter.
classic presentation is amenorrhea or scant bleeding during Estrogen replacement is necessary in pat ients with hyper-
periods (hypomenorrhea) with ovulatory symptoms (cervi- gonadotropic hypogonadism to prevent bone mass loss until
cal mucous changes, adnexal tenderness associated wit h the average age of natural menopause (S0-51 years). Estrogen
follicle formation ) or premenstrual symptoms (mood replacement preparations are available in oral, transdermal,
cha nges or breast tenderness). Some patients will maintain subcutaneous, and vaginal routes of administration. The dose

54
Rep roductiv e Di sord e rs

of estrogen requ ired by young women is tit rated to prevent KEY POINTS
vasomotor symptoms and vaginal dryness and may be h igher
• After excluding pregnancy, the laboratory evaluation of
than that used in an older age group. Because spontaneous
primary and secondary amenorrhea includes measure-
ovulation may occur (although it is infrequent ), counseling on
ments of prolactin, follicle-stimulating hormone, lute-
contracept ive options should also be provided for sexually
in izing hormone, estradiol, and thyroid-stimulati ng
active women not desiring pregnancy. Cyclic progesterone
hormone.
exposure should be considered in patients with an intact
uterus to prevent excessive unopposed endometrial prolifera- • If hormonal evaluation for amenorrhea is negative, the
tion . Oocyte donation may be considered for fe rtility options next step is a progesterone challenge test; if the patient
for this patient population. bleeds within 1 week of completing 7 to 10 days of pro-
gesterone, estrogen deficiency is not the cause and
KEY POINT PCOS should be considered
• The most common causes of secondary amenorrhea are
pregnancy, stmctural abn ormalities, and polycystic
ovary syndrome. Hyperandrogenism Syndromes
Hirsutism and Polycystic Ovary Syndrome
Evaluation of Amenorrhea When hirsutism is presen t, the patient should be assessed for
A thorough history and physical examination, including a virilization, or development of male characteristics. Rapid
pelvic examination, are needed to evaluate both prima ry onset and progression of deepening of the voice, severe acne,
(no h istory of menstruation) and secondary amenorrhea clitoromegaly, and male pattern balding are signs of viriliza-
(cessation of menstruation a fter menarche) . Urine or serum tion and are concerning for an ovarian or adrenal tumor. Age
human chorionic gonadotropin (HCG) testing should be of onset after 30 years is also a risk factor for an androgen-
clone first to exclude p regnancy, as this is t he most common secreting tumor.
cause of amenorrhea. In patients with primary amenor- In patients with hirsutism or virilization, recommended
rhea, a karyotype is recommended if a pregnancy test is initial laboratory tests include measurement of plasma dehy-
negative. Serum leve ls of prolact in , FSH, LH, estradiol, and droepiandrosterone sulfate (DHEAS) level and serum levels of
thyroid -stimulating hormone (TSH) should then be TSH, prolactin , total testosterone, and follicular- phase
obtained in the evaluation of primary and seconda ry amen- 17-hydroxyprogesterone. Normal levels exclude adrenal tumors,
orrhea. Abnormal levels of prolactin and/or TSH support a hypothyroidism, hyperprolactinemia, and ovarian tumor.
nonovarian cause of amenorrhea . Elevations of FSH and LH Common fo rms of late-onset congenital adrenal hyp erplasia,
levels in the presence of a low estradiol level support the often mistaken for PCOS, can be excluded with a normal
d iagnosis of POT. 17-hydroxyprogesterone level. Pelvic ultrasound and adrenal CT
If no elevations in these hormones are found, a progester- should be performed to exclude an ovarian or adrenal neoplasm
one challenge test (oral meclroxyprogesterone acetate, 10 mg if the semm total testosterone level is greater than 200 ng/clL
fo r 7-10 days) may be used to determine if the amenorrhea is (6.9 nmol!L) . Adrenal CT is necessary to exclude an adrenal
due to estrogen deficiency If t he patient has menstrual l;lleed- cortisol-secreting and/or androgen-secreting neoplasm if the
ing within 1 week of completing 7 to 10 days of plasma DHEAS level is greater than 700 !JglmL (18.9 !Jmol/L) .
gesterone, estrogen deficiency is not the cause. In this case Hirsutism is typically a benign condition, most commonly from
PCOS (or a si milar diagnosis) should be considered. If no PCOS. A marked elevation of total testosterone or DHEAS is not
menstrual bleeding occurs, the patient has a low-estrogen compatible with a diagnosis of PCOS.
state, and hypogonadotropic hypogonadism is the diagnosis PCOS has a prevalence of7% to 10% and is one of the most
(see Disorders of the Pituitary Gland). common endocrine disorders in young women. Two sets of
Pelvic ultrasound is helpfu l to identify structural diagnostic criteria are commonly used. The 2003 American
causes of amenorrhea such as mullerian agenesis and Society for Reproductive Medicine and the European Society of
intrau terine synechiae. Saline-infusion sonohysterogram Human Reproduction criteria for PCOS require tvvo of the fol-
can ident ify int rauterine synechiae, and transvaginal or lowing three findi ngs in the absence of other endocrine disor-
t ransabdominal ultrasound can identify absence of a uterus ders: (1) oligo-ovulation or anovulation, (2) clin ical or
in p atients with mulleria n agenesis. Endocrinology consul- biochemical evidence of hyperandrogenism (such as hirsutism
tation for further evaluation and testing of patients with or acne), or (3) ultrasound findings of polycystic ovarian mor-
findings suspicious for a genetic cause of am enorrhea may phology in at least one ovary. The 1990 criteria from the Nat ional
be appropriate. A pituita ry MRT may be indicated to exclude Institutes of Health and the National Institute of Child Health
other int racran ial causes of hypogonadotrop ic hypog- and Human Development require all of the following for diag-
onadism wh en diagnosing HA or FHA, and consultation nosis of PCOS: oligo-ovulation, signs of androgen excess (clini-
w ith an endocr inologist is recommended before imaging is cal or biochemical), exclusion of other disorders that can result
pursued. in menstmal irregulari(y, and hyperandrogenism.

55
Reproductive Disorders

A consta nt stagnant follicular stage is seen in PCOS, KEY P0 I NTS {continued)


resulting in unopposed estradiol secretion from small ovarian
• Polycystic ovary syndrome ca n be diagnosed if two of
follicles. Owing to disordered secretion of LH by the anterior
the following three findings are present: (1) oligo-ovula-
pituitary, intraovarian androgen production is also increased
tion or a novulation. (2) clinical or biochemical evidence
in PCOS, resul ting in the hypera ndrogenism associated with
of hyperandrogen ism (such as hirsutism or acne), or (3)
the disorder. Women wi th PCOS typically have eleva ted rest-
ult rasound fi ndings of polycystic ovarian morphology in
ing LH levels. In patients trying to conceive, this can lead to
at least one ovary.
false-positive indication of the ovulatory LH surge on home
urina ry LH kits for ovulation. • The 1990 criteria from the National Institutes of Health
Estradiol secretion results in prol iferation of the endome- and the Nationa l lnsti tu te of Chi ld Health and Human
trium in the absence of progesterone secretion from a corpus Development require all of the following for diagnosis
luteum. This predisposes patients to endometrial hyperplasia of PCOS: oligo-ovulation . signs of androgen excess
and heavy menstrual bleeding as a result of anovulatory bleed- (clinical or biochemical), exclusion of other disorders
ing. Oligo-ovulation and anovulation result in infertility but that can result in menstrual irregu larity, a nd hyperan -
are typically correctable with clomiphene citrate or letrozole drogcnism.
for ovulation induction if fertility is desired. If fertility is not
desired, oral contraceptives should be considered if not con- Androgen Abuse in Women
tra indicated. 1\ddilion of oral contraceptives will increase Anabolic steroids may be abused by some women to enha nce
secretion of sex hormone- binding globu lin (SHBG) and their athletic performance or physique. Such exogenous
decrease circulating levels of free testosterone. If a patient has a administration may result in absence ofGnRH pulsatil!ty and
contraindication to oral contraceptives, a progestin-secreting resul tant hypogonadotropic hypogonadism and amenorrhea .
intrauterine device or cyclic oral or vaginal progesterone should Adverse effects may include hirsutism, acne. deepening of the
be given to prevent prolonged unopposed estrogen exposure. voice. decreased breast size, and clitoromegaly. Withdrawal of
Hyp era ndrogenism may present as hirsutism. acne, or exogenous androgens does not result in severe hyp ogonadism
androgenic alo pecia. In patients with hirsu tism desiring as it does in men, and most women return to regular men-
trea tment. existing terminal hairs will need to be removed strual cycles.
with depilatory methods, but the rate of hair growth while on
treatment will Spironolactone, an aldosterone and
androgen inhibitor, may be added after 6 months if acne and
Female Infertility
hirsu tism are still cosmetically bothersome. Before initiation Infertility is defined as the absence of conception after 1 year
of therapy, the patient should be counseled about teratogenic of unprotected intercourse (on average twice weekly) in a
effects on a male fetus, and contraceptive cotfnseling sh ou ld woman younger than 35 years of age. Investigation should ·
be provided . begin after 6 months if no conception has occurred in a
Both obese and lean women with PCOS also have insulin woman 35 years of age or older. Infertility evaluation should
resistance, and studies have identified an increased incidence include a careful medical history of both partners w ith special
of metabolic syndrome, obesity, impaired glucose tolerance, foc us on menstrual history, previous exposure to sexually
and type 2 diabetes melli tus in these women. Although insulin transmitted infections. pelvic surgery, and previous obstetric
resista nce may improve with weigh t Joss, the use of insulin- complications such as miscarriage or cesarean delivery. If a
sensitizing agents such as metformin is associated wilh a report of oligomenorrhea is elicited, measurement of serum
decrease in serum androgens; however, it is not very effective TSH and prolactin levels is appropriate to exclude thyroid dis-
as a single agent for ovulation induction. ease and hyperprolactinemia as causes of oligo-ovulation.
Evaluation of patients with PCOS should include assess- Further evaluation of infertility causes typically includes
ment for signs of sleep apnea. hypercholesterolemia. and fatty semen analysis of the male partner. confirmation of ovulation
liver. In w'omen with a t hickened endometrium or menome- w ith measurement of mid luteal progesterone level (>3 ng/mL
trorrhagia, endometrial sampling with endometrial biopsy [9.5 nmol!L]), and because Fallopian tubes may be obstmcted
should be considered to evaluate for endometrial hyperplasia. due to infection such as pelvic inflammatory disease. tubal
We ight loss and exercise should be emphasized. patency evaluation wit h hysterosalpingogram. A common
cause of tubal occlusion and resultant infertili ty is past pelvic
KEY POINTS
inflammatory disease. Laparoscopy for evaluation of pelvic
• Polycystic ovary syndrome has a prevalence of7% to adhesions or mild endometriosis may be warranted in patients
10% and is one of the most common endocrine elisor- with dysmenorrhea, previous exposure to sexually t ransmit-
ders in young women; it is often associated with insulin ted infections, or previous pelvic surgery.
resistance, metabolic syndrome, obesity and type 2 dia- If no abnormalities are found, treatment to enha nce
betes mellitus. endogenous gonadotropin release and increase the numbers
(Cont inued)
of oocytes ovulated monthly may be warranted. Some studies

56
Reproductive Disorders

suppor t moving directly to in vitro fertilization treatment tor Leydig cell production of testosterone occurs in a diurnal
women with infertility at age 40 years. In women treated pattern, with the h ighest concentration observed in the morn-
with ovarian stimulation. oral medications such as clomi- ing. A large percentage of ci rculating testosterone is bound
phene citrate or letrozole are typically used. This therapy is either to SHBG or albumin. A serum total testosterone level
not appropriate in patients with POI. Pat ients should be measured in the ea rly morning is general ly considered to be an
counseled about the 5% to 8'Yo risk of multiple gestation with accurate measurement of a patient's androgen status, but it
these thera pies. Referral to a reproductive endocrinologist is does not account for decreased SHBG as seen in obesity (see
recommended. Evaluation of Male Hypogonadism).
KEY POINT
• Infertility evaluation in women should include a med-
ical history of both partners with special focus on Hypogonadism
menstrual history, previous exposure to sexually Primary Hypogonadism
t ransmitted infections. pelvic surgery, and previous Primary hypogonadism. or testicular failu re, represents a
obstetric complications such as miscarriage or cesar- decrease in testosterone or sperm production. Primary hypo-
ean delivery. gonadism is uncommon and may have congenital or acquired
causes. Klinefelter syndrome is the most common cause of
congenital primary hypogonadism. Klinefelter syndrome is a
Physiology of Male Reproduction common cause of hypergonadotropic hypogonadism and azo-
Control of spermatogenesis a nd testosterone production ospermia. resulting in infe rtility. A 47,XXY karyotype is diag-
depends on the pulsatile secretion of GnRH from the hypo- nostic of Klinefelter syndrome. Mosaic variants of this condi-
thalamus as well as subsequent downstream stimulation of tion exist but typically present with oligoasthenospermia,
the a nterior pituita ry and male gonads. ln the testicle, FSH testicular failure or hypogonadism. Concomitant symptoms
stimulates Sertoli cell spermatogenesis, and LH stimulates often include sexua l dysfunction and generalized fa tigue. Tall
Leydig cell testosterone production. Negative feedback from stature is a common finding. Patients with Klinefelter syn-
testosterone production inhibits FSH and LH secretion at the drome may fail to achieve puberty or may present after sexual
level of the anterior pi tuitary as well as pulsatile hypothalamic maturation with azoospermia. Exposure to chemotherapy or
GnRH secretion. Inhibin B, produced by the Sertoli cells. also radiation may als9 result in primary testicular failure. Local
inhibits FSH (F-igure 12). injury as a result of torsion, orchitis, or trauma may result in
ischemia and necrosis of testicular tissues.

e Secondary Hypogonadism
Typically secondary hypogonadism is a result of insufficient
GnRH production by the hypothalamus or deficient LI-I/FSI-I
secretion by the anterior pituitary. Causes may be congenital
e or acquired. Idiopathic hypogonadoLropic hypogonadism,
with anosmia (Kallmann syndrome) or without anosmia, is
the most common cause of congenital secondary hypog-
onadism. Acquired secondary hypogonadism is most com-
monly iatrogenic due to exogenous testosterone administra-
tion. Untreated sleep apnea and obesity are other common
causes. Other acquired causes include hyperprolactinemia,
Spermatogenesis chronic opioid use, glucocorticoid use, or inflltrative disease
(lymphoma or hemochromatosis) .

Androgen Deficiency in the Aging Male


The natural progression of male aging involves testosterone
level decline. Most men will not become hypogonadal, and the
FIGURE 1 2 . Male reproductive axis. Pulses of GnRH elicit pulses of LH and decline in testosterone production is highly variable with each
FSH. FSH acts on Sertoli cells, which assist sperm maturation and produce inhibin person. ··Low r · has become a part of the popular vernacular,
B, the major negative regulator of basal FSH production. The Leydig cells produce owing to aggressive direct-to-consumer marketing, and
testosterone, which feeds back to inhibit GnRH and LH release. Some testosterone describes many symptoms that may or may not be associated
is irreversibly converted to dihydrotestosterone or estradiol, which are both more
potent than testosterone in suppressing GnRH and LH . FSH =follicle-stimulating
with a low serum testosterone level in men. Both prescription
hormone; GnRH = gonadotropin-releasing hormone; LH = luteinizing hormone; and over-the-cou nter testosterone and derivative sales have
- (circled) = negative feedback. surged in the United States an d many other countries. With

57
Reproductive Disorders

the increased sales of testosterone formulations to treat agi ng hypogonadotropic state is revealed, transferrin saturation and
men, questions have emerged about the potential adve rse ferritin levels should be evaluated to exclude hemochromato-
effects of exogenous testosterone therapy. particularly in men sis. MRI of the pituitary should be performed to evaluate for
who have no biochemical evidence of testosterone deficiency. hypothalamic or pituitary masses as the cause of the hypogon-
Potential adverse effec ts include increased risk of cardiovascu- adotropic state if no confounding medications or reve rsible
lar disease and death, venous thromboembolism, and prostate secondary causes are discovered. Figure 13 shows an algori thm
cancer. tor evaluating male hypogonadism.
KEY POINTS

Evaluation of Male Hypogonadism • Measurement of testosterone levels is not recommended HVC


A thorough history and physical examinalion are essential in if a patient is having regular mom ing erections, does
the evaluation of hypogonadism. A sleep history is especially not have true gynecomastia on examination, and has a
helpful. A large constellation of nonspecific symptoms is asso- normal testicular examination, as it is highly unli kely
ciated with male hypogonadism, which makes diagnosis and that he has testosterone deficiency.
treatment based on symptoms alone not advisable. Nonspecific • Testosterone deficiency is diagnosed with two early
symptoms include fatigue, decreased muscle strength, morning total testosterone levels below the reference
decreased libido. amotivational state. or decreased robustness range.
or frequency of erections. Testosterone measurements are not • Once testosterone deficiency is confirmed, the cause
recommended if only nonspecific symptoms are present; should be further investigated prior to initiation of tes-
rather, investigation of other causes of the patient's symptoms tosterone replacement.
is appropriate. More specific symptoms include gynecomastia.
diminished testicular volumes, and absence of morning erec-
tions. Measurement of testosterone levels is not recommended
if a patient is having regular morn ing erections, does not have
Testosterone Replacement
true gynecomastia on examination, and has a normal testicu- Therapy
lar examination, as it is highly unlikely that he has testoster- Testosterone replacement therapy is a widely used treatment
one deficiency. for men with hypogonadism. Possible benefits seen with
Testosterone deficiency is diagnosed with tvvo early testosterone replacement therapy, such as improved libido,
morning serum total testosterone levels below the refer- energy level, and bone density, have been described but
ence range. Because illness and strenuous activity ca n remain co ntroversial. Testosterone therapy has been associ-
falsely lowe r testosterone levels, measurement should ated with increased hemoglobin and hematocrit levels,
occur in healthy men who have avoided strepuous activity worsened obstructive sleep apnea. and a decrease in HDL
for several days. Measurements of the testosterone level cholesterol levels. LDL cholesterol levels do not appear to be
occurring later in the morning or in the afternoon are not affected.
useful for interpretation. Consultation with an endocri- Although hypogonadism remains an independent risk
nologist should be conside red if two early morning total fac tor for mortality. recent studies have examined the asso-
testosterone levels are low. In certain cli nical scenarios, ciation between testosterone therapy and cardiovascular risk.
such as morbid obesity. total testosterone may be iow but The association betvveen testosterone therapy and mortal ity
free testosterone may be normal. Free testosterone assays has remained controversial. Physicians prescribing testoste-
can be unreliable, and routine measurement of free testos- rone therapy to elderly men with biochemically p roven
terone is not recommended. Free testosterone by equilib- testosterone defi ciency and comorbidities should use it pru-
ri um dialysis is the gold-standa rd assay. dently with close follow-up. Cardiovascular disease risk as
Once confirmed, the cause of hypogonadism (primary or well as risk for thrombosis should be discussed with patients
secondary) should be further investigated prior to initiation of before pursuing therapy. Prescribing testosterone therapy in
testosterone replacement. Serum LH, FSH. prolactin. and TSH the absence of biochemically proven testosterone deficiency
levels should be measured. Primary testosterone deficiency puts the patient at risk for iatrogenic hyperandrogenism with
(hypergonadotropic hypogonadism) is diagnosed when FSH subsequent increased risk of myocardial in farction. stroke,
and LH levels are frankly elevated in the presence of a simul- death, venous thromboembolism, polycythemia, and
taneously low testosterone level. Low or inappropriately nor- obstructive sleep apnea. Prescribing testosterone therapy in
mal FSH and LH levels in the presence of simultaneous low the absence of a full evaluation may delay treatmen t for sec-
testosterone levels are diagnostic of secondary hypogonadism ondary causes such as prolactinoma. hemochromatosis, or
(hypogonadotropic hypogonadism). intracranial mass.
A hypergonadotropic state (elevated LH and FSH levels) Although implantable pe llets and injectable testosterone
should be further investigated with a karyotype if no history preparations are available, the most popular testosterone prep-
of gonadotoxic therapy or testicular insult is elicited. If a arations currently are topical (most commonly hydroalcoholic

58
Re p roductive Disorders

Measure: total serum testosterone level (x2)

Follow
and/or SHBG in appropriate patients

Confirmed low testosterone:


Check FSH and LH levels

High FSH or LH level

Seco ndary hypogonadism:


Check PRL level, iron studies

Elevated PRL level, ot her pituitary deficiencies Elevated transferrin saturation, ferritin level
Signs/symptoms of mass effect, testosterone level <150 ngldl
(5.2 nmoi/L)

Pituitary MRI Hemochromatosis

FIGURE 1 3 . Algorithm for evaluating male hypogonadism. FSH = follicle-stimulating hormone; LH = luteinizing hormone; PRL= prolactin;
SHBG =sex hormone-binding globulin; x 2 = two separate measurements.

gels) . They require daily use and may incur significant cost rone level should be in the mid-normal range. Monitoring of
the patient. but the steady level of testosterone ach ieved the prostate specific antigen and hematocrit level should
within 30 minutes of application is an appealing feature. tollow Endocrine Society guidelines (Table 29).
Inadvertent absorption by patient contacts may occur; users
KEY POINTS
should be informed that virilization of contacts is not uncom-
mon and premature puberty can occur in exposed children. • Testosterone deficiency should be diagnosed biochemi-
The patient should also be counseled t hat decline in endog- cally. and its cause should be definitively determined
enous testosterone production and spermatogenesis may before initiation of testosterone replacement therapy.
occur. If fertility is desired, testosterone therapy should be • Patients requiring testosterone replacement therapy
avoided, and consultation w ith a reproductive endocrinolo- should have testosterone, prostate specific antigen, and
gist is recommended. hematocrit levels monitored.
Patients requiring testosteron e replacement therapy • Goal total testosterone level should be in the mid-
should have testosterone levels monitored at 3 and 6 months normal range for patients requiri ng testosterone therapy.
after initiation and annually thereafter; the goal totaltestoste-

59
Re productive Disorders

KEY POINTS
• Excessive muscle bulk, acne, gynecomastia, and
decreased testicular volume may be seen on physical
Pa rameter Recom mended Alerts
examination in male patients using anabolic steroids.
Screening Sched ule
• Exogenous testosterone use may result in irreversible
Hematocrit Value obtained at Va lue > 54% decline in spermatogenesis and resultant infertility, as
baseline and then at 3
months and 6 months well as permanent inability to produce endogenous tes-
after therapy initiation, tosterone.
followed by yea rly
measure ments
PSA level For patients > 40 years Increase> 1.4 ng/ml
of age with a baseline {1.4 !1g/ L) in 1 year
M ale Infertility
value >0.6 ng/ml or >0.4 ng/ml Physical examination should include assessment for the pres-
{0.6!1g/L), DRE and PSA {0.4 !1g/L) after
ence or absence of the vas deferens, evaluation for congenital
level {determ ined at 6 months of use;
3 and 6 months after abnormal resu lts on bilateral absence of the vas deferens (as seen in cystic tlbrosis) ,
therapy initiation DRE; AUA prostate assessment of testicular volume. and evaluation for the pres-
followed by regu lar symptoms score/
screening) IPSS >19
ence of hernia. varicocele, or tumor. Semen analysis obtained
after 48 to 72 hours of abstinence fro m sexual activity is the
AUA =American Urological A ssociation; O RE • d ig1ta l rectal examination;
IPSS- International Prostate Symptom Score; PSA = prostate-specific antige n.
best test to assess male fertility. For accurate results. analysis of
D ata frorn Bhasin S, Cunningham GR, Hayes FJ, et a l. Testostero ne therapy in men
the sample should occur within 1 hour of ejaculation. Exl·cnded
with androgen def1ciency syndromes: a n Endocri ne Society C linicCI I Practice abstinence periods may diminish fructose in the ejaculate and
Guideline. J Clin Endocrinol Metab. 2010;95(6):2550. (PMID: 20525905]
J artificially lower sperm motility. If the physical exami nation is
abnormal. evaluation by a urologist may be appropriate. If
semen ana lysis results are abnormal, the test should be
Anabolic Steroid Abuse in Men repeated, and referral. if abnormal, to a reproductive endocri-
Testicular testosterone production is suppressed in the pres- nologist is warranted.
ence of exogenous testosterone administration. Many elite
KEY POINT
athletes abuse androgens in injectable form. and herbal prep-
arations of oral testosterone are readily available. Commonly • Semen analysis obtained after 48 to 72 hours of absti-
used androgens include injectable testosterone esters and nence from sexual activity is the best test to assess male
oral alkylated testosterone preparations. HCG injections fertility; if abnormal, t he test should be repeated tor
m imic LH stimulation to the Leydig cells result in ele- confirmation.
vated testosterone levels. Although this therapy is appropri
ate in men with hypogonadotropic hypogonadism. it may
also be abused. Aromatase inhibitors are frequently used Gynecomastia
concurrently with exogenous testosterone preparations to Gynecomastia is glandular breast tissue enlargement in men
preven t ad ipose conversion of estrogens to androgens and due to imbalance in the levels or activity of testosterone and
deve lopment of gynecomastia. And rostenedione supple- estrogen. This imbalance results in an increased estrogen-to-
menls are commonly abused. testosterone ratio. which in turn results in decreased inhibi-
Excessive muscle bulk, acne, gynecomastia. and decreased tory action of testosterone on t he breast tissue. The less testos-
testicular volume may be found on physical examination in terone a nd/or more estrogen the breast t issue is exposed to,
patients using anabolic steroids. Irreversible hypogonadism may the more likely gynecomastia will develop. Although abnor-
result and often presents as male infertility with oligospermia or mal in the postpubertal man. it is usually benign. It is typically
awospermia on spenn analysis. Permanent inability to produce bilateral but not always symmetric. Unilateral gynecomastia is
endogenous testosterone may occur. Extra testicular effects may uncommon and should be evaluated with mammogram as
also be noted, including low HDL cholesterol level, hepatotoxic- soon as possible owing to risk of breast cancer.
ity. erythrocytosis, and increased risk of obstmctive sleep apnea. There are many causes of gynecomastia, ranging from
Mood disorders are common in anabolic steroid users. drug-induced (marijuana. alcohol. Sa-reductase inhibitors,
Laboratory studies showing low or normal gonadotropin H2- receptor antagonists, spironolactone. digoxin , ketocona-
levels and a low testosterone level with eli nical evidence of hyper- zole, calcium channel blockers. ACE inhibitors, antiretroviral
androgenism are consistent with use of a non- testosterone- agents. tricyclic antidepressants, selective serotonin reuptake
containing product, such as one containing androstenedione, inhibitors) and hypogonadism (prima ry, secondary) to chronic
or cessation of long-standing (typically greater than 1 year) illness (hepatic cirrhosis, chronic kidney disease) and endo
anabolic steroid use, with failure to recover endogenous tes- crine disorders (hyperprolactinemia, acromegaly, hyperthy-
tosterone function. roidism. Cushing syndrome). Obesity and aging are associated

60
Calcium and Bone Disord ers

with gynecomastia owing to increased aromatase activity in In both cases, ionized calcium should be measured. It will
t he periphery. Estrogen-secreting tumors (such as Leydig or usually be normal, indicating normal circulating free levels of
Sertoli cell tumors or adrenal cortical carcinoma) and HCG- calcium. There are also instances of artificially increased cal-
secreting tumors (such as germ cell tumors and hepatic carci- cium levels due to high protein states as in multiple myeloma
nomas) are associated with gynecomastia. . (elevated monoclonal immu noglobulins), hyperalbumine-
A th orough history shoul d be obtained. The breasts mia, Waldenstrom macroglobulinemia, and thrombocytosis.
should be examined for glandular enlargement, which typi- In these patients, ionized calcium would be normal with ele-
cally extends concentrically from under the areolae, and is vated total serum calcium.
firm, mobile, and rubbery. The breasts may be tender if the Vitamin 0 is a fat-soluble vitamin, and body sources
time course is acute. Pseudogynecomastia is subareolar adi- include de novo production from the skin, through fo rms
pose tissue. without glandula r proliferation, that is associ- found in food, and through supplementation (Table 30) . There
ated with obesity. True gynecomastia typically distorts the are two forms of vitamin 0 supplementation: vitamin 0 2
normally flat contour of the male nipple, causing it to pro- (ergocalciferol) and vitamin 0 3 (cholecalciferol). Although
trude owing to the mass of glandular tissue beneath it. In both forms are useful in raising vitamin 0 levels, vitami n n 1
pseuclogynecomastia, the n ipple is typically still flat but soft, may be more beneficial because of tighter bonding to vitamin
and nondescript subcutaneous fat tissue is present in the 0 receptors, longer shelf life, greater potency than vitamin 0 2 ,
breast area. and being identical to the vi tamin 0 that naturally occurs in
Mild, chronic, asymptomatic gynecomastia does not war- humans after ultraviolet light exposure.
rant evaluation. Evaluation of gynecomastia that is asymmet- Regardless of the method of ingestion, vitamin 0 3 and 0 2
ric or concerning for malignancy (bloody n ipple discharge, are both inactive forms that must be hydroxyl ated twice before
hard and fixed, associated with regional lymphadenopathy), of becoming active. The first occurs in the liver and converts
rapid and recent onset, or larger than 2 em (>5 em in obese vitamin 0 to 25-hydroxyvitamin 0 [25(01-l)O], also known as
men owing to the known increase in aromatase activity in calcidiol. The second occurs primarily in the kidney and forms
obesity), should include measurement of total testosterone,
LH, FSH, and TSH levels, as well as assessment of liver and
kidney fu nction. lf indicated by fi ndings on history and/or
physical examination, measurement of prolactin, estradiol,
and HCG may also be indicated. lfthe biochemical evaluation
demonstrates abnormalities, further evaluation w ith testicular Food Sources

ultrasound, adrenal CT, or pituitary MRl may be indicated; Cod liver oi l Cholecalciferol 400-1 000 U/
consultation with an endocrinologist is recommended before teaspoon
imaging is ordered. Salmon, wild caught Cholecalciferol 600-1000 U/4 oz.

KEY POINTS Salmon, ca nned Cholecalciferol 300 U/4 oz

• Unilateral gynecomastia in the male patient is concern- Mackerel, canned Cholecalciferol 250 U/4 oz
ing for malignancy and warran ts immediate evaluation Sundried sh itake Ergocalciferol 1600 U/4 oz
with a mammogram. m ush rooms
Egg yolk Ergoca lciferol 20 U/yolk
HVC • Mild. chronic, ;Jsymptomatic gynecomastia in the male
patient does not warrant evaluation. Sunlight (one minimal 20,000 U in
erythermal dose) bath ing suit

Fortified Foods

Milk Cholecalciferol
Calcium and Bone Orange juice Cho lecalciferol I
Disorders lnfantformula Cholecalciferol

Pharmaceutical Sources
Calcium Homeostasis
Vitamin D 2 Ergocalciferol 50,000 U/capsule
and Bone Physiology
Liquid vitamin D2 Ergocalciferol 8000 U/capsule
Serum calcium levels are tightly regulated on a moment-to-
Mult ivitamin Ergocalciferol 400, 500, or1000
moment basis by the actions of vitami n 0 and parathyroid and U/capsule
hormone (PTH). The amount of calcium that is albumin bound cholecalciferol
can be af1ected by hydration and nutritional status. If albumin Vitam in D3 Cholecalcife rol 400,800,
levels decrease, total serum calcium levels may appear low 2000,5000,
1 0,000, 50,000
(pseudohypocalcemia). Conversely, if albumin levels increase,
U/capsule
total calcium levels will appear elevated (pseudohypercalcemia).

61
Calcium a nd Bone Di sord ers

the physiologically active 1,25-clihydroxyvitamin D [1,25(0H)2D],


Hypercalcemia
also known as calcitriol (Figure 14).
Because 25-hydroxyvitamin D has a relatively long half- Clinical Features of Hypercalcemia
life of several weeks, it is the best indicator of whole body Hypercalcemia is marked by serum calcium levels above the
vitamin D status. Active vitamin D acts on three organ systems normal range, usually greater than 10.5 mg/dL (2.6 mmol!L).
to achieve and maintain normal serum calcium : bone, intes- Most patients are asymptomatic, and hypercalcemia may be
tine, and kidney. With adequate vitami n D, bone resorption is noted incidentally on laboratory tests obtained fo r other rea-
increased, intestinal uptake of dietary calcium is increased, sons. Symptoms may occu r w it h any degree of hypercalce-
and excretion of calcium through the kidney is decreased. PTH mia but are more likely when serum calcium levels exceed
is secreted to increase the calcium in the blood in response to 12 mg/dL (3 mmol/ L) . Classic symptoms of polyuria, polydip-
even the slightest degree of hypocalcemia; it acts on the kidney sia, and nocturia sometimes occur w ith elevated calcium
to increase production of active vitamin D and promote cal- levels ofll mg/dL (2.8 mmol/L) or less. Other symptoms such
cium reabsorption in the distal convoluted tubule and loop of as anorexia, nausea, abdominal pain, constipation, increased
Henle, and increased resorption in bones, thereby increasing creatinine levels, and mild mental status changes are more
release of calcium into the blood. likely to occur with levels greater than 11 mg/dL (2.8 mmol!L).
As serum calcium levels cont inue to increase beyond 12 mg/
dL (3 mmol/L) , symptoms become more severe such as pro-
found mental status changes, obtundation, acute kidney
injury due to profound dehydration, and increased creatinine
Sunlight concentration.
( UVB)
KEY POINTS

1
Skin • Classic symptoms of hypercalcemia are polyuria, poly-
dipsia, anorexia, nausea, abdominal pain, constipation,

t,---------11 7-Dehydrocholesterol
and mental status changes; as serum calcium levels
increase and/or the rate of change increases, symptoms
become more severe, with profound mental status
Cholecalciferol
(vitamin D 3) changes, obtundation, and acute kid ney injury.
• 25-Hydroxyvitamin D has a relativefy long half-life of HVC
several weeks, is the best indicator of whole body
Dietary Sources of Vitamin D
vitami n D status, and is the recommended test for

J
3
vitamin D deficiency.
Liver +-L-C-h o- 1-ec_a_lc-if-er_o_I_D_
Ergocalciferol -(f-ish_._m
D2 (suppl _e_a-t)__j
ements)

Diagnosis and Causes of Hypercalcemia


When serum calcium elevation is incidentally noted, repeat
measurement of serum calcium is indicated, and if a second
hypercalcem ic level is noted , furt her evaluation is warranted
to determine the cause (Table 31). The next step is determin -
ing if the hypercalcemia is PTH- or non-PTI-1- mediated by
simultaneous measurement of serum calcium and intact PTH
levels (Figure 15). Ionized calcium may be used in evaluating
1,25-dihydroxyvitamin D3 ,hypercalcemia, but it is rarely helpful in diagnosing hypercal-
Maint ains calcium balance
in t he body cemia in patients with normal albumin levels or no acid-base
disturbances.

Parathyroid Hormone- Mediated Hypercalcemia


Primary Hyperparathyroidism
Primary hyperparathyroidism is the most common cause of
PTH-mediated hypercalcem ia, and is diagnosed with a simul-
taneously elevated serum calcium level, with an inappropri-
ately normal or elevated intact PTH leveL The incidence peaks
in the seventh decade and affects mostly women (75'Yo). Before
the age of 45 years, rates are similar in men and women.
FIGURE 14 . Production of vitamin D. PTH = parathyroid hormone; Approximately 80% of patients will have elevated PTI-1 levels
UVB = ultraviolet B. with simultaneously elevated calcium levels. Most commonly,

62
Ca l ci um and Bone Di sorde r s

TABLE 31 . Causes of Hypercalcemia may facilitate management. Serum phosphorus levels are typi-
Parathyroid Hormone-M ediated Hypercalcemia
cally low or low-normal in these patients. In contrast, phos-
phorus levels will be elevated in patients w ith vitamin D
Primary hyperparathyroidism (adenoma, hyperplasia) toxicity. Approximately SO% of patients w ith primary hyper-
Pa rathyroid cancer . parathyroidism w ill have elevated urine calcium levels, and the
Tertiary hyperparathyroidism other SO% will have normal levels. Occasionally, urine calcium
Fami lial hypocalci uric hypercalcemia
can be low in those patients with concomitant primary hyper-
parathyroidism and vitamin D deficiency. Additionally, patients
Normocalcemic primary hyperparathyroid ism
with vitamin D deficiency convert more 25-hyd roxyvitam in D
Non-Parathyroid -Mediated Hypercalcemia to 1,2S-dihydroxyvitamin D so they may have elevated levels of
Hypercalcemia of malignancy (humoral and local osteolytic) 1,2S-dihyd roxyvitamin D.
Parathyroidectomy is the treatment for primary hyper-
Vitamin D toxicity
parathyroidism. Surgical management is curative in roughly
Vitamin A toxicity
90'?;, of patients, but evidence that the benefit outweighs the
Milk alkali synd ro m e risk of the surgical procedure is present under only certain
Thyrotoxicosis circumstances. There have been several long-term observa-
Prolonged immobi lization tional studies that found stability in biochemical markers and
Granulomatous diseases (sarcoidosis, tuberculosis) bone density in patients who do not meet the surgical inter-
vention criteria listed in Table 32. When one or more of these
Lymphomas
criteria are met, surge1y is recommended. Surgery can be
Total parentera l nutrit ion
cons·idered when surgical criteria are not mel, bul patients
should be cautioned that there are no robust data to support
that intervention.
primary hyperparathyroidism is due to a single parathyroid It is crit ical t hat an experienced surgeon perform the sur-
adenoma; however, rarely it may be attributed to multigland gery to avoid increased risk of postoperative hypoparathy-
hyperplasia (typical in patients with end-stage kidney disease roidism and damage to the recurrent laryngeal nerve.
or multiple endocrine neoplasia syndromes) or parathyroid Historically, a bilateral neck dissection was done to identify
gland carcinoma (calcium is typically >14 mg/dL [3.5 mmol/L] parathyroid glands t hat appeared to have irregular appearance
and intact PTH'levels >250 pg/mL [250 ng/L] on presentation ; or increased size. With the increased use of sestami bi scans,
d iagnosis is made histopathologically given the overlap with high -detlnition ultrasound, and intraoperative measurement
benign primary hyperparathyroidism). of PTH levels, the minimally invasive technique is now pre-
Once diagnosed, measurement of serum phosphorus, ferred. Minimally invasive surgery allows tor a smaller incision
24-hour urine calcium, and serum 25-hydroxyvitamin D levels and a shortened surgical duration.

200

180

160

:::;- 140
E Prim ary h yper parathyroid ism
a.
3 120
:r:
1--
...
c...
v
100 Vitamin D
deficiency
E 80

60
Normal
40

20 Hypopar athyroid ism

Hy percalcemia of m alignan cy
0
6 7 8 9 10 11 12 13 14 15

Serum calcium (mg/ dl)

FI GURE 15 . Relationsh ip of calcium, PTH, and vitamin D status in normal conditions and in several diseases. PTH =parathyroid hormone.

63
Calcium and Bone Disorders

benign pri mary hyperparathyroidism, parathyroid ca rcinoma


is equally prevalent in both sexes, more commonly presents
with kidney and bone involvement and a neck mass, and fre-
Increase in serum calcium 1 mg/dl(0.25 mmoi/L) above
upper limit of normal• quently is associated with a total serum calcium level greater
Creatinine clearance must be <60 mUm in (0.06 Um in)•
than 14 mg/dL (3.5 mmol/L) and very high pa rathyroid hor-
mone levels, typically greater than four times the upper limit
T-score (on DEXA scan) o f -2.5 or worse at the lumbar spine,
total hip, femoral neck, o r distal rad ius• of normal. Most patients present with single gland involve-
ment. Because parathyroid carcinomas may not appear histo-
Age 50 years or younger•
logically different from benign adenomas, local spread from
Surgery also ind icated in patients in whom medical surve illan ce
the capsule, distant metastasis, or lymph node involvement
is neither desired nor possible, including those with significant
bone, kidney, gastrointestina l, or neuromuscular symptoms must be present for carcinoma diagnos is. rn these patients,
t ypica l of primary hyperparat hyroidism surgical resection is the treatment of choice with calcimimet-
DEXA- dual-energy x- ray absorpt1ometry ics used for residual disease or in patients who are poor surgi-
;tin otherwise asymptomatic patients.
cal candidates. Any patient found to have parathyroid carci-
Recommendations from Bilczikian JP. Khan AA, Potts JT Jr. Third International
noma should be screened for the HRPT2 gene. and if positive.
Wo rkshop on the M ana gement of Asympto matic Primary Hype rparathyroidism. family members should be screened as well.
Guidelines for the m anagement of asymptomatic primary hyperparathyroidiSm:
summary statement from the Third International Workshop. J Cfin Endocrinol
Metab. 2009 Feb;94(2):335-9. 1PMID: 19193908)
Tertiary Hyperparathyro id ism
Tertiary hyperparathyroidism is the result of the prolonged
PTH stimulation needed to mainta in normocalcemia resulting
In patients with osteoporosis who are poor surgical can- from dec-reased 1,25-dihydroxyvitamin D levels from kidney
didates or refuse surgery, intravenous bisphosphonate therapy impairment. This prolonged stimulation resu lts in increased
will slow bone resorption and temporarily decrease serum calcium levels and severe hyperparathyroid hyperplasia and
calcium levels. Intravenous bisphosphonate should be redosed elevated PTH levels that do not respond to phosphate binders
when hypercalcemia recurs. and calcitriol therapy. Severe bone loss and other symptoms
In patients who do not meet the criteria for surgery, su r- make surgical resection the treatment of choice.
veillance is recommended. Patients should have annual meas-
urement of serum calcium and creatinine levels. A th ree site Normocalcemic Primary Hyperparathyroidism
dual-energy x-ray absorptiometry (DEXA) scan should be Nonnocalcemic primary hyperparathyroidism is detlned as
performed every 1 to 2 years to evaluate bone mineral density increased PTH levels in the absence of elevated calcium levels.
of the lumbar spine, hip. an d distal radius. The frequency of This is a diagnosis of exclusion used in patients being evalu-
DEXA sca nning should increase to yearly whe11 any treatment ated for low bone density in which all secondary causes have
has been initiated for bone health or other medications have been ruled out, including vitamin D deficiency. Approximately
been added that may affect bone health (antiandrogen. anties- 20°/o of these patients will develop hypercalcemia \•Vithin
trogen, antiseizure med ications, or glucocorticoids). 3 years, so they should be monitored closely.
In patients with asymptoma tic primary hyperparathy-
roidism, other precautions can be taken to prevent disease- Familia l Hypocalciuric Hyperca lcemia
related complications. These patients should ma intain The most common fom1 of fam ilial hypercalcemia is fa milial
adequate vitamin D (400-600 U daily) intake to prevent fur- hypocalciuric hypercalcemia (FHH). ll is a rare autosomal dom-
ther PTH stimulation. fn patients w ith concomitant vitamin D inant condition with a high penetrance that often occurs in
deficiency, repletion is recommended to replete patients childhood. These patients are frequently asymptomatic, but in
whose levels are below 30 ng/dL (75 nmol!L) with careful rare cases the calcium- sensing receptor (CASR) gene mutation
attention to urine calcium excretion and serum calcium once can increase the risk of pancreatitis. Elevated serum calcium
values a re greater than 30 ng/dL (75 nmol!L). A large study did levels are caused by a mutation in the G-coupled protein CASR
not show worsening calcium levels when repleting vitamin D gene. These receptors are in the parathyroid glands and the kid-
in patients with levels less than 30 ng/dL (75 nmol/L). neys. The sensor mutation results in a shift upward in the "nor-
Adequate physical activity to prevent bone resorption and mal" range of calcium that the receptor recognizes, resulting in a
adequate hydration to prevent kidney damage are imperative. mildly elevated serum calcium level (usually less than 11.0 mg/
dL [2.8 mmol/L]) and high nom1al or mildly elevated PTH level.
Parathyroid Ca rci noma An elevated PTH level is more commonly seen in patients with
Parathyroid carcinoma is very rare, accounting for less than 1% concomitant vitamin D deficiency. The diagnosis is made by
of all persons with primary hyperparathyroidism. Mutations measuring 24-hour urine calcium excretion levels. Typically,
in the HRPT2 gene are thought to be the major genetic link in patients with FHH will have a 24-hour urine calcium level less
parathyroid carcinoma, and inactivation of th is gene leads to than 200 mg/ 24 h (5. 0 mmol/24 h). The preferred standard is the
fa milia l hyperparathyroidism as well. When compared with calcium-creatinine clearance ratio, using the following formula:

64
Ca lcium and Bon e Di sorde rs

Ca/Cr clearance ratio = [24-hour urin e Ca x serum Cr] 7 [serum hypercalcemia through increased bone resorptio n .
Ca x 24-hour urine Cr]. A ratio tess than 0.01 confirms the diag- Reso lution of the thyrotoxicosis should lead to normaliza-
nosis if all other causes of hypocalciuria (thiazides, lithium, tion of calcium levels. Prolonged immobilization and
vitamin D deficiency) have been excluded. FHH is usually a increased vitamin A levels can lead to increased bone
benign condition that requires no intervention but should be resorption . Increased levels of calcium absorpti o n fro m the
recognized to prevent unnecessary parathyroidectomy. gut can be from markedly high vitamin D levels or increased
int ake of calciu m carbonate products (milk-alkali syn -
Other Fami l ial Hypercalcemias drome). Granulomatous d iseases, such as sa rcoidosis and
Fami lial hyperparathyroidism is another rare cause of hyper- Wegener granulomatosis, and malignan t lymphomas cause
calcemia. The disease presentation is almost identical to spo- hypercalcemia through increased 1-a-hydroxylation activ-
radic primary hyperparathyroidism, and a careful family h is- ity that increases 1,25-di hyd roxyvi tam in D levels and cal-
tory will suggest the diagnosis. Once a diagnosis of primary cium reabsorption in the gastroin testinal tract.
hyperparathyroidism is made, screening fo r familial causes
should be done if the patient (1) is younger than 30 years of age Treatment of Hypercalcemia
at t he t ime of diagnosis, (2) has a family history of hypercalce- The treatment of hypercalcemia should focus on decreasing the
mia, or (3) has a medical history of other endocrinopathies. serum calcium level by increasing calcium excretion and
These patients should be tested for multiple endocrine neopla- decreasing bone resorption or intestinal calcium absorption. as
sia syndrome types 1 and 2 (MEN1 and MEN2) . well as volume repletion. Polyuria, due to the decreased concen-
MENl is characterized by functional pituitary adenoma, tration ability of the distal tubule, is the main cause of dehydra-
functional pancreatic tumors, and primary hyperparathyroidism. tion in t hese patients. Although many patients do not requi re
MEN2A is characterized by medullary thyroid cancer, pheochro- hospitalization, those with marked mental status changes, acute
mocytoma, and parathyroid gland hyperplasia with the associ- kidney injmy, or calcium levels greater than 12 mg/dL (3 mmoi/L)
ated RET oncogene mutation. Patients with MEN would follow should be hospitalized for t reatment. First-line therapy is aggres-
similar guidelines for surgical removal of parathyroid glands. sive intravenous flu id resuscitation. Once the patient is volume
replete, an intravenous loop diuretic should be added if the cal-
Medications Causing Hypercalcemia
cium level has not normalized. Intravenous bisphosphonate
Thiazide diuretics decrease the reabsorption of calcium in the
therapy is usually given for longer-term control of hypercalce-
kidney and result in elevated levels. Primary hyperparathy-
mia. Caution shou!C! be exercised with these agents in the setting
roidism, howeyer, should also be considered if the patient
of kidney dysfu nction. Zoledronic acid, while more expensive, is
remains hype rcalcemic despite the discontinuation of the thi-
a more effective therapy for patients with malignancy-related
azide diuretic. In these patients, the thiazide may have been
masking the PTH-mediated hypercalcemi.a. hypercalcemia. In patients resistant to or intolerant of
phonate therapy, off-label use of denosumab, which also reduces
Lithium decreases the parathyroid glands' sensitivity to
osteoclast-mediated bone resorption can be used. Attention
calcium and may also reduce urine calcium excretion.
should be turned as quickly as possible to treatment of the
Non- Parathyroid Hormone- Mediated Hypercalcemia underlying cause of the patient's hypercalcemia to ensure long-
In contrast to PTH-mediated hypercalcemia, non'-PTH- term maintenance ofnormocalcemia. If the underlying cause is
med iated hypercalcemia is associated with very-low PTH lev- increased 1,25-dihydroxyvitamin D hydroxylation, glucocot1i-
els, typically less than 10 to 15 pg/dL (10 -15 ng/L) . coids can be effective therapy but may need to be dosed on a
regular basis. For patients who present with semm calcium
Ma I igna ncy-Associated Hypercalcemia levels greater than 18 mg/dL (4.5 mmoi!L) with neurologic
There are two mechanisms of hypercalcemia of malignancy: symptoms or compromised kidney function, hemodialysis is an
local osteolytic and humoral. When lytic bone metastases are appropriate choice to quickly reduce calcium levels.
present, hypercalcemia is the result of increased mobilization
KEY POINTS
of calcium from the bone. Humoral hypercalcemia is less com-
mon and occurs w hen the tumor itself produces parathyroid- • Primary hyperparathyroidism is the most common
related p rotein (PTHrP) that binds to and activates the cause of parathyroid hormone-mediated hypercalcemia
parathyroid receptor, raising serum calcium levels. Squamous and is diagnosed with simultaneously elevated serum
cell carcinomas, breast cancers, and renal cell carcinomas are calcium levels, w ith an inappropriately normal or ele-
the tumors most commonly associated with hypercalcemia of vated intact parathyroid hormone level.
malignancy. In multiple myeloma, the hypercalcemia is caused • Parathyroidectomy is curative in approximately 90% of
by t he release of factors t hat stimulate osteoclast activity. patients with primary hyperparathyroidism, but should
be performed by an experienced surgeon using mini-
Other Causes mally invasive techniques.
Non - PTH- mediated hypercalcemia ca n be caused by sev-
(Continued)
eral other mechanisms. Thyrotox icosis can lead to mild

65
C alci um and Bo ne Disord ers

KEY P 0 I NT S (continued) KEY POINT


• ln contrast to parathyroid hormone-mediated hypercal- • The most common cause of hypocalcemia is hypopar-
cemia, nonparathyroid hormone-mediated hypercalce- athyroidism , which is most often due to trauma during
mia is associated with very low parathyroid hormo ne surgery to the neck
levels, typically less than 10 to 15 pg/dL (10-15 ng/L) .
• The acute treatment of hypercalcemia focuses on Treatment of Hypocalcemia
decreasing the serum calcium level by increasi ng cal- Mild, asymptomatic hypocalcemia (serum calcium 8.0-8.5 mg/
cium excretion w ith vigorous volume replacement, dL [2.0- 2.1 mmol!L)), is a common fi ndi ng and does n ot
decreasing bone resorption with bisphosphonates. requ ire treatment.
Calcium carbonate and calcium citrate are the most com-
mon oral calcium formulations. Calcium carbonate requires
Hypocalcemia an acidic environment to be absorbed; therefore, all patients
Clinical Features of Hypocalcemia who are on proton pump inhibitors should be prescribed cal-
Hypocalcemia , defined by serum calcium levels below the cium citrate. Adequate levels of both 25-hydroxyvitamin D
normal range, may be asymptomatic if mild. As calcium levels and 1,25-di hyd roxyvitamin D are also essential. In the acute
decrease, particularly below 8.0 mg/dL (2 .0 mmol/ L), symp- phase of repletion, 1,25-dihydroxyvitamin D is more effective
toms may develop, including paresthesias (numbness/tingling than 25-hydroxyvitamin D. If the patient is symptomatic or if
around the mouth , tingli ng in fi ngers a nd toes), muscle the hypocalcemia is acute, calcium gluconate and calcium
cramping (Trousseau and Chvostek signs), decreased muscle citrate are both available in int ravenous form. Goal calcium is
strength, electrocardiogram changes (prolonged Q/T interval) , 7.0 to 7.5 mg/dL (1.8-1.9 mmol /L) with intravenous repletion,
tetany, and seizures. and oral forms can be used once that goal is achieved. The
overall goal of repletion is the low to low-normal range (serum
Diagnosis and Causes of Hypocalcemia calcium 8.0-8.5 mg/dL [2.0- 2.1 mmol/L]) .
Asymptomatic hypocalcemia may be noted incidentally on If a patient requires chronic replacement, usually due to
routine laboratory tests. When this occurs, the calcium level hypoparat hyroidism, care must be taken to avoid hypercalciu-
should be repeated in conjunction w ith a serum albumin leveL ria as calcium nephrolithiasis and decreased glomerular filt ra -
If hypocalcemia is confirmed, simultaneous intact PTH and tion rate can occur. Chronic replacen1ent typically includes
serum calcium mtlst be measured to confirm if PTH is calcitriol, calcium, and occasionally magnesium. Calcitriol is
responding appropriately. The appropriate physiologic the vitamin D source of choice because PTH is needed for
response to lower calciu m levels is an elevation in PTH levels. optimal conversion of 25-hydroxyvitamin D to 1, 25-dihydrox-
yvitamin D. Serum calcium, magnesium, creatinine, and urine ·
I:J Hypoparathyroidism calcium levels should be measured at each foll ow-up visit. The
goal calcium levels should be low-normal without hypercal-
Hypoparathyroidism. commonly due to trauma during neck
surgery (thyroidectomy or parathyroidectomy), is the most ciuria. The magnesium level should ideally be greater than
common cause of hypocalcemia. During head and neck sur 2 mg/dL (0.83 mmoi/L) and creatinine levels should remain in
geries. the parathyroid glands can be inadvertently removed or the normal range. If t he urine calcium level is greater than
parathyroid hormone produclion can be transiently decreased 300 mg/ 24 h (hypercalciuria), calcium and/or vitami n D replace-
due to disruption of blood supply. Only serial measurements of ment needs to be decreased. Calcium is usually decreased flrst if
calcium levels vvill determine whether the damage is transient. the vitamin D levels are within the normal sufflciency range.
Hypoparathyroidism can also be caused by damage from radi KEY POINT
ation exposure. parathyroid gland infarclion. inf11trative dis
• If hypoparathyroidism is the cause of hypocalcemia, cor-
eases (hemochromatosis. Wilson disease, granulomas). or
rection of any coexisting hypomagnesemia to serum
autoimmune hypoparathyroidism. C]
magnesium 2 m g/dL (0.8 mmoiiL) or higher is necessary.
Other Causes of Hypocalcemia
Other, less common causes of hypocalcemia include poor
calcium intake, activating mutations in the CASR gene, PTH M etabolic Bone Disease
resistance, increased phosphate binding in vascular space O st eopenia and Osteoporosis
(rhabdomyolysis or tumor lysis syndrome), increased citrate Physiology
chela tion with large volume blood transfusions, sepsis, vita- Bone m ineral density begins to increase w ith and peak
min D deficiency, and hypomagnesemia. Low levels of mag- bone mass is ach ieved in early aclullhood. Sex hormones, estro-
nesium (due to alcohol abuse or malnutrit ion) activate gen and testosterone, are crucial to increasing bone mineral
G-proteins that stimu late calcium-sensing receptors and density in women and men, respectively. Specifically, estrogen
decrease PTH secretion. has impact on osteoclast and osteoblast activity in both men and

66
Calcium and Bone Disorders

women. Towards the end of puberty, estrogen halts bone resorp- Diagnosis
tion and signals the closure of epiphyseal plates. Bone mass The diagnosis of osteopenia is based on BMD testing.
begins to decline in women after menopause. with decreased Osteoporosis, however, can be diagnosed by BMD testing or
estrogen levels, and in men over age SO years. In men, the loss clinically in the patient with history of fragility fracture, hip
of testosterone typically accelerates bone loss after 70 years of fracture, or vertebral compression fracture.
age. Early cessalion of sex hormone production in either sex, for DEXA assesses the density of the vertebral and hip bones
any reason, may accelerate the loss of bone mineral density. compared with healthy young-adult sex-matched reference
Bone loss occurs when the removal of old bone (osteoclastic values. The distal one- third of the radius can be used in
activity) exceeds the replacement with new bone (osteoblastic patients when the hip or verteb ral BMD cannot be measured.
activity). Accelerated bone Joss can often be attributed to hypo- The score is based on the number of standard deviations above
gonad ism or medications that promote bone Joss. or below the mean reference value and is known as the T-score.
Th ose patients with T-scores at - 1.0 and above have normal
Risk Assessment and Screening Guidelines bone density. A T-score betvveen -1.0 and - 2.S is defined as
Declining bone mi neral density is associated with futu re frac- low bone mass (osteopenia). Osteoporosis is defined as a
ture risk and therefore is an important component of fracture T-score below -2.5. Severe osteoporosis is defined as aT-score
risk assessment. Individual peak bone mass is determined by of - 2.S or below with one or more fractures. In women and
genetic factors, nutrition. changes in hormone (estrogen. tes- men younger than SO years, the International Society for
tosterone. thyroxine) levels. concomitant health conditions. Clinical Densitometry recommends that ethnic- or race-
and physical activity level. adjusted Z-scores be used. The Z-score compares a patient's
The National Osteoporosis Foundation recommends that BMD with others of their same age and ethnicity, and osteopo-
all postmenopausal women and men older than SO years of age rosis/osteopenia cannot be diagnosed in these patients. A
be evaluated for osteoporosis. This evaluation includes a thor- Z-score of -2.0 or lower should be described as ··tow bone
ough history For potential risk factors and physical examination. mineral density for chronologie age" or "below the expected
This preliminary screeni ng w ill determ ine if bone mineral den- range for age. " Patients with Z-scores above - 2.0 are "within
sity (BMD) or vertebral imaging is necessary. Table 33 lists sev- the expected range for age."
eral common risk facto rs for osteoporosis. !fa patient is deemed In 2008, the World Health Organization (WHO) created
high risk. a study of BMD with DEXA may help further assess the Fracture Risk Assessment Tool (FRAX) calculator t hat fur-
frac tu re risk The DEXA is designed to measure BMD and estab- ther defines the 10-year fractu re risk for patients with osteope-
lish risk of fracture in postmenopausal women. In young men nia, defined as aT-score betvveen - 1.0 to - 2.S on DEXA. The
and premenopausal women, assessment of BMD for fracture FRAX score notes the probability of major osteoporotic fracture
risk is not advised or validated. Table 34lists the U.S. Preventive and hip fracture in the next 10 years. If the risk of major
Services Task Force recommendations for BMD testing. porotic fracture is greater than or equal to 20% or the risk ofhip

TABLE 33. Low Bone Mineral Density Associations


Lifestyle Comorbid Illness Hormonal Stat es Medications Nonmodifiable Risk
Factors

Alcohol use Vitamin D insufficiency Premature menopause Anticonvulsants Race


BM I < 17 Hyperca lciuria Premature ovarian Glucocortico ids mg/d Age
insufficiency of prednisone or
Low calcium intake Osteogenesis imperfect Gender
equivalent for ;::3 months)
Panhypopituitarism
Smoking Homocystinuria First-degree relative
GnRH antagonists and
Hyperprolactinemia with low bone mineral
Immobilization Hemochromatosis agon ists
density
Androgen insufficiency
Weight loss Glycogen storage SSRis
disease Thyrotoxicosis
M alabsorptive bariatric Thiazolidinediones
surgery Cystic fibrosis
Aromatase inibitors
Gastric bypass surgery Celiac disease Anticoagulants
Recurrent falls Cushing syndrome Lithium
Inflammatory bowel
disease
Diabetes mellitus
(types 1 and 2)

GnRH = gonadotropin-releasing hormone; SSRI = selective serotonin reuptake inhibitor.

67
Calcium and Bone D i sorders

TABLE 34. U.S. Preventive Services Task Force hypercalciuria) is an appropriate set of laboratory tests to
Recommendations for Measurement of Bone Mineral screen for secondary causes ofBMD loss. These are modifiable
Density and Vertebral Imaging conditions that, if corrected, will result in increased BMD.
Bone Mineral Density Testing• These patients are typically young with either markedly low
BMD for age or a new fracture or patients of any age wit h mu l-
Women age 65 and older and men age 70 and old er
tiple fractures.
Postmenopausal women and m en age 50 to 69, based on risk
factor profile
With all secondary causes of decreased BMD, reversal of
the cause should be the first line of therapy and subsequent
Those who have had a fracture, t o det ermine degree of d isease
severity DEXA should be performed. If the BMD has n ot improved or
the FRAX score suggests increased risk of fracture, treatment
Radiographic findings suggestive of osteoporosis or vertebral
def orm ity should be started. In the event of a fracture, the underlying
G lucoco rticoid therapy for m ore than 3 months
cause should sti ll be addressed in addition to initiating phar-
macologic therapy for osteoporosis.
Prima ry hyperpa rathyroid ism
Treatment for o st eoporosis (t o m onitor therapeutic response) Pharmacologic Treatment Options
Vertebrallmagingb Currently there are six categories of pharmacologic agents that
are FDA approved for the treatment of postmenopausal osteo
Women;::, 70 and men ;:::80 ifT-score at the spi ne, total hip, or porosis. These medications are bisphosphonates, calciton in,
femoral neck is.,; - 1.0
estrogens, estrogen agonists, parathyroid hormone, and the
Women aged 65-69 and men aged 75-79 ifT-score at the
receptor activator of nuclear factor KB (RANK) ligand inhibitor
spine, total hip, or femoral neck is .,;1 .5
family. Bisphosphonates are usually first-line therapy unless
In postmenopa usal women age 50-64 and m en aged 50-69
with the following risk factors:
there is a compelling reason why another therapy should be
used. The other modalities are typically used after a bisphos-
Low-trauma fractures
phonate failure or inability to use the medication. Most of these
Historic height loss of 1.5 in or more (4 em )
medications can also be used for prevention of osteoporosis.
Height loss of 0.8 inches o r more (2 em ) Prevention therapy may be used in patients with osteopenia
Recent or ongoing long-term glucocorticoid treatment who do not meet FRAX standards for therapy but have multiple
' BMD testing should be performed at DEXA facilit<es ustng accepted quality
risk factors such as high-risk medications (glucocorticoids or
assurance measures. antiestrogen, antiandrogen, or antiseizure medications) in
bVertebral imag ing should be repeated w hen a new loss of height is noted or new combination with a strong family history of osteoporosis.
back pain is reported.

Bisp hosphonates
fracture is greater than or equal to 3%, the patient's benefit Bisphosphonate medications work by inhibiting osteoclastic
from therapy exceeds the risk, and treatment should be offered. activity. Before starting any bisphosphonate therapy, vitamin D
The FRAX was validated fo r use in persons 40 to 90 years status and calcium levels should be evaluated, as bisphospho-
of age who are not currently or previously treated with phar- nates can lead to hypocalcemia. For oral bisphosphonates,
macotherapy for osteoporosis. The WHO has a web.site that integrity of the esophageal lining and ability to swallow pills
offers an online FRAX calculator at vvvvw.shef.ac.uk/ FRAX. are important. Kidney function should be assessed as bispho-
In additional to BMD testing, vertebral imaging using radi- sphonates are contraindicated in patients with an estimated
ographs is recommended in high-risk groups since many ver- glomerular filt ration rate less than 35 mL/m in /1. 73 m 1 •
tebral fractures are asymptomatic. The presence of a vertebral Although rare, osteonecrosis of the jaw has been reported with
compression fracture establishes t he clinical diagnosis of osteo- , bisphosphonate usage, particularly with high-dose intrave-
porosis, regardless ofT-score on DEXA, and treatment is rec- nous administration and increased duration of the bisphos-
ommended. Once a vertebral image is obtai ned, it only needs to phonate. Additionally, atypical femur fractures have been
be repeated if there is noted height loss or new back pain. reported with long-term usage. Regular questioning about
pain in the thigh or groin area is recommended for patients on
Evaluation of Secondary Causes of bisphosphonates. If patients report discomfort, a radiograph
Bone Mineral Density Loss should be obtained. To reduce the risk of these side effects, a
Most cases of osteoporosis are due to declining levels of sex drug holiday has been suggested in patients with low-risk
hormones which are non- modifiable, such as age, sex, meno- osteoporosis (T-score greater than - 2.5 or single fractures) who
pause, height, and build. Some patients, however, have osteo- have been on therapy for 3 to 5 years with stable BMD. During
porosis caused by secondary causes. Measurement of complete bisphosphonate therapy for prevention or treatment, men SO
blood count (for malignancy), complete metabolic panel (for to 70 years of age should consume 1000 mg/d of calcium and
calcium levels and kidney function), thyroid-stimulating hor- women aged Sl years and older and men aged 71 years and
mone, 25- hydroxyvitamin D, and urine calcium (screening for older consume 1200 mg/d of calcium.

68
Calcium and Bone Disord e rs

Alendronate has been approved by the FDA for preven- Parathyroid Horrnone
tion and treatment of osteoporosis. The prevention dose is a Teriparatide (recombinant human PTH [1-34]) has been FDA
S-mg tablet daily or a 3S-mg tablet weekly. The treatment dose approved for the treatment of osteoporosis in postmenopausal
is a 10-mg tablet daily or a 70- mg tablet weekly. Alendronate is women and men who are at high risk for fracture. "High risk"
also approved for treatment of men with osteoporosis as well is defined as patients with aT-score of -3.0 or less or patients
as treatment of both women and men with glucocorticoid- who have either had a fracture or decreased BMD while on
induced osteoporosis. Alendronate has been shown to reduce bisphosphonate therapy. It is also approved for men and
the incidence of spine and hip fractures by approximately SO% women at high risk offracture due to long-time glucocorticoid
over 3 years in patients with previous fractures. use. Teriparatide has bone-building properties in addition to
lbandronate is FDA approved for the treatment and pre- the antiresorptive prope1iies of the other agents. It is the only
vention of postmenopausal osteoporosis. The dosage is a bone-building treatment option fo r osteoporosis.
lSO-mg tablet monthly or 3 mg every 3 months by intravenous Teriparatide is an anabolic steroid that is administered by
injection. There is FDA approval for the oral formulation for a 20-!J.g daily subcutaneous injection; it is approved for up to
osteoporosis prevention only. lbandronate primarily reduces 24 months over a patient's lifetime. After the 24-month dura-
risk of vertebral fractures by SO% in 3 years. tion, an antiresorptive agent (such as b isphosphonates or den-
Risedronate is FDA approved for the prevention and treat- osumab) can be administered to maintain BMD gains achieved
ment of osteoporosis. The treatment dose is a 5-mg tablet daily, with teriparatide.
a 3S-mg tablet weekly, a 7S-mg tablet on two consecutive days
every month, or a 1SO-mg tablet monthly. Risedronate is also Receptor Activator of Nuclear Factor KB (RANK)
approved for treatment of osteoporosis in men and women Ligand Inhibitors
with glucocorticoid-induced osteoporosis. Risedronate Denosumab is a receptor activator of nuclear fac tor KB (RANK)
reduces the incidence of vertebral fracture by approximately ligand inhibitors that is FDA approved for the t reatment of
4S% and nonvertebral fractures by one- third over 3 years. osteoporosis in postmenopausal women who are at high risk
Zoledronic acid has been FDA approved for the preven- of fracture. lt is an antiresorptive agent, like the bisphospho-
tion and treatment of postmenopausal osteoporosis in women, nates with much the same effect and outcomes. It is also
for improvement of bone mass in men with osteoporosis, and approved fo r treatment of osteoporosis in men and those
for the prevention and treatment of glucocorticoid-induced undergoing treatment of certain cancers, such as prostate can-
osteoporosis in men and women. Recently zoledronic acid has cer, who are at high risk for fractures. Denosumab is given by
been approved ·for secondary prevention of fractures in subcutaneous injection (60 mg every 6 months) .
patients who have had recent low-trauma hip fracture. The
treatment dose ofzoledronic acid isS mg intravenously annu- Annual Reassessment of Patients with Low Bone Mass
ally or once every 2 years for prevention. Once an initial DEXA scan has been obtained, every effort
should be made to have subsequent scans done on the same
Calcitonin machine. Once a repeat scan has been done, change in BMD,
Calcitonin is FDA approved for the treatment of osteoporosis not T-score, from year to year is the appropriate way to inter-
in women who areS or more years postmenopausal. pret whether there has been a significant change in BMD. Most
(200 U) is delivered in a single daily intranasal spray. reports will not show a statistically significant change in the
Subcutaneous administration is also available but is used less BMD from the previous test. If not noted on the report, a cal-
frequently. Calcitonin should be used with caution with culated change of about 4% likely represents a statistically
patients who have an allergy to salmon, allergic rhinitis, or significant change. Annually, a complete clinical evaluation of
epistaxis. Very rarely, patients can also have an anaphylactic the patient, determination of risk fac tors for bone loss, and
response that requires emergency attention and discontinua- evaluation for development of secondary causes of bone loss
tion of the medicalion. should be performed, starting with a history and physical
examination .
Estrogen Agonists and Antagonists
The use of estrogen to maintain bone health in postmenopau- Vitamin D Deficiency
sal women has fallen out of favor because of data indicating In promoting absorption from the gut, vitamin D enables proper
that estrogen increases the risk of cardiovascular disease and bone mineralization by mainlenance of calcium and phospho-
breast cancer. Therefore, estrogen use fo r osteoporosis preven- rus levels. Vitamin D also modulates the actions of osteoblasts
tion should be limited to younger women with premature and osteoclasts to ensure proper bone growth and remodeling.
ovarian failure and postmenopausal women who also require Chronically low levels of vitamin D can lead to rickets in chil-
its beneficial effects for hot flushes or vaginal dryness. dren and osteomalacia in adults (see MKSAP 17 Nephrology).
Raloxifine has been approved for the treatment of post- In addition to bone health, vitamin D plays a role in
menopausal osteoporosis. The treatment dose is a 60-mg tab- inflammation reduction, growt h regulation of various cell
let to be taken with or without food. types, immune function, and neuromuscular signaling.

69
Calcium and Bone D isorders

In assessing serum levels of vitamin 0, concentrations of Vitamin 0 toxicity is a very rare entity but one to be aware
25-hydroxyvitamin 0 are the best indicator of vilamin 0 sta- of. The effects of vitamin 0 levels greater than 90 ng/mL (225
tus. It reflects vitamin 0 produced cutaneously and that nmol!L) include hypercalcemia. As a fat-soluble vitamin,
obtained from food and supplements and has a circulating decreasing vitamin 0 levels that are once elevated can be a
half-life ofl5 days. slow process requiring continued monitoring.
There are three levels of vitamin 0 status: sufficient
KEY POINTS
(25-hydroxyvitamin 0 ng/m L [75 nmol/L]) , insuflicient
(25-hydroxyvilamin 0 21-29 ng/mL [52.4-72.4 nmol!L]), and • The U.S. Preventive Services Task Force recommends
deficient (25- hydroxyvitamin 0 s20 ng/mL [50 nmol/ L]). screening for osteoporosis in women aged 65 years and
Because vitamin 0 levels can be affected by sun exposure, fail older and in younger women whose fracture risl< is
through winter months are ideal times to measure vitamin D equal to or greater than that of a 65-year old white
levels. It is best to measure vitamin 0 levels at the same lime women who has no additional risk fac tors.
each year unless treatment is being followed. In general. the • The presence of a vertebral compression fracture makes
optimal levels of vitamin Dare those that prevent PTH levels the clinical diagnosis of osteoporosis, regardless of
from increasing to above normal levels. Increased PTH levels T-score on dual-energy x-ray absorptiometry (DEXA)
will lead to increased calcium withdrawal from the bones. In scan, and treatment is recommended.
an attempt to find the optimal vitamin 0 level. several studies • Fracture Risk Assessment Tool (FRAX) score can help
have looked at vitamin 0 levels related to cancer incidence, identify which patients are most likely to benefit from
muscle stability and falls, immune status, and mood, in addi- osteoporosis treatment; bisphosphonate therapy is first-
tion to bone health. Among most experts, a level between 30 line therapy for postmenopausal osteoporosis treatment
and 40 ng/mL (75-100 nmoi/L) is deemed sufficient for pre- and prevention.
ventive health. Based on these levels, about 30% to 60% of
• A 25 D level between 30 and 40 ng/mL HVC
Americans have low vitamin 0 levels, therefore most expert
(75-100 nmoL L) is deemed su fflcient for bone
grou ps recommend screening all patients at least once. Those
most expert groups recommend screening all groups at
with darker skin , decreased sun exposure, or increased
least once for evidence of deficiency since U.S. inci-
demands (pregnancy) often have low levels.
dence is 30% to 60% of the population, however, it
Special populations will have lower levels of vitamin 0
should not be a serial, recurring screening test.
owing to medical conditions or medica lion side effects. In addi-
tion, obesity has been correlated with lower vitamin 0 levels
possibly related to fat sequestration. Certain antiseizure medica- Paget Disease of Bone
tions (phenobarbital and phenytoin) may increase the metabo- Paget disease of bone is characterized by rapid and chaotic
lism of vitamin 0 to inactive forms. Glucocorticoids can decrease bone remodeli ng leading to disorganized bone microarchitec:
vitamin 0 metabolism. Agents that decrease absorption such as ture. This disease more commonly affects persons of European
orlistat and cholesterol lowering agents can decrease vitamin 0 descent during the sixth decade, with a prevalence of 3'Yo to
absorption. Similarly, patients with malabsorption disorders, 10% in the elderly: Paramyxovirus infection of osteoclastic
including those with celiac disease and those who had bariatric precursors is thought to be one possible cause, although about
surgery, can have decreased levels of vitamin D. In these special 15% of people with Paget disease have a family member with
patient populations, not only does screening for deficiency need the disease. Paget disease appears to be inherited in an autoso-
to be more frequent, but repletion may be more challenging.ll is mal dominant manner with incomplete penetrance.
recommended that these populations be given at least two to
three times more vitamin 0 to maintain adequate levels. Clinical Presentation
The most common clinical manifestation is asymptomatic
Recommendations elevated serum alkaline phosphatase levels; only 30% of
The current National Osteoporosis Foundation and Endocrine patients have symptoms at diagnosis. In some patients, the
Society recommendation for adults 19 to 70 years of age is at dysfunctional bone structure creates expansion in the bone
least 600 U/d of vitamin D to maximize bone health: however, leading to pain, swelling, and warmth. Bones of the axial skel-
to raise blood levels consistently above 30 ng/dL (75 nmol/ L) eton are most freque ntly affected. namely the pelvis (70%),
may require 1500 to 2000 U/d. In adults older than 70 years of femur (55%) ,lumbar spine (53%), skull (42'Yo), and tibia (30%).
age, 800 U of supplemental vitamin 0 per day is recom- As a result, patients tend to have headache, sensorineural
mended to maximize bone health; however, 1500 to 2000 U/d hearing loss, and bowing of the long bones (Figure 16). The
may be required to keep levels consistently above 30 ng/dL abnormal bone growth may also lead to nerve impingement
(75 nmol/L). ln treating the deficient patient. 50,000 U of causing pain or neurologic deficits. Rarely, patients can
either ergocalcife rol or cholecalciferol is recommended, once develop increased vascular shunting to bones with resultant
weekly for 8 weeks. Once sufficie ncy is attained , maintenance right-sided heart fail ure, cellular transformation to osteosar-
therapy of 1500 to 2000 U/d is recommended. coma, and hypercalcemia of immobilization.

70
Bibliography

Bibliography
Disorders of Glucose Metabolism
Bergensral RM, Klonofi DC. Garg SK, et al; ASI-'!RE In-HOME Study Group.
Threshold-based insulin pump interruption for reduction of hypoglyce-
mia. N Eng! J Med. 2013 Jul18;369(3) :224-32. [PMID: 23789889]
Bergenstal RM, Tamborlane WV, Ahmann A, et a!; STAR 3 STUDY Group.
Effectiveness of sensor-augmented insulin-pump therapy in type 1 diabe-
tes. N Engl J Med. 2010 Jul22;363(4):3ll-20. [PMID: 20587585]
Committee on Practice Bulletins-Obstetrics. Practice Bulletin No. 137:
Gestatio nal diabe tes mellitus. Obstet Gynecol. 2013 Aug;122(2 Pt 1):406-16.
[PM!D: 23969827]
FIGURE 1 6 . Radiograph showing Paget disease of bone in the left lateral Cowie CC. Rust Byrd-Holt DO, et al. Prevalence of diabetes and high risk
tibia of a 71-yearold woman. Note the anterior bone, cortical thickening, and for diabetes using A\, criteria in the U.S. population in 1988- 2006. Diabetes
bone enlargement as compared with the normal radiograph on the right. The Care. 2010 Mar;33(3J:S62-8. [PMID: 20067953]
long-standing Paget disease resulted in a left leg that was 2.5 em (1 in) shorter Cryer PE, Axelrod L, Grossman AB. ct al. Evaluation and management of adult
than the right. hypoglycemic disorders: an Endocrine Society Clinical Practice Guideline.
J Endocrinol Metab. 2009 Mar;94(3) :709-28. [PMID: 19088155]
Gregg EW, Chen H, Wagenknecht LE, eta]. Association of an intensive lifestyle
intervention with remission of type 2 diabetes. JAMA. 2012 Dec
19;308(23) :2489- 96. [PMID: 23288372]
Diagnosis Malanda UL, Welschen LM, Riphagen !1, et al. Self-monitoring of blood glu-
The diagnosis of Paget disease should be suspected in asymp - cose in patients with type 2 diabetes mellitus who are not using insulin.
Cochrane Database Syst Rev. 2012 Jan 18;l:CD005060. [PMID: 22258959]
tomatic patients w ith a n isolated elevation of alkaline phos-
Nathan OM; DCCT/EDIC Research Group. The Diabetes Control and
phatase without evidence of liver disease. In these patients Complications Trial/EpidemiolOi:,'Y of Diabetes Interventions and
the most sensitive test is a nuclear bone scan which will Complications Study at 30 years: Overview. Diabetes Care. 2014 Jan;37(1):9-
16: [PMID: 24356592]
detect areas of increased metabolic activity. Plain films of Qaseem A, Humphrey LL, Sweet DE, et al. Oral pharmacologic treatment of type
these areas should be obtained to identify pathognomonic 2 diabetes mellitus: a clinical practice guideline from the American College
pagetic lesions such as focal osteolysis with coarsening of the of Physicians. i\nn Intern Med. 2012 Feb7;156(3) :218 31. [PMID: 22312141]
Yeh HC, Brown TT, Maruthur N, et al. Com parative effectiveness and safety of
t rabecular pattern and cortical thickening. In symptomatic methods on insulin delivery and glucose monitoring for diabetes mellitus.
patients with bone pain, plain films of painful areas may be Ann Intern Met\. 2012 Sep 4;157(5):336-47. [PMlD: 22777524]
the init ial imaging test, although many experts recommend a Disorders of the Pituitary Gland
baseline bone scan once the diagnosis is confirmed prior to Colao A. Bronstei n MD. Freda P, et al. Pasireotic!e versus octreotide in acro-
initiating treatment. megaly: A head-to-head superiority stui'iy. J Clin Endocrinol Metab. 2014
Mar;99(3):791-9. [PM!D: 24423324]
Colao A, Petersenn S, Newell-Price J, et al. A 12-month phase 3 study of
Treatment pasireotide in Cushing's disease. N Engl J Med. 2012 Mar 8;366(10):914-24.
lPM I0 : 22397653]
The main therapies for Paget disease ofbo!le are the nitrogen-
Freda PU, Beckers AM, Katznclson L, eta!. Pituitary incidentaloma: an endo-
containing bisphosphonate medications (alendronate, crine society clinical practice guideline. J Clin Endocrinol Metab. 2011
pamidronate, risedronate, and zoledronic acid). The main Apr;96(4):894-904. [PM!D: 21474686]
indications for antiresorptive therapy are (1) pain caused by Katznelson L, Laws £ R Jr, Melmed S, et al. Acromegaly: an endocrine society
clinical practice guideline. J Clin Endocrinol Metab. 2014 Nov;99(11):3933-
the increased metabolic activity; (2) planned surgery at site of 51. [PMID: 25356808]
pagetic bone disease, and (3) hypercalcemia due to multiple Klibanski A. Clinical Practice. Prolactinomas. N Eng! J Med. 2010 Apr
affected sites. There is no evidence that antiresorptive therapy 1;362(13):1219-26. [PMID: 20357284]
Korbonits M, Storr H, Kumart AV. Familial pituitary adenomas-who should be
is beneficial in asymptomatic patients. These medications are tested for AlP mutations? Clin Enclocrtnol (Ox!). 2012 Sep;77(3):351 6.
ideal because they suppress the rapid bone turnover that is [PMID: 22612670]
characteristic of Paget disease. Decreases in alkaline phos- Malchiodi E. Profka E, Ferrante E, et al. Thyrotropin secreting pituitary adeno-
mas: Outcome of pituitary surgery and irradiation. J Clin Enclocrinol Metab.
phatase can be noted within 10 to 14 days after the initiation of_ 2014 Jun;99 (6):2069-76. [PMI D: 24552222]
therapy with a nadir reached in 3 to 6 months. NSAIDs and Melmed S, Casanueva FF, Hoffman AR, et al. Diagnosis a nd treatment of
antineuropathic medications can also be used for pain control hyperprolactinemia: An Endocrine Society clinical practice guideline.
J Clin Endocrinol Metab. 2011 Feb;96(2):273-88. [PM ID: 21296991]
in these patients. For patients w ith pseudofractures, orthope- Nieman LK, Biller BMK, Findling JW, et al. The diagnosis of Cushing's syn
dic stabilization may be required. drome: an Endocrine Society clinical practice guideline. J Clin Endocrino l
Metab. 2008 May;93(5):1526-40. [PMID: 18334580]
KEY POINTS Rajasekaran S. Vanderpump M. Baldeweq S, et al. UK guidelines for the man-
agement of pituitary apoplexy. Clin Endocrinol (Ox!). 2011 Jan;74(1):9-20.
• The most common clinical manifestation of Paget dis- [PM!D: 21044119]
ease of bone is asymptomatic elevated alkaline phos- Vasilev V, Da ily AF, Petrossians P, et a!. Fam ilial pituitary tumor syndromes.
phate levels. Endocrine Practice. 2011 Jui-Aug;l 7 Suppl 3:41 -6. [PMID: 21613050]

• The main therapy fo r Paget disease of bone is the Disorders of the Adrenal Glands
Cordera F; Grant C, van Heerden J, eta!. Androgen secreting adrenal tumors.
nitrogen-containing bisphosphonate medications Surgery. 2003 Dec;l34(6):874-80. [PMID: 14668717]
(alendronate, pamidronate, risedronate, and zole- Eisenhofer G. Goldstein D, Walther M, eta\. Biochemical diagnosis of pheo-
dronic acid). chromocytoma: How to distinguish true- from false-positive test results. J
Clin Endocrinol Metab. 2003 Jun;88(6) :2656-66. [PMID: 12788870]

71
Bibliography

Fishbein L, Orlowski R, Cohen D. Pheoch romocytoma/ Paraganglioma: Review Barbieri RL. Makris A. Randall RW, Daniels G, Kistner RW. Ryan KJ. Insulin
of perioperative management of blood pressure and update on genetic stimulates androgen accumulation in incubations of stroma
mutations associated with pheochromocytoma. J Clin Hypertens obtained from women with hyperandrogenism. J Clin Endocrinol Metab.
(Greenwich). 2013 Jun;15(6):428-34. [PMID: 23730992] 1986 May;62(5):904-10. [PMID: 3514651]
Funder JW, Carey RM , Fardella C. et al. Case detection, diagnosis. and treat- Belvisi L, Bombelli F, Sironi L. Doldi N. Organ specific au toimmunity in
ment of patients with primary aldosteronism: an endocrine society clinical patients with premature ovar ian failure. J Endocrinol Invest. 1993
practice guideline. J Clin Endocrinol Metab. 2008 Sep;93(9):3266 81. Dec;16(11) :889-92. [PMID: 8144865.1
[PMID: 18552288] de Moraes Ruehsen M, Bli7.7ard RM, Garcia-Bunuel R, Jones GS. Autoimmu n ity
Lenders JW, Duh QY, Eisenhofer G. et al. Pheochromocytoma and panlgan- and ovarian failure. Am J Obstet Gynecol. 1972 Mar ;l12(5):693 703. [PMID:
glioma: An en docrine society clin ical practice guideline. J Clin Endocrinol 4551032 1
Metab. 2014 Jun;99:1915- 1942. [PMIO: 24893135] r:em anclez-Balsells MM. Murad Mil, Lane M, et al. Clinical review 1: Adverse
Morelli V, Reimond o G. Giordano R, et a!. Long term follow up in adrenal ellects of testosterone therapy in adult men: a systematic review and meta-
incidentalomas: an Italia n multicenter study. J Clin Endocrinol Metab. 2014 analysis. J Clin Endocrinol Metab. 2010 Jun;95(6):2560-75. (PM lD: 20525906]
Mar:99(3):827-34. [PMTD: 24423350] Hoek A, Schoemaker J, Drexhage HA. Premature ovarian failure ancl ovarian
Neary N, Nieman L Adrenal insumciency: etiology, diagnosis and treatment. Curr autoimmunity. Endocr Rev. 1997 Feb;18(1):107- 34. [PMID: 9 034788]
Opin Endocrinol Diabetes Obesity. 2010 Jun;17(3):217-23. [PMID: 20375886] Kauffman RP. Castraeane VD. Premature ovarian failure associated with auto-
Nieman LK, Biller BM. FindlingJW, eta!. The diagnosis ofCush.ing's syndrome: immune polyglandular syndrome: pathophysiological mechanisms and
an Endocrine Society Clinical Practice Guideline. J Clin Endocrinol Metab. future fertility. J Womens Health (Larchmt). 2003 Jun;12(5):513-20. [PMID:
2008 May;93(S): t526-40. [PMID: 18334580] 12869299]
Sprung CL. Annane D. Keh D. et al. Hydrocortisone therapy for patients with Kong MF. JeiTcoate W. Eighty-six cases of Addison's disease. Clin Endoerinol
septic shock. · Engl J Med. 2008 Jan 10;358(2):1ll- 24. IPMID: 1818-1957] (oxn. 1994 Dec;41(6):757- 61. [PMI D: 7889611]
Young Jr. Clinical practice. The incidentally discovered adrenal mass. N Lcgro RS, Barnhart HX, Schlaff WD, et al; Cooperative Multicenter
Engl J Med. 2007 Feb 8;356(6):601-10. [PMID: 17287480] Reproductive Medicine Network. Clomiphene. metformin, or both for
infertility in the polycystic ovary syndrome. N Engl J Med. 2007 Feb
Zeiger MA, Thompson GB. Duh QY, et al. The American Association of Clinical
8;356(6):551 66. [PMID: 17287476]
Endocrinologists and American Association of Endocrine Surgeons medi
cal guidelines for the management of adrenal incidentalomas. Endocr Mignot :\1H, Schoemaker J. Kleingeld M. Rao BR, Drexhage HA. Premature
Pracl. 2009 Jui-Aug;15 Suppll:l-20. [PMID: 19632967] ovarian failu re. I: The association with autoimmu nity. Eur J Obstet Gynecol
Reprod Bioi. 1989 Jan;30(1):59- 66. [PM I]): 2647538]
Disorders of th e Thyroid Gland
Moncayo-Naveda H. Moncayo R. Benz R. Wolf A. Lauritzen C. Organ specific
American Thyroid Association (ATA) Guidelines Taskforce on 1l1yroid Nodules and antibod ies against ovary in patien ls with systemic lupus erythematosus.
Dit1erentiated Thyroid Cancer. Cooper OS, Doherty GM, Haugen BR. el a!. Am J Obstet Gynecol. 1989 May;160(5 Pt1):1227-9. [PMID: 2729399]
Revised American Thyroid Association management !,'llidelines for patien ts
witb thyroid nodules and differentiated thyroid cancer. Thyroid. 2009 Plymatc SR, Matcj LA, Jones RE, Fried l KF.. lnhibilion of sex hormone-bind ing
Nov;19(U):U67-214. Erratum in: Thyroid. 2010Aug;20(8):942. [PMID: 19860577] globulin production in rhe human hepatoma (llep G2) cell line by insulin and
prolactin. J Clin Endocrinol Metab. 1988 Sep;67(3):460 4. [PMlD: 2842359]
Barta lena L. Diagnosis and management of Graves disease: a global overview.
Nat Rev Endocrine!. 2013 Dcc:9 (12):724 -34 . [PM ID: 24126481] Ryan MM. Jones HR Jr. Myasthenia gravis and premature ovarian fa ilure.
Muscle Nerve. 2004 Aug;30(2):231-3. [PMID: 15266640]
Biondi B. Natural history, diagnosis and management of subclinical thyroid
dysfunction. Best Pract Res Clin Endocrinol Metab. 2012 Aug;26(4):431 46. Vigen R, O'Donnell CI, Baron AE. et al. Association of testosterone thera py
[PM ID: 22863386] ' with mortality. myocardial infarction. and stroke in men with low testos-
terone levels. JA..\IIA. 2013 6:310(17): 1829 36. Erratum in: JAMA. 2014
Bogaui F. Bartalena L, Martino E. Approach to the patient wilh amiodarone- Mar 5;311(9):967. [P:\110: 24193080]
induced thyrotoxicosis. J Clin Endocrinol Metab. 2010 Jun:95(6):2529-35.
[PMI D: 2052590-1) Calcium and Bone Disorders
Cooper OS, Biondi B. Subclinical thyroid disease. Lancet. 2012 Mar i\1-Azem H, Khan AA. Hypoparathyroidism. Best Pract Res Clin Endocrtnol
24;379(9821):ll42-54. [P:\110: 22273398] :\1etab 2012 Aug;26(4):517-22. [P:\110: 22863393]
Demers LM, Spencer CA. Laboratory medicine practice guidelines: laboratory Bischoff-Ferrari H A, Wille It WC. Orav E J, et al. A pooled analysis of vitamin
support for the diagnosis and monitoring of thyroid disease. Clin D dose requirements for fracture prevention. N Engl J Med. 2012 Jul
Endocrinol (Ox(). 2003 Feb;58(2):138-40. [PMID: 12580927] 5:367(1),40 9. [PMJD, 22762317]
Devdhar M, Ousman YH, Burman KD. Hypothyroidism. Endocrinol Metab Holick ME Vitamin D deficiency. N Engl J Med. 2007 Jul 19;357(3):266-81.
Clin North Am. 2007 Sep;36(:l):595-615, v. [PMID: 17673121] [PMlD: 17634462]
Farwell AP. Nonlhyroidal illness syndrome. Curr Opin Endocrinol Diabetes Holick MF, Binkiey N, Bischoff Ferrari HA, el al. Evaluation, treatment and
Obes. 2013 Oct;20(5):478-84 . [PMlD: 23974778] prevention of vitamin D deficiency: an Endocrine Society clinical practice
Klubo Gwiewzinska J, Wartofsky L. Thyroid emergencies. Med Clin North guideline. J Cli.J1 Endocri.nol Metab. 2011 Jui;96(7):1911-30.[PMID: 21646368]
Am. 20L2 Mar;96(2):385-403. [PMID: 22443982] Marcocci C, Cetani F. Primary hyperparathyroidism. N Eng! J Med. 20 ll Dec
Pearce EN, Farwell AP, Braverman LE. Thyroiditis. N Eng! J Med. 2003 Jun 22;365(25) :2389- 97. [PMID: 22187986]
26;348(26):2646-55. Review. Erratum in : N Engl J Med. 2003 Aug Institute of Medicine. Dietary Reference Intakes lor Calcium and Vitamin D.
7;349(6):620. [PMID: 12826640] Washington, DC: National Academy Press, 2010.
Siegel RD, Lee SL. Toxic nodular goiter. Toxic adenoma and toxic multinodular ;\lational Osteoporosis Foundation. Clinician's Guide to Prevention and
goiter. Endocrinol Mctab Clin North Am. 1998 Mar;27(1):151-68. [PMJD: 9534034] Treatment of Osteoporosis. Washington, DC: Nalion al Osteoporosis
Stagnaro-Green A, Abalovich :\1, Alexander E, et al; American Thyroid Foundation, 2014.
Association Taskforce on Thyroid Disease During Pregnancy and Ralston SH. Paget"s disease of the bone. N Eng! J Med. 2013 Feb 14;368(7):644
Postpartum. Guidelines of the American Thyroid Association for the diag- SO. [PMID: 23406029]
nosis and management of lhy ro id disease during pregnancy and postpar- Sharma OP. Hypercalcemia in granulomatous d isorders: a clinical review. Curr
tum. Thyroid. 2011 Oct;21(10):108H25. [P:\1!0: 21787128] Opin Pulm Med. 2000 Sep:6(5):442-7. (PMID: 10958237]
Reproductive Disorders Shoback D. Hypoparathyroidism. N Eng! J Med. 2008 Jul 24;359(4):391- 403.
Bakalov VK. VanderhoofVH. Bondy CA, Nelson L\1. Adrenal antibodies detect [PMID: 18650515]
asymptomatic auto-immune adrenal insufficiency in young "'"Omen with Wolpowitz D. Gilchrest BA. The vitamin D questions: how much do you need
spontaneous premature ovarian failure. Hum Reprod. 2002 Aug;l7(8):2096- and how should you get it? JAm Acad Dermatol. 2006 Feb;S4(2) :301-17.
100. [PMID: 12151-143] [P\110: 16,143061]

72
Endocrinology and Metabolism
Self-Assessment Test
This self-assessment test contains one-best-answer multiple-choice questions. Please read these directions carefully
before answering the questions. Answers, critiques, and bibliographies immediately follow these multiple-choice
questions. The American College of Physicians is accredited by the Accreditation Council for Continuing Medical
Education (ACCME) to provide continuing medical education for physicians.

The American College of Physicians designates MKSAP 17 Endocrinology and Metabolism for a maximum of14 AMA
PRA Category 1 CreditsrM. Physicians should claim only the credit comm ensurate with the extent of their partici-
pation in the activity.

Earn "Instantaneous" CME Credits Online


Print subscribers can enter their answers online to earn CME credits instantaneously. You can su bmit your answers
using online answer sheets that are provided at mksap.acponline.org, where a record of your MKSAP 17 credits will
be available. To earn CME credits, you need to answer all of the questions in a test and earn a score of at least SO%
correct (number of correct answers divided by the total number of questions) . Take any of the following approaches:
:;.... Use the printed answer sheet at the back of this book to record your answers. Go to mksap .acponline.org,
access the appropriate online answer sheet, transcribe your answers, and submit your test for instantaneous
CME credits. There is no additional fee for this service.
:;.... Go to mksap.acponline.org, access the appropriate online answer sheet, directly your answers, and
submit your test for instantaneous CME credits. There is no additional fee for this service.
:;.... Pay a $15 processing fee per answer sheet and submit the printed answer sheet at the back of this book by
mail or fax, as instructed on the answer sheet. Make sure you calculate your score and fax the answer sheet.
to 215-351-2799 or mail the answer sheet to Member and Customer Service, American College of Physicians,
190 N. Independence Mall West, Philadelphia, PA 19106-1572, using the courtesy envelope provided in your
MKSAP 17 slipcase. You will need your 10-digit order number and 8-digit ACP ID number, which are printed
on your packing slip. Please allow 4 to 6 weeks for your score report to be emailed back to you. Be sure to
include your email address for a response. ·

If you do not have a 10-digit order number and 8-digit ACP ID number or if you need help creating a username and
password to access the MKSAP 17 online answer sheets, go to mksap.acponline.org or email custserv@acponline.org.

CME credit is available from the publication date of December 31, 2015, until December 31, 2018 . You may submit
your answer sheets at any tim e during this period .

73
Directions
Each of the numbered items is followed by lettered answers. Select the ONE lettered answer that is BEST in each case.

Item 1 Item 3
A 23-year-old woman is evaluated because of a 1-week his- A.31-year-old woman is evaluated following her recent
tory of palpitations. She also reports some heat intolerance discovery that she is pregnant at approximately 10 weeks'
and mild anxiety during t he last several weeks, but she oth- gestation . Medical history is significa nt for a prolactinoma
erwise feels well. She is in the firs t trimester of an otherw ise diagnosed 2 years ago during an evaluation for amenor-
uncomplicated first pregnancy. Her only medication is a rhea. At the time of diagnosis, her serum prolactin level
prenatal vitamin. was 184 ng/mL (184 and a 1.4-cm pituitary adenoma
On physical examination , she is afebrile, b lood pres- extendi ng above the sella was detected on MRl without
sure is 110 /72 mm Hg, pulse rate is 105 /m in, and respi- evidence of mass effect. She was treated with bromocrip-
ration ra te is 13/ min. BMI is 20. "The skin is warm and tine with return of regular menses. She disco ntinued the
moist. There is no proptosis or lid lag. Examina tion of the bromocript ine when she found that she w as pregnant. She
neck shows a d iffusely enlarged thyroid with an audible is currently without symptoms. She does not have new
bruit over both Jobes. Cardiopulmonary and abdomina l or severe headache. Medical history is otherwise u nre-
examinations are unrem arkable. Neurologic examina- markable, and her only current medication is a prenatal
t ion reveals a fine resting tremor of the hands a nd brisk multivitamin.
reflexes. On physical examination, vital signs are normal. Visual
Laboratory studies: fields are full to confrontation, and the remainder of her
Thyroid-stimulating hormone <0.008 examination is normal.
(0.008 mUl L) Which of the following is the most appropriate next step in
Free thyroxine (T4 ) 5.5 ng/dL (70.9 pmol/L) management?
Total triiodothyronine (T3) 400 ng/dL (6.2 nmol/L)
Thyroid-stimulating 4.5 (normal <1.3) (A) Check serum prolactin level
immunoglobulin index (B) Formal visua l field testing
(C) Repeat pituitary MRI
Which of the following is the most appropriate treatment? (D) Restart bromocriptine
(A) Methimazole
(B) Propylthiouracil Item 4
(C) Radioactive iodine A 48-year-old woman returns for a fo llow-up visit for man-
(D) Thyroidectomy agemen t of type 1 diabetes mellitus. She reports doing well
since the last visit. Overall, she believes that most of her
blood glucose levels are at goal, but is concerned about occa- ·
Item 2 sional episodes of hyperglycemia occurring in t he morning
A 34-year-old man is evaluated for episodic palpitations of before breakfast. She eats a bedtime snack every night that
8 months' duration. The palpitations last 5 to 10 minutes is not covered with mealtime insulin. Review of her blood
and then resolve spontaneously. They are usually associated glucose Jog demonstrates morning fasting blood glucose
with sweating an d anxiety. Medical history is significant'for values from 80 to 190 mg/dL (4.4-10.5 mmol!L). Her other
thyroidectomy for medullary thyroid carcinoma-diagnosed premeal and bedtime values range from 100 to 120 mg/dL
at 12 years of age. His father has also undergone thyroidec- (5.5- 6.7 mmol!L). She exercises two to three times per week
tomy for medullary thyroid cancer. His only medication is in the evening. Medical history is significant for hypelien-
levothyroxine. sion and hyperlipidemia.
On physical examination, blood pressure is 164/ Medications are insulin glargine, insulin lispro, rami-
92 mm Hg, pulse rate is 106/ min, and respiration rate is 12/ pril, simvastatin , and aspirin.
min. Auscultation of the heart reveals a regular tachycardia On physical examination, blood pressure is 130/72
without murm urs. The remainder of his examination is mm Hg and pulse rate is 67/ min. BMI is 24. The remainder
unremarkable. of the examination is unremarkable .
Laboratory studies show a 24- hour urine excretion of Results of laboratory studies show a hemoglobin
catecholamines of 310 h (1832.1 nmol/m 2/24 h) A1c level of 6.9% and serum creatinine level of 1.0 mg/ dL
and metanephrines of3400 h (17,238 nmol/24 h). Serum electrolytes are normal.

In addition to the presenting diagnosis, which of Which of the following is the most appropriate manage-
the following disorders is this patient most likely to ment of this patient's occasional fasting hyperglycemia?
develop? (A) Add insulin lispro at bedtime
(A) lnsulinoma (B) Add metformin
(B) Neurofibroma (C) Increase insulin glargine dose
(C) Primary hyperparathyroidism (D) Measure 3 AM blood glucose level
(D) Prolactinoma (E) Continue current regimen

75
Self-Assessment Test

Item 5 (C) Repeat measurement of antibodies to glutamic acid


A 70-year-old woman is seen for follow-up evaluation for decarboxylase 65 and islet antigen 2
possible Cushing syndrome. She presented with new -onset (D) Repeat measurement of fasting C-pept ide and glucose
diabetes mellitus and a 9.1-kg (20-lb) weight gain over th e levels
last 6 months. Medical history is otherwise unremarkable,
and she is currently taking no medications and has had no
exposure to exogenous glucocorticoids in the past year. Item 7 CJ
On physical examina tion, blood pressure is 160/90 A 43-year-old man is evaluated in the emergency depart-
mm Hg, pulse rate is 80/min, a nd respiration rate is 12/ ment fo r the "worst headache of my life." ll occurred sud-
min. BMI is 30. Facial plethora, central obesity, and bilat- denly without warning. He has had mild headaches that
eral supraclavicular fat pads are noted. There are viola- come and go over t he past 3 years, but nothing this severe.
ceous abdominal striae measuring 1 em wide and multiple Soon after the headache began, he lost vision in his lefl eye.
ecchymoses on th e extremities. and the vision in his right eye became blurry. lle vomited
Initial laboratory studies show a serum cortisol level of twice in the emergency department. His medical history is
9 11g/dL (248.4 nmol!L) following a 1-mg dose of dexameth- significant for progressive erectile dysfunction and loss of
asone the night before, and a 24-hour urine free cortisol libido over the past 3 years.
level that is greater than 3 times the upper limit of normal, On physical examination, temperature is 37.4 oc (99.3 °F),
which is confirmed on a second measurement. A plasma blood pressure is 156 92 mm llg, pulse rate is 1041min, and
adrenocorticotropic hormone (ACTH) level is undetectable. respiration rate is 16 min. BMI is 28. He has loss of vision
in his left eye and in the upper quadrants of his right eye.
Which of the following is the most appropriate diagnostic He also has left eye ptosis. Other cranial nerves a re intact.
test to perform next? Strength and sensation in all extremities are normal as are
(A) CT scan of the adrenal glands his speech and gait.
(B) Inferior petrosal sinus sampling ct of the head shO\·vs acute pituitary hemorrhage. Pitu-
itary MRI shows a 3.1 x 2.5 x 2.2-cm pituitary mass with
(C) Late night salivary cortisol measurement
central hemorrhage. The mass compresses the optic chiasm
(D) MRI of the pituitary gland and the left cavernous sinus.
After administering high-dose glucocorticoids, w hich
Item 6 of the follow ing is t h e m ost appropriate im m ediate
A 44-year-old man is evaluated for management of type 2 man agement?
diabetes mellitus. He was diagnosed with diabetes 6 months (A) Assess pituitary function
ago after being 1ldrnitted to the hospital vvith diabetic keto-
(B) Repeat imaging in 2 weeks
acidosis. He was discharged from the hospital on a basal and
preprandial insulin regimen. Medications are regular insulin (C) Urgent transsphenoidal pituitary decompression
before meals and neutral protamine Hagedorn (NPH) insulin (D) Whole brain external beam radiation
at bedtime. He completed diabetes educat'ion and nutrition
classes and has been adherent with lifestyle modifica tions.
His insulin doses have been decreased gradually over the last Item 8
4 to 5 months. His most recent hemoglobin level is 6.7%. A 42-year-old woman is evaluated d uring an annual
Blood glucose values from his log book average 130 mg/dL physical examination. She feels well. She has no per ti-
(7.2 mmol/ L) . nent personal or family medical history, and she takes no
On physical examination, temperature is 37.2 oc (99.0 °F), medications.
blood pressure is 128/68 mm Hg, a nd pulse rate is 72/ min. On physical examination, vital signs are normal. Pal-
BMI is 30. His physical examination is unremarkable. pation of the thyroid reveals a possible nodule in t he right
Laboratory studies at t he time of hospital admission: lobe th at is not mobile with swallowing. The remainder of
Glucose, fasting 825 mg/dL (45.8 mmol!L) the gland is unremarkable, and there is no palpable cervical
Antibody to glutamic Negative lymphadenopathy. Other physical examination fi nd ings
acid decarboxylase 65 are normal.
(GAD-65) Laboratory studies revea l a serum thyroid-stimulating
Antibody to islet antigen Negative hormone level ofl.7 (1.7 mUl L).
2 (IA-2) Ultrasound of the neck shows a right 1.5-cm hypoechoic
C-peptide, fasti ng 0 .5 ng/ mL (0.16 nmol/ L) nodule with internal microcalcifications.
Normal range: 0.8-3.1 ng/ mL
(0.26-1. 03 nmol!L) Which of the following is the most appropriate next step in
management?
Which of the following is the most appropriate n ext step in (A) CT with contrast of the neck
his management? (B) Fine-needle aspiration of the nodule
(A) Discontin ue current insulin regimen, initiate sliding- (C) Levothyroxine therapy
scale insulin (D) Measurement of serum t hyroglobulin level
(B) Discontinue insuli n, initiate metformin (E) Thyroid scan with technetium

76
Self-Assessme nt Te st

Item 9 tw ice. Her las t m enstr ual period was 4 months ago. She
An 18-year-old woman is evaluated for primary amenor- also notes low libido and dyspareunia. She has not had
rhea. Her cognitive fu nction is normal, and she is not sex- weight changes, const ipat ion, ha ir loss or h irsu tism , or
ually active. Her personal and fam ily medical history is skin changes.
unremarkable. She lakes no medications. Her medical history is significant for primary hypothy-
On physical examination, temperature is 36.1 oc (97.0 °F), roidism and bipolar disorder. Medications are levothyrox-
blood pressure is 110/70 mm Hg, pulse rate is 72 /min, and ine, lithium, and risperidone. She reports that she has been
respiration rate is 16/ min; BMI is 20. Her height is 147 em stable on these medications for a few years and feels well.
(58 in). Physical examination and secondary sex character- She plans to discuss her medications with her psychiatrist
istics are normal, with Tanner stage IV breast and pubic hair prior to pregnancy.
development. On p hysical examinat ion, blood pressure is 118/72 mm
Pregnancy t esting is negat ive. On subsequent lab- Hg and pulse rate is 82/ mi n. BMI is 24. The thyroid is normal.
oratory studies estradiol level was undetectable, serum Visual fields are intact.
follicle-stimu lating horm one level is 72 m U/ mL (72 Laboratory studies:
U/ L), and serum luteinizing hormone leve l is 46 m U/ mL Follicle-stimulating hormone 1.3 mU/ mL (1.3 U/L)
(46 U/ L). Luteinizing hormone 2.0 mU/ mL (2.0 U/L)
Prolactin 102 ng/ mL (102 J.!giL)
Which of the following is the most appropriate manage- Thyroid-stimulating hormone 1.1 ).lU/mL (1.1 mUlL)
ment?
(A) Initia te estrogen and progestin therapy Which of the following is the most likely cause of her
(B) Measure serum prolactin hyperprolactinemia?
(C) Measure thyroid-stimulating hormone (A) Hypothyroidism
(D) Perform pituitary MRI (B) Lithium
(C) Pituitary adenoma
Item 10 (D) Risperidone
A 32-year-old man is evaluated for a 1-week history of
severe neck pain. He also has heat intolerance, palpitations, Item 12
and insomnia. Medical history is significant only for a viral A 55-year-old woman is evaluat ed for a new-patient
upper respiratory tract infection 3 weeks ago. He takes no visit. Medical history is signiftcant_for an eating disorder.
medications. Although she has maintained a normal weight fo r the past
On physical examination, he appears an.;d ous and is 20 years, she notes that prior to that time her weight wou ld
sweating. There is no proptosis or lid lag. Examination of fluctuate in a range correlating with BMis of 17 to 19. She
the thyroid reveals a normal-sized gland that is very tender has otherwise been healthy and currently feels well. .
to palpation. There are no thyroid nodult:s. The heart rate is is postmenopausal and a never-smoker. Family h istory is
regular but tachycardic. The lungs are clear. significant for postmenopausal osteoporosis in her mother.
Laboratory studies: Her medications are over- the-counter calcium and vitamin
Thyroid-stimulating <0.008J.!U/mL (0.008 mUlL) D supplements.
hormone On physical examiJ1ation, temperature is 36.3 oc (97.3 °F),
Free thyroxine (T) 3.2 ng/dL (41.3 pmol/L blood pressure is 137/81 mm Hg, pulse ra te is 76/ min, and
Total triiodothyronine (T) 310 ng/ dL (4.8 nmol/ L) respiration rate is 11/ min. BMI is 21. She has mild thoracic
TI1yroid-stimulatin g <1.3 (normal, <1.3) kyphosis but no skeletal tenderness. TI1e remainder of the
immunoglobulin index examination is unremarkable.
24-Hour radioactive iodine 5% (low) Results of laboratory studies are significan t for a serum
uptake calcium level of9.1 mg/dL (2.3 mmol/L) and 25-hydroxyvi-
tamin D level of 40 ng/ mL (99.8 nmol!L); thyroid function
Which of the following is the most appropriate treatment? studies are normal.
Dual-energy x-ray absorp tiometry (DEXA) scan shows
(A) Methimazole T-scores of - 1.8 in the femoral neck and - 1.9 in the lumbar
(B) Metoprolol spine. Ten-year fracture risk using the Fracture Risk Assess-
(C) Propylt hiouracil ment Tool (FRAX) is 6.9% for major osteoporotic fracture
(D) Radioactive iodine and 0.7% for hip fracture. Plain radiographs of the spine
show no evidence of compression fracture.
Which of the following is the most appropriate manage-
Item 11 ment of this patient?
A 30-year-old woma n is evaluated for amenorrhea . She (A) Begin raloxife ne
a nd her husband are in terested in pregnancy in t he
next year, and t hey are concerned that they will not be (B) Repeat DEXA scan in 2 years
able to conceive. Her me nses became irregular about (C) Replace calcium with cholecalciferol
2 years ago. In the past 12 months, she has had menses (D) Start bisphosphonate therapy

77
Self-Assessment Test

CJite m 1 3 weekly, 6 weeks ago. Medical history is significant for vit-


/1. 64 year old man in the lCU is evaluated because of abnor iligo and chronic fatigue. Medicat ions are vitamin D2 and
mal thyroid function tests. He was admitted 3 days ago calcium carbonate.
for commun ity-acquired pneumonia requiring in tubation, On physical examination, temperature is 36.1 oc (96.9 °F),
mechanical ventilation. int raveno us fluids. and dopa mine blood pressure is 132/71 mm Hg, pulse rate is 83/min , and
support for his blood pressure. respiration rate is 12/ min. BMl is 19. The remainder of her
On p hysical examina tion . tem perature is 38 .8 oc examination is unremarkable.
(101.8 °F), blood pressure is 95 60 mm 1-!g, and pulse rate Laboratory studies:
is 130. min. The skin is warm and dry. There is no proptosis. Calcium 9.1 mg/dL (2.3 mmol!L)
Examination of the neck shows a normal sized thyroid Creatinine 0.9 mg/dL (79 .6 1-lmol/L)
w ithout nodules. Ca rdiovascular examination reveals reg Parathyroid hormone 101 pg/mL (101 ng/ L)
ular tachycardia. On neurologic examination. reflexes are 25-Hydroxyvitamin D, 7 ng/m L (17.5 nmol/ L)
slightly delayed. after 6 weeks
The serum thyroid stimulating hormone (TSll) level is 24-Hour urine calcium 150 mg/24 h (3.7 mmol/24 h)
0.1 11U mL (0.1 mU L) , the serum free thyroxine level (T)
is 0.9 ng. tiL (11.6 pmol Ll. and the serum total triiodothy Which of the following is the most appropriate next step in
ronine 0 ) level is SO ng elL (0.8 nmol L) . management?
Which of the following is the most likely cause of this (A) Parathyroid sestamibi scan
patient's abnormal thyroid function?
(B) Refer for parathyroidectomy
(II.) Euthyroid sick synd rome (C) Switch to vitamin D3 (cholecalciferol)
(B) Graves disease (D) Tissue transglutaminase antibody testing
(C) Hashimoto thyroiditis
(D) Subacute thyroiditis
Item 16
A 54-year-old woman is evaluated because of fatigue.
Item 14 Although she follows a daily 1400-kcal diet and exercises
A 43-year-old man is evaluated during a follow-up visit for 3 to 4 nights per week for 30 minutes, she has gained
management of type 1 diabetes mellitus. He was diagnosed at 2.3 kg (5.0 lb) in the last month. She has hypercholesterol-
18 years of age and has multiple chronic complications from emia requiring statin therapy. Her mother was diagnosed
his diabetes, including end -stage kidney disease requiring with hypothyroidism shortly after the birth of her last child.
hemodialysis, gastroparesis, frequent hypoglycemia w ith On physical examination, blood pressure is 145/90 mm Hg,
hypoglycemic unawareness, painful peripheral neuropathy, pulse rate is 80/min, and BMI is 25. The skin is d.ty. The thyroid
and proliferative retinopathy. The patient uses an insulin is mildly enlarged with a diffusely nodular texture. No discrete
pump and a contin uous glucose monitoring, system to man- t hyroid nodules are palpated. Reflexes are normal.
age his diabetes. He is adherent with his regimen and per- Laboratory studies:
forms multiple fingers tick blood glucose measurements with Thyroid-stimulating hormone 6.5 (6.5 mUlL)
values ranging from 65 to 250 mg/ dL (3.6-13.9 mmol!L). His (TSH)
most recent hemoglobin A1c level is 7.5%. Free thyroxine (T) 0.9 ng/dL (11.6 pmol! L)
Which of the following is the most appropriate next step in Thyroid peroxidase antibody Positive
the management of this patient?
Similar results fo r TSH and T4 were obtained 4 months
(A) Alter insulin pump settings to attain a hemoglobin A1 c ago.
goal of less tha n 7. 0%
Which of the following is the most appropriate next step in
(B) Alter insulin pump settings to decrease the insulin management?
doses
(C) Discontinue the insulin pump, start subcutaneous (A) Initiate levothyroxine therapy
insulin injections (B) Measure thyroid-stimulating immunoglobulins
(D) Start gabapentin for treatment of painful peripheral (C) Repeat serum TSH measurement in 12 months
neuropathy (D) Schedule thyroid radioactive iodine uptake and scan

Item 15 Item 17
A 55-year-old woman is seen in follow- up fo r low bone A 74-year-old woman is evaluated for a diagnosis of primary
mass and vitamin D deficiency. Cortical bone thinning was hyperparathyroidism made after an elevated serum calcium
noted on radiographs of her right ankle following a fall 3 level was incidentally discovered on laboratory studies. She
months ago. Subsequent eva luation included a dual-energy has no symptoms associated with hypercalcemia. Medical
x- ray absorptiometry (DEXA) scan showing osteopenia. Her history is significant for hypertension and chronic kidney
serum 25-hydroxyvitamin D level is 4 ng/ mL (10 nmol!L) . disease. Her only medication is amlodipine. Sh e has never
She was star ted on 50,000 U of vitamin D2 (ergocalciferol) smoked.

78
Self-Assessment Test

On physical examination, temperature is 36.8 oc (98.3 °F), hot flushes and fatigue. She has noted galactorrhea. She
blood pressure is 134/87 mm Hg, pulse rate is 92/ min, and began having headaches 2 years ago. In addition, she notes
respiration rate is 14/min. BMl is 27. The remai nder of her blurry peripheral vision .
examination is unremarkable. The rest of her medical history is unremarkable. She
Laboratory studies: takes no medications.
Calcium 11.3 mg/dL (2.8 mmol!L) On physical examination, blood pressure is 112/72
Creatinine 1.3 mg/dL (114.9 J.l.mol/ L} mm Hg and pulse ra te is 68/ min. BMI is 21. White milky
Parathyroid hormone 76 pg/ ml (76 ng/L} substance is expressed from her breasts bilaterally. Ocular
Estimated glomerular 40 mL/mi n/1.73 m2 movements and cranial nerves are intact. 1here are no stig-
filtration rate mata of Cushing disease or acromegaly.
Laboratory studies:
Dual-energy x- ray absorptiometry (DEXA) scan shows Cortisol8 A'-'1 after 1 mg of 16 J.l.g/dL (441.6 nmol!L)
a T-score of - 1.3 in the right femoral neck. Her Fracture dexamethasone the night
Risk Assessment Tool (FRAX) score indicates a 2.1 'Yo 10-year before
probability of hip fracture and 17% 10-year probability of Estradiol <32 pg/ mL (117.4 pmol/L}
any fracture. Follicle-stimulat ing 11 mU/mL (1.1 U/ L}
Which of the following is the most appropriate therapy to hormone
recommend to this patient? Luteinizing hormone 0.8 mU/mL (0 .8 UJL)
Prolactin 472 ng/ mL (472 !1g/L}
(A) Alendronatc Thyroid stimulating 1.1 J.l.Uim L (1.1 m Ul L}
(B) Cinacalcet hormone
(C) Parathyroidectomy
MRI shows a 2.4-cm pituitary tumor that elevates the
(D) Clinical observation optic chiasm and surrounds the left carotid artery.
Which of the following is the most appropriate treatment?
Item 18 (A) Cabergoline
A 74-year-old woma n is evalua ted because of new-onset (B) Octreotide
anxiety and insomnia. For the last 6 weeks, she has been (C) Radiation
waking up multiple times each night. She does not have
(D) Surgery
heat intolerance, change in bowel habits, palpitations, or
dyspnea on exertion. She takes no medications.
On physical examination, blood pressure is 125/68 mm Item 20
Hg and pulse rate is 89/ min. BMl is 18. There is no proptosis
or lid lag. Exam ination of the thyroid reveals a 1.5-cm fi rm A 74 year old woman is evaluated in the emergency depart Cl
nodule in the left lobe that moves upward with swallowing. menl for several hours of altered mental status. She is from
A fine resting hand tremor is present out -of state and is visiting \Yith relatives. One of her young
Laboratory st ud ies reveal a serum thyroid-stimula t- relatives was recently ill with gastrointestinal symptoms.
ing hormo ne level of 0.05 J.l.U/ mL (0.05 mU / L} , a serum The patient developed anorexia 3 days ago and vomiting
free thyrox ine (T) level of2.9 ng/ dL (37 .4 pmoi/L} , and 2 days ago. She has been unable to tolerate any liquid or solid
a serum total triiodothyro ni ne (T3) level of 250 ng/ dL foods for the last 24 hours. Medica l history is significant for
(3.8 nmoi/ L). IJ. pe 2 diabetes mellitus. hypertension, hyperlipidemia, and
Ultrasound of the neck shows two thyroid nodules, a hypothyroidism. 'v1cdications are aspirin, lisinopril. glime
1.5-cm nodule in the right lobe and a 2.0-cm nodule in the piridc.le\·othyroxine, and atorYastatin. Her last dose ofmed
left lobe. ications was 48 hours ago.
On physical examination, her temperature is 37.5 oc
Which of the following is the most appropriate next step in (99.5 °F). blood pressu re is 115 65 m m Hg, and pulse
management? rate is 95 min . She is arousable hut confused. Mucous
membranes are dry. Her neck is supple. Cardiac exam
(A) Fine-needle aspirat ion of both thyroid nodules
ination reveals no murmurs. Her chest is clear to auscul
(B) lnitia tion of methimazole tation. Bowel sounds are present. and mild tenderness
(C) Radioactive iodine (123!) uptake and scan of the thy- to palpation is noted throughout the abdomen. ·n1ere is
roid no rebound or guarding. There arc no focal neurologic
(D) Total thyroidectomy deficits.
Laboratory studies are pending.
Which of the following is the most likely cause of this
Item 19 patient's altered mental status?
A 34-year-old woman is evaluated for amenorrhea. head-
(A) Cerebrovascular accident
ache, and fatigue. She reports that from the time of menarche
until 2.5 years ago, her menses were regular and predictable. (B) I Iypoglycemia
Two and a hal f years ago, her menses became irregular and (C) Hypothyroidism
then stopped completely 6 mon ths ago. She has had a few (D) Statin toxicity

79
Self-Assessment Test

Item 21 with a round face. She has terminal hairs on her chin, upper
A 38-year-old woman is evalua ted because of a 3-week lip, chest, and back. Mild facial acne, central obesity, and a
history of palpitations. She notes that her heart '·races" few wide purple striae on the back of her arms are also noted.
at night and after minimal exertion. She also report::: She has supraclavicular fat. Her skin has psoriatic plaques.
in to lerance but has no change in bowel habits or menses. Muscle strength in the upper and lower extremities is 4/5.
On physical examination, the patient is restless and Which of the following is the most likely diagnosis?
has pressured speech. Temperature is 36.8 oc (98.2 °F),
blood pressure is 130/ 60 mm Hg, pulse rate in 110/min , and (A) Adrenocortical carcinoma
respiration rate is 12/min. Her skin is warm and moist, and (B) Cushing disease
a bilateral hand tremor is present. There is no proptosis or (C) Ectopic adrenocorticotropic hormone p roduction
lid lag. The thyroid is enlarged without nodules or bruits. (D) Iatrogenic Cushing syndrome
Serum thyroid-stimulating hormone level is 0.08!-!U/ mL
(0.08 mUl L), and the serum free thyroxine (T) level is 1.7
ng/ dL (21.9 pmol/L).
Item 24 C]
Which of the following is the most appropriate next step in A 20 year-old woman is evaluated in the emergency depart-
management? ment for polyuria. polydipsia. polyphagia. and an uninten-
(A) Measure serum triiodothyronine (T) level tional 5.4-kg (11.9-lb) weight loss over the past month.
She has had increasing lethargy over the last 2-l hours. Her
(B) Measure serum thyroid peroxidase antibody titer
medical history and family history are unremarkable. She
(C) Repeat thyroid function tests in 6 weeks takes no medications.
(D) Schedule ultrasound of the neck On physical examination . temperature is 37.5 oc
(99 .5 °f) . blood pressure is 98 52 mm llg. pulse rate is 120
min. and respiration rate is 30 min. BMl is 17. She is lethar
Item 22 gic with dry mucous membranes. tachypnea. and tachy
A 34-year-old woman is evaluated for a diagnosis of hyper- cardia. Chest auscultation is clear. Abdominal examination
calcem ia after presenting to the emergency departmen t 3 shows difruse mild tenderness and normal bowel sounds.
days ago fo r treatmen t of a kidney stone. She presented with There is no rebound tenderness or guard ing with palpation.
severe right fla nk pain with nausea and vomiting. A 2-mm Laboratory studies:
kidney stone was identified in the right ureter by ultraso- Hemoglobin 17 g dl.(l70g L)
nography t hat passed spontaneously; multiple additional l.eukocyte count 14.200 IlL (14.2 X 10QIL)
intrarenal calcifications were noted to be present. Labora- Blood gases. a rterial
tory studies at t hat time showed normal kidney func tion pH 7.25
and a serum calcium level ofll.S mg/ dL (2.9 mmol / L). Med- PC02 21 mm Hg (2.8 kPa)
ical history is otherwise significant for hypertension and Creatinine 1.3 mg dL (111.9!-!mol L)
sarcoidosis. Her only medication is hydrochlorothiazide. Electrolytes
On physical examination. temperature is 36.8 °C (98.2 °f ), Sodium 130 mEq L (1:30 mmol L)
blood pressure is 138/87 mm Hg. pulse rate is 89/ min. and Potassium :3.0 mEq L (3.0 mmol L)
respiration rate is 12. B\-1"! is 32. 1he remainder of the exam Chloride 99 mEq L (99 mmol L)
ination is unremarkable. Bicarbonate 9 mEq L (9 mmol L)
Laboratory studies are significant for a parathyroid Glucose 620 mg dL (34 .4 mmol L)
hormone level of 4 pg/ mL (4 ng/L) . Lactic acid 8 mg dL (0.89 mmol L)
Which of the following is most likely responsible for caus- Intravenous 0.9% saline is initiated through a central
ing this patient's hypercalcemia? venous catheter.
(A) Calc ium-sensing receptor mutation An e lectrocardiogram shows si nus t<lchycardia 120, min.
Chest radiograph is normal.
(B) Elevated 1,25-dihydroxyvitamin D levels
(C) 25-1lyd roxyvitamin D level Which of the following is the most appropriate next step
(D) Tl1iazide- induced renal calcium reabsorption in the man agem ent?
(A) Administer intravenous ceft riaxone
(B) Administer intravenous potassium chloride
Item 23
(C) Ad minister intra\·enous sodium bicarbonate
A 40-yea r-old woman is evaluated for amenorrhea of 4 (D) Initiate intravenous insulin therap)
months' duration. She has had weight gai n. facia l hair,
alopecia, and debilitating fatigue. Her medical history is
significant for psoriasis. She seems to be gain ing weight in
her tace, abdomen, and neck. She also bruises easily. Her Item 25
only medication is clobetasol fo r psoriasis. A 65-year-old woman is evaluated following a recent diag-
On physical examination, temperature is 37.6 oc nosis of osteoporosis discovered on a screening dual -energy
(99.7 °f). blood pressure is 148/ 90 mm Hg, pulse rate is 88/ x-ray absorptiometry (DEXA) scan that showed T-scores
min, and respiration rate is 12/m in. BMl is 38. She is obese of -2.5 at the femoral neck and - 2.7 at the hip. Overall she

80
Self-Assess m ent Test

feels well, although she notes a S-cm (2-in) loss of height st riae measuring 8 to 12 mm wide are noted on his u pper
over the past 15 years and a 2.3-kg (5 -lb) weight loss over a rms and abdomen. T11ere is 1+ bilateral lower extremity
the last year. She is poslmenopausal. Medical history is edema. Multiple ecchymoses and acanlhosis n igrica ns
unremarkable, and she is a never-smoker. Family history a re presen t.
is negative for osteoporosis or non trau matic fraclures. She Laboratory studies:
takes no medications. Adrenocorticotropic <5 pg/ mL (l.l pmoi/L)
On physical examinat ion, temperature is 36.8 oc hormone
(98.2 °F), blood pressure is 144/ 68 mm Hg, pulse rate is 92/ 24-Hour urine cortisol
min, and respiration rate is 14/ min. BMI is 22. The remainder excretion
of the examination is unremarkable. Ini tial measurement 280 J.!g/24 h (771.6 nmol/ 24 h)
Laboratory studies are signi ficant for a normal basic Repeat measurement 300 llg/ 24 h (826.7 nmol/24 h)
met abolic profile and complete blood count. Serum calcium Cortisol, serum 46 1-lg/dL (1269.6 nmol! L)
is 9.5 mg/dL (2.4 mmol/ L) and serum phosphorus is 3.8 mg/ Urine
dL (1.2 mmol/L). Serum 25-hydroxyvitamin D level is 32 ng/ Calecholamines 40 llg/ m 2/ 24 h (236.4 nmollm 2/
dL (79.9 nmol / L) . 24 h)
Which of the following tests is indicated prior to initiation Metanephrines 1000 llg/ 24 h (5070 nmol/24 h)
of pharmacologic therapy?
CT scan of the abdomen with and withou t contrasl
(A) Serum and urine markers of bone turnover reveals a 5.6-cm heterogeneous right ad renal mass with
(B) Serum estradiol level focal areas of calcifica lions and hemorrhage. T11e density of
(C) Serum parathyroid hormone level the mass is SO Hounsfield units, and the contrast was hout
at 10 minutes is 20%.
(D) Serum thyroid -stimulating hormone level
Which of the following is the most appropriate next step in
the management of this patient's adrenal mass?
Item 26
(A) Chemotherapy
A 24-year- old woman is evaluated for excessive men-
strual bleed ing. She was recently diagnosed with polycys- (B) Fine- need le biopsy
ti c ovary syndrome during an evalua tion for h irsulism. (C) Radiation therapy
Menarche occurred at age 11 years, and she has always (D) Surgical excision
had irregular menses occurring approximately every 60
days. However, her periods over the past year have been
associated witl'i. heavy bleeding that is increasi ngly both- Item 28 C]
ersome. Medical history is otherwise unremarkable. and A 47 year old man presents to the emergency department
she takes no medications. She curren tly does not desire wi th weakness and shaki ness. l ie has a long history of a leo
fertility. hoi abuse but significantly decreased his usual daily a lcohol ·
On physical examination, she is afebrile. Blood pres- intake over the past week because of gastroin testinal upset.
sure is 110/70 mm Hg, pulse rate is 78/min, and respirat ion nausea. and diarrhea. Medical histor) is o therwise unre
rate is 14/ min. BMl is 32. Excess terminal hair growth is markable, and he takes no med ications.
present on the upper lip, chin, and chest. T11e physical On physical exa mina tion. the pa tient is awake and
examination is otherwise normal. orien ted bu t trem u lous. Mucous membranes are dry .
A urine pregnancy tesl is negative. Temperature is 37.3 oc (99.1 °F), blood pressu re is 139 76
mm Hg, pulse rate is 101 min, and respirat ion rate is 15
Which of the following is the most appropriate therapy for
min. BMI is 19. Cardiopulmonary examination is unre
this patient?
ma rka ble. The abdomen is ditrusely tender to palpation .
(A) Combined oral contraceptive pills Periodic spon ta neous twit ching is no ted in the major
(B) Levonorgestrel in trauterine system muscle groups.
Laboratory stud ies arc significant for a ser um cal
(C) Metform in
cium level of 6.5 mg dL (1.6 mmol L) , serum albumin
(D) Periodic progest in w it hdrawal level of 2.6 g dL (26 g L), serum potassium of 3.4 m Eq L
(3 .4 mmol L). and normal kidney fu nction sludies. Seru m
Item 27 pa rathyroid hormone and 25 hydroxyvi ta min D le,·els
are pending.
A 55-year-old man is evaluated for abdominal fullness and An electrocardiogram shows pro longatio n of the QT
nausea of 2 weeks' duration. He has no vomiting or fever. interval.
One month ago, he was diagnosed with type 2 diabetes
mellitus. He reports an unintentional weight loss of 5 kg (11 Which of the following is the most appropriate next diag-
lb) over the past month, generalized weakness, and poor n ostic test t o evalu ate this patient's hypocalcemia?
appetite. Metform in is his only medication. (A) 1.25 Di hydroxyvitamin D level
On physical examina tion , blood pressure is 158/90
mm Hg and pu lse rate is 90 / min. BMI is 29 . His face (B) 24 Hour uri ne calciu m excretion
is round and red. A do rsocervical fat pad is present. (C) Ionized calcium level
His abdome n is distended. but non tender. Violaceous (D) Magnesium level

81
Self-Assessment Test

Item 29 (C) Serum ferritin level and transferri n satu ration


A 52-year-old woman presents for follow-up evaluation (D) Testicular ultrasound
after being diagnosed w ith type 2 diabetes mellitus 6 weeks
ago. Her initial hemoglobin A1c level was 8.0°1.•. Management
at this time is with lifestyle modifications. She has worked Item 31
closely with a diabetes educator and a nutritionist since her A 59-yea r-old man is evaluated for hypercalcemia. He was
diagnosis. She has lost 3.2 kg (7 !b) by making changes to her recently diagnosed with mu ltiple myeloma. He does not
diet and activity level. Review of her blood glucose log for have ano rexia, nausea, constipation, polydipsia , polyuria,
the past 2 weeks shows preprandial blood glucose values or confusion. Medical history is o therwise unremarkable,
in the 150 to 160 mg/dL (8.3-8.9 mmol! L) range and several and he takes no medications.
2- hou r postprandial blood glucose values of190 to 200 mg/dL On physical examination, temperature is 36.4 oc
(10.5-11.1 mmoi!L). Her only other medical problem is (97.5 °F), blood pressure is 134/ 80 mm Hg, pulse rate is 80/
hypertension for wh ich she takes lisinopril. min, and respiration rate is 12/min. BM! is 30. 1he remainder
On physical examination. blood pressure is 125/70 mm of his physical examination is norma l, and no weakness is
Hg and pulse rate is 74/min. BMI is 28. There is no evidence noted on neurologic examina lion.
of diabetic retinopathy. She has normal monofi lament and Serum calcium level is 10.8 mg/dL (2.7mmoi/ L).
vibratory sensation in her extremities.
Which of the following is the most appropriate next labora-
Except for her blood glucose parameters. basic labo-
tory test for evaluating this patient's hypercalcemia?
ratory studies obtained at the time of her in itial diagnosis
were normal. (A) 1.25-Dihydroxyvitamin D level
In addition to continuing lifestyle modifications, which of (B) Ionized calcium level
the following is the most appropriate management for this (C) Parathyroid hormone level
patien t's diabetes? (D) Parathyroid hormone- related protein level
(A) Initiate dapagliflozin
(B) Initia te glipizide
Item 32
(C) Init iate metformin
A 37-year-old man is evaluated for a 2-yea r history of low
(D) Initiate sitagliptin libido, loss of morn ing erections: fa tigue, and decreasing
muscle mass. His medical history is otherwise unremark-
Item 30 able. He ta kes no medications.
On physical examination. vital signs are normal. BMI
A 28-year-old man is eval ua ted for fatigue and erectile is 35. The remainder of the examination. including genital
dysfunction. His symptoms have been progressive over examination. is normal.
the past year. He notes decreased libido and reports loss
of morning erections. He also feels tin;d. has d ifficulty Laboratory studies:
concentrating. and notes diffuse joint aches. He believes Follicle-stimulating hormone 12 m U/ mL (12 U/ L)
he has less strength and has had to decrease his level of Luteinizing hormone 10 mU/ mL (10 U/ L)
exercise. Prolactin Normal
Medical history is unremarkable. He had normal Morning testosterone (total) 148 ng/dL (5.1 nmoi!L)
puberty and normal grovvth. He takes no medications. Confirmatory morning 137 ng/dL (4.7 nmoi!L))
On physical examination. temperature is 37.4 oc (99.3 °F). testosterone (total)
blood pressure is 108/72 mm Hg. pulse rate is 68/ min. and Thyroid-stimulating hormone Normal
respiration rate is 14/ min. BMI is 23 . ·n1e liver edge is pal - A pituitary MRI is normal.
pable 4 em below the costal margin. 1he penis is normal.
and t he testes are normal volume but soft and freely mobil e Before initiating therapy for tl1is patient, which of the fol-
w ithout masses. Visua l fields are intact. lowing should be determined?
Laboratory studies: (A) Bone mineral density
Follicle-stimulating hormone 3.0 m U/ mL (3.0 U/ L)
(B) Desire for fertility
Luteinizing hormone 2.2 mU/ mL (2.2 U/ L)
Prolactin 12 ng/ mL (12 11g/ L) (C) Fasting plasma glucose level
Testosterone. total (8 AM) 178 ng/ dL (6.2 nmoliL) (D) Scrotal ultrasound
Testosterone. total (8 AM) . 162 ng/dL (5.6 nmol/ L)
repeated
1hyroid stimulating hormone 2.3!1U/ mL (2.3 mUl L) Item 33 CJ
:\ .'55 ) ear-old woman is evaluated in the emergency
Pituitary MRl is normal. department because of a I week histor) of palpitations.
Which of the following is the most appropriate next step in chest pain. shortness of breath. diarrhea. and weight loss.
management? \ledical history is significant for an episode of chest pain
·l weeks ago: the chest pain was e\"aluated with an exer
(A) Begin testosterone replacement therapy cise stress test." hich was posit he. A subsequent cardiac
(B) Karyotyping catheterization vvas negative ror epicardial coronary artery

82
Self-Assessment Test
------------------------------

1'1'1 disease. She also has a history of Graves disease treated (C) Continue curren t treatment regimen
1.1.11 with daily methimazole: she stopped this medication on (D) Decrease calcium supplementat ion
CONT the day of her catheterization and has not restarted it since
that time.
On physical examination, she is restless and confused. Item 35
Temperature is 38.9 oc (102.0 °f), blood pressure is 175
A 40-year-old man with type 1 diabetes mellitus presents to
94 mm llg. pulse rate is 135/ min and regular. and respira -
the office. He seeks advice on his diabet es management as
tion rate is 20/min. 1l1e skin is warm and moist. "!here is
he intensifies his exercise routine in preparation fo r partic-
mild proptosis with scleral injection . Examination of the
ipation in a 10-K race. He reports prolonged hypoglycemia
thyroid reveals a diffusely enlarged gland that is non tender
during intense exercise, despite eat ing a meal prior to the
to palpation. A thyroid bruit is heard. Abdominal examina
activity. His insu lin regimen is insulin glargine and insulin
tion is unremarkable. There is edema of the lower extremi-
glulisine. His most recent hemoglobin A1c level was 7.0%.
ties to the mid lower leg. On neurologic examination, she is
oriented to place but not to time, as she does not knmv the Which of the following is the most appropriate manage-
correct year. ment of this patient's hypoglycemia on the days that he
Laboratory studies: exercises?
Leukocyte count 10,500 ).IL (10.5 X lOQ L) (A) Decrease meal- time insulin glulisine dose prior to
Creatinine 1.3 mg dL (115 ).!mol L) exercise, continue insulin glargine dose
Thyroid stimulating <0.008 ).IU mL (0.008 mUl L)
(B) Discontinue insulin glargine, continue insulin glulis-
hormone
Free thyroxine (T 1) 7.5 ng. dL (96 .7 pmol L) ine dose
(C) Increase meal-time protein prior to exercise, continue
current insulin doses
Which of the following is the most likely diagnosis?
(D) Switch insulin glulisine to a.sliding-scale regimen,
(A) Euthyroid sick syndrome continue insulin glargine dose
(B) Pheochromocytoma
(C) Subacute thyroiditis
Item 36
(D) Thyroid storm
A54-year-old man is eva luated because of fa tigue. He also notes
constipation and cold intolerance. Medical history is significant
Item 34 for tonsillar squamous cell carcinoma treated vvith radiation
3 years ago. Th.ere is no family history of thyroid disorders.
A 39-year-old' man is seen for a follow-up examination. On physical examination , the skin is dry. Mild peri-
1l1ree months ago, he underwent a n organ -sparing pro- orbital edema is present. The thyroid is of normal size and
cedure for squamous cell carcinoma of the throat. As part without nodules. Reflexes are delayed.
of his surgery, his thyroid and all four parathyroid glands Laboratory studies show a h,emoglobin level of ll gld [
were removed. He was started on 1,25-dihydroxyvitamin D (110 gi L), a serum sodium level of129 mEqi L (129 mmoli L),
(calcitriol) and supplemental calcium carbonate fo llowing and a serum thyroid-stimulating hormone (TSH) level of
surgery. He subsequently completed a course of concur- 1.4 )..lU/ mL (1.4 m Ul L).
rent cisplatin-based chemotherapy and rad iation therapy
without significant complications and currently feels well Which of the following is the most appropriate next step in
with no new signs or symptoms. Medical history is oth- management?
erwise unremarkable except fo r a 25-pack-yea r smoking
(A) Free thyroxine (T) measurement
history; he discontinued tobacco use at the t ime of diag-
nosis. Medicat ions are calcitriol, calcium carbonate. and (B) Repeat TSH measurement in 4 weeks
levothyroxine. (C) Thyroid scin tigraphy
On physical examination, temperature is 37.0 ac (98.6 of), (D) Thyroid ultrasound
blood pressure is 125/84 mm Hg, pulse rate is 8Simin, and
respiration rate is 121min. BMJ is 24. Well-healed surgical
incisions are noted on the anterior neck. The remainder of Item 37
the examination is unremarkable. A 24-year-old woman is evaluated for hypercalcemia inci-
Laboratory studies: dentally discovered on laboratory studies performed fo r
Calcium 8.5 mg/dL (2.1 mmol/L) another indication. She reports no hypercalcemia-related
Creatinine 0 .8 mgl dL (70.7 symptoms. Medical history is significan t for gastroesopha-
1,25-Dihydroxyvitamin D Within therapeutic range geal reflux disease and menstrual migraine. Family history
24-Hour urine calcium 375 mg/24 h (9.4 mmoll 24 h) is notable for a brother who has a "calcium·· problem. Medi-
cations are omeprazole and sumatriptan as needed.
Which of the following is the most appropriate manage- On physical examination, temperature is 36.2 oc (97.2 of ),
ment of this patient's postsurgical calcium therapy? blood pressure is 127168 mm Hg, pulse rate is 731min, and
respiration rate is 121min. BMI is 25. Chest, heart, and
(A) Check 25- hydroxyvitamin D level abdomi nal examinations are normal, as is the remainder of
(B) Check parathyroid hormone level her examination.

83
Self-Assessm e nt Test

Laboratory studies are significant for a serum calcium serum cortisol following a low-dose overnight dexameth-
level of11. 2 mg/dL (2.8 mmo!IL), parathyroid hormone level asone suppression test. Subsequent testing revealed a sup-
of 55 pg/ mL (55 ng/ L), and 25- hyd roxyvitamin D level of35 pressed adrenocorticotropic hormone level.
ng/m L (87.4 nmo!IL) . Kidney and thyroid function studies A contrast-enhanced adrenal CT scan revealed a
are n ormal. well-circumscribed 3.7 cm right adrenal mass with a con
tnist washout of greater than 50'X, at 10 minutes.
Which of the following is the most appropriate next step in
management? Which of the following is the most appropriate perioper-
a tive m anagement?
(A) Bone densitometry
(B) Measurement of urine calcium and creatinine levels (A) Postoperative hydrocortisone
(C) Parathyroid sestamibi scan (B) Postoperative mitotane
(D) Referral fo r parathyroidectomy (C) Postoperative norepinephrine
(D) Preoperative phenoxybenzamine

Item 38
A 25-year-old woman w ith type 1 diabetes mellitus is eval- Item 40
uated fo r recent-onset glycemic fl uctuations without symp- A 62-year-old man is evaluated fo r right leg pain. The pain
tomatic hypoglycemia. She was diagnosed with diabetes 7 has developed progressively over the past 6 months and
years ago. Her hemoglobin A1c levels since diagnosis have worsens w ith prolonged activity, such as playing golf. Med-
ra nged from 6.4% to 7.3%, with th e most recent value at ical history is unremarkable, and he ta kes no medications.
7.3%. She reports eating a carbohydrate-consistent diet at On physical examination, temperature is 36.3 oc (97.3 °F),
each meal, w ith little variation in her selection of meals blood pressure is 128/67 mm Hg, pulse rate is 73/ min, and
or snacks. She started a new job several months ago but respiration rate is 10/min . BMI is. 29. There is no pain to
contin ues her daily exercise routine and sleep schedule. She pal pation over the femoral region, and he has normal range
has no other medical problems or symptoms. Her diabetes of motion.
treatment regimen is insulin glargine once daily and insulin Laboratory studies are significant fo r a serum alkaline
lispro three times daily. phosphatase level of200 U/ L and serum calcium level of 9.0
Physical examina tion findings and vital signs are nor- mg/ dL (2.3 mmol/L).
mal. Plain radiograp h of the right femur is shown.
Estimate glomerular fi ltration rate, serum creatinine
level, and urine album in-creatinine ratio are normal. Her
blood glucose values fro m the previous week are shown
below.
Blood glucose values:
Breakfast Lunch Dinner Bedtime
(mg/dL (mg/dL (mg/dL (mg/dL
[mmol/L]) [mmol/L]) [mmol!L]) [mmol/L])
124 (6.9) 190 (10.5) 109 (6.1) 210 (11.6)
110 (6.1) 92 (5.1) 112 (6.2) 126 (7.0)
115 (6.4) 118 (6.5) 112 (6. 2) 126 (7.0)
117 (6.5) 127 (7.0) 204 (11 .3) uo (6 .1)
108 (6.0) 101 (5.6) 122 (6.8) 114 (6.3)
101 (5.6) 111 (6.2) 106(5.9) 72 (4.0)
126 (7.0) 187 (10.4) 102 (5.7) 196 (10.9)

Which of the following is the most likely cause of the fluc-


tuating glycemic control?
(A) Antibodies to exogenous insulin
(B) Gastroparesis
(C) Inadequate insuli n doses
(D) Inappropriate insulin timing

Which of t he follow ing is the most appropriate n ext step in


Cl Item 39
A 20-year-old wo man is admitted to the hospital for an
management?
(A) Antiresorpt ive t herapy
elective adrenalectomy. Her history is significant fo r recent
onset of hypertension. weight ga in, generalized weakness. (B) Bone biopsy
and easy bru ising. Laboratory evaluation demonstrated (C) Evaluation for multiple myeloma
an elevated urine free cortisol and lack of suppression of (D) Clin ical observation

84
Self-Assessment Test

Item 41 Item 43
A 29-year-old woman is evaluated during her fi rst prenatal A 44-year-old woman is evaluated for weight gain, muscle
visit. She feels well. Medical history is significant only for weakness. and metabolic syndrome. She has hirsutism but
hypot hyroid ism, for which she has taken levothyroxine, also notes hair loss on her head. She has been amenorrheic
100 11g/ d, for the last 3 years. Her only other medicat ion is a for 2 years.
prenatal vitamin. Medical history is significan t for hyperlipidemia, type
On physical examination , she is afebrile. Blood pres- 2 diabetes mellitus, hypertension, and obesity. Medications
sure is 98/72 mm Hg, pulse rate is 88/min, and respiration are atorvastatin, metfonnin. and lisinopril.
rate is 12/ min. The thyroid is of normal size and without On physical examination, blood pressure is 156/92 mm
nodules. There is no cervical lymphadenopathy. Cardio- Hg and pulse rate is 78/min. BMI is 42. She has a cushingoid
vascular and pulmonary examinations are unremarkable. appearance, acne, and moderate hirsutism affecting the chin,
Abdominal examination reveals normal bowel sounds. The upper lip, breasts, back, and chest. There are several wide vio-
uterus is not palpable. laceous striae across the abdomen and the back of her arms.
Laboratory studies show a serum thyroid-stimulating Laboratory studies:
hormone level of 3.6 !JUi mL (3.6 mUl L) and a serum total Adrenocorticotropic 52 pg/ mL (11.4 pmol!L)
thyroxine (T) level of 4.5 !JgldL (58 nmol!L) . hormone (ACTH)
Which ofthe following is the most appropriate treatment of Cortisol8 AM after 1 mg of 5.2 11g/ dL (143 .5 nmol/ L)
this patient's hypothyroidism? dexamethasone the night
before
(A) Continue levothyroxi ne dose 24-Hour urine cortisol
(B) Decrease levothyroxine dose excretion
(C) Discontinue levothyroxine Initial measurement 124 pg/ 24 h (341.7 nmol/24 h)
Repeat measurement 98 f.(g /24 h (270.0 nmol/24 h)
(D) Increase levothyroxi ne dose
lntrapetrosal sin us sampling identifies a pituitary
microadenoma as the source of t he high ACTH level.
CJitem 42 Which of the following is the most appropriate test to per-
A 62 year old man is evaluated in the hospi tal for several formnext?
hours of nausea, lig htheaded ness. and back and abdomi nal
pain. I lc underwen t uncomplicated mechan ical aortic \'al\'e (A) 8-mg dexamethasone suppression test
replacement 3 days ago. Be had been doing well postop (B) 24-Hour urine free catecholamine and metaneph rine
eratively until t he development of his current symptoms. measurement
Medical history is significant on ly for hypothyroidism. Med- (C) Dual-energy x- ray absorptiometry scan
icat ions are therapeutic unfractionated heparin. levothy- (D) PET scan
rox ine. and as needed oxycodone.
On physical exa minat ion. te mperature is 37.2 oc
(99 .0 °F), blood pressure is 80 SO mm Hg, pulse ra te
is 110 'm in , a nd respirat ion ra te is 18 m in. BMl is 26. Item 44 CJ
Exa m inat ion of the ches t shows a clean and dry sterna l A 28-year-old woman is being discharged from the hospital
wound. Ca rdiac exam inat ion reveals regu la r tachyca r w ith a diagnosis of autoimmune adrenalit is. Medical history
dia and mechan ical S, . The re is no pain w it h palpation is otherwise unremarkable, a nd she was on no medica tions
of t he abdomen o r lowe r bac k. His skin p igmen tat io n is prior to admission.
no rma l. On physical examination, temperature is 37.0 oc (98.6 °F),
blood pressure is 120 80 mm Hg. pulse rate is 80 mi n. and
Laboratory studies:
respiration rate is 12 min. BMI is 22. Increased skin pigmen
Hemoglobin 7.3 g!dL (73 g L)
tation is noted over the extensor surfaces of the extremit ies
9.0 g dl. (90 gl.)
bilaterally. The remainder of her examination is normal.
postoperatively
Leukocyte coun t 11.000 (II X 10" L) Which of the following is the most appropriate long-term
Act ivated pa rtial 70s medication regimen for this patient?
thromboplastin time
Creatinine 1.0 mg/ dL (88.4 11mol L) (A) Dexamethasone, 1 mg once daily
Sodium 130 mEq L (130 mmol L) (B) Flud rocortisone. 0.05 mg once daily
Potassium 6.0 mEq I. (6.0 mmol L) (C) Hyd rocortisone . 10 mg three times daily
Cort isol, random <2 !Jg. dL (55.2 nmol L) (D) Predni sone, S mg once daily, and fl udrocortiso ne ,
0.05 mg once daily
Which of the following is the most likely diagnosis?
(A) Au toimmu ne ad renalitis Item 45
(B) Bilateral adrenal hemorrhage A 78-year-old woman is evaluated for a 2-week history of
(C) Opiate ind uced ad renal insufficiency unintentional weigh t loss, night sweats, and neck swelling.
(D) Pituitary apoplexy She has more recently also had difficulty swallowing solid

85
Self-Assessment Test

foods and positional shor tness of breath. She does not have Which of the following is the most appropriate manage-
hoa rseness, palpitations, or cha nge in bowel habits. Prior ment of this patient?
to the development of these symptoms, she had been in her
usual state of health with no illnesses. She has hypothyroid- (A) Refer for parathyroidectomy
ism and has been taking levothyroxine for 40 years, with a (B) Start alendronate
stable dose for the past 10 years. (C) Start calcitonin
On physical examination, temperature is 36.9 °( (98.4 °F), (D) Start cinacalcet
blood pressure is 140/ 88 mm Hg, pulse rate is 75 / min, and (E) Start vitamin D3 (cholecalciferol)
respiration rate is 12/m in. The thyroid is symmetrically
enlarged, finn, and fixed. There is no t hyroi d bruit.
CT scan of the neck is shown. Item 47
A 57-year-old man is admitted to the hospital for evaluation
of substernal chest pain. His medical history is significant
for type 2 diabetes mellitus. coronary artery disease, hyper
tension, and hyperlipidemia. He manages his diabetes as
an outpatient with diet, exercise, and metformin. His other
medications are aspirin. metoprolol. atorvastatin, and sub -
lingual nitroglycerin as needed. His inpatient plasma glu
cose values are 170 to 210 mg/dL (9.4-ll.6 mmoi /L). Resu lts
of all other laboratory studies are normal.
Which of the following is the most appropriate treatment
for this patient's diabetes while
(A) Basal and prandial insulin
(B) Glipizide
(C) Metformin
(D) Sliding-scale insulin

Item 48
Which of the following is the most likely diagnosis? A 43-year-old woman is evaluated (or progressive weight
(A) Graves disease gain over t he past 2 years. Her previous weigh t was 72 .6 kg
(16 0 lb) but has steadily risen to her current weight ofl06.6
(B) Papillary thyroid cancer
kg (235 lb). She notes a slight increase in her a ppetite but
(C) Primary thyroid lymphoma minimal change in her lifestyle or activity level. She has .
(D) Subacute thyroiditis t ried to lose weight with increased exe rcise and n utritional
counseling but without sign ificant results. More recently
she reports having trouble sleeping and decreased exercise
Item 46 tolerance w ith activi tes such as walking up steps. Medical
A 67-year-old man is evaluated for a recent diagnosis of h istory is sign ifica nt for impaired fas ting glucose, hyper-
prima ry hyperparathyroidism a fter an elevated serum cal- tension, and hyperlipide mia. Medications are hydrochlo-
cium level was incidentally detected on laboratory testing. rothiazide and atorvastatin. She has not been prescri bed
Medical history is significant only for hypertension, and his glucocorticoids or had glucocorticoid joint injections.
only medication is ramipril. On physical examination, temperatureis37.2 oc (99.0 °F),
On physical examination, temperature is 35.8 oc (96.4 °F). blood pressure is 136/86 mm Hg, pulse rate is 88/min, and
blood pressure is 120/ 68 mm Hg, pulse rate is 62/ min. and respiration rate is 12/min. BMl is 38. She has rounded facies,
respiration rate is 14/ min. BMI is 32. 1he rema inder of his thin hair, mild hirsutism, and prominent fat deposition in
examination is unremarkable. the dorsocervical and supraclavicular areas. Her skin is thin ,
Laboratory studies: and she bruises easily, although striae are not present. Her
Calcium 10.9 mg/dL (2.7mmol!L) exami nation is otherwise unremarkable.
Creatinine 0.9 mg/dL (79.6 pmol! L) Laboratory studies are significant for a fasting plasma
Parathyroid hormone 98 pg/mL (98 ng/L) glucose level of 120 mg/dL (6.7 mmoi/L) and norma l thy-
25-Hydroxyvitamin D 19 ng/mL (47.4 nmol! L) roid-stimulating hormone level.
Estima ted glomerular >60 mL/min /1.73 m 2 Which of the following is the most appropriate next step in
filtration rate evaluation?
A dual-energy x- ray absorptiometry (DEXA) scan (A) Ad renocorticotropic hormone measurement
shows T-scores of - 1.3 in the righ t femoral neck, - 1.0 in the (B) 1-mg dexamethasone suppression test
lumbar spine, and - 1.4 in the non-dominant fo rearm. Frac-
ture Risk Assessment Tool (FRAX) score indicates a 13'Yo risk (C) 8- mg dexamethasone suppression test
of major osteoporotic fracture and a 1.9% risk of hip fracture (D) Pituitary MRI
over the next 10 years. (E) Serum cortisol measurement

86
Self-Assessm e nt Te st
- - - - - -- - - - -

Item 49 Item 51
A SO-year-old man undergoes follow-up eval ua- A 64-year-old man w ith type 2 diabetes mellitus and stage 4
tion fo r type 2 d iabetes me ll it us . His daily log demon- chronic kidney disease is evaluated fo r continued glycemic
strates average blood glucose levels of 120 to 1SO mg/dL management. He is followed closely by the nephrology ser-
(6 .7- 8.3 mmo!IL), with hypoglyce mia in the SO mg l vice in preparation for impending hemodialysis, with in iti-
dL (2.8 mmol!L) range noted once or tw ice per week ation of erythropoietin therapy within the last 3 months. His
w ithou t a discernible pattern . He is unable to detect the average fasting and preprandial blood glucose values are in
hypoglycemia. the 14S to 190 mg/ dL (8.0-10.S m mo!IL) range. He does not
The patien t has a med ical history of diabetic ret- have hypoglycemia. His insulin regimen consists of insulin
inopa thy, chronic kidney disease, peripheral neurop- detemir at bedtime and insulin glulisine before meals. His
at hy, hypertension, hype rlipide mia. obstruct ive sleep most recen t hemoglobin A1c value has decreased from 7.S%
a p nea (on bil evel positive a irway p ressure), gast ro- to 6.2%.
esop hagea l refl ux d isease, and os teoarthritis in both
Which of the following is the most appropriate manage-
k nees. He repor ts intolerance to s trenuous exercise due
ment for this patient's diabetes?
to knee pain. He is a ble to walk 1S minutes d aily. He has
worked closely w it h a n u tritionist, resu lting in a s.o- (A) Contin ue current therapy
kg (11-lb) weight loss over 1 year, w hich has plateaued (B) Decrease insulin detemir dose
recently .
(C) Discontinue preprandial insulin glu lisine
Medicatio ns are insuli n glargine, insu lin as part , lis-
inopril, carvedilol, pa ntoprazole, aspirin, a nd atorvas- (D) Measure postprandial glucose level
tatin.
On physical examina tion, blood p ressure is 140 /
90 mm Hg and pulse rate is 6S / min. BMI is 37. Bilateral Item 52
p roliferative retinopathy is present. There are no carotid A 34-year-old woman is evaluated for hirsutism and acne
bruits or cardiac murmurs. Bilateral loss of monofi la ment that began 4 months ago. She also reports that her voice
a nd vibra tory sensation on the feel and decreased ankle has become deeper. Her menstrual periods have remained
re flexes are noted. TI1e rema inder of the exam ination is regular, occurring every 28 days.
normal. She has one son 5 years of age. She reports that her
Results of laboratory studies show hemoglobin pregnancy was uncompl icated and she had no difficulty
A1c level of 8.2°,{, a nd serum creatinine level of 1.7 mg/dL conceiving. She had a tubal ligation following the birth of
(1S0.3!J.mol!L). her child. She has a sister with polycystic ovary syndrome.
She takes no medications.
Which of the following is the m ost appropriate treatm en t
On physical examination, blood pressure is 140/84 mm
of this patient?
Hg, and pulse rate is 62/min. BMI is 21. She has hyperpig-
(A) Bariatric surgery mented terminal ha irs on the chin, neck, abdomen , and
(B) Increase insulin lower back and comedones and pustules on the face and
(C) Initiate metformin upper back. TI1ere is fro ntotemporal hair loss. The remain-
der of the examina tion is unremarkable.
(D) Initiate pra mlintide
Laboratory studies:
Cortisol, free (urine) 26 !J.g/24 h (71.6 nmol/ 24 h)
Dehydroepiandrosterone 8.2 !J.gl mL (22.l!J.mol!L)
Item 50
sulfate
A 47-year-old woman is evaluated for an inciden tally dis- Prolactin 8 ng/ mL (8 !J.g/L)
covered righ t adrenal mass. Testosterone 96 ng/dL (3.3 nmol/L)
On physical examination. blood pressure is 120/80 mm
Hg in both arms and pulse rate is 84/min. The abdomen is
Which of the following is the most appropriate diagnostic
nontender, and there are no palpable masses. The remain-
test to perform next?
der of the examination is unrema rkable.
Noncontrast Gr of the abdomen demonstrates a 3.2-cm (A) Abdominal CT scan
well-circumscribed, partia lly cystic right adrenal lesion (B) Low-dose dexamethasone suppression test
w ith a density of 30 Hounsfield units. A low-dose dexa-
(C) Pelvic u ltrasound
methasone suppression test is nega tive for evidence of cor-
tisol hypersecretion. (D) Pituitary MRT

Which of the following is th e most appropriate next step in


m anagement? Item 53
(A) Adrenalectomy A 27-year-old woman is evaluated for management of her
(B) CT-guided transcutaneous biopsy type 1 diabetes mellitus. She was diagnosed 10 years ago. She
has no known complicat ions from her diabetes. She eats a
(C) Plasma aldosterone to plasma renin ratio healthy diet and exercises an average of60 min utes per day in
(D) Plasma free metaneph ri nes the evening. She takes insuli n glargine and insulin as part. She
(E) No additional testing is indicated is adherent with her in sulin regimen and checks her blood

87
Self-Assessment Test

glucose level three to five times per day. Her average blood erections but reports low libido and occasional erectile dys-
glucose value is 125 mg/ dL (6.9 mmoi/ L) , with fasting glucose function during intercourse. He wakes once during t he night
values ranging from 80 to 150 mg/dL (4.4 8.3 mmoi/ L) . She to urinate and drink water. He estimates that he urinates 5to
routinely measures her 2-hour postpranclial glucose values. 8 times during the day. which is unchanged.
and they are consistently less than 150 mg/dL (8.3 mmol!L). Medical history is significant for transsphenoidal
She has several overnight blood glucose values ranging from resection of a craniopharyngioma at age 18 years. He has
90 to 140 mg/ dL (5.0-7.8 mmol/L). Her hemoglobin A1e values required anterior pituitary hormone replacement and des-
over the last 6 months have been 7.3'Y., to 7.5%. She is cliscour- mopressin since that time. Medications are desmopressin,
aged that her hemoglobin A1c values remain above 7.0%. hydrocortisone, levothyroxine, somatrop in. and testoster-
Laboratory studies, including creatinine and complete one ena nthate.
blood count, are normal. On physical exa mination, temperature is37.0 oc (98.6 °F),
blood pressure is 118/ 64 mm Hg, pulse rate is 74 /m in. and
Which of the following is the most appropriate manage-
respiration rate is 14/ min. No orthostatic changes are noted.
ment of her elevated hemoglobin A1c level? BMI is 24. There are no facial changes suggestive of acromeg-
(A) Begin continuous glucose monitoring aly or Cushing syndrome. The thyroid is normal w ithout
(B) Increase exercise goiter or nodules. Hair distribution and skin turgor are
normal. There is no gynecomastia or striae. Normal penis
(C) Increase insulin aspart
and testicular volume arc noted. Visual fields are intact on
(D) Increase insulin glargine neurologic examination.
Laboratory studies:
Item 54 Sodium 138 mEq/ L (138 mmol!L)
Insulin-like growth factor 1 Normal
A 25-year-old woman is evaluated after a recent diagnosis of Prolactin ·18 ng/mL
polycystic ovary syndrome. She is concerned about hirsut- 1l1yroid- stirnula ting hormone (0.8 mUl L)
ism and irregular menses. She has no desire to be pregnant Thyroxine (T), free 0.7 ng/dL (9.0 pmol!L)
at this time. She takes no medications. Testosterone (11 days after 482 ng/dL (16.7 nmoi/L)
On physical examination, she is afebrile. Blood pres- injecti on)
sure is 110/ 60 mm Hg, pulse rate is 68/ min, and respiration
rate is 16/ min: BMI is 37. Coarse hair is noted on the chi n. Follow-up MRTs show no residua l tumor.
jawline, and periumbilical area. The remainder of her phys-
ical examination. including pelvic examination, is normal. Which of the following is the most appropriate manage-
ment?
Which of the following is the most appropriate treatment?
(A) Increase desmopressin
(A) Combined oral contraceptive pills (B) Increase hydrocortisone
(B) Intermittent progestin withdrawal (C) Increase levothyroxine
(C) Levonorgestrel intrauterine system (D) Increase testosterone ena nthate
(D) Spironolactone therapy (E) Stop somatropin

Item 55 Item 57
A46-year old man is evaluated following a diagnosis of pheo- A 55-year-old man is evalua ted following a screening for
chromocytoma. He has no signs or symptoms at this time. type 2 cliabetes mellitus. He is asymptomatic. He has a h is-
Except fo r hypertension, his medical history is unremark- tory of hypertension and hyperlipidemia. There is no h istory
able. Medications are lisinopril and hydrochlorothiazide. of anemia, liver disease. or kidney disease. Medications are
On physical exam ination, blood pressure is 170/ 90 mm lisinopril and rosuvastalin.
1-!g and pulse rate is 90/min. The remainder of the physical On physical examination. blood pressu re is 123/
examination is unrevealing. 76 mm Hg and pulse rate is 72/ min. BMI is 28. The remainder
of the examination is unremarkable.
Which of the following is the most appropriate next step in
management? Laboratory stuclies:
Hematocrit 45.6%
(A) Increase lisinopril dosage Creatinine 1.0 mg/dL (88.4 11moi/ L)
(B) Perform contrast-enhanced adrenal CT scan Glucose, fasting 128 mg/dL (7.1 mmoi!L)
(C) Start phenoxybenzamine Hemoglobin A1e 5.6'Yo
(D) Start propranolol
Which of the following is the most appropriate diagnostic
test to perform next?
Item 56
(A) Fasting plasma glucose
A 38-year-old man w ith panhypopituitarism is evaluated for
worsening fatigue and weight gain over the past 3 months. He (B) Hemoglobin A1e
is sleeping 9 hours each night, but he feels tired during the day (C) Oral glucose tolerance test
and has decreased his usual exercise level. He has morning (0) Random blood glucose

88
Self-Assessment Test

Item 58 Her medical history is significant for diet-con trolled


A 34-year-old man is evaluated tor a 2 year h istory of type 2 diabetes mellitus diagnosed l year ago and oste-
fatigue, low libido. and infertility. His family history is nota- oporosis diagnosed 4 yea rs ago. Her only med ication is
ble for a brother with infertility. His medical history is unre- alendronate.
markable. and he Lakes no medications. On physical examination. temperature is 37.0 oc (98.6 oF),
On physical examination. vital signs are normal. His blood pressure is 120/80 mm Hg, and pulse rate is 70/min.
height is 195.6 em (77 in). and his weight is 86.2 kg (190 lb). BMI is 26. The remainder of the physical examination is
Genital examination reveals small, firm testes bilaterally. normal.
Laboratory studies revea l a morning serum total tes- Laboratory evaluation reveals a serum sodium level
tosterone level of 140 ng/ dL (4.9 nmol/L) (which was con- of 139 mEq/L (139 mmo l/L) and serum potassium level of
firmed on repeat testing), a serum follicle-slimulating hor- 4.1 mEq/ L (4.1 mmol!L). The previously obtained CT scan
mone level of 24 mU / mL (24 U/ L), and a serum luteinizing shows a 2.0-cm well-circumscribed, left adrenal lesion with
hormone levelof18 mU / mL (18 U/ L). Repeated semen anal- a density of 5 Hounsfield units.
ysesrevealazoosperrrtia. In addition to screening tests for pheochromocytoma,
Which of the following is the most appropriate diagnostic w hich of the following is the most appropriate diagnostic
test to perform next? test to perform next?

(A) Karyotype
(A) Adrenal vein sampling
(B) MRl of the pituitary (B) Low-dose dexamethasone suppression test
(C) Scrotal ultrasound (C) Plasma renin activity and aldosterone concentration
measurement ·
(D) Serum prolactin measurement
(D) o further testing

Item 59
Item 61
An 84-year-old man is evaluated for moderate fatigue. He
otherwise feels well and does not have constipation. cold A 68-year-old man is seen in follow-up for a recent diagno-
intolerance, we ight ga in or loss, anx iety. tremor. palpita- sis of acromegaly. He presented with chronic fatigue, joint
tions, or dyspnea. Medical history is significant for hyper- and back pain, and an increase in his shoe size over the past
tension, and his only medication is felodipine. 2 years. Medical history is significant for hypertension and
On physical examina lion. he is alert and oriented. Blood type 2 diabetes mellitus. Current medications are lisinopril,
pressure is 144 mm llg: other vital signs are normal. The metformin. and as- needed acetaminophen.
thyroid is not palpable. Cardiopulmonary examination is On physical examination. blood pressure is 146/88 mm
normal. There is no peripheral edema. Neurologic examina- Hg and pulse rate is 90/ min. BMI is 29. He has a prominent
tion is nonfocal, and deep tendon reflexes are norma l. brow. Macroglossia is present. Lung and heart exarrtinations
are unremarkable. Musculoskeletal examination
Laboratory studies:
large hands and knees with bone swelling and crepitus.
Complete blood count Normal
Skin is thickened, and there is excessive perspiration. On
Comprehensive metabolic Normal neurologic examination. bitemporal hemianopsia is noted.
profile
Laboratory studies are significant for an elevated serum
Thyroid-stimulating hormone 6.4!1U mL (6.4 mU L)
insulin-like grovvth factor 1 level of 996 ng/ mL (996 11g/L)
(TSH)
and serum prolactin level of 42 ng/ mL (42 ).lg/ L).
Free thyroxine (T) 1.3 ng/dL (16.8 pmoi/L)
MRI shows a 2.5 x 1.8-cm pituitary macroadenoma that
elevates the optic chiasm and appears to envelop the left
Which of the following is the most appropriate manage- carotid artery and invade the left cavernous sinus.111e optic
ment? chiasm is mildly atroph ied.
(A) Levothyroxine therapy Which of the following is th e most appropriate next step in
(B) Measurement of serum total triiodothyronine (T) therapy for this patient?
level
(A) Dopamine agonist
(C) Measurement of serum total T 1 level
(B) Growth hormone receptor blockade
(D) Repeat TSH and free T.1 measurement in 6 Lo 12 weeks
(C) Somatostatin analogue
(D) Stereotactic radiation therapy
Item 60 (E) Transsphenoiclal pituita ry surgery
A 62-yea r-old woman is evaluated for an incidentally dis-
covered left adrenal mass. Two weeks ago. the patient was
evaluated in the emergency department for dilluse abdom- Item 62
inal pain and vomiting. A CT scan was obtained that was A 47-year-old man is evaluated postoperatively following
normal except tor the adrenal mass. Three hours after pre- thyroidectomy for papillary thyroid cancer. Preoperative
sentation to the emergency department, the pain resolved evaluation showed no evidence of distant metastatic dis-
spontaneously. ease, and he total thyroidectomy w ith central

89
Self-Assessment Test

and left lateral neck dissections. The patient's medical his- Which of the following is the most appropriate treatment?
lory is otherwise unremarkable.
On physical examination, vital signs are normal. The (A) Clomiphene citrate
neck surgical sites are clean and dry. 1he remainder of the (B) in vitro fertilization
examination is unremarkable. (C) Injectable gonadotropin
The surgical pathology report reveals a 3.5-cm papillary (D) Metformin
thyroid cancer in the left lobe of the thyroid and sLx malig-
nant lymph nodes out of 35 dissected. There is evidence of
minor extrathyroidal extension and vascular invasion by Item 65
the primary tumor.
A 64-year-old woman is seen for follow -up evaluation. Two
Which of the following is the most appropriate postopera- weeks ago, she was in a car accident, and an incidenta l pitu-
tive treatment? itary adenoma was found on a cervical spine CT sca n. She
has no residual injuries from the car accident.
(A) Doxorubicin chemotherapy She is otherwise healthy and takes no medications. She
(B) External beam radiotherapy went through menopause at age 51. She has night sweats
(C) Radioactive iodine therapy two to three times per month and occasional hot flushes.
(D) No additional therapy These have improved over the past decade and are not
bothersome. She is not sexually active. She has never taken
hormone replacement therapy. She has had no change in
vision, headaches, or galactorrhea.
Item 63 On physical examin'ation, temperature is 37.5 oc
A 66-year old man is evaluated in the office after being (99.5 °F), blood pressure is 110/ 63 mm Hg, pulse rate is
treated in the emergency department for an exacerbation of 82 min. and respiration rate is 14/ min. BMI is 26 . There
chronic obstructive pulmonary disease. While in the emer- is axillary and pubic hair loss. Visual fields are intact.
gency department, he was noted to have a random blood There are no findings suggestive of Cushing syndrome or
glucose level of 211 mg/ dL (11.7 mmol!L). His hemoglobin acromegaly.
Ale was 7.8% at the time. A repeat random fingers tick blood
Laboratory studies:
glucose level in office is 204 mg/dL (11.3 mmol!L) .
Estradiol <20 pg/ mL (73.4 pmoi!L)
The patient reports recent polyuria and polydipsia. l-Ie
Follicle-stimulating hormone 6.4 mU /mL (6.4 U/ L)
has lost 6 kg (13.2 lb) over the last 3 months. l-Ie has chronic
Luteinizing hormone 3.2 mU/ mL (3.2 U/ L)
epigastric pain associated with loose. oily stools due to Prolactin 53 ng/ mL (53 J..lg/ L)
chronic pancreatitis.
Thyroid-stimulating hormone 3.2 J1Ui mL (3.2 mUl L)
l-Ie has a 20- pack-year history of tobacco use and
Thyroxi ne (T), free 1.1 ng/dL (14.2 pmol!L)
prior alcohol use, however, he does not currently use alco-
hol. Current medications are enteric coated pancreatic Pituitary MR! shows a 7- mm adenoma in the anterior
enzymes, vitamins, tiotropium inhaler. and an albuterol sella. The tumor is not invasive. It does not approximate the -
inhaler as needed. · optic ch iasm. TI1e p ituitary stalk is mid-line.
On physical exa mination, temperature is 37.1 oc (98.8 °F) ,
blood pressure is 130/75 mm 1-lg, and pulse rate is 90/ min. Which of the following is the most appropriate manage-
BMI is 22. l-Ie has mild epigastric pain on palpation without ment?
rebound tenderness or guarding. The rest of his examination (A) Begin dopamine agon ist
is unremarkable.
(B) Gamma knife stereotactic radiosurgery
Which of the following is the most appropriate treatment (C) Repeat testing in 12 months
for his diabetes? (D) Transphenoidal resection
(A) Exenatide
(B) Glipizide
(C) Insulin
Item 66
(D) Metfonnin A 57-year-old man with a 15-year history of type 2 diabe-
tes mellitus is evaluated for bilateral burning sensation in
his feet for the last 6 to 12 months. The sensation worsens
at night. His hemoglobin A1c levels have remained less
Item 64 than 7.0% for the last 2 years but were between 8.0% and
A 34-year-old woman is evaluated because she and her male 9.0% before implementing significa nt lifestyle changes
partner have been trying to conceive for 13 months without and transi tioning to insulin t herapy from metf'o rmin ther-
success. Her medical history is notable for a 6-year history apy 2 years ago.
of irregular men ses and a recent diagnosis of polycystic His medical history includes coronary artery disease,
ovary syndrome. Her only medication is a prenatal vitamin. first-degree atrioventricular block, nonproliferative dia-
On physical examination. vital signs are normal. BMI is betic retinopathy. hypertension. and hyperlipidemia.
36. Terminal hair growth on the chin, upper lip. and sides Medications are regular insulin, neutral protamine
of the face is noted. No evidence of abdominal or pelvic Hagedorn (NPH) insulin, aspirin, metoprolol. atorvastatin ,
masses, clitoromegaly, or galactorrhea is detected. and lisinopril.

90
Self-Assessment Test

On physical examination, findings are compatible with rate is 16/min . BMI is 24 . Pelvic examination reveals a non-
distal polyneuropa thy. tender, mobile uterus.
Which of the following is the most appropriate manage- Laboratory studies:
ment of this patient's neuropathy? Estradiol <80 pg/m L (293.6 pmol!L)
Follicle-stimulating hormone 6.2 mU/ mL (6.2 U/ L)
(A) Amitriptyline Luteinizing hormone 4.1 mU/ rnL (4.1 U/ L}
(B) Duloxetine Prolactin Normal
(C) Nerve conduction study Thyroid-stimulating hormone Normal
(D) Vitamin measurement Urine human chorionic Negative
gonadotropin

C]ltem 67 Which of the following is the most appropriate diagnostic


A 72 year old man presents to the emergency department test to perform next?
overnight. His wife noted that he was diaphoretic and (A) MRI of the pituitary
restless during the night. and his initial blood glucose level
(B) Peripheral karyotype
measured b) emergency medical sen·ices was 11 mg dL
(2.3 mmol Ll. He was gi\en a single dose of intravenous (C) Progestin withdrawal test
glucose, which incrca<>ed his blood glucose level to 85 mg dL (D) Transvaginal ultrasound
(4 .7 mmoi ' L). His symptoms recurred en route to the hos-
pital, and he responded to a second dose of intravenous
glucose. While in the emergenc) department. he was Item 69
provided food and his glucose level has remained above A 29-year-old woman comes to t he office for a follow-up
80 mg. dL (·l.l mmol L) without further treatment. The evaluqtion. She was diagnosed with type 2 diabetes mellitus
patient's wife reports seYeral recen t episodes of overnight 2 years ago. Her fasting and premeal blood glucose values
diaphoresis over the last several weeks. I lis last meal range from 120 to 150 mg/dL (6.7-8.3 mmol!L). She has had
prior to this episode was approximate)) 7 hours before no hypoglycemic events. She has been adherent with her
sym ptom onset. insu lin regimen along with diet and exercise modifications.
His medica l history is significant tor hypertension and ller hemoglobin A1c level has decreased from 9.0% to 7.5%
ostcoarthritis. Medications are lisi nopril and as needed ibu- over the last 12 months. She expresses frustration over the
profen. need for multiple medications to treat her diabetes. Medical
On physical examination, the patient is alert and ori history is otherwise unremarkable.
en ted. Blood pressure is 135 8-t mm llg, and pulse rate is 82 Medications are insulin glargme. insulin aspart. and
min. is 23. lhe general physical and neurologic exam · metformin.
inations are normaL On physical examination. blood pressure is 120/72 mm
Laboratory studies arc significant for a normal com Hg and pulse rate 80/ min. BMI is 27. The remainder of the
plete metabolic profile and blood count. An insulin secre examination is normal. ·
tagogue screen is obtained, and hypoglycemic studies con Hemoglobin A1c level is currently 7.4%. Results of all
sisting of measurement of insulin, C peptide. proinsulin, other laboratory studies are normal.
and p hydroxybutyrate levels are planned.
Which of the following is the most appropriate manage-
Which of t he following is the most appropriate diagnostic ment of this patient's diabetes?
strategy for this patient?
(A) Increase insulin glargine dose
(A) l lypoglycemic studies now (B) Measure 3 A M blood glucose level
(B) Mixed meal testing with hypoglycemic studies at the (C) Measure postprandial blood glucose levels
lime of symptomatic hypoglyccmia (D) Switch metformin to sitagliptin
(C) Oral glucose tolerance testing with hypoglycemic
studies at the lime of symptomatic hypoglycemia
(D) 72-Hour fast with hypoglycemic studies at the time Item 70
of symptomatic hypoglycemia A 32-year-old woman is evaluated for a 2-week history of
foreign body sensation in her eyes. Despite flushing the eyes
numerous times with saline. the sensation remains. She also
Item 68 feels pressure behind her eyes. She is not pregnant. Medical
A 24-year-old woman is evaluated for a 3-month history of h istory is unremarkable.
amenorrhea accompanied by cyclic pelvic pain. Preceding On physical examination. scleral injection and peri-
t he onset of amenorrhea, she had a recent dilatation and orbital edema a re present bilaterally, right worse than left.
curettage to remove retained products of conception after Diplopia is noted on lateral gaze. There is no lid lag or
a first-trimester spontaneous abortion. Her personal and proptosis. The thyroid is diffusely enlarged, and a bruit is
family medical history is un rema rkable. She takes no med- noted. 1eurologic examination discloses brisk reflexes and
ications. a bilateral hand tremor.
On physical examination, she is afebrile. Blood pres- Laboratory studies reveal a serum thyroid-stimulating
sure is 110/ 60 mm Hg, pulse rate is 68 /min, and respiration hormone level of less than 0.008 11U/mL (0.008 mU/ L), a

91
Self-Assessment Test

serum free thyroxine (T) level of 4.5 ng/dL (58.0 pmol!L).


and a serum total triiodothyronine (T3) level of 365 ng/dL
(5.6 nmol/L}.
Which of the following is the most appropriate next step in
treatment?
(A) External-beam radiotherapy to the orbits
(B) Methimazole
(C) Radioactive iodine
(D) Total thyroidectomy

Item 71
A 42-year-old man is evaluated for resistant hypertension.
He was diagnosed with hypertension at age 35 years and
reports t hat his blood pressure has never been well con-
trolled. He is taking his medications as prescribed. He does
not have headaches, chest pain, palpitations, shortness of
breath. or symptoms of panic attack. He has no history of There is diffuse enlargemen t of the thyroid, and the
cardiovascular disease, does not smoke. and does not drink inferior edge of the gland ca nnot be visualized. CT scan
alcohol. Medications are lisinopril, am lodipine, hydrochlo- shows substernal extension of the goiter in the left lobe
rothiazide, metoprolol. and potassium chloride supplemen- with mild tracheal narrowing and tracheal d eviation to
tation. lie is not taking any over-the-counter medications. the rigtlt.
On physical examination. blood pressure is 150/86 mm Which of the following is the most appropriate treatment?
Hg and pulse rate is 65/ min. BMI is 24. Examination of the
heart is significant for an s, but no murmurs. The remainder (A) External-beam radiotherapy
of his examination is unremarkable. (B) Levothyroxi ne
Laboratory studies are significant for a serum creati- (C) Radioactive iodine
nine level of 1.0 mg/dL (88.4 J.lmol!L). fas ting plasma glu-
(D) Thyroidectomy
cose level of 82 mg/ dL (4.5 mmol/ L), and serum potassium
level of 3.2 mEq/L (3.2 mmol / L).
Which of the following is the most appropriate next diag- Item 73
nostic step? A 26-year-old woman is evaluated because she and her
(A) Dexamethasone suppression test husband have been trying to conceive for 14 months wi th-
out success. Her husband fathered a child in a previous
(B) Plasma aldosterone-plasma ren in activity ratio marriage. Her medical h istory is notable for pelvic inflam-
(C) Plasma metanephrines and catecholamines matory disease, which was diagnosed and successfully
(D) Rena l artery Doppler flow study treated at age 18 yea rs. Her only medication is a prenatal
vitamin.
Physical examination findings are unremarkable.
Item 72
Which of the following studies is most likely to be diag-
A 65-year-old man is evaluated because of painless neck
nostic?
swelling and difficulty swallowi ng that has progressively
worsened over the last year. He does not have hoarseness, (A) Diagnostic laparoscopy
but he feels as though his voice is not as strong as it was in (B) Hysterosalpingogram
the past. Medical history is significant for multiple thyroid (C) Peripheral karyotype of the patient
nodules. Fine-needle aspiration of three of the largest nod-
ules 5 years ago showed that all nodules vvere benign on (D) Semen analysis of the patient's husband
cytologic examination. Medical history is othervvise unre- (E) Transvaginal ultrasound assessment of follicle count
markable, and he takes no medications.
On physical examination, vital signs are normal. The
thyroid is diffusely enlarged and mobile with swallowing. Item 74
He has facial flushing when raising his hands above his head A 46-year-old man is evaluated for a right lateral neck mass.
as shown (see top of next column). He first noted the mass while shaving 2 weeks ago. He does
No cervical lymphadenopathy is noted. Cardiovascular not have hoarseness or difficulty swallowing. Medical his-
and pulmonary examinations are normal. Reflexes are nor- tory is significant only for hypertension. His only medica-
mal, and there is no visible tremor. tion is hydrochlorothiazide.
Laboratory studies show a serum thyroid-stimulating On physical examination. blood pressure is 165/95 mm
hormone (TSH) level of 4.0 ).lU/ mL (4.0 mUl L). Hg, pulse rate is 88/ min, and respiration rate is 12/ min.
Thyroid ultrasound shows innumerable coales- There is a firm 2-cm mass in the right lateral neck and a
cent nodules with no suspicious sonographic features. mobile 1-cm right thyroid nodule.

92
Se lf-Assessment Test

Laboratory studies: ranging from SO to 190 mg/ dL (2.8-10.5 mmol!L) . His over-
Calcium 10.5 mg/dL (2.6 mmol!L) night blood glucose va lues range fro m 120 t o 140 mg/dL
Creatinine 1.8 mg/dL (159.1 (6 .7-7.8 mmo l!L) . He is unable to detect hypoglycemia.
Thyroid -stimu lating 4.8 (4.8 m Ul L) The patien t is concerned abou t hyperglycemia, a nd he
hormone desires to reach a hemoglobin A1c level ofless than 7%.
· Medical history is significant for dia betic retinopathy,
Ultrasound of the neck reveals a hypoechoic thyroid peripheral neuropathy, hypertension, and hyperlipidemia.
nodule wit h microcalcificalions measuring 1.6 em. 111ere is Medications are neutral protamine Hagedorn (N PH) insulin,
a 2.2-cm mass w ith internal calcifications in the area of the regular insulin , losartan, chlorthalidone, metformin, rosu -
palpable abnormality. vasta tin , and aspirin .
Fine-needle aspiration of the t hyroid nodule and the On physical exam ina tion, blood pressure is 125 / 82
lateral neck mass reveals medullary thyroid cancer. RET mm Hg a nd pu lse rate is 80/ m in. BMT is 24 . Reti na l
testing is posit ive for a mutation in codon 634. examinat ion demons trates nonprolifera tive retinopa-
Which of the follow ing is the most appropriate next step in thy. His lower ext remities have diminished sensa tion
management? to a 10-g monofila m ent a nd vibra tion w ith a 128- Hz
tuning fork. ·
(A) 18-Fiuoro-deoxyglucose positron emission tomogra- Hemoglobin A1, level is 7.2%, and the results of all other
phy scan laboratory studies are normal.
(B) Plasma frac tionated metanephrine levels
Which ofthe following is the most appropriate treatment of
(C) Serum parathyroid hormone level this patient's diabetes?
(D) Total thyroidectomy and lateral neck dissection
(A) Continue current insulin and metform in doses
(B) Continue current insulin, increase metform in dose
CJ Ite m 75 (C)
(D)
Decrease mea l-time insulin, add pramlintide
Decrease meal-time insulin, continue metformin
A 55 year-old woman is evaluated in the emergency
departmen t for altered menta l status that has developed
over the past week. Her fam ily notes that she has become Item 77
increasingly confused , and her oral intake has decreased
sign ificantly. Her med ical history includes breast cancer A 28-year-old woman is evaluated for a 6-month history of
that was d iagnosed 5 years ago. Addit ional medical his- infertility. In order to predict her fertile period. the patient
to ry is significant for hypertensio n. Her only medication is has been using a home urinary luteinizing hormone (LH)
ch kit. The kit can identify an LH s urge and therefore pred ict
On physical exam inatio n. the pat ient is minimally ovulation and the optimal time fo r in tercourse. Her uri -
responsive. Mucous membranes are d ry. Temperatu re is nary LH kit result has been consistently positive but her
36.2 ac (97.2 °F). blood pressure is 100 59 mm Hg. pulse pregnancy test result consistently negative. She has been_
rate is 110 min. and respiration rate is i2 min. B.Ml is 22. exercising recently in an attempt to lose weight. Her med-
There is mild, diffuse abdominal tenderness to palpation. ical history is otherwise unremarkable, and she takes no
The remainder of the examination is unremarkable. medications.
Laboratory studies are sign ifican t for a serum sod ium On physical examina tion , vital signs are normal. BMI is
level of 148 mEq L (148 mmol L) . creatinine le\·el of 38. Coarse terminal hairs are noted on t he chin, neck, and
4.0 mg 'dL (353.6 baseline creatinine leve l is anterior chest. Pelvic examination is normal.
1.6 mg dL (141.4 L) . and calcium level of18.2 mg elL Pelvic ultrasound reveals a thickened endometrium
(4.6 m mo l L). and polycystic-appearing ovaries bilaterally.
Chlorthalidone is d isconti n ued, and intravenous nor- Which of the following is the most likely diagnosis?
mal saline is started .
(A) Hypothyroidism
Which of the following is the most appropriate immediate
(B) Late-onset congenital adrenal hyperplasia
next step in management?
(C) Polycystic ovary syndrome
(A) Calcitonin (D) Prolactinoma
(B) Cinacalcet
(C) Hemodialys is
Item 78
(D) Zoledronic acid
A 72-year-old woman is eval uat ed in follow-u p for oste-
oporosis. Medical history is significa nt for a hip fracture
5 years ago sus tained a fter a mild fall. Evaluation a t that
Item 76 time in cluded a d ual-energy x-ray absorptiometry (DEXA)
A 60-year-old man is evaluated d uring a rout ine fo llow-up scan showing a left hip T- score of -2.8 a nd vertebral
examination. He has type 2 d iabetes mellitus. Review of T-score of - 2.7. She has been main ta ined on alendronate
h is blood glucose log demons trates fasting blood glucose t herapy since that t ime. Medical history is a lso significant
va lues rang ing fro m 120 to 160 mg/d L (6.7-8.9 m moi / L) for hypertension. Medications a re alendronate, hydro-
and variable 2-ho ur post pra ndial blood glucose values chlorothiazide, calcium, and vitamin D. Family h is tory

93
Self-Assessme nt Te st

is significant for osteoporosis in her mother, sister, and On physical examination, her temperature is 37.2 oc
maternal aunt. She has a 35 -pack-yea r tobacco u se history (99.0 °F), blood pressure is 1.18/68 mm Hg, pulse rate is
a nd continues to smoke. 72/ m in. and respiration rate is 10/min. BMJ is 28. Other-
On physical examination, temperature is 36.9 oc wise. her physical exa min ation is unremarkable.
(98.4 °F). blood pressure is 138/87 mm Hg, pulse rate is Laboratory studies:
89/ min, and respiration rate is 12/ min. BMI is 28. She Human chorionic gonadotropin Negative
has marked thorac ic kyphosis and increased central Prolactin 68 ng/mL (68).lg/ L)
adiposity. Thyroid-stimula ting hormone 15 ).1 U/mL (15 m U/ L)
Labora tory studies show a serum calcium level of Thyroxine (1). free 0.82 ng/dL (10.6 pmoi/L)
8.6 mg/dL (2.2 mmol!L) and 25- hydroxyvitamin D level
of 44 ng/ mL (109 .8 nmoi/ L}; kidney function studies are
norma l. Which of the following is the most appropriate manage-
Repeat DEXA shows a stable bone mineral density. ment?

Which of the following is the most appropriate treatment of (A) Begin cabergoline
this patient's osteoporosis? (B) Begin levothyrQxine
(C) Hold sertraline and retest
(A) Change to denosumab
(D) Order pituitary MRI
(B) Change to teriparatide
(C) Continue alend ronate therapy
(D) Initiate a drug holiday
Item 81
A 33-year-old man is eval uated for infertility. As part of
Item 79 this evaluation. azoospermia is identified on two sequential
A 55-year-old man presents for follow-up of a recent diag- semen analyses. His medical history is otherwise notable for
nosis of hyperaldosteronism. Medical and family history chronic sinusitis. He takes no medications.
are unremarkable. Medications are losa rtan. metoprolol. On physical examination, vital signs are normal. BMI
amlodipine, and ch lorthalidone. is 24 . There is no evidence of inguina l hernia. A small left-
On physical exa mination, blood pressure is 150/90 mm sided varicocele is present that empties when the patient is
Hg and pulse rate is 70/ min. The rest of the examination is recumbent. Testicles are of normal size. Abse nce of the vas
unremarkable. deferens is noted bilaterally.
Additional studies include a noncontrast cr scan of the
Which of the following is most likely responsible for this
abdomen that shows a 1.2-cm homogeneous left adrenal
patient's infertility?
lesion with a density of - 5 Hounsfteld units. Adrenal vein
sampling indicates similar ratios of aldosterone to cortisol (A) Anabolic steroid abuse
bilaterally. (B) Cystic fibrosis
Which of the following is the most therapy for (C) Klinefelter synd rome
this patient? (D) Varicocele
(A) Bilateral adrenalectomy (E) Y chromosome microdeletions
(B) Dexamethasone
(C) Left adrenalectomy
(D) Spironolactone Item 82
A 22-year-old woma n is evalua ted for fatigue and weigh t
loss over the last 2 mon ths. She describes a lack of energy,
Item 80 decreased appetite, a nd an u nint ent ional 3.6-kg (8-lb)
A 24-year-old woman is evaluated for 3 months of galac- weight loss. Over the last week she has experienced na u-
torrhea. worseni ng fatigue. and ma laise. She was well sea and occasional vomiting. She notes no other focal
6 mon ths ago and without explanation began feeling symptoms. She has no significant medical history. Family
tired and letha rgic. She has noted milk leaking spon ta - history is remarkable for hypothyroidism in her mother
neously from her breasts. Her menses have been unpre- and type 1 diabetes mellitus in her brother. She takes no
dictable. She had a normal cycle 3 months ago and medications.
4 months ago, but then her next cycle was delayed. Her On physical examination, temperature is normal,
last menstrual period was 6 wee ks ago, and sh e had blood pressure is 100/68 mm Hg. pulse rate is 90/ min, and
excessive bleeding. She has stopped exercising and has respiration rate is 12/ min. BMl is 19. Hyperpigmentation
gained 3.6 kg (8 lb). She also reports headaches that is pronounced over the knuckles of th e hands and elbows
occur about 2 to 3 times per week. She has noted consti- bilaterally. The remainder of t he physical examination is
pation and breast tenderness. She has no libido. which unremarkable.
is a sign ificant cha nge. She took a home p regnancy test. La boratory evaluation demonstrates an 8 AM serum
which was negative. She has a history of depression. cortisol level of 2 ).lg/dL (55.2 nmol! L) , a se rum sodium
There is no family history of p it uita ry d isease. Her only level ofl33 m Eq/ L (133 mmol!L), and a serum potassium
medication is se rtraline for de pression. level of 5.8 mEq/ L (5.8 mmol / L).

94
Self-Asse ssment Test

Which of the following is the most appropriate next step in (C) Discontin ue insulin glargine, change insulin aspart to
management? a sliding-scale regimen
(A) Cosyn tropin stim ulation test (D) Discontinue insulin glargine, discontinue insulin
as par t
(B) Hydrocortisone and fl udroco rtisone
(C) Plasma adrenocorticotropic hormone (ACTH) mea-
s urement
Item 84
(D) Prednisone
A 34-year-o ld ma n is eva luated for a 1- year hist ory
of decreased lib ido and severe e rectile dysfu nction ,
Item 83 including abse nce of m orning erections. He and his
A 21-year-old woman is evalua ted for management of type fe m ale partner woul d like to conceive. His medical
1 diabetes. She was diagnosed3 months ago after presenting history is otherwise unrem arkable , and he takes no
to the emergency department w ith diabetic ketoacidosis. medicat ions.
Her hemoglobin A1c level at the time of diagnosis was 15.2%. On physical examination, vital signs are normal. BMI is
She was discharged fro m the hospital on a basal and pran- 27. Atroph ic testes are noted bilaterally.
dial insulin regimen w ith minor adjustments required as Laboratory studies:
an outpatient. Her blood glucose log demonstrates a recent Follicle-stimulating hormone 1.2 m U/mL (1. 2 U/ L)
change manifesting as frequent symptomatic episodes of Luteinizing hormone 0.8 mU/ mL (0.8 U/ L)
bot h fasting and postprandial hypoglyce mia with blood glu- Prolactin 110 ng/ mL (110 Jlg!L)
cose levels ranging from 50 to 65 mg/ dL (2.8-3.6 mmol/ L) . Morning testosterone (total) 120 ng/dL (4.2 nmol!L)
Medications are insulin glargine and insulin aspart. Thyroid-stimula ting hormone Normal
On physical examination, blood pressure is ll0/70 mm
Hg and pulse rate is 70/ min. BMI is 19 . The remainder of the MRI of the pituitary reveals a 0.8-cm anterior pituitary
examination is unremarkable. mass consistent w ith an adenoma.
Hemoglobin A1c level is 6.2%. Which of the following is the most appropriate
Which of the following is the most appropriate manage- treatment?
ment for this patient's hypoglycemia? (A) Cabergoline
(A) Decrease insulin glargine, decrease insulin aspart (B) Clomiphene citrate
(B) Decrease insulin glargine, decrease insulin aspart, add (C) Sildenafi l
pramlintige (D) Testosterone replacement therapy

95
Answers and Critiques

Item 1 Answer: B roidism clue to multiple gland hyperplasia frequently occurs


Educational Objective : Treat hyperthyroidism during in patients with MEN2A. Patients with primary hyperparathy-
pregnancy.
roidism may present with symptoms related to hypercalcemia
(polydipsia, polyuria, and constipation), or the hyperparathy-
This patient has hyperthyroidism occurring during pregnancy roidism may be found during an evaluation for osteoporosis
and should be treated with propylthiouracil (PTU) . Based on or nephrolithiasis. Pheoch romocytomas in MEN2A are usu-
this patient's age and sex, the most likely cause of her hyper- ally benign and intra-adrenal in location, but can be multiple
thyroidism is Graves disease. Clinical findings support ing or bilateral.
this diagnosis are the diffuse thyromegaly, thyroid bruit, and lnsulinoma ahd prolactinoma occur in multiple endo-
elevated thyroid-stimulating immunoglobulin index. Treat- crine neoplasia type 1 (MENl). In MENl, one mutated allele
ment of hyperthyroidism during pregnancy is typically with of the MENI ge ne is usually inherited, and a somatic muta-
medical management, and PTU, rather than methimazole, tion in the other allele is later acquired and results in the for-
is the drug of choice during the firs t trimester. Although mation of neoplasia. The most common endocrine disorder
rare, methimazole has been associated with development of in MENl is primary hyperpa rathyroidism resulting from one
aplasia cutis (absence of a portion of skin on the scalp in a or more parathyroid adenomas.
localized or widespread area) and choana! atresia (blockage Neurofi broma is not a clinical feature of MEN2A. Neu-
of the posterior nasal passage due to tailed recanalization of rofib romas, cate-au-lait spots, and pheochromocytoma are
the nasal fossae du ring fetal development) when used in the among the clinical features of t he autosomal dominant dis-
fi rst trimester. Once organogenesis is complete. methimazole order neurofib romatosis type 1.
can be substituted for PTU. Methimazole is typically the pre-
KEY POINT
ferred agent in patients with hyperthyroid ism because it has
a longer intrathyroidal half-life than PTU and usually can be • Medullary thyroid cancer, pheochromocytoma, a nd
administered once daily. pri mary hyperparathyroidism occur in patients with
Radioactive iod ine is co ntraindicated during pregnancy multiple endocrine neoplasia type 2A (MEN2A) .
because it can cause destruction of the fetal thyroid.
lhyroidectomy is generally avoided during pregna ncy Bibliography
unless the hyperthyroidism cannot be controlled or the 1\.rampitz GW, Norton JA. RET gene m utations (genotype and phenotype) of
patient cannot tolerate either PTU or methimazole. vVhen multip le endocrine neoplasia type 2 ancl fami lial medullary thyroid ·
carcinoma. Cancer. 2014 Jull :l20(13):1920 31. [PMm: 24699901)
needed, surgery is typically performed during the second
trimester, if possible.
KEY POINT lte m3 Answ er: B
• In pregn ant patients with hyperthyroidism, propylthi- Educational Objective: Treat prolactinoma during
ouracil, rather than methimazole, should be used in pregnancy.
the fi rst trimester because of the teratogenic effects of The patient should undergo formal visual field testing. During
methimazole during this time. pregnancy, there is concern that prolactinomas can grow
due to estrogenic stimulation. "Ihe risk of significant growth
Bibliography depends on the size of the prolactinoma prior to pregnancy:
Lazarus Jll. 'vlanagement of hyperthyroidism in p regnancy Endocrine. 2014
Mar;45(2):190-4 . [PMID: 24174179)
With microadenomas (<10 mm). the risk is considered to
be low. whereas the risk is higher wit h macroprolactinomas
mm). Significa nt expansion may cause vision loss by
ltem2 Answer: c compressing the optic chiasm. It is therefore appropriate to
obtain visual field testing during pregnancy in women with
Educational Objective: Diagnose the multiple endocrine
macroadenomas even without symptoms to diagnose vision
neoplasia type 2 (MEN2) syndrome.
field loss (such as bitemporal hemianopsia) that may be due
This patient is most likely to develop primary hyperparathy- to an enlarging prolact inoma. Testing in these women is rec-
roidism. He has symptoms related to catecholamjne excess ommended every trimester of pregnancy. Because this patient
from a pheochromocytoma and has a personal and family is in her fi rst trimester and has a history of a macroadenoma,
history of medullary thyroid cancer. This is typical of multi- visual field testing now is indicated.
ple endocrine neoplasia type 2A (MEN2A) resulting fro m a It is not helpful to check the semm prolactin level. Pro-
mutation in the RET proto-oncogene. Primary hyperparathy- lactin is normally elevated in pregnancy and can be greater

97
Answers and Critiques

than 200 ng/mL (200 pg/L). Additionally. this patient has a measuring the glucose level at 3 A:VI. Medications that affect
known prolactinoma, so her prolactin will likely be elevated the overnight glucose level need to be decreased if the 3 AM
for both reasons. Elevated prolactin itself is not necessari ly glucose level is low. Medications that aflect the overnight
ha rmful. Prolactinomas cause concern when they cause glucose should be increased or added if the 3 AM glucose level
hypogonadism or mass effect. In this patient. an elevated is e levated.
prolactin level w ill not change the treatment plan because it Fast-acting insulin such as insulin lispro at bedtime
is expected to be elevated and does not help clarify whether increases the risk of hypoglycemia.
the prolaclinoma is causing harm. Metformin will decrease gluconeogenesis from the liver
Routine monitoring of women with prolactinomas with and improve fasting hyperglycemia. However, for similar
MRI during pregnancy is not indicated because the absolute reasons, overnight hypoglycemia must be excluded before
risk of significant enlargement of pituitary adenomas is this treatment could be safely initiated.
low. llowever. MRI is indicated in women with a pituitary Increasing the insulin glargine dose could also worsen
macroadenoma and abnormalities on visual field testing or overnight hypoglycemia if that is the cause of the fasting
change in headache possibly attributable to expansion of the hyperglycemia.
adenoma. Despite the hemoglobin A1c level of less than 7'Yo, the
Bromocriptine is avoided in pregnancy when possible etiology of the patient's fasting hyperglycemia should be
because its safety during gestation has not been established, investigated. Detection of overnight hypoglycemia would
although when a patient has symptoms of mass effect during necessitate immediate changes in her insulin regimen or
pregnancy, bromocriptine may need to be restarted. How- food intake regardless of her hemoglobin A1c value.
ever, it would not be appropriate to restart dopamine agon ist
KEY POINT
therapy in this patient without a clear indication for treat-
ment such as an enlarging pituitary adenoma. • Overnight blood glucose monitoring can help detect
hypoglycemia or dawn phenomenon.
KEY POINT
• Because prolactinomas may increase in size in preg- Bibliography
nant women and lead to loss of vision, close clinical Cryer PE. Axelrod L Grossman AB. et al. Evaluation and management of
monitoring and formal visual field testing should be adult hypoglycemic disorders: an Endocrin e Society Clinical Practice
Guideline. J Clin Endocrinol Metab. 2009 Mar;94(3):709 28. [PMID:
performed during each trimester. 190881551

Bibliography
Melmcd S. Casanueva FF. lloffman AR. et al. Diagnosis and t reatment of
hyperprolactinemia: an Endocrine Society clinical practice guideline. J Item 5 Answer: A
Clin Endocrinol Metab. 2011 Feb;96(2):273 88. [PMID: 21296991)
Educational Objective: Evaluate the cause of Cushing
syndrome.

ltem4 Answer: D The most appropriate diagnostic test to perform next is a Cf


scan of the adrenal glands. This patient was suspected to have
Educational Objective: Manage overn ight
Cushing syndrome (CS) based on pathognomonic clinical
hypoglycemia.
findings and is not taking exogenous glucocorticoids. Two
This patient should measure her blood glucose level at 3 A\1. screening tests for CS are abnormal, which is adequate to
The etiology of fluctuating fasting glucose values in diabe- establish the diagnosis. The next step in evaluation is mea-
tes can be multifactorial, including overnight hypoglycemia, surement of the plasma adrenocorticotropic hormone (ACTH)
dawn phenomenon, or inadequate insulin doses. To maintain level to differentiate between ACfH-dependent and ACfH-in-
normal blood glucose levels upon rising, an early morning dependent CS. With ACTH-dependent causes, the plasma
physiologic release of catecholamines. cortisol. and growth ACfH is usually greater than 20 pg/mL (4.4 pmol /L). I low-
hormone occurs to stimulate endogenous glucose produc- ever, the low plasma ACfH reported in this patient is consis-
tion from the liver. Overnight hypoglycemia caused by over- tent with ACTH-independent CS. ACT scan ofthe abdomen to
treatment of diabetes or prolonged effects of recent physical evaluate the adrenal glands is therefore indicated because the
exertion can lead to low-normal fasting glucose values and most common causes of ACTH-independent CS are adenomas
amplify the release of catecholamines. cortisol. grovvth hor- and carcinomas of the adrenal cortex.
mone, and glucagon to increase endogenous glucose produc- Inferior petrosal sinus sampling and MRI of the pituitary
tion, which can lead to hyperglycemia (Somogyi effect). With gland are used to confirm the presence of a corticotroph ade-
the dawn phenomenon, fasting hyperglycemia occurs in the noma of the pituitary gland in ACfH-dependent CS, which
setting of inadequate basal insulin coverage to maintain the is the most common cause of CS overall. However, neither is
endogenous glucose value within a normal range. Food intake indicated in this patient because biochemical testing has not
in the evening can also contribute to fasting hyperglycemia if revealed ACTH-dependent CS.
it is inadequately covered with insulin. Overnight hypogly- 1h e diagnosis of CS requi res that at least tvvo first-
cemia and the dawn phenomenon can be distinguished by line screening tests be abnormal, including the low-dose

98
Answers and Critiques

dexamethasone suppression test (LDST) (both standard and development of ketoacidosis, which would require restarting
overnight), 24-hour urine free cortisol (UFC), or la te-night insulin therapy.
salivary cortisol. This patient failed to suppress cortisol levels Determining autoimmunity, in conjunction with beta
following an overnight LDST and had an elevated 24-hour cell fu nction, is helpful in assessing whether a patient has
UFC level confirmed by a repeat collection. Therefore, the potential to become insulin-independent in the fut1.lre.
measurement of late night salivary cortisol is u nnecessary His autoantibodies were negative at t he time of his presen -
because the diagnosis of CS has already been established. tation, and it is unlikely that these would now be positive. It
is not necessary to retest antibodies in this setting.
KEY POINT
• CT of the adrenal glands should be performed in KEY POINT
patients with ACTH-independent Cushing syndrome • Prior to switching from insulin to oral therapy in
because adenomas and carcinomas of the adrenal cor- patients with ketosis-prone diabetes, fasting
tex are common causes. C-peptide and glucose levels should be checked .

Bibliography Bibliography
Nieman LK. Bi ller BM, Findling JW. e L al. The diagnosis of Cushing·s \1aldonado \1R. Otiniano \1£, Cheema F, et al. Factors associated with insu-
syndrome: an Endocrine Society Clinical Practice Guideline. J Clin lin discontinuation in subjects with ketosis-prone diabetes but preserved
Endocrinol Mctab. 2008 May:93(5):1526-40. [P\1ID: 18334580] 13-cell function. Diabet Med. 2005 Dec;22(12) :1744-SO. [PMID: 16401322]

ltem6 Answer: D
Educational Objective: Manage ketosis-prone type 2
Item 7 Answer: C
Objective: Treat pituitary apoplexy.
Cl
diabetes mellitus. The patient has acute apoplexy caused by pituita!) bemor
This patient should have a repeat measurement of fast- rhage and requires urgent transsphenoiclal decompression
ing C-peptide and glucose levels. He has ketosis-prone of the hemorrhage to preserve vision. Given his history of
type 2 diabetes mellitus. Patients w ith ketosis-prone type previous headache, loss of libido. and erectile dysfunction.
2 diabetes do not fulfill the classic phenotype associated he likely had a preexisting prolactinoma that acutely bled.
w ith autoimmune type 1 d iabetes. These patients are often Prolactinomas are almost always treated with dopamine ago
older, overvveight or obese, and of black or Latino ethnic- nists. but this patient requires urgt;nt surge!)' to decrease
ily. Patients with new-onset ketosis-prone diabetes require pressure on the optic chiasm to save his vision. Transsphe
insuhn therapy initially but might be able to be managed noidal resection is the preferred method of pituitary surgery.
w ith oral agents in the future. Prior to switching from He also requires urgent stress-dose glucocorticoids because
insulin to oral therapy, his pancreatic beta-cell function of risk of secondal)' cortisol deficiency, which would be-
should be assessed with fasting C-peptide and glucose mea- life threatening.
surements. Ketosis-prone type 2 diabetes is heterogeneous The patient has appropriately received glucocorticoids
condition in that the presence of autoantibodies is variable for possible adrenocorticotropic hormone deficiency. There
across the population, as is the degree of pancreatic beta cell is no indication to assess other pituitary function. In the
fu nction . His initial C-peptide level in the setting of hyper- next 2 to 4 weeks. thyroid function will need to be assessed.
glycemia and diabetic ketoacidosis is not an accura te indi- but thyroxine (T) has a long half-life and does not need to
cation of his pancreatic fu nction. Due to the toxic effects be emergently measured or replaced. of hypogo-
of prolonged hyperglycemia on the pancreatic beta cells. nadism or growth hormone deficiency is nOt urgent; surgical
the fasting C-peptide and glucose or a glucagon-stimu lated decompression is.
C-peptide should be measured 7 to 14 days after the correc- Waiting 2 weeks to repeat imaging could lead to per-
tion of the acidosis in order to better assess function. If his manent vision loss. He needs immediate intervention to
repeat fasting C-peptide value is greater than or equal to 1.0 preserve his vision. Simi larly, whole brain external beam
ng/m L (0.33 nmol/L) or his glucagon-stimulated C-peptide radiation is not appropriate for th is patient who needs rapid
value is greater than or equal to 1.5 ng/mL (0. 5 nmol!L), h is surgical decompression.
beta cell function is preserved. KEY POINT
A sliding-scale insulin regimen that does not include
• Patients with pituitary apoplexy and vision loss
basal insulin and does not begin insulin administration
should receive immediate stress-dose glucocorticoids
u nless the blood glucose level is at or above a threshold level
in addition to u ndergoing urgent tran ssphenoidal
will cause wide swings from hyperglycemia to hypoglyce-
pituitary decompression.
mia, and this is inappropriate treatment.
Discontinuation of his insulin and switching to an oral
agent such as metformin could be attempted with evidence Bibliography
Rajasekaran S, Vanderpump M, Baldeweg S, et al. UK guidelines for the
of beta cell fu nction preservation. Close follow-up would management of pituitary apoplexy. Clin Endocrinol. 2011 Jan;74(1):9-20.
be n ecessary to monitor for worsening hyperglycemia or [PMlD: 21044119]

99
Answers and Crit iques

Item 8 Answer: B ltem9 Answer: A


Educational Objective: Manage a sol itary thyroid Educational Objective: Manage primary amenorrhea.
nodule.
TI1e most appropriate management is to initiate estro-
Fine-needle aspiration (FNA) is the most appropriate next step gen and progestin thera py in this patient with primary
in the evaluation to determine whether this patient's thyroid amenorrhea. Primary amenorrhea is defined as the lack
nodule is malignant or benign. She has already had an ultra- of menses by age 16 yea rs accompanied by a normal body
sound examination. Ultrasonography is a sensitive means of hair pattern and normal breast development. Pregnancy
identifying nodules and providing further characterization of must be ruled out in all patients with primary amenor-
the nodules, which is more predictive of malignancy than size rhea. Approximately SO% of patients with prima ry amen-
alone. The ultrasonographic features that are considered more orrhea have a chromosomal abnormality. Primary ovarian
suspicious for malignancy include hypoechogenicity, micro- insufficiency due to Turner syndrome. a syndrome char-
calcifications, irregular margins, and increased intranodular acterized by short stature and the loss of a portion or all
Doppler flow. According to American Thyroid Association of one X chromoso.me, is one of the most common causes
guidelines, this patient with a hypoechoic nodule that is of primary amenorrhea. The diagnosis of Turner syn-
1.5 em should have an FNA to rule out malignancy. drome can be made on the basis of a karyotype, and this
Nodular features are not readily identified on CT, and should be the next d iagnostic test for this patient. Such a
this study has a lower sensitivity than ultrasonography for patient may also have fragile X premutation; however, no
identifying the presence of nodules. CT of the neck would cognitive impairment is typically seen in this patient pop-
therefore not provide any additional information about this ulation. Diagnosing Turner syndrome is critical because
patient. who has already had an ultrasound examination. In affected patients have a higher incidence of cardiovascular
addition , CT would expose this patient to unnecessary radi- disease. metabolic syndrome, and thyroid dysfunction.
ation. If a patient has a substernal goiter. CT is beneficial for Patients with Turner syndrome may have either primary
determining the extent of the goiter. However, this patient or secondary amenorrhea and common ly have normal
has no extension of the thyroid into the mediastinum. secondary sexual characteristics. The mechanism involved
Levothyroxine therapy to suppress growth of benign nod- appea rs to be ea rly follicular depletion, such that ovaries
ules is no longer recommended. Randomized clinical trials have are devoid of follicles and oocytes. Serum evaluations in
failed to show a significant effect on nodule volume. Addition- these patients wi ll reveal low estradiol levels (typically
the required dose can induce thyrotoxicosis, which is asso- below the detectable level in the assay) and marked ly
ciated with signifkant risk for cardiovascular complications. elevated gonadotropin levels. This constellation of find-
Measurement of the serum thyroglobulin level is ings is consistent with hypergonadotropic hypogonadism.
reserved for patients who have had a total thyroidectomy Such patients should receive hormone replacement ther-
and is useful as a tumor marker for detection of residual or apy with estrogen and cyclic progestin to prevent endo- ·
recurrent thyroid cancer. In a patient with fll1 intact thyroid, metria! hyperplasia, osteoporosis, and other sequelae of
serum thyroglobulin measurement is an insensitive and hypoestrogenism.
nonspecific means of testing for malignancy. A pituitary prolactinoma causes secondary amenor-
Thyroid scanning with technetium is unlikely to be rhea through direct inhibition of gonadotropin releasing
helpful in this patient. 1hyroid scanning is used to deter- hormone secretion by prolactin. Because this patient has
mine the functional status of the nodule. Isotope scanning is elevated levels of gonadotropins, neither a pi tuitary MRI nor
most useful in the setting of a nodule accompanied by a low a prolactin measurement is necessary.
serum thyroid-stimulating hormone level because toxic (or Both hypothyroidism and hyperthyroidism also cause
hyperfunctioning) nodules typically do not require FNA as seconda1y amenorrhea. Hypothyroidism results in increased
the vast majority are benign. This patient has a normal serum levels of thyrotropin-releasing hormone through negative
thyroid-stimulating hormone level and is likely to have an feedback, and this hormone, in turn, stimulates prolactin
indeterminate result ('·cold'' or "warm'') on thyroid scanning. secretion and suppresses gonadotropin secretion. Hyper-
thyroidism can cause rapid weight loss. which is known
KEY POINT
to cause functional hypothalamic amenorrhea. Since this
• Fine-needle aspiration is the most appropriate next patient has elevated gonadotropin levels and no signs of
step in the management of a patient with a solid thy- hyperthyroidism, thyroid stimulating hormone measure-
roid nodule measuring greater than 1 em on ultra- ment is not needed.
sonography.
KEY POINT

Bibliography • Patients w ith primary amenorrhea associated with


American Thyroid Association (ATA) Guidelines Taskforce on Thyroid hypergonadotropic hypogonadism should receive
Nodules and Diffcrcmiatcd Thyroid Cancer. Cooper DS. Doherty
Haugen BR. et al. Revised American Thyroid Association management
hormone therapy with estrogen to prevent endome-
guidelines for patients with thyroid nodules and differentiated thyroid trial hyperplasia and osteoporosis and cyclic progestin
cancer. Thyroid. 2009 Nov:19(ll):ll67 214. Erratum in: Thyroid. 2010 to prevent endometrial hyperplasia.
Aug:20(8):942.[PMID: 19860577]

100
Answers and Critiques

Bibliography hyperprolactinemia but should only be done under the super-


Cord ts EB. Christofolini DM. Dos Santos AA. Bianco B. Barbosa CP. Genetic vision of a psychiatrist. The patient's risperidone should not
aspects of premature ovaria n failure: a lileralure review. Arch Gy nccol
Obstet. 2011 Mar:283(3):635 43. [PMID: 21188402] be disconti nued unless her psychiatrist is consulted.
Hypothyroidism can cause hyperprolactinemia when
uncontrolled, but her hypothyroidism is well treated. She
Item 10 Answer: B has no symptoms of hypothyroidism, and her thyroid fu nc-
tion tests are normal, so uncontrolled hypothyroidism is not
Educational Objective: Treat subacute thyroiditis.
the cause of her hyperprolactinem ia.
A such as metoprolol, is the most appropriate treat- Antipsychotic agents cause hy perprolactinemia by
ment for t his patient with thyrotoxicosis w ho has subacute blocking dopamine, but lithium does not cause hyperpro-
granulomatous (de Quervain) thyroiditis based on the low lactinemia.
rad ioactive iodine uptake (RAIU) and the painful thyroid on Although the patient has a likely explanation for her
exam ination. An antecedent event. in th is case a viral ill- hyperprolactinemia, a prolactinoma is still possible. Risperi-
ness, destroys the thyroid follicles and triggers the release of done can cause prolactin levels above 200 ng/ mL (200 ).!gi L) ,
preto rmed thyroid hormones into the bloodstream, creating so her level of 102 ng/mL (102 ).!g/L) is not unreasonable.
the thyrotoxic phase. During this phase. further release of Remeasuring the prolactin level following discontinuation
thyroid hormones ceases, resulting in a low RAlU as seen in of the drug is recommended before fu rther evaluation for
th is pat ient. are beneAcial in the thyrotoxic phase a pituitary adenoma; the patient's psychiatrist should be
to block the ad renergic effects of the high circulating thyroid consulted before withholding the risperidone tor testing. A
hormone levels. Treatment of subacute thyroiditis is other- pat ient w ith hyperprolactinemia without a clear secondary
wise typically supportive, with NSA!Ds for pain as needed. In or drug-induced cause should be assessed by a n imaging
patients w it h severe pai n. glucocorticoids may occasionally study (preferably, MRI of the pituitary gland) to a exclude
be used. pituitary lesion.
Blocki ng further release of thyroid hormones w ith a
KEY POINT
t hionamide (either meth imazole or propylthiouracil) is inef-
fective beca use the thyroid has al ready released preformed • Antipsychotic agents block dopamine and decrease
thyroid hormone into the bloodstream and is currently not inhibition of prolactin release at the pituitary, causing
producing or secreting additional thyroxine. hyperprolactinemia.
Admi nistration of radioactive iodine for treatment of
the hyperthyroW.ism is also ineffective because this patient's Bibliography
thyroid is not currently taking in iod ine, as evidenced by the S. Casanueva FF. Hoffman AR. et al. Diagnosis a nd treatmen t of
hyperprolactinemia: a n Endocrine Society clinical prac tice gu ideline.
low uptake o n his thyroid sca n. More importantly, radioac- J Clin Endocrinol Metab. 2011 Feb:96(2) :273 88. [PMID: 21 296991 ]
tive iodine w ill not treat th is patient's underlying problem of
damaged thyroid follicles and release of p refo rmed thyroid
hormone into the bloodstream.
Item 12 Answer: B
KEY POINT Educational Objective: Manage low bone mass.
• Treatment of subacute gra nu lomatous (de Quervain)
A repeat dual-energy x-ray absorptiometry (DEXA) scan
thyroiditis typically is supportive. with NSA!Ds and
should be repeated in 2 years in this patient with low bone
occasionally glucocorticoids for severe pain; mass and relatively low 10-year fracture risk. Th e Fracture
are beneficial in the thyrotoxic phase to Risk Assessment Tool (FRAX) calculator defines the 10-year
block the adrenergic effec ts of the h igh circulating fractme risk for patients wit h T-scores in the - 1.0 to - 2.5
t hyroid hormone levels. ranges. The FRA)( calculator (wv.""l. shef.ac.uk/FRAX) incor-
porates multiple risk factors including sex, fracture history,
Bibliography temoral neck bone mineral density. glucocorticoid use, smok-
Sweeney LB. Stewart C. Gailo nde DY. Thyroiditis : an integrated approach. ing, BML age. and alcohol intake to determine projected frac-
Am Fam Physician. 20 14 Sep 15:90(6):389-96. [P.MlD: 25251231]
ture risk. If the risk of major osteoporotic fracture is greater
than or equal to 20% or the risk of hi p fracture is greater than
Item 11 Answer: D or equal to 3%. then the patient's benefi t fro m therapy exceeds
the risk, and she should be offered treatment. Because of her
Educational Objective: Diagnose hyperprolactinemia
history of low body weight and limited nutritional intake
caused by an antipsychotic agent.
during the ti me of development of peak bone mass, she is at
Hyperprolactinemia is a known side effect of antipsychot ic increased risk for low bone mass or osteoporosis Hnd is there-
agents, and this patient's hyperprolactinemia is likely due tore an appropriate candidate for early screening. Her DEXA
to risperidone. Antipsychotics block dopamine and decrease scan shows low bone mass. Spine film shows no evidence of
inh ibition of prolactin release at the pituitary, causing hyper- fracture. Additionally, her calcium and vitamin D levels are
prolactinemia. Stopping the medica tion can reverse the normal. Continuing lifestyle activities (such as maximizing

101
._
Answers and Critiques ____

weight-bearing exercise and avoidance of tobacco or excessive occur during an acute illness is lowering of the serum total
alcohol) in addition to calcium and vitamin D supplementa- triiodothyronine (T 1) level. lhe semm thyroid <;timulating
tion is appropriate management of this patient. hormone (TSH) and serum free lh)Toxine (T 1) Je,·eJs may also
Raloxifene is a selective estrogen receptor modulator decline as the illness increases in scYerity. 'lhe pattern of these
(SERM) that is a treatment option for women with osteo- test results is often indistinguishable from that seen with
porosis because it has been shown to increase bone mineral central hypothyroidism. In fact. some contrm·ersy exists about
density and reduce the risk of vertebral (but not nonverte \Vhether the clinical picture of a low serum TSH level and low
bra!) fractures. However. raloxifene is also associated w ith serum T1 and T 1 levels is an adaptive response to the critical
an increased risk of thromboembolic events and vasomotor illness in order to alter the body's metabolism and thereby aid
symptoms. There is limited data supporting use of ra lox ifene in recovery from the acute illness.
or other SERMs for treati ng patients w ith low bone mass. This patient does not have any definite findings of
alt hough some guidelines recommend considering treat underlying Graves disease. TI1e tachycardia and fever are
ment in patients with low bone mass and 10-year fracture likely the result of his se,·ere infection. If he was also
risk determined by the FRAX calculator of greater than or experiencing thyr9id storm. high serum T 1 and T 1 levels
equal to 20'Yo for a major osteoporotic fracture or greater than would most likely be associated v. ith the IO\\ serum TSII
or equal to 3% for hip fracture. Raloxifene would therefore concentration.
not be appropriate therapy for this patient. The most common cause of hypothyroidism is
Cholecalciferol (D). a metabolite of vitamin D. is com- Hashimoto thyroiditis. Physical examination findings can
monly used to supplement low serum vitamin 0 levels in include a reduced basal temperature. dia<>tolic hypertension.
patients with vitamin 0 deficiency. This patient has normal an enlarged thyroid gland. bradycardia. pallor. dry and cold
serum vitamin D levels; therefore, there is no indication for skin. brittle hair. hoarseness. <md a delayed recovery phase
treatment w ith vitamin D metabolites. of deep tendon reflexes. Patients with Ha<>himoto thyroiditis
Bisphosphonates are considered first-line therapy fo r have low T 1 and T 1 levels and cle,·atcd TSII. lhis patient's
osteoporosis, although they are not used routinely in women low TSH level is not compatible with Ha<;himoto thyroiditis.
w ith low bone mass. Si milar to the use of SERM therapy. This patient is unlikely to have subacute t hyroiditis
guidelines recommend consideration of treatment with a based on the timing of the illness. Although the infection
bisphosphonate for low bone mass only if there is 10-yea r could have triggered destruction of the thyroid. the subse-
frac ture risk determined by the FRAX calculator of greater quent release of preformed Ihyroid hormom.·s into the serum
than or equal to 20% for a major osteoporotic fractu re or should result in an elevation of<;erum T 1 and T 1 levels in the
greater than or €qual to 3'i'o for hip fractu re. early phase of the disease. which is not consistent v. ith the
findings in this case.
KEY POINT
• Treatment for low bone mass in postmenopausal KEY POINT
women involves lifestyle modification tmaximizing • Unless there is a strong suspicion of an underlying
weight-bearing exercise and aYoidance of tobacco or thyroid disorder that may be contributing to a
excessive alcohol) and vitamin D and calcium supple- patient's clinical findings. thyroid function tests
mentation: the need for pharmacologic therapy is should not be performed during critical illness
based on the 10-year estimated fracture risk for because test results are highly likely to be abnormal.
a major osteoporotic fracture or for hip fracture).
Bibliography
Bibliography farwell AP. Nonthyroidal illness syndrome. Curr Opin Endocrinol Diabetes
Obes. 2013 Oct;20(5):478 84. [PMII1: 2:3974778)
Kling JM, Clarke BL. Sandhu NP Osteoporosis prevention. screening. and
treatment: a review. J Womens Health. 2014 Jui:23(7):S63 72. [I'M))):

Item 14 Answer: 8
Educational Objective: Liberalize glycemic targets in
Item 13 Answer: A
Cl Educational Objective: Diagnose euthy ro id sick
a patient with multiple diabetic complications and
advanced microvascular disease.
syndrome.
The most appropriate next step in management of this
lhis patient's clinical picture is consistent with euthyroid patient is to decrease the ris k of hypoglycem ia by decreas-
<;ick <;yndrome (also called nonth)Toidal illness syndrome). ing the insulin doses delivered by the pump. This patient
Unless there is a strong suspicion of an underlying thyroid has had type 1 diabetes for 25 years with subseq uent devel-
disorder that may he contributing to a patient's clinical find opment of advanced microvascu lar disease. His freque nt
ing<;. thyroid function tests should not be perfonned during hypoglycemic events a nd hypoglycem ic unawareness
critical illnc'>'>. In hospihllized patients. especially ones as ill increase the risk of morbidity and mortality from recur-
as the patient described here. thyroid function test results rent hypoglycemia that may occur with stri ngent glycemic
are highly likely to be abnormal. The first deviation that may goals. Glycemic goals should be individualized to account

102
Answers and Critiques
-----------------------

for patient-specific fac tors, such as age and comorbidities. Parathyroid imaging w ith a sestamibi scan is not indi-
The American Diabetes Association suggests a hemoglo- cated because the patient does not have primary hyper-
bi n A1c goa l of tess than 8.0% in patients with a decreased parathyroid ism. Her parathyroid hormone level is elevated
life expectancy, history of severe hypoglycemia, multiple as an appropriate physiologic response to her markedly
comorbidities, or advanced microvascu lar or m acrovascular l9w vitam in D levels. Once her vitam in D levels are suffi-
disease. The less stringent hemoglobin A1e goal should be cient (>30 ng/dL [75 nmol!L]), her parathyroid hormone
implem ented to avoid recurrent hypoglycemia; however, level shou ld be remeasured to ensure that it has returned to
the goal may need to be increased above S.Oo/o if it cannot the normal range. The parathyroid hormone level should be
he achieved safely. rem easured in approxim ately 4 weeks.
Altering the pump settings to deliver more insu lin to Referral for parathyroidectomy would also not be indi-
attain a hemoglobin A1c goal of less than 7.0o/o will increase cated in this patient without an established d iagnosis of
his risk of hypoglycemia. His hemoglobin A1c goal should be p rimary hyperparathyroidism.
liberalized to avoid hypoglycem ia . A person her age with normal diet and minimal sun
The risk of hypoglycemia can be reduced with lower exposure should about 1000 U daily of vitamin D to
insulin doses delivered by either an insulin pump or sub- maintain adequate vitamin D stores. The choice to use chole-
cutaneous injections. Since the patient is already using an calciferol versus ergocalciferol is otten based on the level of
insulin pump, alteration of his insu lin pump settings to vitam in D deficit. Since t he ergocalciferol is more readily
deliver less insulin shou ld occur next. available in the 50.000 U form and has a shorter half- life, it
Gabapentin fo r treatment of his painful peripheral neu- is recommended when a patient's vitamin D level is less than
ropathy is appropriate, but avoidance of recurrent hypogly- lO ng/mL (25 nmoi /L) . Cholecalciferol is often used when
cemia is the most serious issue that needs to be addressed the level is between 20 and 30 ng/m L (50-75 nmol/L) or for
next due to the associated increased risk of morbidity and maintenance and therefore wou ld not be ideal to replete t h is
m ortality. patient. Clinical discretion can be used for levels between 10
and 20 ng/mL (25 -50 nmol/ L).
KEY POINT
Although vitamin D3 (cholecalcife rol) is a reasonable
• A less stringent hemoglobin A1e goal is appropriate fo r option for treatment of vitamin D deficiency, as mentioned
persons with diabetes m ellitus with a decreased life above, cholecalciferol is best used for maintenance of vita-
expectancy, history of severe hypoglycemia. multiple min D levels or repletio n when the 25-hydroxyvitamin
comorbidities, or advanced microvascular or macro- D level is between 20 ng/m L (SO l!mol!L) and 30 ng/ m L
vascu lar disease. (75 nmol!L). Neither for m of repletion, however, w ill be
in the p resence of significant malabsorption.
Bibliography
KEY POINT
American Diabe1es Association. (6) Glycemic targets. In: Standards of
Medical Ca re in Diabetes Care. 2ot; Ja n:38 Su ppl 1:533 • When vitamin D repletion efforts fail, secondary
40.1PMID: 25537705]
causes, such as m alabsorption, should be considered.

Bibliography
Item 15 Answer: D
Tavakkol A. DiGiacomo D. Green PH. Lcbwohl B. Vitamin D status and
Educational Objective: Determine causes of vitamin comitant autoimm unity in celiac disease. J Clin Gastroenierol. 2013
[P:.HD: 23328299]
D deficiency.

This patient with refractory vitamin D deficiency. despite


aggressive attempts at repletion. should be screened for celiac Item 16 Answer: A
disease. The fact that supplemen tation with a therapeutic
Educational Objective: Treat subclinical
dose of vitamin D has failed to replete her body stores should
hypothyroidism.
raise concern for a malabsorption disorder. Based on her his-
tory of another autoimmune disorder (vitiligo). lower range The most appropriate next step in management is to initiate
BMI, and nonspecific s igns and symptoms such as fatigue. tevothyrox.ine therapy. This patient has subclinical hypothy-
subclinical celiac disease would be a reasonable cause of roidism with a mild elevation of the serum thyroid-stimulat-
her malabsorption. Celiac disease m ay present w ith classic ing hormone (ISH) level and normal serum free thyroxine (I)
symptoms of diarrhea, overt malabsorption, and weight loss. level. Her family history of hypothyroidism and her thyroid
but may also exist in a very m ild form and may go largely peroxidase antibody positivity increase the likelihood of pro-
undetected since patients have only nonspecific symptoms gression to overt thyroid fa ilure. In patients with a mild serum
and subclinical malabsorption. If this patient tests positive fo r ISH elevation (betvveen the upper limit of normal and 10
celiac disease, removing gluten from her diet will improve her mL [10 m UlL]). beginning levothyroxine therapy is reasonable
intestinal lining and improve absorption of vitamin D. Even if symptoms suggestive of hypothyroidism are present.
with therapy. patients with malabsorption will likely require Thyroid-stimulating immunoglobulins (ISls) are highly
increased doses of vitamin D supplementation. associated with Graves disease. When the diagnosis of

103
Answers and Critiques

Graves disease cannot be made clinically in a patient \·v ith A bisphosphonate. such as alcndronate. would not be
hyperthyroid ism, measurement of the semm level of these the treatment of choice fo r this patient as she does not
antibodies is recommended, especially if radioactive iodine have osteoporosis nor does she meet Fracture Risk Assess-
uptake studies are not available or if radioactive iodine expo- ment Tool (FRAX) criteria for therapy. The FRAX calculator
sure is contraindicated, as in pregnancy and breastfeeding. defines the 10-year fracture risk fo r patients with T-scores
This pa tient does not have hyperthyroidism. and these tests in the - 1.0 to -2.5 range. The FRAX calculator (wvvw.shef.
are therefore not indicated. ac.uk/FRAX) incorporates multiple risk factors including
Repeating a serum TSH measurement in this patient gender. fracture history, femo ral neck bone mineral den -
was reasonable. since up to 30% of patients w ith an initially sity. glucocorticoid usage. smoking, BMJ. age, and alcohol
abnormal serum TSH level w ill have a normalization of intake to determine projected fracture risk. Tf the risk of
this value upon retesting. Since this patient has persistent major osteoporotic fracture is greater than or equal to 20'Yo
elevation ofTSH and symptoms that may be allribu table to or the risk of hip fracture is greater than or equal to 3%,
hypothyroidism. waiting 12 months before initiating therapy the patient's benefit from therapy exceeds the risk, and she
is not appropriate. should be offered t,reatment. Additionally. bisphosphonates
A radioactive iodine uptake (RA IU) scan is reserved for s hould be used with caution in patients with compromised
patients with hyperthyroidism. Patients with Graves disease kidney function.
typica lly have an elevated RAlU. Conversely. in patients with If the patient declines surgical intervention, a calci-
thyroiditis or exposure to exogenous thyroid hormone. the mimetic agent such as cinacalcet would be an appropriate
RAIU will be low (<5%) despite biochemical hyperthyroid- therapy. Cinacalcet has recently been FDA approved as an
ism. Obtaining a thyroid RAIU in this patient is not indicated. alternative for patients u nable or unwilling to undergo para-
thyroidectomy. Cinacalcet lowers calcium levels by stimulat-
KEY POINT
ing the calcium-sensing receptors of the parathyroid glands
• Levothyroxine therapy is reasonable in a patient with and inhibiting parathyroid hormone secretion. However, it is
subclinical hypothyroidism and symptoms suggestive expensive and has multiple d rug interactions, wh ich make it
of hypothyroidism. less desirable in this patient.
Because this patient is considered to be at increased risk
Bibliography of complications due to untreated hypercalcemia, clinical
Garber JR. Cobin RH. Gharib H, et al: America n Association of Clinical observation alone would not be appropriate.
Endocrinologists and American Thyroid Association Taskforce on
llypothyroidism in Adults . Clinical practice guidelines fo r hy pothyroid KEY POINT
ism in adults: cosponsored by the American Association of Clinical
Endocrinologists and the American Thyroid Associa tion. Endocr Pmct. • Decreased estimated glomerula r filt ration rate (<60
20 12 Erratum in : Endocr Pmct. 2013 Jan
Feb;l9(1):!75. 23246686] mL'min /1.73 m 2) is an inclicalion for surgical treat-
ment of primary hyperparathyroidism in otherwise
asymptomatic persons.
Item 17 Answer: C
Bibliography
Educational Objective: Define surgical indications for Bilezikian JP. Brandi \1 L. East e ll R. Sil\·erberg SJ. Cdelsman R. Marcucci C.
primary hyperparathyroidism. cr al. Guidelines for the of Asymptomatic Primary
Hyperparathyroidism: Summary Statem ent from the Fo urth
The most appropriate treatment recommendation for this International Workshop. J Clin Endocrinol l>lctab. 20l ,t:99(10):3561 9.
[PMID: 251626651
patient is parathyroidectomy. She has primaty hyperpara
thyroidism as shown by her elevated serum calcium and
parathyroid hormone levels. She also has evidence of kidney Item 18 Answer: c
compromise w ith an elevated creatinine level and decreased
Educational Objective: Evaluate a toxic thyroid nodule.
estimated glomerular ftltration rate (eGFR). Impaired kid-
ney fun ction (defined as eGFR <60 mL minl l.73 m 2• 24- h Performing a radioactive iodine (ll:ll) uptake and scan is the
urine calcium >400 mg/24 h [10 mmol /24 h]. or the presence most appropriate management for this patient, who most
of nephrolithiasis or nephrocalcinosis by radiograph. ultra- likely has a toxic nodule or nodules as the cause for her mild
sound. or CT) is an indication for surgical treatment of hyper hyperthyroidism. Since multiple nodules were identified on
parathyroidism in otherwise asymptomatic patients. Other ultrasound. it is important to ascertain which of the nodules
indications for surgery in asymptomatic patients include age are autonomous. as the likelihood of malignancy in such
younger than SO years; a semm calcium level greater than lesions is very low. Performing a thyroid scan wi ll help iden-
or equal to 1 mg/dL (0.25 mmol/L) above upper limit of nor- tify whether one or more of the nodu les is responsible for her
mal: a T-score of - 2.5 or worse at the lumbar spine. total hip. thyroid function abnormalities.
femoral neck. or distal radius; or in those in whom medical A thyroid scan w ill also determine if fine-needle aspi-
surveillance is neither desi red nor possible. Patients with ration is indicated elsewhere in the gland. Nodu les identi-
these indications are considered to have the highest potential fied as autonomously fun ctioning (or "hot") do not require
benefit from surgery. fine-needle aspiration, whereas nodules that are either '·cold"

104
Answers and Critiques

or "warm" (of similar uptake to the surrounding non-nodular specific ki nd of radiation therapy) would be considered in
thyroid tissue) may require cytologic evaluation. a patient with a tumor that is refractory to medical therapy
Th is patient may ultimately benefit from methimazole and incompletely resectable.
therapy. but since her hyperthyroidism is mild, it is sate to Because t his patie nt has a prolactinoma, the treatment
wait to initiate medical therapy until after the thyroid scan of choice is a dopamine agonist, not surgery. There is an
is pertormecl. In patients with a greater degree of thyrotoxi- excellent chance that the p rolactinoma will respond to cab-
cosis. beginning methimazole to lower the thyroid hormone ergoline, and the patient can avoid surgery.
levels prior to performing additional testing is reasonable.
KEY POINT
Once the hormone levels are nearing the normal range.
t he thionamide should be withheld for 5 days prior to the • Dopamine agonists, such as cabergoline, are first- line
thyroid scan. therapy for symptomatic patients with prolactinomas.
If the patient is fou nd to have a toxic nodule or multi ple
toxic nodules, surgical removal is a reasonable treatment Bibliography
approach, particularly if there are compressive symptoms Mclmed s. Casanueva FF. Hoffman AR. et a!. Diagnosis and treatmen t of
hyperprolaclinemia; .-\n Endocrine Society clinical practice guideline. J
from the goiter. Since th is is considered an elective proce- Clin Endocrinol Metab. 201l Feb:96(2):273 -88. (PMID: 21 2969911
dure. ad ministering low-close methimazole to normalize
of her thyroid hormone levels would be advisable prior to
surgery. B
KEY POINT
Item 2 0 Answer:
Educational Objective: Diagnose hypoglycemia in a
Cl
• Performing a radioactive iodine (1231) thyroid uptake patient with d iabetes mellitus taking a sulfonylurea.
and scan is the most appropriate initial management Hypoglycemia is the most likely cause of this patien rs
tor a patient who has mild hyperthyroidism most altered mental status. In patients taking a sulfonylurea tor
likely caused by a toxic nodule or nodules. diabetes who develop dehydration. such as this patient with
decreased oral intake in conjunction with nausea and vom-
Bibliography iting, impaired kidney perfusion may lead to altered phar-
Jameson JL. Minimizing unnecessary surgery for thyroid nodules. i\ Eng! J macokinetics and an increased risk of hypoglycem ia related
Med. 2012 Aug 23;367(8):765- 7. [PMID: 22731671]
to ongoing effects of t he medication and mi nimal carbo
hydrate intake. Although sulfonylureas are very effective
Item 19 Answer: A anti hyperglycemic medications. most agents have relatively
long half- lives. allowing convenient daily dosing. However.
Educational Objective: Treat a macroprolactinoma.
this slower clearance predisposes to hypoglycemia com
The patient has a macroprolactinoma, whjch is best treated pared with other antiglycemic medications. particula rly·
with a dopamine agonist such as cabergoline. TI1e most com- when kidney fu nction is impaired. Glyburide. in particu lar.
mon cause of hyperprolactinemia is a prolaclinoma, which is has a longer half life than other su lfo nylureas and its use is
a benign adenoma. Microprolactinomas are less than 10 mm recommended against in older patients by the Beers Crite-
in diameter, and macroprolactinomas are 10 mm or greater ria. a list of medications that should be avoided or used with
in diameter. Prolactinomas a re the most common type of caution in older patients. Al though she last look glimepiride
secretory pituitary adenoma. The patient's tumor is causing more than 24 hours before her presentation with altered
signillcant mass effect, including compression of the optic mental status, she was dehydrated, thus prolonging the
chiasm, invasion into the cavernous sinus, secondary hypo- glucose lowering effects of this insuli n secretagogue for
thyroidism. and likely growth hormone deficiency; however. several clays.
even with these complications. the macroprolactinoma is Allhough patients with diabetes are at increased risk
best treated with medica lion instead of surgery. A dopamine for atherosclerotic cardiovascular disease. this patien t does
agonist can cause a rapid decrease in the serum prolactin not have focal abnormalities suggesting a cerebrovascular
level and shrinkage of the prolactinoma. More specifically. in accident. and stroke itself is not a common cause of isolated
as many as 90% of patients, it can normalize prolactin levels. altered mental status.
reverse hypogonadism, and shrink tumors by at least SO%. Lcvothyroxine also has a rela tively long half-life allow-
Because of these rapid decreases in tumor size. dopami ne ing once daily closing in most patients. Missing one or sev-
agonists can be used as fi rst-line therapy. even in patients eral doses of Ievothyroxi ne would therefore not likely lead
with mild visual field defects, as long as visual acuity is not to a degree of hypothyroidism causing acute mental status
threatened by rapid progression of the tumor or recent tumor changes.
hemorrhage. Stat in toxicity is unusual. and the most common toxic
Octreotide is used to treat acromegaly but has no role in ily associated with stalin use is muscu loskeletal symptoms.
the fi rst-line treatment of prolactinoma. Increased stat in levels are not typically associated with men
Medical therapy is preferred over surgery and radiation tal status changes and would likely not be the cause of this
for t he treatment of prolactinomas. Stereotactic surgery (a patienrs mental status changes.

105
Answers and Critiques

KEY POINT Item 22 Answer: B


• Sulfonylureas with long half-lives, such as glimepir- Educational Objective: Diagnose hypercalcemia in a
ide, may lead to acute kidney inju ry and hypoglyce- patient with sarcoidosis.
mia in older persons with diabetes mell itus. This patient most likely has hypercalcemia due to increased
1.25-dihydroxyvitamin D levels. She has an elevated cal-
Bibliography cium level with a low parathyroid hormone level, indicating
American Diabetes Associat ion. (7) Approaches lo g lycem ic treatmen t. In:
non-para thyroid hormone (PTH)- mediated hypercalcemia.
Standards of Medical Care in Diabetes- 2015. Diabetes Care. 2015 Jan:38
Su ppl !:S4! 8 . [P\Il!D: 25537707] The differential diagnosis of non-PTH-mediated hypercalce-
mia includes can cer-related hypercalcemia caused by osteo-
lytic lesions of bone, humorally mediated by tumor-secreted
Item 21 Answer: A parathyroid hormone-related protein (PTHrP) , or granuloma-
Educational Objective: Evaluate triiodothyronine {T3 ) tous diseases. such as sarcoidosis. In granulomatous diseases,
hyperthyroidism. the hyd roxylase in disease-associated macrophages actively
converts 25-hydrmcyvitamin D to the highly active 1,25-
The serum triiodothyronine (T3) level should be measured dihydroxyvitamin D metabolite. The increased levels of active
next. This patient exhibits signs and symptoms of hyperthy- vitamin D lead to increased absorption of calcium in the
roidism. Even though the available laboratory data are consis- gut. promotion of increased bone resorption of calcium, and
tent with subclinical hyperthyroidism. the diagnostic evalu- decreased calcium and phosphate excretion by the kidney.
ation is not complete until the T3 level is checked. Although Elevated serum calcium can be d ue to a mutation in the
rare, an elevated T3 level and a normal serum free thyroxine G-coupled protein calci um- sensing receptor (CASR) gene.
(T) level may be present in patients w ith hypetthyroidism. These receptors are in the parathyroid glands and the kid-
Measurement of the T3 concentration should therefore be neys. The sensor mutat ion results in a shift upward in the
obtained in all patients suspected of having thyrotoxicosis. "normal" range of calcium that the receptor recognizes.
Measurement of the T3 level is not indicated in patients w ith resulting in a mildly elevated serum calcium level (usually
hypothyroidism because the T3 concentration is conserved <11.0 mg/dL [2.7 mmol!L]) and high normal or mildly ele-
and may remain within the normal range. even in patients vated PTH level. unlike this patient whose PTH level was low.
with significant hypothyroidism. 25-Hydroxyvitamin D is the major circulating form of
Measuring the thyroid peroxidase (TPO) antibody titer vitami n D and is minimally metabol!cally active relative to
will not provide additional information about this patient. 1,25-dihyd roxyvitamin D. Levels in this patient are likely to be
Determining TPO a ntibody status is helpful in patients w ith low clue to excessive conversion to the 1.25-clihyclroxyvitamin
a mildly elevated serum thyroid-stimulating hormone (TSH) D form, and would not identifY the likely mechanism of
level and is associated with Hashimoto thyroid itis and future hypercalcemia in this patient.
risk of developing permanent hypothyrOidism. However, Hydrochlorothiazide, a thiazide diuretic, reduces blood
th is patient demonstrates signs and symptoms consistent volume by acting on the kidneys to reduce sodium (Na+) reab-
with hyperthyroidism. sorption in the distal convoluted tubule. Thiazides increase
Repeat ing the thyroid function tests (TFTs) in 6 the reabsorp tion of calcium in the distal convoluted tubule
weeks may be appropriate if the total or free T3 level is by their action on a Na+-cl- calcium co-transpot1er. However,
fo und to be normal. If the T3 is normal. the P?tient has any increase in calcium caused by thiazides is mild and rarely
subcli nical hyperthyroidism, and repeating the TFTs may reduces the PTH level below the lower range ofnonnal.
be indicated to determine if the abnormality is transient
or permanent. KEY POINT
Ultrasound of the n eck is appropriate for th e eval- • Granulomatous diseases, such as sarcoidosis, cause
uation of this patien t if a nodule is suspected. However, hypercalcemia through increased !-a - hydroxylation
th is patient's diffusely e nlarged thyroid gland is not activity that increases 1,25-dihyd roxyvitamin D levels
suggestive of nodu lar disease. In addition, even if the and calcium reabsorption.
physica l examinatio n were suggestive of nod ula r dis-
ease, the first step wou ld be evaluation of the funct ional Bibliography
thyroid statu s. Sharma OP. Hy pe rcalcemia in granulomato us disorders: a din ical revk w.
Curr Opin Pulm 'vied. 2000 Sep;6(5):44:l-7. [PMID: 10958237]
KEY POINT
• Most patients w ith thyrotoxicosis w ill have elevations
of both free thyroxine (T) and triiodothyronine (T), Item 23 Answer: D
but isolated T3 elevation is rare. Educational Objective: Diagnose Cushing syndrome
from exogenous glucocorticoids.
Bibliography
Vaidya B. Pearce SH. Diagnosis a nd management of thyrotoxicosis. llMJ. The pa tient has iat rogen ic Cushing syndrome cau sed
2014 Aug 21 ;349:g5128. [PMID: :l$1463901 by use of topical glucoco rticoids in the treat ment of

106
Answers and Critiques

psoriasis. Cushing synd rome presents similarly whether it KEY POINT


is due to a pituitary adenoma (Cushing disease), adrenal
• During the in itial management of diabetic ketoacido-
tumor cortisol production , ectopic adrenocorticotropic
sis, hypokalemia should be corrected before initiation
hormone production, or excessive use of glucocorticoids.
of intravenous insulin therapy to avoid significant
Her p resentation would be consistent w it h any of these
diagnoses; however, she is on high- potency topical gl u- worsening of the serum potassium levels that could
cocorticoids, so this alone expla ins her symptoms and cause cardiac arrhythmias.
presentation. Exogenous glucocort icoid use as a cause of
Cushing syndrome is common, whereas the other causes Bibliography
a re rare. Wilson JF. In the clinic. Diabetic ketoacidosis. An n Intern Med. 2010 Jan
5:152(J):ITCHS. [PMID: 20048266]
KEY POINT
• The most common cause of Cushing syndrome is an
Item 25 Answer: D
elevated level of cor tisol resulting from both endoge-
nous and exogenous exposure to glucocorticoids. Educational Objective: Identify secondary causes of
bone loss prior to initiating bisphosphonate therapy.
Bibliography Checking this patient's serum thyroid-stimulating hormone
Nieman LK, Biller BMK, Findling JW, eta!. The diagnosis of Cu shing's syn- (TSH) level would be the most appropriate test to obtain
drome: an Endocrine Society clinical practice f,'l.lideline. J Clin Endocri nol
Metab. 2008 May;93(5):1526-40. [PMID: 183345801 prior to initiation of pharmacologic therapy. Although osteo-
porosis in postmenopausal women is most commonly asso-
ciated with nonmodifiable risk factors such as age, sex,
B menopausal status, height, and build, it is always important
Cl Item 24 Answer:
Educational Objective: Treat a patient with diabetic
to assess for possible secondary causes of bone loss that
might be amenable to treatment, particularly ifthere is clin-
ketoacidosis.
ical suspicion in a specific patient. Appropriate laboratory
This patient has diabetic ketoacidosis (DKA) and a low serum testing for most patients with newly diagnosed osteoporosis
potassium level, so potassium chloride should be adminis- includes complete blood count (for malignancy) , complete
tered. The patient should be making adequate urine before metabolic profile (for calcium levels and kidney fu nction),
administration of potassium replacement. Low potassium TSH, 25-hydroxyvitamin D, and calcium (screening fo r
stores in the body will need correction prior to initiation hypercalciuria). most of which were normal in this patient.
of insulin therapy to avoid cardiac arrhythmias. Potassium However, this patient's h istory of unintentional weight loss
chloride should be added to each liter of intravenous fluids to over the past year may be her only symptom of hyperthy-
maintain the serum potassium level in 4.0 to 5.0 mEq. L roidism. which could be contributing to her osteoporosis·
(4.0-5.0 mmoLL) range with the continued use of intrave and would be important to treat in addition to therapy
nous insulin therapy. directed toward her osteoporosis. 111erefore, measuring this
Antibiotic therapy is not warranted at this time. The patient's ISH level would be appropriate prior to starting
mild leukocytosis upon presentation is most likely related to therapy tor osteoporosis.
stress from DKA rather than an infectious process. In addi- Both serum and urine markers of bone turnover mea-
tion, the chest radiograph is normal. sure collagen breakdown products and other chemicals
Sodium bicarbonate provides no added benefit when released from osteoclasts and osteoblasts as part of bone
the arterial blood pH is greater than 6. 9 and may be asso- metabol ism. However, they are not commonly used in most
ciated with harm. A 2011 systematic review found that patients with osteoporosis primarily because there is signif-
bicarbonate ad ministration worsened ketonemia. Several icant variability in the different measures in an individual
studies found higher potassium requirements in patients patient or between different patients, making standard-
receiving bicarbonate. Studies in children found a possi ization of results difticult. 1heretore, their use is typically
ble association between bicarbonate therapy and cerebral limited to research settings or in management of specific
edema. patients who have failed to respond to usual therapy tor
Du ring the treatment of DKA, initiation of insulin osteoporosis.
therapy and correction of acidosis will shift the extracel- There is evidence that estrogen is effective for preven-
lular potassium back into the intracellular space. Signifi- tion and possibly treatment for osteoporosis, although the
cant worsening of the hypokalemia noted at presentation significant nonskeletal risks associated w ith this therapy
could occur if this is not corrected to a serum potassium have led to its not being used in favor of bisphosphonates.
level higher than 3.3 mEq tL (3.3 mmol·L) prior to init i- As this patient is postmenopausal and has no clinical sug-
ation of insulin therapy, wh ich could lead to the devel- gestion of excess estradiol secretion, and bisphosphonate
opment of arrhythmias. Insulin t herapy can be safely therapy would be considered preferable to estrogen despite
initiated once the seru m potassium level is greater than serum levels, testing for estradiol would not be indicated in
3.3 mEq;L (3 .3 mmoi /L). this patient.

107
Answers and Critiques

Hyperpara thyroidism should always be considered as KEY POINT


a possible secondary cause of bone loss. However, since
• In women with polycystic ovmy syndrome, heavy
this patienL had normal calcium and 25- hydro.\.yvitamin
menstrual bleeding, and hirsutism who do not desire
D levels, hyperparathyroidism would be highly unli kely,
fertility. estrogen-progestin oral contraceptive pills are
and checking a parathyroid ho rmone level would not be
indicated. first-line therapy to provide endometrial protection
and suppress androgen production.
KEY POINT
• Testing for secondary causes of bone loss is appropri- Bibliography
ate before beginni ng pharmacologic therapy for newly Legro RS. Arslanian SA. Ehrmann DA. ct al: Endocrine Sociery. Diagnosis
and treatment of polycystic ovary syndrome: an Endocrine Society clini
diagnosed osteoporosis. cal practice guideline. J Clin Endocrinol Metab. 2013 Dcc:98(L2):-1565 92 .
[PM!D: 24151290)

Bibliography
lludec SM. Camacho PM. Secondal}· causes of osteoporosi;. Endocr Pract.
201:!: 19: 120 H. IPMID: 231869-19) Item 27 Answer: D
Educational Objective: Treat adrenocortical
carcinoma.
Item 26 Answer: A
Adrenocortical carcinoma (ACC) is the most likely cause of
Educational Objective: Manage heavy menstrual
this patient's Cushing syndrome, and surgical excision is the
bleeding caused by polycystic ovary syndrome.
most appropriate management. TI1e patient has classic clinical
This patient w ith polycyst ic ova ry syndrom e (PCOS) manifestations of Cushing syndrome (CS). and the 24-hour
with heavy menstru al bleeding and hirsu tism shou ld be urine cortisol is markedly elevated on repeated measure-
treated wi th combined est rogen-progestin oral con tra- ments. Plasma adrenocorticotropic hormone (ACTH) is sup-
ceptive pills. Pati ents w ith PCOS rema in in a stagnant pressed consistent with an ACTH-independent cause. The
follicular stage resul ting in u nopposed estradiol secretion imaging characteristics of ACC include a large mass with
fro m small ovaria n follicles, causing proliferatio n of the irregula r borders or shape, calciOca tion , high attenuation
endome triu m in t he absence of progesterone secretion (high Hounsfield units) on CT. and delay in contrast medium
!'ro m a co rpu s lu teum. This predisposes pat ients to e ndo- v,rashout (less than 50'1., at 10 minutes). findings all present in
met ria l hyperpl asia a nd heavy menstrual bleedi ng as a this patient.
result of anovulatory bleeding. lntraovarian a nd rogen The treatment of ACC depends on the extent of dis-
prod uction is also increased in PCOS, resulting in the ease at presentation. Surgical removal after appropriate bio-
hyperandrogenism and hirsutism associated wi t h the chemical assessment remains the best option, especially in
disorder. Estrogen- progestin oral contraceptive pills a re patients \Nith early disease. Even after appa rent complete -
first- line t herapy for the menstrual irregularities and resection, adjuvant therapy with mitotane, a known adrenal
hirsutism associated with PCOS. This t herapy prevents cytotoxic drug. may be beneficial. Treatment with mito-
unopposed estrogen- induced proliferatio n of the endo- tane is recommended tor patients with persistent disease
metrium and suppresses the excess androgen production and others with known metastases and is associated with
associa ted with PCOS. This would be appropriate therapy objective remissions in approximately 25% of patients. The
to treat both issues in this patient, who does not currently main factors limiting the use of mitotane include nausea,
des ire fertility. vomiting, lethargy, and neurologic side cftects. Experience
Progest in therapy alone. either through periodic pro- with other cytotoxic chemotherapy is li mited, but has usu-
gestin w it hdrawal or use of a progesterone-eluting intrauter- ally been ineffective. A poorer prognosis is associated with
ine device, w ill provide endometrial protection and treat this advanced stages of the disease, the presence of metastasis at
patient's menstrual irregularity. However. progestin therapy diagnosis. an older age. and cortisol hypersecretion by the
alone does not suppress androgen production and would not tu mor. In patients w ithout clinically evident disease after
treat this patient's hirsutism. initial surgery. the median survival rate is 60% at 5 years.
Metformin has several favorable metabolic effects Fine-needle biopsy cannot distinguish a benign ade-
in patients with PCOS, including increased insulin sen noma from a carcinoma and is not used in the evaluation
si tivity and reduced serum free testosterone. However. it of ACC. It is sometimes used to distinguish ACC from meta-
has been shown to be less effective than oral contracep- static disease. Its use in this patient is both unnecessary and
tives tor improving the m enstrual pattern and reducing inappropriate.
serum androgens. It also does not provide e ndometrial Radiation therapy is not used as the pri ncipal initial
protection a nd is considered a second li ne therapy for treatment in ACC; however, it may be used as adjuvant ther-
pa tients with PCOS w ith significa nt menst rual irreg- apy after surgery to prevent tumor recurrence. Radiation
ularities a nd hirsutism who are unable to tolerate oral therapy can also be used to treat areas of metastasis, such as
co ntraceptive p il ls. to the bones or brain.

108
Answers and Critiques

KEY POINT Bibliography


Shoback. [).Hypoparathyroidism. N Eng! J Med. 20 08 Jul24:359(4):391-403.
• Surgical removal after appropriate biochemical lPMID: 18650515]
assessment is the most appropriate treatment for
adrenocortical carcinoma, especially in patients with
early disease. Item 29 Answer: C
Educational Objective: Manage early type 2 diabetes
Bibliography
mellitus.
llerruti A. 13audin E. Gelderblom II. el a!. Adrenal cancer: ESMO Clinical
Practice Gu idelines fu r diagnosis. treatmen t and follow-up. Ann Oncul.
2012:23 Suppl 7:vii131 138. [PM!D: 22997446) The most appropriate management for this patien t is to
initiate metformin. The pat ient is early in her diabetes
disease course without evidence of m icrovascular dis-
I:::J Item 28 Answer: D ease. For otherwise healthy adults meeting these cri teria ,
Educational Objective: Diagnose hypomagnesemia the American Diabetes Association recommends a hemo-
as a cause of hypocalcemia. oJobin
o · Atc level of less than 7.0%, preprandial glucose
values of 70 to 130 mg/dL (3.9-7.2 mmol/L), and 1- to
This patient's magnesium level should be checked. He has a 2- hour postprandial glucose values of less than 180 mg/clL
very low calciu m level that is likely contributing to his clinical (10 mmoli L) . Because the patient has not met these goals,
findings of tremulousness. muscle irritability. and electro a pharmacologic agent should be added at this time. Life-
cardiogram changes. He struggles with alcohol abuse. and style recommendations consisting of increased physical
his low albumin level suggests malnutrition. likely due to his activity. dietary modifications, and weight loss (if 13M! is
chronic alcohol intake as a primary source of calories. Mag elevated) are the initial fi rst step in treating diabetes. When
nesium deficiency is common in persons who abuse alcohol. lifestyle modifications fail to meet glycemic goals wit hi n
Furthermore, he has had diarrhea for several days, which w ill 6 weeks, metformin is the recommended first-line t her-
a lso deplete his magnesium stores. '! his patient's low calcium apy to be started in conj unction with contin ued lifestyle
level should promote parathyroid hormone (PTH) secretion modi fications. If glycemic goals are not met after 3 months
to help correct the hypocalcemia. However. decreased levels of lifestyle modifications a nd mettorm in use, additional
of magnesium impair release of PTH; levels in patients with agents should be added lo the regimen every 3 months
significant magnesium deficiency are either low or inappro until glucose goals are met.
priately in the t_lOrmal range. Low magnesium levels are also Dapagli flozin, a sodium-glucose transporter- 2 (SGLT-
associated w ith resistance to PTH activity at the level of bone. 2) inhibitor, increases excretion of glucose through the
further contributing to hypocalcemia. Therefore. in patients kidney. It is a second-line agent that should be used after
with hypocalcemia and hypomagnesemia. it is crucial to cor lifestyle modifications and metformin fail to reach glyce-.
rect the magnesium level (to at least 2mg dL [0.83 mmol L]) . mic goals.
as it is difficult to increase calcium levels until this is done. The sulfonylurea glipizide stimulates insulin secretion
1,25 Dihyd roxyvitamin 0 has a short half life. and from the pancreatic beta cells. This agent could improve
its measurement only reflects the active levels of vitamin the patient' s postprandial hyperglycemia, but it may also
D. Since active levels tend to vacillate frequently based on induce weight gain in a patient actively working on weight
immediate need. this level would not be diagnostically help- loss. Glipizide is a second-line agent that should be used
ful. 25-Hydroxyvitamin D levels have a longer half life and. after lifestyle modifications and mettormin fail to reach
therefore. more accurately reflect the total body stores of glycemic goals.
vitamin D. Sitagliptin , a dipeptidyl peptidase- 4 (DPP-4) inhib-
Measuring 24-hour urine calcium excretion in this itor, improves glycemic control by slowing gastric emp-
patient would not be diagnostically helpful as significant tying and suppressing glucagon secretion. It is also con-
urinary loss of calcium is uncommon. and the urine calcium sidered a second- line agent tha t might be considered if
level in this patient wou ld be expected to be low due to com life style modifications and metfo rmin fa il to reach glyce-
pensatory kidney retention of calcium. mic goals.
Checking the ionized calcium level is indicated in settings
KEY POINT
where abnormal serum protein binding of calcium is possible.
'!his test is not needed in this symptomatic patient vvho has no • For most patients with type 2 diabetes mellitus, life-
suggestion of excessive protein binding of calcium. style modifkations and metformin therapy are the
most appropriate initial treatments.
KEY POl NT
• Low serum magnesium levels impair parathyroid hor- Bibliography
mone secretion and require repletion before serum American Diabetes Association. (7) Ap proaches to glycemic treatmen t. ln:
calcium levels may be corrected. Standards of Medical Ca re in Diabe tes-2015. [)iabetes Ca re. 20 15 Jan:38
Suppl l :S'11 8 . [PMID: 255377071

109
Answers and _ __ __ _ _ _ _ __

Item 30 Answer: C symptoms associated with hypercalcemia. Approximately


Educational Obj ective: Identify hemochromatosis as 40'Yo to 45% of the calcium in serum is bound to protein,
a cause of hypogonadotropic hypogonadism. principally albumin. although the physiologically active
form of calcium is in an ionized (or free) state. In most
The patient has a clinical history suspicious for hemochro- patients with relatively normal serum albumin levels, the
matosis and should be further evaluated by measuring serum total calci um usually accurately reflects the ionized calcium
transferrin saturation and ferritin levels. 1he patient has fractio n. However, in clinical settings where increased pro-
hypogonadism based on his clinical symptoms of decreased tein binding of calcium may occur, the serum total calcium
libido and erectile dysfunction, associated with a low morn- level may be elevated without a rise in the actual serum
ing serum testosterone level. A hypogonadotropic etiology ionized calcium concentration. This may occur in patients
is indicated by his low luteinizing and follicle-stimulati ng with hyperalbuminuria (as may occur in those who are
hormone levels. Causes of hypogonadotropic hypogonadism severely dehydrated) , and in patients with a paraprotein
include infiltrative diseases such as hemochromatosis. sar- capable of binding calcium (such as occasionally occurs in
coidosis, cancer metastatic to the pituitary, and lymphoma. some patients with multiple myeloma). This phenomenon
Pituitary tumors that impair gonadotropin function may is sometimes p seudohypercalcemia (or factitious
also be a cause. This patient has several clinical findings hypercalcemia). If present, a normal ionized calcium level
suggestive of possible hemochromatosis, including a report may indicate that the elevated total calcium levels are due
of arthralgia and hepatomegaly on physical examination. to excessive protein binding and potentially eliminate the
Therefore. the next step in evaluation of this patient's hypo- need for further evaluation for hypercalcemia.
gonadotropic hypogonadism is measurement of serum fer- Measuring the 1,25-dihydroxyvitamin D level is useful
ritin level and transferrin saturation to evaluate for possible in further assessing patients with non- parathyroid hormone
hemochromatosis. (PTH)-mediated hypercalcemia to assess for excess vita-
'!he cause of hypogonadism must be evaluated prior min D production. PTH testing is indicated in patients with
to the initiation of testosterone replacement. If testosterone hypercalcemia to differentiate between PTH-mediated and
therapy is sta rted without testing for hemochromatosis. the non- PTH-media ted hypercalcemia.
diagnosis may be missed. The hypercalcemia associated with multiple myeloma
Although genetic disorders such as Kli nefe lter syn- is caused primarily by tumor-induced, osteoclast-mediated
drome (47,XXY) may cause hypogonadism. patients with bone resorption due to cytokines released by myeloma cells
th is syndrome have hypergonadotropic hypogonadism with and is not PTH-mediated or due to excessive vitamin D lev-
elevated luteinizing and follicle-stimulating hormone values. els. Measurement of PTH level and 1,25-dihydroxyvitamin
unlike this patient. Therefore, karyotyping is not indicated. levels may be indicated as part of this patient's evaluation
A testicular ultrasound is used to evaluate the cause of only after true hypercalcemia has been established.
prima ry testicular failure and is not indicated in the eval- Parathyroid hormone- related protein (PTHrP) level
uation of hypogonadotropic hypogonadism. Th is patient's measurement is useful in evaluating patients with non-
low gonadotropin levels indicate either a hypothalamic or PTH-mediated hypercalcemia but would not be indicated as
pituitary disorder. instead of testicular disease. Although a next study in this patient in whom pseudohypercalcemia
hemochroma tosis may also directly affect testicular func- has not been excluded.
tion in addition to its central hypogonadal effect, testicular
KEY POINT
ultrasound is not helpful in establishing the diagnosis of
hemochromatosis as a cause of hypogonadism. • In patients with conditions in which increased pro-
tein binding of calcium may occur, such as hyperal-
KEY POINT
buminemia or paraproteinemia, an artificially ele-
• Patients with symptoms of hypogonadotropic hypo- vated total serum calcium level must be excluded.
gonadism should have serum transferrin saturation
and ferritin concentration levels measured to identify Bibliography
hemochromatosis prior to initiating any therapy. Clines GA. and treatment of hypercalcemia of malignancy.
Curr Opin Endocrinol Diabetes Obes. 2011 Dec;l 8(6):339 -16. [P:v!I D:
21897221]
Bibliography
B:1con BR. Adam PC. Kowdley Kv: et al. Diagnosis and management of
hemochromatosis: 2011 Practice guidelines by the American Association
for t he Study of Liver Diseases. Hepatology. 2011:54(1):328 -13. Item 32 Answer: 8
21-1522901
Educational Objective: Manage androgen therapy in
the setting of hypogonadism.
Item 31 Answer: 8
Before initiating therapy for this patient with hypogonad-
Educational Objective: Diagnose pseudohypercalcemia.
ism, his desire for fe rtility should be explored. Testoster-
This patient' s ionized calcium level should be checked. one replacement therapy can be associated with decreased
His total calcium level is elevated, but he is without clear spermatogenesis and infertility. Exogenous testosterone

110
Answers and Critiques

suppresses both hypothalamic gonadotropin-releasing hor- D


mone a nd pituitary follicle-stimulating hormone and lutein-
Item 33 Answer:
Educational Objective: Diagnose thyroid storm.
Cl
izing hormo ne (LH) production, resulting in depletion of
intratesticula r testostero ne. The eftect is suppression of sper- '!his patient has thyroid storm. most likely precipitated by the
matogenesis so pronounced that testosterone replacement iodine contained in the dye load from the cardiac catheteriza
therapy has been studied as a male hormonal contraceptive. lion in the context of undertreated Graves disease f(Jllowing
Based on Endocrine Society gu idelines, men with hypogo- her dbcontinuation of methimazole. Graves disease is the
nad ism should be treated with exogenous testosterone when most common underlying condition associated with thyroid
they have consistent signs and symptoms of hypogonadism storm. and onset may be triggered by a number of factors in
and low serum testosterone levels. Symptomatic men may addition to iodine, including infection, surgery. myocardial
report reduced libido, erec tile dysfunction, mood changes. infarction. trauma. or parturition. ']his patient has typical
irritability, fatigue, or memory loss. Although this patient's features ofthjroid srorm: fever. tachycardia. heart failure. gas
symptoms would likely be improved wi th exogenous ad min- trointcstinal dysfunction. and mental status changes. Scoring
istration of and rogen. replacement therapy may also result in exist that .can be used to provide a more objectiYe
infertil ity due to oligospermia. Patients w ith hypogonadism measurement of the severity of the thyrotoxicosis. Treatment
who desire fertility may require treatment with human cho- of thyroid storm is directed toward supportive care. reduction
rionic gonadotropin (HCG). HCG has LH-like activity and of th) raid hormone production, decreasing peripheral con
stimulates the production of intratesticular testosterone, \ersion of thyroxine (T 1) to triiodothyronine (T) . addressing
resulti ng in the high concentrations required for induction adrenergic symptoms and thermoregulatory changes, ami
and maintena nce of spermatoge nesis. treating any idenlified precipitating factors. Thionamides and
Asymptomatic men w ith low serum testosterone levels P blockers are the mainstay of treatment to reduce thyroid
may experience decreased bone mineral density and oste- hormone production and control adrenergic symptoms.
oporosis. Hormone replacement therapy will decrease the ·n,is patient's clinical presentation is not consistent with
risk of osteoporosis. A bone mineral density measurement euthyroid sick syndrome. Although this patient's low level
prior to the initiation of hormone replacement therapy is of serum thyroid stimulating hormone (TSH) is consistent
not needed. with euthyroid sick syndrome. her elenlted serum thyroid
Male hypogonadism is associated w ith increased vis- hormone level is not. In euthyroid sick syndrome, serum
ceral fat and insulin resistance, and hormone replacement thyroid hormone levels are typically low, creating a clinical
therapy improves these metabolic parameters. 'lliere is no picture similar to that seen in hypothyroidism.
recommendatie n that hypogonadal patients in itia ting hor- Pheochromocytoma typically presents with hyperten
mone replacement therapy be screened tor diabetes mel- sion and tachycardia but thyromegaly is not a clinical feature
litus w ith fasting plasma glucose measurement or other of this disorder. Additionally. confus ion and altered mental
testing. status are not presenting signs of the disease.
A 2010 systematic review of hypogonadal men receiv- Subacute thyroiditis is typically precipitated by a recent
ing testosterone therapy found no evidence of increased viral illness. An iodine load. such as this patient had. is an
risk of prostate cancer when compared with the placebo/ unlikely cause. Although patients with subacute thyroiditis
noninter vention group. The Endocrine Society guideline on may present with significant thyrotoxicosis. the thyroid is
testosterone replacement therapy recommends a digital rec- typically painful to palpation. a clinical finding that was not
tal examination and prostate-specific antigen (PSA) level present in this patient.
determination at 3 and 6 months following the initiat ion of KEY POINT
replacement therapy. Continued regular screening is recom-
mended for men older than 40 years of age with a baseline • In patients with underlying Graves disease, thyroid
PSA level greater than 6 ng/mL (6 Scrotal ultrasound storm may be precipitated by the iodine content
is unnecessary prior to inilialion of testosterone therapy; a fou nd in contrast media.
clinical testicul ar examination to rule out abnormali ties or a
testicular mass is sufficient. Bibliography
Klubo-Gwiezdzinska J I. L. Thyroid eme rgencies. Med Clin North
KEY POINT Am. :1012 :Vlar:96{1):3S5-403.IPM ID: 224439821

• Prior to ini tiation oftestosterone therapy for hypo-


gonadism, the desi re for fe rtility should be ascer-
Item 34 Answer: D
tained because exogenous testosterone replacement
Educational Objective: Manage postsurgical
therapy may result in oligospermia and infertility.
hypoparathyroidism.

Bibliography Th is patient without parathyroid function should be advised


Samphtski M K. l.oai Y, Wong K. l.o KC. Grober ED, Jarvi KA. Testosterone use to decrease his calcium supplementation intake. In patients
in the m ale in.tertilily popu lation: prescribing patterns and effects on
semen and hormonal parameters. Fertil Steril. 2014 Jan: l0l (l):6-l 9.
without parathyroid function, stimulation of the kidney to
IPMID: 24094422] convert 25-hydroxyvitamin D (from the liver) into the active

11 1
Answers and Critiques

form, 1,25- dihydrO>..'YVitamin D (calcitriol). is lost, as is the rapid-acting insulin, before intensive exercise. Since the dura-
signal to increase reabsorption of calcium in the kidney in the tion of action of insulin glulisine ca n extend up to 4 hours, cov-
distal convoluted tubule and loop of Henle. As a result, both ering the meal consumption prior to exercise w ith a smaller
calcitriol and calci um supplementation must be prc,:idPd. dose of insulin glulisine can reduce the risk of hypoglycemia
as was done in this patient. However. without reabsorption in the setting of intense or prolonged exercise.
of calcium by the kidney. oral calcium will be absorbed and Discontinuation of insulin glargine in a patient with
passed through the kidney, resulting in higher levels of urine type 1 diabetes will lead to hyperglycemia if" the rapid-acting
calcium than in patients with normal parathyroid hormone insulin isn't adjusted to provide basal insulin coverage. The
levels. In mon itoring calciu m status in hypoparathyroid hyperglycemia and insu lin deficiency that develop in the
patients. the recommended goal should be a 24-hour urine absence of basal insuli n coupled with the stress associated
calcium level or less than 300 mg/24 hours (7.5 11111101124 h) with exercise w ill lead to an increase in the release of coun-
with a concomitant serum calcium level in the low normal terregulatory hormones. In this scena rio. there may not
range (8.0 8.5 mg/dL l2.0-2.1 mmol!L]). If the urine calcium be sufficient insulin to decrease lipolysis and subsequent
levels are surpassed. it is appropriate to decrease the calcium oxidation of free tatty acids. This could lead to diabetic
intake first: in patients with serum calcium levels greater ketoacidosis.
than 8.5 mg/dL (2.1 mmoi /L) and urine calcium greater than The meal- time insuli n prior to exercise should be
300 mg/24 hours (7.5 mmoll24 h). concurrent decreases in decreased; however, modification of the diet with increase
both calcium supplementation and calcitri ol are indicated. in carbohydrates. rather t han protein, can also help avoid
Although 25 hyd roxyvitamin D levels are the best indi- exercise-induced hypoglycemia. Consumption of 15 to 30
cator of total body vitamin D stores in patients with normal grams of carbohydrates prior to exercise and/or a snack with
parathyroid function. it would not be helpful for therapeutic complex carbohydrates after prolonged exercise can help
decision-making in this patient as the lack of PTH requires mitigate the risk of hypoglycemia. Carbohydrates. especially
treatment with activated (1.25-d i.hydroxy) vitami n D. In this simple ones. can rapidly provide glucose to the bloodstream
patient, 1.25-dihydroxyvitamin D levels measured concur and maximize glycogen stores in the liver that can be utilized
rently with the serum and urine calcium levels are the most for fuel during exercise. TI1e digestion time for protein is
appropriate indicators of therapeutic eftect. prolonged compared with carbohydrates, thus providing a
Removal of the parathyroid glands does not allow the slower source of energy during exercise
PTH level to be used to monitor appropriateness of therapy. A sliding-scale regimen of insulin glulisine is a reactive
It would be expected to be low or undetectable in this patient management plan for glucose control. In this scenario, it
who has had resection of his parathyroid glands and chemo- is possible that the patient could have an increased risk of
therapy and radiation. hyperglycemia or hypoglycem ia prior to. during, or after
It would be inappropriate to continue the current exercise secondary to insufficient or excessive doses of insu-
regimen given t he elevated 24-hour calcium levels. lin from the sliding-scale regimen.
Persistently e levated levels could lead to nephrolithiasis or
KEY POINT
nephrocalcinosis.
• Because exercise can increase glucose utilization by
KEY POl NT
the muscles. reducing the doses of mealtime insulin in
• The urine and serum calcium goals are different in a patiem wilh diabetes mellitus will decrease the risk
patients with hypoparathyroidism; the urine calcium of hypoglycemia w ith intense or prolonged exercise.
goat is less than 300 mg/24 hours (7.5 mmol 24 h)
and serum calcium goal is between 8.0 and 8.5 mg/dL Bibliography
(2 -2.1 mmol!L) in these patients. America n Diabetes Foundations of care: education.
tion. physical activity. smoking cess:1tio n. psychosocial care, and
nizalion. In: Standa rd!> of vtedical Care in Diabetes 2015. Diabetes Care.
Bibliography 20 15 Jan::lS Suppl l :520 :JO. [PMID: 25537702)
1-Jlan MS. SG. Hu Medical managemen t of postsurgical
hypoparathy roidism. Endocr Pract. 2011 Apr: 17 Suppl
21134871 1
Item 36 Answer: A
Educational Objective: Diagnose central
Item 35 Answer: A hypothyroidism.
Educational Objective: Manage exercise-induced
Measurement of the serum l"ree thyroxine (T) level is the
hypoglycemia.
most appropriate next step in management for this patient
This patient should decrease his meal time insulin glulisine who has clinical evidence of hypothyroidism (fatigue, con-
dose prior to exercise and continue his insulin glargine. Exer- stipation. cold intolera nce. clry skin, delayed reflexes, ane-
cise can increase glucose utilization by the muscles. which can mia, and mild hyponatremia) but has had radiation to the
induce hypoglycem ia in the setting of exogenous insulin. This base of the skull. including the pituitary gland. Although
patient consumes a meal and administers insulin glulisine. a measurement of thyroid-stimulati ng hormone (TSH) is the

112
Answers and Critiques

most accurate reflection of thyroid status in patients with an rarely associated with hypercalcemia and therefore does not
intact hypothalamic-pituitary- thyroid axis, it is not a reliable require lherapy to lower serum calcium levels. Screening
measure of t hyroid function in patients in whom there is other family members for the d isorder is ind icated.
loss of hypot halamic-pituitary function, such as seen in this Bone densitometry is not indicated in this age group in
patient. His low-normal TSH in the context of clinical hypo- the absence of fragility fractures or other risk fac to rs such as
thyroidism suggests possible central hypothy roidism. and long-term high-close glucocorticoid use or prima ry hyper-
measurement of the circul ating level of thyroid hormone, the parathyroidism.
free serum T4 , wou ld therefore be a more accura te indication A parathyroid sesta mibi scan is a very useful nuclea r
of his thyroid status. imaging study for localization of adenomas in patients witll
Repeating the TSH measurement would not be appro- primary hyperparathyroidism or parathyroid cancer. How-
priate in th is patient with signs and symptoms of hypothy- ever. pri mary hyperparathyroidism has not been confirmed
roidism, as untreated hypothyroidism leads to increased in this patient w ith suspicio n for FHH, making this study
card iovascular morbid ity and mortality. In addition. because premature.
of likely central hypothyroid ism, the TSH level would remain 1he need for surgical treatment in this patient has also
an inaccurate ind icator of thyroid fu nction. not been established. Therefore, surgical referra l fo r parathy-
Thyroid scintigraphy is unlikely to distinguish t he roidectomy wou ld not be appropriate.
source of the hypothyroidism. as patien ts with primary
KEY POINT
or secondary hypothyroid ism have decreased radioactive
uptake. Thyroid scan ning is most helpfu l in elucidating the • Measurement of 24-hour uri ne calcium and
cause of hyperthyroidism. crea tinine levels will distinguish between primary
Ultrasound of t he neck is normal in patients with cen- hyperpa rat hyroidism and fam ilial hypocalciuric
tral hyperthyroidism and would be un likely to provide any hypercalcemia.
additional information about this patient's thyroid status.
Bibliography
KEY POINT
Shinall :V!C Jr. Dahir K:V!. Broome JT. Differentiating fam ilial hypocalciuric
• When central hypothyroidism is suspected. measure- hypercalcemia from primary hyperparathyroidi sm. Endocr Pract. 2013
Ju l-Aug: 19(-1) :697-702. [PMJD: 234256441
ment of the serum free thyroxine (T) level is essential.

Bibliography
Item 38 Answer: D
Persani L. Clin ical (evicw: Central hypothyroidism: palhogenic. diagnostic.
and therapeutic challenges. J Clin Endocrinol Metab. 2012 Sep:97(9):3068 Educational Objective: Evaluate timing of prandial
78.[PMID: 228514921
insulin in a patient with diabetes mellitus.

The mismatch of timing of insulin administration to food·


Item 37 Answer: 8 intake with meals, possibly related to the ti me demands of
her new job. is the most likely explanation for the erratic
Educational Objective: Diagnose familial hypocalciuric
glycemic fluctuations noted on this patient's blood glucose
hypercalcemia.
log. 'n1e adequacy or ller nocturnal long-acting insulin is
This patient's urine calcium and creatinine levels should be reflected in her near-goal pre-breakfast blood glucose levels.
measured. Her laboratory values are consistent with hyper However. the major fluctuations occurring around mealtimes
calcemia, and her parathyroid hormone level is toward the are best explained by inconsistent use of her immediate-act-
upper end of the normal range. In her age group and with ing insulin relative to food intal<e. Meal coverage with insulin
a fam ily member w it h a suspicious history. it is important shou ld mimic the physiologic pattern seen with endogenous
to distinguish between primary hyperparathyroidism and insulin secreted from pancreatic beta cells. Admi nistration of
fam ilial hypocalciuric hypercalcemia (FHH). The distinc- inunec!iate-acting insu Uns should therefore ideally occur just
tion between primary hyperparathyroidism and FHH can prior to or at the time of the meal consumption. Because of
be made by a 24-hou r urine collection for calcium and cre- the rapid onset of action with these agents, sh ifting the tim-
ati nine. which will establish the amount of kid ney calcium ing of admi nistration away from this physiologic pattern may
excretion and w il l allow evaluation of the calcium-creati- resul t in the blood glucose fluctuations seen in th is patient.
nine ratio. Total urine calcium of less than 200 mg/ 24 h (5 An important aspect of diabetes education is helping patients
mmol/ 24 h) and a calcium-creatinine ratio less than 0.01 are understand the actions of their prescribed insulin regimen
highly suggestive of fa milial hypocalciu ric hypercalcemia. and the importance of timing issues when using them.
FHH results from a mutation in a specific calcium-sens- Antibodies can develop in response to exposure to
ing receptor in the parathyroids and kidneys, and results exogenous insu lin ; however. these a ntibodies are rarely
in an upward shift in the range of calcium and PTH leading cli nically significa nt and would not adequately explain the
to these clinical fi ndi ngs. Although a rare entity, maki ng blood glucose pattern seen in this patient.
this d iagnosis is crucial because it may prevent unnecessary Gastroparesis ca n cause erratic blood glucose values due
parathyroidectomy for the patient. The course of FHH is to either rapid transit or delayed emptying of food within the

113
Answers and Critiques

digestive system. However. the lack of gastrointestinal symp PostoperatiYe norepinephrine is nor indicated. If the
toms, absence of cl inical evidence of other d iabetes related patient were to experience hypotension or shock postop
complications. and her hemoglobin A1c history suggesting erati\'ely. treatment with vasopressors in addition to gluco
good diabetic control make gastroparesis a less likely cause corticoid replacement \\ould be considered: howe\·cr. this
of her erratic blood glucose readings. \\ou ld not replace the administration of glucocorticoids in
Inadequate insulin dosi ng can cause fl uctuations in gly· th is population at risk for acute hypocortisolism.
ccmic control. However. this patient reports little variabili ty Preoperat in' phenoxyben1.ami ne is indicated in the
in her da ily diet. She also has several days w ith evidence of m anagement of patients with pheochromocytoma. not CS.
adequate glyce mic control on her current insul in regimen ·rhe purpose of preopcrati\·e a blockade is to pn)\'ide blood
doses. Therefore, inadequate dosing is less likely to be the pressure cont rol and decrease the risk or card iovascular
cause of her glycem ic variabili ty. com plications related to exccssiw catecholamine release
du ri ng intraopemti\l' man ip ul ation or the tum or.
KEY POINT
• Because meal coverage with insulin s hould m imic the KEY POINT

physiologic pattern seen w ith endogenous insulin • Following adrenalectomy. patien ts with adrenal
secreted from the pancreatic beta cells as closely as Cushing syndrome should be trea ted with stress -dose
possible. insulin administration in patients with dia- glucocorticoids during the postoperative period to
betes mellitus s hould ideally occur prior to or at the avoid the risk of acute hypocortisolis m.
time of the meal consumption.
Bibliography
Bibliography Di Datmazi G. Berr 01. Fassnacht et at. Adrenal fu nct ion after adrenal-
ectomy for subclinical hypcrcortisolism and Cushing·s syndrome: A
DeWi ll DE. Hirsch lB. Ou tpalielll insulin therapy in type I and rype 2 diabe systematic review of the literatu re. 201-1;99:2637-26-15. [PMID:
tes mel titus. JAMA. 2003 May 7:289(17):2251 [P\111): 12n4 137]
2.J878052]

Item 39 Answer: A Item 40 Answer: A


Educational Obje ctive: Manage a patient with Educational Objective: Manage Paget disease of bone.
Cushing syndrome following adre nalectomy.
Initiating antiresorptive therapy in th is patient with symp-
The most appropriate management nf thh patient under tomatic Paget disease ofbone is the most appropriate ne:\.'t step
going right ad renalectomy for ( ushing ndrome (( <.,) in management. Paget d isease of bone is characterized by focal
i-; po'itopenlli\·e hydrocortisone. The patient has adre areas of accelerated bone remodeling that ultimately causes
nocorticotropic hormone (.\CTH) independent ( <\. and ovcrgrovvth of bone at one or more sites that may impair the
a contrast enhanced adrenal CT scan .d emonstrated <1 integrity of affected bone. Areas common ly aflected include -
right adrenal mass \\ith imaging characteristics consis the skull. spine. pelvis. and long bones of the lower extrem-
tent \\ ith a benign adenoma. Adrenocortital <1denom<1'i ities. such as in this patient who has thickened cortical bone
typically ha\'e low attenuation on unenhanced cr scan and coarsening of the trabecular bone of the femur. The main
(d ens ity less than 10 Hounsfield unit-;) and exhibit rapid indications for ami resorptive therapy in most patients include
was hou l o f int ra\'enous iodine cont rast media (>SO", pain caused by increased bone metabolic act ivity (as in this
at 10 m inutes) . Following patients with patient) and hypercalcemia due to multiple affected sites. The
adrena l CS may de\'Ciop acu te adrenal became of most commonly used treatm en t agents arc nitrogen-contain-
hypo! hala mic pi tuita ry adrenal (H PA) axis su pp ressio n ing bisphosphonates (alendronate. pamidron<lte, ri sedronate,
and co ntra lateral acl rcnal atrophy. All patients should be and zoledronic acid): these are the newer bisphosphonates
trea ted vvith stress dos e glu cocorticoids <llld tapered to and have been the most extensively studied for treatm ent of
physiologic re placem en t until H PA axis is con Paget disease of bone. Bisphosphonates stabilize bone turn-
firmed . \ lost pa tients have adrenal im ufTicicnC\ lasting over by suppressing bone resorption and new bone formation
as long <lS 12 mont hs. with a resulting reduction in semm alkaline phosphatase
Postopcrati\e admi nistration of mitotane. an <ldrcno levels. There is no evidence that anliresorpt ivc therapy is ben-
lytic d rug. is recommended as adjm·ant therapy fi>r patients eficial in asymptomatic patients.
,,·ith locally per<>istent or metastatic adrenocortical carci Bone biopsy is rarely needed to establish the diagnosis
noma In these patients. mitotane is <lssociatcd with of Paget disease of bone when there arc characteristic radio-
objecti\'C remissions in approximate!\ 2.'>" of pat ients. 1his graphic fi ndings of bone turnover (concurrent osteolytic and
patient's CT scan find ings are not consistent with ACC Adre osteoblastic changes) and consistent laboratory studies (such
nal cancers are typically large (>-1 6 em) \\ith irregul<1r mar as elevated semm alkaline phosphatase levels). Bone biopsy
gi ns and areas of necrosis or calcification. L' nenhanced ( VI' may be usefu l in certain si tuations in which bone lesions are
wi ll dcmon<>trate high attenuation (density >10 Hounsticld pri marily osteoblastic (suggesti ng possible metastatic dis-
u nits). and washou l of intra\·enous iodine con trast media is ease) or osteolytic (possibly indi cating multiple myeloma).
\e<;s than so·:;, at lO m inutes. neither of which are present in this patient.

114
Answers and Critiques

The bone lesions of multiple myeloma are primarily proteins. ll1e increased thyroid hormone requirements of the
osteolytic, in which case further evaluation for that diagnosis fetus also contribute to this change in serum T4 levels.
w ith a serum and urine protein electrophoresis would be In this patient, with a serum TSH value above 2.5 J.!.UimL
appropriate. However. this patient's radiographic find ings (2.5 m UIL) during the first trimester, the dose of levothyrox-
are not consistent w ith a diagnosis of multiple myeloma, ine needs to be increased rather than decreased or discon-
and further evaluation for this disorder would therefore not tinued. The serum TSH level should be rechecked in 4 weeks
be indicated. to ensure that the close adjustment continues to be adequate.
Most patients with Paget disease of bone are asymp- Likewise, thyroid function tests should be repeated at least
tomatic and are identified only by elevated serum alkaline once during each trimester to ensure that additional adjust-
phosphatase levels detected on laboratory studies obtained ments in the levothyroxine dose are not needed.
for other reasons. In many patients w ith mild disease, clini-
KEY POINT
cal observation without initiation of therapy is appropriate.
However, in this patient with symptomatic disease in a • On confirmation of pregnancy in a patient taking lev-
critical weigh t-bea ring skeletal area, clinical observation othyroxine, semm thyroid-stimulating hormone
without treatment would not be appropriate. should be checked to determine the need for levothy-
roxine dose adj ustment.
KEY POINT
• Treatment of Paget disease of bone with anti resorptive Bibliography
agents is indicated in symptomatic patients, those Stagnaro-Green A. Abalovich M, Alexander E. et al : American Thyroid
with elevated calcium levels, or patients with involve- Association Taskforce on Thyroid Disease During Pregnancy and
Postpartum. Guidelines of the American Thyrnid Association for the
ment of skeletal areas at high risk of complications. d iagnosis and management of thyroid d isease during pregnancy and
incl uding fractures. postpartum . Thyroid. 2011 Oct :21(10 ):108 1- 12S. [P\110: 21787128]

Bibliography
Item 42 Answer: B
Ralston SH. Paget's disease of the bone. :\ Eng! J Med. 2013 Feb 14:368(7):6-14
SO. [PMID: 23406029] Educational Objective: Diagnose the cause of adrenal
Cl
failure.

Item 41 Answer: D ll1is patient most likely has primary adrenal failure due to
bilateral adrenal hemorrhage. He has acute onset nausea.
Educational Treat hypothyroidism in
lightheacledness. back and abdom inal pa in. and hypoten
pregnancy.
sion. vVhilc nonspecific. these findings arc consistent with
ll1 is pregnan t patient's levothyroxine dose should be adrenal failure. Additionally, laboratory studies show hypo
increased to lower the serum thyroid-stil1)ulati ng hormone. natremia. hyperkalemia. and hypocortisolemia. which are-
Maternal thyroid hormone production typically increases by also consistent with the diagnosis. A sudden drop in the
30% to 50% during pregnancy; therefore. in pregnant patients hemoglobin and hematocrit. as seen in this patient. may he
requiring levothyroxine supplementation, the replacement present w ithout C\'idence of bleeding elsewhere. Risk fac
dose usually needs to be increased to provide adequate thy- tors fo r adrenal hemorrhage include ant icoagulant therapy
roxine (T) for the neurologic development of the fetus. The (and may occur w ith treatment level'> within the therapeutic
combination of this patient's elevated thyroid-stimulating range), the postoperative state. abnormalities of hemostasis
hormone (TSH) level and low total T4 level suggest that her (such as heparin induced thrombocytopenia or ant iphos
levothyroxine dose should be increased. During pregnancy, pholipicl antibody syndrome) . and sepsis. Because adrenal
the physiologic changes in thyroid hormone levels include a hemorrhage is uncommon and the associated fi ndings may
reduction in the serum TSH level and an increase in the serum be relatively nonspecific. an increased level of suspicion is
total T., level. ll1is change in the serum TSH level is partly required for the diagnosis in at-risk patients: failure to iden-
due to the rise in the serum human chorionic gonadotropin tify acute adrenal failure in a timely manner may lead to car-
(HCG) level; both hormones share sequence homology in diovascular collapse. Adrenal hemorrhage can often be visu
their a subunit. As the serum HCG level rises w ith progression alized on abdominal CT imaging. Treatment of acute adrenal
of pregnancy, the hormone can bind to the TSH receptors, failure is wit h stress close glucocorticoid<> (hydrocort isone.
resulting in a reduction in serum TSH levels. Consequently. SO 100 mg intravenously every 6 8 hours) and supportive
the normal reference range for serum TSH during pregnancy care vvit h intravenous fluids and ,·asopressors as needed for
shifts to a lower value, from 0.5 to 5.0 J.! UimL (0.5-5.0 mUlL) hemodynamic compromise.
pre-pregnancy to 0 .03 to 2.5 J.!UimL (0.03- 2.5 m UlL) during Although autoimmune acl renalitis may cause primary
the first trimester. Additionally, the serum total T.1 level rises adrenal fai lure and is associated with the presence of other
1.5- told above the normal nonpregnant reference range. Part autoimmune d iseases (such as hypothyroid ism in this
of this increase is due to the higher levels of estrogen associ- patient) . the onset of symptoms related to hypocortisolism
ated with pregnancy, which cause an increase in serum total are usually more gradual. ;mel skin hvperpigmentation is
protein levels, including serum thyroid hormone- binding often seen on examination.

115
Answers and Critiques

Cl lionsChronic
CONT.
administration of long acting opiate medica
is a kno'A n cause of hypogonadotropic hypogonad
Measuring 24-hour urine free catecholamine and
nephrine levels would be used to screen for
ism and a potential etiology of secondary adrenal insum cytoma. If this patient had an adrenal adenoma. she would
ciency. 1 he administration of relatively short acting opiates certainly need to be screened for pheochromocytoma prior
(oxycodone) on an as needed basis for postoperative pain to surgery. but she does not have a known adrena l lesion. Her
is unlikely to cause clinically significant hypothalamic- sympto ms and comorbidities are explained by her diagnosis
pituitary adrenal axis disturbance. Hypogonadotropic of Cushing disease, and she has no specific signs or symp-
hypogonadism would also not present with hyperkalemia toms that require an evaluation for pheochromocytoma.
because mineralocorticoid secretion is preserved. A PET scan is not necessary or indicated in the evalu-
Pitui tary apoplexy results from acute hemorrhage ation of Cushing syndrome. Cushing disease is caused by a
into the pituitary and may result in adrenal insuflkiency. pituitary adenoma and is not malig nant. Cushing syndrome
Although this patient might be at increased risk due to due to ectopic ACTH productio n is often caused by a can-
anticoagulation. he does not have headache or visual clistur cer. PET scan may be indicated to t ry to loca lize the source
bances. which are common in pituitary apoplexy. Because of ectopic ACTH. but the source of the excessive ACTH is
pituitary apoplexy causes secondary adrenal insufficiency. it al ready know n to'be from a pituitary adenoma.
would not result in hyperkalemia.
KEY POINT
KEY POINT • Patients with hypercortisolism should have a screen-
• Patients with bilateral adrenal hemorrhage typically ing dual-energy x-ray absorptiometry scan because
present with clinical features of acute cortisol and they are at a high risk of osteoporosis and fracture.
aldosterone deficiency, including gastrointestinal dis-
turbance, lethargy. weakness, hypotension, shock, Bibliography
hypoglycemia, and electrolyte imbalances. such as r\'ieman LK. Biller BMK. Findling JW. et aL The diagnosis of Cushing's syn-
drome: an Endocrine Society clinical pmctice guideline. J Clin l'ndocrinol
hyponatremia and hyperkalemia. b. 2008 \1ay:93(5): 1526 40. [PM!D: 18334580]

Bibliography
Rosenberger Ll f. Smith PW, Sav.yer RG. et aL Bilateral adrenal hemorrhage:
the unrecognized cause of hemodynam ic collapse associated wirh hepa
rin induced thrombocytopenia. Crit Care Med. 2011 Apr:39(4):833 8
Item 44 Answer: D
Educational Objective: Manage primary adrenal failure.
Cl
[PMIJ): 212427991
'Ihe most appropriate regimen for the long term treatment of
this patient's primary adrenal failure would be prednisone,
ltem43 Answer: C S mg once daily, and fludrocortisone. 0 .05 mg once daily. In
primary adrenal failure, there is a failure in the production -
Educational Objective: Evaluate a with a new
of all the hormones of the adrenal cortex. Patients therefore
diagnosis of hypercortisolism for osteoporosis.
require both glucocorticoid and mineralocorticoid replace
This patient with newly diagnosed hypercortisolism is at ment Because she is no longer ill. physiologic replacement
high risk for osteoporosis and fracture. and a screening dual- doses are appropriate. Prednisone primarily has glucocor
energy x ray absorptiometry (DEXA) scan is indicated. 1he ticoid activity, with 5 mg being considered a physiologic
mechanism of osteoporosis in patients with Cushing syn- replacement dose. It is also long acting so may be adminis
drome is related to decreased intestinal calcium absorp- tered once daily in combination with fludrocortisone. which
tion, decreased bone forma tion. increased bone resorption, has almost pure mineralocorticoicl properties. Physiologic
and decreased rena l calcium reabsorption. Patients with low replacement doses offludrocortisone arc 0.05 to 2 mg per clay.
bone density should be considered for bisphosphonate treat- Dexamethasone is primarily a glucocorticoid and
ment to reduce the risk of frac ture. could be used as the glucocorticoid portion of combination
An 8 mg dexa methasone suppression test can be used replacement therapy with a mineralocorticoid such as
to help localize the source of adrenocorticotropic hormone rocortisone. However, dexamethasone alone would not be
(ACTH) in a patient w ith Cushing syndrome. Patients with appropriate therapy for primary adrenal failure due to its
pituitary Cushing disease respond to 8 mg of dexamethasone intrinsic lack of mineralocorticoid activity.
w ith a suppressed cortisol leveL In comparison. patients Fludrocortisone alone is also inappropriate because it
with ectopic ACTH production do not respond to 8 mg of would not provide glucocorticoid replacement.
dexamethasone. a nd their cortisol levels remain elevated. Hydrocortisone has both glucocorticoid and mineralo
However. this test is associated with a number of false corticoid properties. with primarily glucocorticoid activity
positive results. The 8-mg dexamethasone suppression test is at physiologic replacement doses of 12.5 to 25 mg in two to
not indicated for this patient because she has already had a three divided doses daily. At total daily doses above so mg,
more defi nitive test. Intrapetrosal sinus sampling has better hydrocortisone has adequate mineralocorticoid activity to
sensitivity and specificity when completed by a skilled inter- allow for its use as monotherapy. However, treatment w ith
vcntional radiologist. hydrocortisone, 10 mg three times daily, does not provide

116
Answers and Critiques

CJ aadequate mineralocorticoid replacement. while it supplies


supmphysiologic amount of glucocorticoid that could lead
CT are typically a patchy infiltrate with minimal lymphade-
nopathy. This patient's image reveals marked diffuse enlarge-
CONT. to iatrogenic Cushing syndrome if administered on a long ment of the thyroid. and she d id not have a history of prior
term hasis. illness.
Patients w it h pri mary adrenal failure require additional
KEY POINT
glucocorticoid at times of physiologic stress. Treatment fbr
minor stress (upper respira tory in fection, fever. minor su r • Primary thyroid lymphoma most often occurs in
gery under local anesthesia) is typically two to three times elderly women with underlying hypothyroidism; the
the b<Jsal dose of hydrocortisone (or equivalent), fo r moder typical presentation includes rapidly enlarging goiter,
ate stress (mi nor or modera te su rgery with general anest he weight loss, and n ight sweats, and imaging reveals a
si<l) usually IS to 75 mg day. and major stress (major surgery. d iffusely enlarged thyroid.
trauma. critical illness. or childbir th) up to ISO to 200 mg
day wit h a gradual taper fbllowing resolut ion of the stress. Bibliography
Kim HC. Han MH. Kim KH. et aJ. Primary thyroid ly mpho ma: CT findings.
KEY POINT l::ur J Radio!. 2003 Jun:46(3):233-9. 12758117]
• Patients with primary adrenal failure require both
glucocorticoid and mineralocorticoid replacement in
physiologic doses.
Item 46 Answer: E
Educational Objective: Treat primary hyperparathy-
Bibliography roidism and concomitant vitamin D deficiency.
Neary , . .'Heman L. Adrenal insufficiency: diagnosis and treatment. Th e fT\OSt appropriate treatment of this patient is to replete
Curr Opin Endocrinol Diabetes Obes. 2010 Jun:l7t3):217- 23.[P:\11D:
2037S886I his vitamin D deficiency with a supplement such as vitamin
D:l (cholecalciferol). He has primary hyperparathyroidism as
shown by his elevated serum calcium and parathyroid hor-
ltem45 Answer: c mone levels. However. there is a high prevalence of concurrent
vitamin D deficiency and insufficiency in patients with pri-
Educational Objective: Diagnose primary thyroid
mary hyperparathyroidism. and low levels of 2S-hydroxyvi-
lymphoma.
tamin D can sti mulate parathyroid hormone secretion in
The patient has primary thyroid lymphoma. which most often non-adenomatous glands. Because of this. measurement of
occurs in elderly women with a long-standing histOI}' of vitamin D levels should be ordered as part of the evaluation of
I Jashimoto thyroiditis. The clinical presentation is typically primary hyperparathyroidism. and repletion should be pro-
one of rapid onset (weeks) of an enlarging goiter with vveight vided if identified. In these patients. it is important to replace
loss and night sweats. The diagnosis is made by biopsy of the their vitamin D to a level of at least 30 ng/dL (7S nmol/L).
thyroid with flow cytometry. Treatment typically involves After this level is reached. the patient should be placed on a
chemotherapy and /or radiation therapy. Thyroidectomy is vitamin D dosage to maintain that value. The choice to use
usually not needed. cholecalciferol versus ergocalciferol is often based on the level
CT scan of the neck. rather than ultrasound. was of vitamin D deficiency. Since ergocalciferol is more readily
ordered in th is patient because of the compressive symp- available in the SO,OOO U form and has a shorter half life, it
toms and positional breathing issues. CT scan all ows visual- is recommended when a patient's vitamin D level is less than
ization of the enl arged thyroid gland a nd assessment of the 10 ng/mL (25 nmol/L). Cholecalciferol is often used when
patency of the trachea. ln this image. the "dough nu t'' sign the level is between 20 and 30 ng/mL (SO 7S nmo l/L) or for
ca n be seen. whereby the enlarged thyroid extends behind maintenance. Since this patient already has hypercalcemia
and completely encircles the trachea. and low-dose repletion is desired, the lower doses (400 U
New-onset Graves disease is unlikely to occur in a daily) of vitamin D3 (over-the-counter cholecalciferol) should
patient of this age, particula rly with her long-standing his- be used. This patient's semm calcium s hould be mon itored at
tory of hypothyroidism. Furthermore. there is no bruit or least monthly.
other clinical sign of Graves disease. and the thyromegaly This patient does not meet the threshold for surgery. His
associa ted with Graves disease is not acute in onset. semm total calcium level is less than 1 standard deviation
This patient is unlikely to have papillary thyroid cancer. from upper limit of norma l. his bone density score is not in
as these tumors typically grow very slowly. in contrast to the osteoporotic or treatment range. he is older than SO years
the acute onset of her findings. Additionally. the thyroid of age, and his glomemlar filtration rate is preserved.
is not typical ly diffusely en larged. as seen on th is CT scan. Alendronate would be an excellent option for calcium
Rather, a distinct nod ule and potentially concomitant cervi- reduction and simultaneous treatment of osteoporosis if
cal lymphadenopathy would be expected. his T-scores were lower or Fracture Risk Assessment Tool
Subacute (de Quervain) thyroiditis is associated with (FRAX) scores were h igher. The FRA.X calculator cleflnes the
acute onset of anterior neck pai n. It is typically seen follow- 10-year fracture risk for pat ients with T-scores in the - 1.0
ing a vira l illness in the preceding months. The changes on to - 2.S range. The FRAX calculator (vvvvvv.shef. ac.uk/ FRAX)

117
Answers and

incorporates mult iple risk fac tors including sex, frac ture orga n fu nction is stable a nd gl ucose levels have returned
history, femoral neck bone mineral density, steroid usage, to baseli ne values.
smoking, BMI. age, and alcohol intake to determi ne pro- Sliding scale insulin is nonphysiologic and can result
jected fracture risk. If the risk of major osteoporotic fracture in large fluctuations in blood glucose levels. Sliding scale
is greater than or equal to 20% or the risk of hip fracture is insul in is not recommended as the sole insuli n therapy in
greater than or equal to 3%, the patient's benefit from ther- the hospital setting.
apy exceeds the risk and t reatment should be offered.
KEY POINT
Calcitonin is an option for reducing his calcium levels,
but he is currently asymptomatic and does not warrant cal- • For non-critically ill hospi talized patients with diabe-
cium lowering. tes mellitus and hyperglycemia, a weight-based treat-
Ci nacalcet, a calcimimetic agent, is another option for ment plan that includes basal and prandial insulin is
lowering calcium for symptomatic patients with kidney recommended.
involvement. Th is patient has preserved kidney func tion
and no symptoms. Due to t he cost and potential side effects Bibliography
of cinacalcet, it would not be indicated at this time. American Diabetes Association. (13) Diabetes care in Ihe hospital. n ursing
home. and skilled nursing facility. In: Stan dards o f Medica l Care in
KEY POINT Diabctcs- 2015. Diabetes Care. 2015 Jan;38 Su ppl:SS0-5 . [PM ID:
25537715]
• In patients with primary hyperparathyroidism and con-
comitant vitamin D deficiency, 25-hydroxyvitamin D
levels should be repleted to at least 30 ng/dL (75 nmol!L)
Item 48 Answer: B
to prevent further parathyroid honnone stin1ulation.
Educational Objective: Diagnose Cushing syndrome.
Bibliography The most appropriate next diagnostic test for this patient is
Ho lick Bin k ley N, Bischoff-Ferrari HA. Gordon C. \lanky D. Heaney R. the 1-mg dexamethasone suppression test. She has the typical
Weaver C. Evaluation, treatment and prevention of vitamin D deficiency:
an Endocrine Sociery clin ical p ractice guideline. J Clin Endocrinol .'vletab. clinical features and findings of cortisol excess, or Cushing
201! Jul:96(7):1911-30. [PMID: 21646368] syndrome. The most common cause of Cushing syndrome is
exogeneous glucocorticoid use; however, she has not received
glucocorticoids. To evaluate for Cushing syndrome, biochem-
Item 47 Answer: A ical evidence of hypercortisolism must be confirmed by use
of several screening tests. Three screening tests are used for
Educational Objective: Manage diabetes mellitus in a
Cushing syndrome: the 1-mg dexamethasone suppression
hospitalized patient.
test (given late at night with assessment of cortisol suppres-
The most appropriate treatment for this diabetes sion the next morning) , 24-hour urine free cortisol excretion ·
mellitus while hospi talized is a weight based treatment (to quantifY total daily cortisol secretion), and measurement
plan that incl udes basal and prandial insulin . Hypergly of evening salivary cortisol (which normally reaches a nadir
cem ia in the hospital is associated w ith poor outcomes. at that time but remains elevated in patients with Cushing
Acco rd ing to the American Diabetes Associati on and syndrome). At least two abnormal first-line screening tests
American Association of Clinical Endocrinologists. glu are required for diagnosis. Only after establishing biochemical
cose goals in hospitalized patients in a non ICU setting are hypercortisolism should the source of excess cortisol produc-
premeal values less than 140 mg dL (7.8 mmol ' L) a nd ran tion be sought.
dom values less than 180 mg 'dL (10 mmol L) . The Amer Measurement of adrenocorticotropic hormone (ACTH)
ican College of Physicia ns recommends avoiding values is not a screening test for Cushing syndrome. After docu-
less tha n 140 mg tdL (7.8 mmol L) to decrease the risk of mentation of excess cortisol production. ACTH levels may
hypoglycem ic complications. The patient's plasma glucose be useful in determining if hypercortisolism is ACTH-de-
values exceed the recommended gu idelines and require pendent or - independent; however, it is not an appropriate
treatment. initial screening test.
Ora l agents do not have safety or efficacy data for use An 8-mg dexamethasone suppression test is helpful in
in the hospilal. Glipizide is an insulin secretagogue that can differentiating between Cushing disease (pitui tary tu mor-
potentially induce hypoglycemia in the hospital setting, par secreting ACTH) and ectopic ACTH production. However,
ticula rly with unpredictable changes in oral intake. it is not a screening test for Cushing syndrome and would
With metformin u se, hosp italized patie nts ca n be appropriate only in specific situations after Cus hing syn-
develop poor organ perfusion. which can increase the drome is diagnosed.
risk of lactic acidosis. Intravenous cont rast dye ca n also A pituitary MRI should be ordered only after hypercor-
impa ir kid ney fun ction in the setting of metto rmin use in tisolism and Cushing syndrome are diagnosed and a pitu-
the hospital. Reinit iation of the patient's home regimen itary adenoma is suspected as a cause.
of metformin at or near the lime of discharge is most Measurement of serum cortisol levels lacks sensitivity
appropria te after all procedures have been co mpleted and and specificity fo r diagnosing Cush ing syndrome, primarily

118
Answers and Critiques

due to the pulsa tile nature of cortisol secretion, and is not Item 50 Answer: D
used as a screening test. Educational Objective: Diagnose an incidentally
KEY POINT noted adrenal mass.
• To screen for Cushing syndrome, biochemical evi- The most appropriate next step in management is to evalu-
dence of hypercortisolism must be confirmed by a ate for pheochromocytoma, preferably by measurement of
1-mg dexamethasone suppression test. 24- hour urine plasma free metanephrines in this patient. Approximately
free cortisol testing, and/or measurement of evening 10% to 15% of incidentally discovered adrenal masses are
salivary cortisol levels. functional, although most have no overt cli nical manifesta-
tions. Therefore, all patients with an incidentally noted adre-
Bibliography nal mass should be evaluated fo r the autonomous secretion
Niema n J.K. Biller BMK. Find ling JW. et al. The diagnosis of Cushing's syn of cortisol and catecholam ines. and those with hypertension
drome: An Endocrine Society clinical practice guideline. J Clin should also undergo testing for primary hyperaldosteronism.
Entlocrinol Metab. 2008 May:93(S):IS26--IO. [PMID: 18334580]
The low-close suppression test should be
performed to evaluate for subclinical Cushing syndrome
ltem49 Answer: A given its superior sensitivity compared with other screen-
ing tests (24-hour urine free cortisol and late- night salivary
Educational Objective: Treat an obese patient with
cortisol): this test was negative in this patient. Measurement
type 2 diabetes mellitus with bariatric surgery.
of 24- hour urine metanephri nes and catecholam ines is the
Patients with a BM I between 35 and 40 with one or more usual fi rst test in most asymptomatic patients to evaluate
complica tions associated with obesity should be con- for catecholamine hypersecretion. although in those vvith
sidered for bariatric surgery, with the goal of sign ifica nt imaging suggestive of pheochromocytoma, measurement of
weight loss and improvement in metabolic abnorma li- plasma free metanephrines is the preferred study because of
ties. ·lhis obese patient has type 2 d iabetes mell itus with its very high sensitivity and high negative pred ictive value fo r
advanced microvascu lar disease along w ith other compli- a normal study Imaging in this patient reveals a well-circum-
ca tions associated w ith obesity including hypertension, scribed partially cystic lesion with high allenuation on non-
hyperlipidemia. obstructive sleep apnea, gastroesophageal contrast CT scan. which is in keepi ng with a pheochromocy-
acid re flu x disease. and osteoa rth ritis. His attempts at li fe toma and is not typical of an ad renocortical adenoma, wh ich
style management with d iet and exercise d id not result characteristically has low allenualion on CT scan (density <10
in weight loss- that substantially improved his metabolic Hounsfteld units) due to relatively high lipid content.
abnormalities or obesity. Management of an adrena l inciden taloma depends
Increasi ng the insul in doses could potentially exac- on its size. imagi ng characteristics (phenotype) , and hor-
e rbate the weekly hypoglycemic even ts occu rring in a monal fun ctioning. Almost all adrenal tumors that are
patient w it h hypoglycemic unawareness. This could also overtly func tional are larger than 6 em in size or have unfa -
lead to more weight gain due to frequent treatments of vorable imaging characteristics should be considered fo r
hypoglycemi<l. surgical removal. However. biopsy or surgical resection of
Metformin is contraindicated in men with a serum any adrenal mass prio r to ruling out a pheoch romocytoma
creati nine level above 1.5 mg/dL (132.6 J.lmol/L) due ro the is not recommended. as any manipu lation of a catechol-
possibility of lacti.c acidosis and will not address the under amine-secret ing tumor w ithout appropria te preoperative
lying problem of obesity. management can p recipitate a hypertensive crisi s.
Pra ml in tide slows gastric emptying, which can decrease Measurement of the plasma aldosterone to plasma renin
appetite. 1he weight loss associated w ith the use of pram lin- ratio is indicated as part of the evaluation of an inciclentally
tide is modest. and it may not be sufficient to improve the discovered adrenal mass in a patient wit h hypertension, but
metabolic abnormalit ies and obesity-related complications not in this individual w ho has normal blood pressure.
in th is patient. In addition. the patient's hypoglycemia may Not performing additional testing may miss a subclin-
be exace rbated by pramlintide. ical pheochromocytoma and would therefore not be an
appropriate next step in management.
KEY POINT
• Obese persons (BMl between 35 and 40) w ith type 2 KEY POINT
diabetes mellitus and associated complications should • A patient with an incidentally noted adrenal mass
be considered for bariatric surgery. should undergo biochemical testing for subclinical
Cush ing syndrome and pheochromocytoma. and
Bibliography those with hyper tension should also be evalua ted for
Meehn nick Jl. Youdim !I. Jones DB. et al. Clinical practice guidelines for the primary hyperaldosteronism.
perioperative n utrition al. metabolic, and nonsurgical support of the
barintric surgery pat ient 2013 u pdate: cosponsored by American
Association of Clinical Endocrinologists. The Obesity Society. and
Bibliography
American Society for Metabolic & Bariatric Surgery. Endocr Pract. 2013 Arnaldi G. Boscaro M. Adrenal incidentaloma. Best Pract Res Clin Endocrinol
Mar Apr:J9(2):337 72. [PMJI): 23529351 ] Metab. 2012; 26:405-419. 1PMID: 22863384]

119
Answers and Critiques

Item 51 Answer: D androgen-producing adrenal tumor. Approximately 50% of


Educational Objective: Manage the limitations of androgen-producing adrenal tumors are ben ign adenomas,
hemoglobin A 1 c measurements in a patient with while the oilier half are malignant. Symptoms are usually
diabetes mellitus and chronic kidney disease. minimal or absent in adult men. Women typically present
with rapidly progressive signs and symptoms of androgen
l h is patient should measure his postprandial glucose level. excess, including acne. hirsutism, and virilization (deepen ing
ll1e hemoglobin A1e measurement is not al\.vays reliable in the of the voice, clitoromegaly. and male- pattern hair loss), and
setting of chronic kidney disease: thus fingerstick blood glu- may also have irregular menses. In patients with clinical evi-
cose measurements should be closely evaluated to help guide dence ofhyperandrogenism, biochemical testing is performed
therapy. This patient's slightly elevated tasting and premeal prior to imaging and should include measu rement of serum
blood glucose values may be indicative of postprandial hyper- testosterone and clehyd roepiandrosterone sulfate (DHEAS),
glycemia tllat could be detected with postprandial glucose an adrenal androgen. This patient's biochemical evaluation
measurements and used to guide therapy. For patients rrying has revealed marked elevation of DHEAS and mild eleva-
to achieve hemoglobin A1e levels less than 7.0%, fasting and tion of testosterone, making an androgen producing adrenal
premeal glucose targets usually are set at approximately 80 to tumor the most likely diagnosis. DHEAS levels above 8
130 mg/dL (4.4-7.2 mmol/L). This patient's slightly out of range (21.6 are diagnostic of an androgen-producing adre-
fasting and premeal glucose measurements seem at odds with nal tumor. The elevated semm testosterone level seen in this
the recent drop in his hemoglobin A1, level to 6.2'Yc,. Specific patien t is likely a consequence of the perip heral metabolism
scenarios UJlique to end-stage kidney disease can affect the of adrenal androgens to testosterone. ll1e serum testosterone
accuracy of the hemoglobin A1e measurement. Hemoglobin A10 level would be more than 150 to 200 ng/dL (5.2 6.9 nmol/ L)
can be falsely elevated in the setting of chronic kidney disease in the setting of an androgen producing ovarian n1mor.
due to carbamylated hemoglobin secondary to uremia inter- Pe.rforming a low-dose dexamethasone s uppression test
fering with some of the assays. Hemoglobin A1e can be fa lsely or pituitary MRl is not indicated because Cushing syndrome
decreased in the setting of a reduced erythrocyte li fespa n. iron is unlikely w ith a normal 24 hour urine free cortisol level
deficiency, blood transfusions. and increased erythropoiesis and in the absence of speci fic feat ures of hypercortisolism
with erythropoietin use. In th.is patient, the fingerstick blood (facial plethora, violaceous striae. and su praclavicular or
glucose measurements do not correlate with the most recent dorsocervical fat pads).
hemoglobin A1c after initiation of erythropoietin. The hemo- Pelvic ultrasound is not an appropriate initial imag-
globin A1e value is falsely decreased after erythropoietin ther- ing test because the marked elevation of DHEAS makes an
apy as a result of a change in the proportion of young and old androgen-producing adrenal tumor much more likely than
erythrocytes and a change in the rate of glycation. an ovarian neoplas m. Although the patient has a fami ly
The patient's fingcrstick blood glucose values are ele- history of polycystic ovary syndrome (PCOS) , the tempo and
vated, which further increases risk for n1icrovascular and severity of her clin ical presentation are not in keeping with
macrovascular damage. The current regimen should be this disorder. Al though ad renal and rogen excess occurs in
adjusted to decrease hyperglycemia. 30% to 40'Yo of women with PCOS. on ly a mild elevation of
Decreasing the insulin detemir close wou ld increase DH EAS (3 mL [8.1 is expected.
hyperglycemia based on daily blood glucose data provided
KEY POINT
by the patient.
Discontinuation of the preprandial insulin glulisine based • In female patients, signs of androgen excess such as
on the falsely decreased hemoglobin A1, level would increase progressive hirsutism and vi rilization over a short
the hyperglycemia noted in the fingerstick blood glucose values. period of time suggest the d iagnosis of an androgen -
producing ad renal or ovarian tumor.
KEY POINT
• End-stage kidney disease in patients with diabetes mel- Bibliography
litus can affect the accuracy of the hemoglobin A1c meas- Cavlan D. Bharwani X Grossman A. Androgen and estrogen secreting
urement, which should not be used to guide adrenal cancers. Semin Oncol. 2010 Dec:37(6):638-·18.IPMID: 21167382]

Bibliography
Ng JM. Cooke M. Bhandari S. ct a! The effect of iron and erythropoietin Item 53 Answer: A
treatment on the Ale of patients with diabetes and chronic kidney Educational Objective: Manage diabetes mellitus
disease. Diabetes Care. 2010 Nov:33(11):23 10 :J. [PMID: 20798337]
with continuous glucose monitoring.

Item 52 Answer: A 1l1ere is a discrepancy between this patient's flngerstick blood


glucose values and her hemoglobin A1e values that can be
Educational Objective: Diagnose an androgen-pro-
quickly reconciled with a 72-hour continuous blood glucose
ducing adrenal tumor.
monitoring system. Continuous blood glucose monitoring
The most appropriate diagnostic test to perform next is systems use an electrochemical enzymatic sensor to mea-
an abdominal CT sca n to confirm the diagnosis of an su re the glucose content of interstitial fluid via insertion of

120
Answers and Critiques

a subcutaneous needle. In some systems data recording can terminal hair growth w ill be slowed with combined oral con-
be made avai lable in real time to the patient, whereas other traceptive use. Oral contraceptives that contain 30 to 35 !Jg
models store the data for later access and analysis. Since she of ethinyl estradiol appear to be more effective in managing
does not have kidney disease or an emia that could affect hirsutism than formulations containing less ethinyl estradiol.
the accuracy of hemoglobin A1c measurements. she likely has Six months of treatment is considered the minimal interval in
episodes or hyperglycemia not detected by her current moni- which to determine the level of response. Adherence to an oral
toring effo rts. r:ingerstick blood glucose values only provide a contraceptive regi men will provide this patienl wilh predict-
small snapshot of the glucose variability that occurs through- able menses as well as contraceptive beneflt. In addition. the
out the day. Undetected hyperglycemia or hypoglycemia can risk of endometrial hyperplasia is diminished.
lead to significa nt diffe rences between the fingerstick blood Intermittent progesterone w ithd rawal, although effec-
glucose va lues and the expected hemoglobin A1c level. Her tive for decreasing the risk of endometrial hyperplasia,
wide ra nge of !asli ng blood glucose values could be indicative would have no effect on this patient's concern regarding
of undetected overnight hypoglycemia. Intermittent continu- hirsutism.
ous glucose mon itoring is recommended when postprandial 'Ihe levonorgestrel intrauterine system is a long-acting,
hyperglycem ia. dawn phenomenon. or overnight hypoglyce- reversible contra.ceptivc device that diminishes long term
mia is suspected. risk of endometrial hyperplasia in patients with PCOS; how-
Lifestyle modifications are recommended for glycemic ever, it provides no benefit for hirsutism and has no effect on
management: however, because this patient exercises in the androgen production.
even ing, overnight hyp oglycemia should be considered and Spironolactone is a potent anliandrogen and is very
evaluated with continuous glucose monitoring. Additional eflectiYe against male-pattern hirsutism in patients with
exercise may exacerbate the hypoglycem ia. PCOS. However, it otters no benefit fo r co ntrol of the men-
This patient docs not have evidence of postprandial strual -cycle. When spi ronolactone is prescribed. patients
hyperglycemia on her tingerstick blood glucose measure- should be counseled regarding the potential teratogenicity
ments. although it could be missed since she on ly measures in male fetuses, and a concu rrent reliable cont raceptive
after her meals periodically. Given the discrepancy in her method should be established to prevent Jetal exposure.
blood glucose values and A1,. level. hypoglycemia should be Pregnancy can still occur in patients with oligo-ovulatory
ruled out first before increasing her insulin doses that may PCOS. and reliance on mcnstmal irregularity is not a substi
increase the risk of hypoglycemia. lute for a more proven contraceptive plan.
KEY POINT KEY POINT
• Continuous glucose monitoring may be useful in per- • The combined oral contraceptive pill is the optimal
sons w ith postprandial hyperglycemia. dawn phe- treatment to address both irregular menses and hir-
nomenon. or overnight hypoglycemia. sutism in patients with polycystic ovary syndrome.

Bibliography Bibliography
Klonoff DC. Buckingham B. Christiansen JS. ct al. Continuous glut'Ose Amsterdam ESHRE Sponsored 3rd PCOS Consensus \Norkshop
monitoring: An Endocrine Society Clinical Practice Guideline. J CUn Group. Consensus on women's health aspects of polycystic ovary S) n
Endocrinol Metab. 2011: Oct:96(10):2968 2979. 219767-15) drome (PCOS). llum Reprod. 2012 Jnn;27(1):14 2-1. 22147920]

Item 54 Answer: A Item 55 Answer: C


Educational Objective: Treat irregular menses Educational Objective: Manage a patient with
and hirsutism in a patient with polycystic ovary pheochromocytoma.
syndrome.
The most appropriate next step in management is to begin
111e most appropriate treatment is combined oral contracep- treatment with an a -adrenoceptor antagonist. such as phe-
tive pi lls. Hirsutism is present in approximately 70% of women noxybenzamine. The purpose of preoperative a.- blockade is
with polycystic ovary syndrome (PCOS) . The combined oral to provide blood pressure control and decrease the risk of car-
contraceptive pill is the optimal treatment to address both this diovascular complications related to excessive catecholamine
patient's concerns of menstrual irregularity and hirsutism. release during intraoperative manipulation of the tumor. Most
TI1e estrogen component increases hepatic production of sex patients are treated for 1 to 2 weeks before surgery with phe-
hormone bi nding globulin. decreasi ng the patient's circulat- noxybenzamine with upward titration based on blood pres-
ing free testosterone level. For women in whom hirsutism is sure. The target blood pressure is below130/80 mm Hgseated
a major concern, treatment is focused on reducing androgen and greater tha n 90 mm Hg (systol ic) sta nding. Because of
production. decreasing the fraction of circulating free tes- phenoxybenzamine's side effects including orthoslasis, nasal
tosterone. and limi ting androgen bioactivity to hair follicles. stuffi ness. fa tigue, and retrograde ejaculation, some clinicia ns
Coarse. thick hairs tha t are already noted on examination use short-acting specific a -antagonists. such as prazosin, dox-
will need to be removed with a depilatory method: however. azosin. or terazosin. In patients with tachycardia,

121
Answers and Critiques

can be added after a-blockade is achieved. Labetalol. a com- hydroco rtisone is adjusted based on symptoms, such as
bined a - and agent. also can be used, especially orthostasis. weight loss, nausea. vomiting, and lighthead-
in patients with tachyarrhythmias. A heart rate of 60 to 70/ edness. He does not have these symptoms, so h is cortisol
min seated and 70 to SO/ min standing can be targeted in most deficiency is adequately treated. Increasi ng his hydrocor-
patients. Patients with pheochromocytoma who are normo- tisone to higher than necessa ry doses increases the risk of
tensive also should be treated with a -blockers because they iatrogenic Cushing syndrome and glucocorticoid-induced
often become hypertensive during surgical resection. osteoporosis.
Increasing the dosage of lisinopril does not address the His testosterone value is normal. and he has normal
need fo r preopera tive pharmacologic management of the morning erections. These are two signs that his hypogonad-
patient's pheochro mocytoma with a -blockade. ism is adequate ly treated. Possibly. h is erectile dysfunction
Although indicated for tumor localization following is a result of taligue from hypothyroid ism or is functional
the biochemical diagnosis of pheochromocytoma. a con- instead of physiologic.
trast-enhanced adrenal cr scan should not be performed ·n1ere is no reason to stop somatropin. l ie has no evi-
until after an a -adrenoceptor antagonist has been in itiated. dence of residual t.umor. Growth hormone (G H) replacement
Administering iodine contrast media to a patient who has can improve lean mass distribution and quality of lire in a
not received a -blockade could incite a hypertensive crisis. patient with true GH deficiency, so it is reasonable to con-
Similarly. the antagonist propranolol tinue. Discontinuing GI-l will likely worsen his fatigue.
should not be given prior to a -blockade because unopposed
KEY POINT
a -adrenoceptor stimulation could also precipitate a hyper
tensive crisis. • Patients with seconda ry hypothyroidism from pitui-
tary dysfunction have low or low-normal thyroid-
KEY POINT
stimulating hormone values, so levothyroxi ne dose
• In patients with a contirmed diagnosis of pheochro- should be adjusted based on free thyroxine (T) leve l.
mocytoma. a - blockade should be instituted before
surgery to reduce the risk of cardiovascular complica- Bibliography
tions and to control blood pressure. Schne ider HJ. Aimaretti G. Kreitschmam1-Andermahr. el al.llypopituitarism.
Lancet. 2007 Apr 28;369(9571):1461 70. [!'MID: 174675171

Bibliography
Lenders JW. Duh QY. Eisenhofer G, et al. Pheochromo,-ytoma and parJgan-
glio ma: an endocrine society clinical practice guideline. J Clin Endocrinol Item 57 Answer: A
Metab. 2014 Jun:99(6):1915 42. IPMIO: 2·1893135]
Educational Objective: Diagnose type 2 diabetes
mellitus.

Item 56 Answer: C A fasting plasma glucose measurement is the most appro-


priate diagnostic test for this patient. Diabetes mellitus
Educational Objective: Manage hormone replace-
ca n be diagnosed with an abnormal result of one screen-
ment therapy in a patient with panhypopituitarism.
ing test performed on two separa te occasions. Although
The patient's hypothyroidism is inadequately treated. causing the hemoglobin A1c is normal in this patient, t he lasti ng
symptoms of fatigue and weight gain and a low free thy- plasma glucose is abnormally elevated w ithin the diag-
roxine (T) .level. Therefore. his levothyroxine dose should nostic range for diabetes mellitus. When discrepa nt results
be increased. P'alients with secondary hypothyroidism from occur a mong diffe rent screening tests for diabetes. the
pituitary dysfunction have low or low normal thyroid-stimu- American Diabetes Association recommends repeating the
lating hormone (TSH) values which cannot be used to assess abnormal screening test. If the repeat plasma glu-
the adequacy of thyroid hom1one replacement. Because of cose measurement is abnormal. the diagnosis of diabetes
this, the levothyroxine dose is adjusted based on free T 1 levels is confirmed. Screening lor type 2 d iabetes should begin in
instead orTSH values. His free is low, suggesting inadequate all asymptomatic patients at age 45 years. In ad ult patients
treatment as the likely cause of his symptoms. with a BMl greater than o r equal to 25. screening should
The patient's desmopressin dosing is adequate to treat occur at any age if one or more additional risk factors tor
his diabetes insipidus. His symptoms are well con trolled diabetes is present.
without evidence of excessive urination. a nd his serum Use of the hemoglobi n A1c as an initial screening test in
sodium level is normal. In addition. increasi ng the close this patient is appropriate as there is no evidence for anemia
would risk potentially causing water retention and hypo- or kidney or liver disease that could decrease the reliability
natremia. of the test. The value was normal and docs not warrant a
He is on a physiologic dose of hydrocortisone. Hydro- repeat measurement as the next diagnostic test to perform
cortisone dose is not adjusted based on laboratory test results in this scenario.
beca use his endogenous adrenoco rticotropic hormone A 2 hour 75 g oral glucose tolerance test can be used
(ACfH) and cortisol levels will remain low on adequate as a screening tool for diagnosing diabetes. Since this lest
therapy and are therefore not used to alter therapy. Instead. was not initia lly used for screeni ng in this patient, it is most

122
_ _ _A_n_s_w
_ ers C ritiques

appropriate to repeat the abnormal screening test (fasting nadism w ith normal prolactin levels. Therefore, measmement
plasma glucose) that was already used for comparison. of serum prolactin is not the most useful test for this patient.
A random blood glucose measurement would be useful
KEY POINT
in this patient if he presented w ith classic hyperglycemic
symptoms in the setting of a blood glucose level of 200 mg/ • Klinefelter syndrome is a com mon cause of hypergo-
dL (11.1 mmol!L) or above. as that would be diagnostic of nadotropic hypogonadism and azoospermia.
d iabetes. This patient is not symptomatic.
Bibliography
KEY POINT
Krausz C. Chianese C. Genetic testing and counselling for male inferlility.
• When discrepant results occur among different Curr Opin Endocrinol Diabetes Obes. 201-t Jun:2l(3) :244 ·SO. IP.MID,
24739313]
screening tests for diabetes mellitus, the American
Diabetes Association recommends repeating the
abnormal screening test. Item 59 Answer: D
Educational Objective: Interpret thyroid function test
Bibliography results in an elderly patient.
American Diaberes Associatio n. (2) Classifkalion and diagnosis of diabetes.
In: Standards of Medical Care in Diabetes 2015. Diabetes Care. 2015;38 Clinical follow-up with repeat measurement of thyroid-stim-
Suppli:S8 16. [PMID: 255377141
ulating hormone (TSH) and free thyroxine (T) is the most
appropriate management for tl1is patient. In persons over
80 years of age, the serum TSH level may be mildly elevated
Item 58 Answer: A above the typical reference range for younger adults. The
Educational Objective: Diagnose Klinefelter upper. limit of the normal range in this elderly population
syndrome. without thyroid dysfunction may be as high as 8.0 ).l U/mL
(8.0 mU L). Since this patient's TSH level is 6.4 ).lU/mL (6.4
The most appropriate diagnost ic test to perform next is a
mUlL). his clinical symptoms are nonspecific, and his physi-
karyotype. 1his patient has evidence of hypergonadotropic
cal examination is normal, additional evaluation or treatment
hypogonadism based on elevated gonadotropin levels a nd
is not ind ic?ted at this time. However, his clinical symptoms
low testosterone level. Klinefelter syndrome is a common
should be monitored, and his serum TSH and free levels
cause of hypergonadotropic hypogonadism and azoosper-
should be measured repeatedly several times over a period of
mia, resulting in infertility. A 47.XXY karyotype is diagnostic
months to ensure that the TSH value-is not part of a trend to
of Klinefelter syndrome. Mosaic variants of this condition
the development of overt thyroid dysfunction.
exist but typically present with oligoasthenospermia. testic-
Because th is patient's serum free level is normal and
ular failure. or hypogonadism. Concomitant symptoms often
a diagnosis of hypothyroidism has not been established. he
include sexual dysfunction and generalized fatigue. TaUstat-
does not require levothyroxine therapy.
ure is a common finding. Patients with Klinefelter syndrome
Measurement of the serum total triiodothyron ine (T)
may fail to achieve puberty or may present after sexual
level is not typicalJy helpful in diagnosing hypothyroidism
maturation with azoospermia. Fertility may be achieved
because total T3 levels may remain within the normal range
from ejaculated spenn, if present, or extracted testicular
well into the evolution of hypothyroidism. However. total T3
sperm: however. advanced reproductive techniques such as
levels are useful in evaluating patients with possible hyperthy-
in vitro fertilization and in tracytoplasmic sperm injection
roidism because the value may be elevated out of proportion to
are necessary to achieve pregnancy. Some couples may opt to
the level. and failure to recognize an elevated T3 value may
include genetic testing by preimplantation genetic diagnosis
underestimate the degree of hyperthyroidism present.
and embryo biopsy to avoid transmission of the disorder to
Total T.1 measures both the bound and unbound thyroid
subsequent generations. Typically. gonadotropin levels a re
hormone fractions. whereas the free T reflects the unbound
high in patients with Klinefe lter syndrome, representing
portion of hormone. and may more accurately reflect avail-
testicular hypofunction. After plans for conception are com-
able hormone levels in patients who may have an abnormal-
pleted, supplementation with exogenous androgens may be
ity in protein metabolism (such as liver or kidney disease).
considered to prevent osteoporosis. Conception vvith donor
However. measurement of total T, in addition to free T 1 would
sperm is an alternative fertility treatment option.
not provide additional diagnostic information in this patient.
MRI of the pituitary would be needed only to rule out
a pituitary mass in the setting of hypogonadotropic hypogo- KEY POINT
nadism. Because this patient's gonadotropin levels are high, • In patients over 80 years of age, the serum thyroid-
a pituitary mass is unlikely. stimulating hormone level may be mildly elevated
Scrotal ultrasound would identify smalJ testicles in a above the typical reference range.
patient with suspected Klinefelter syndrome. but it would nol
identify the cause ofthis patient's elevated gonadotropin levels. Bibliography
Serum prolactin level would likely be normal. as Tabatabaie V. Surks MI. The aging lhyroid. Curr Opin Endocrinol Diabetes
Kli nefelter syndrome is characterized by primary hypogo- Obes. 2013 Oct;20(S):.ISS 9. [PM lD: 2397.!775 ]

123
Answe rs and Critiques

Item 60 Answer: 8 this patient's tumor is invading the left cavernous sinus and
Educational Objective: Evaluate an incidentally compressing the optic chiasm. complete resection will likely
noted adrenal mass. not be possible; however, surgery can eftectively debulk
the tumor and preserve vision in addition to significantly
The most appropriate diagnostic test to perform next is the decreasing growth hormone (GH) secretion as measured by
low-dose dexamethasone suppression test to screen for the insulin-like growth factorl (lGF-1) levels. In patients in whom
autonomous secretion of cortisol. The increasing use of imag- complete resection is not possible. such as this patient, addi-
ing stud ies tbr various medical indications has revealed oth- tional therapy may be required such as stereotactic radiation
erwise unrecognized adrenal masses in less than 1% of the therapy or medical therapy to inhibit GH secretion or block its
population younger than 30 years and up to 7% of those older effect on the tissues. I Iowever, surgical resection remains an
th<Jn 70 years. Ten to lS'Yo of adrenal incidentalomas are func- essential first step in the treatment of acromegaly
tional , although most have no overt clinical manifestations. A small number of GH-secreling pituitary adenomas
Therefore. testing is usually necessary to identi fY functional co-secrete prolactin. Although dopamine agonist therapy
tumors secreting catecholamines, cortisol, or aldosterone. Of with agents such as bromocriptine would treat the associ-
the functional adenomas, most secrete excessive amounts of ated prolactin elevation, it is minimally efleclive in acromeg-
cortisol. In subclinical Cushing syndrome (CS). classic signs aly and would not adequately treat GH secretion or add ress
or symptoms of cortisol excess are not observed: however. the mass effect of a GH-secreting adenoma.
complications of long-standing hypercortisolism may result. A GH receptor blocker, pegvisomant. is avai lable. Peg-
The patient has two disorders that can be seen in association visomant works in the peripheral tissues as an antagonist to
with subclinical CS: type 2 diabetes mellitus and osteoporosis. GH but does not decrease its production by the tumor. This
Obesity and hypertension are also common. The low-dose pat ient needs intervention to treat mass efTect at this ti me
overnight dexamethasone suppression test is recommended because the tumor is damaging the optic chiasm and the
as the initial screening test fbr this condition due to its high patient"s vision. and this treatment would not be expected
sensitivity. Screening fo r pheochromocytoma, such as by to decrease the tumor size.
measuring 24-hour urine fractionated metanephrines and analogues, such as octreotide and lan-
catecholam ines, is also indicated in all patients with an inci- reotide, in hibit GH secretion and are helpfu l in treating
dentally noted adrenal mass. some patients with acromegaly. They are used prima rily in
Adrenal vein sampling (AVS) is not needed. AVS is most patients with unresectable tumors without significant mass
often performed to evaluate for a bilateral versus unilateral effect or those with a contraindication to surgery. They may
ad renal cause of primary hyperaldosteronism. also be used in patients with continued GH secretion fol-
Measuremet;t of plasma renin activity and ald osterone lowi ng incomplete transsphenoidal resection. However, they
concentration is not indicate d in patients without would not be an appropriate treatment in this patient with a
hypertension. large. invasive. vision- threatening pHuitary tumor.
No furt her testing is also inappropriate. Although the Radiation therapy may be added to surgical or medical
imaging characteristics of the mass are in keeping with a therapy to help increase the chance fb r remission or cure.
benign adrenal adenoma, further diagnostic evaluation is Rad iation to the pi tuitary carries a high risk of causi ng
needed. This includes testing for autonomous hormonal pituita ry insufficiency and damage to surrounding tissues
secretion and subsequent radiograph ic surveillance (first at (particularly the optic nerves); therefore, it is not usually an
3 6 months and then an nually for 1-2 years). initial treatment for acromegaly in most patients. In those in
KEY POINT whom it is used. stereotactic surgery (gamma knife) is the
preferred approach to minimize potential complications.
• Ten to 15% of adrenal incidentalomas are ftrnctional; bio-
chemical testing is needed to identify functional tumors KEY POINT
secreting catecholamines, cortisol, or aldosterone. • The primary therapy for acromegaly is transsphenoi-
dal surgery to remove the causative grov.rth hormone-
Bibliography secreting pituitary adenoma.
Zeigt!r Thompson GB. Duh QY, et al. The American Association of
Clinical End ocrinologists a nd American Association of Endocrine
Surgeo ns mt!dical guidelines for the managemt!n t of adrenal inciden talo- Bibliography
mus. Emlocr Pract. 2009 Jul Aug: IS Su ppl t:l -20. [PMID: t96J2967] Melmed. S. Acromegaly pathogenesis and trt!atment. J Clin Invest. 2009
:-lov:ll9:3189 202. IP\110: 19884662]

Item 61 Answer: E
Educational Objective: Treat acromegaly with Item 62 Answer: C
transsphenoidal pituitary surgery. Educational Objective: Treat high-risk thyroid cancer
postoperatively.
Transsphenoidal resection of the pituitary adenoma is the
initial treatment of choice in pat ients with acromegaly. It is Radioactive iodine (RAI) therapy is the most appropriate post-
also the only treatment that is potentially curative. Because operative treatment in this patient who is at high risk of

124
Answers and Critiques
- ----- - -----------
J
cancer recurrence based on his age, the size of the primary insulin is the recommended treatment. Unlike autoimmune
tumor, the presence of vascular invasion and extrathyroidal type 1 diabetes, clrronic pancreatitis also destroys the pancre-
extension, and the number of involved lymph nodes. He atic alpha cells causing a glucagon deficiency that increases
therefore may benefit from adjuvant RAJ therapy, which may the risk of spontaneous hypoglycemia. Glucagon acts on the
decrease the likelihood of recurrent disease in patients with li,ver to increase glucose production through glycogenolysis
nodal metastases. 111 is is given in conjunction with levothy- and gluconeogenesis. The recovery from hypoglycemia is also
roxine suppression therapy, which is indicated in all patients impaired with alpha cell destruction. Early recognition of
who have had a total thyroidectomy. Because of his high-risk hypoglycemic symptoms and strategic hypoglycemic lreat-
disease, it would be appropriate to lower the thyroid-stimu- ment plans should be emphasized with patients with pancre-
lating hormone level to less than 0.1 J.lU/mL (0.1 m UIL) in the oprivic diabetes.
absence of contraindications such as pre-existing cardiovas- Exenatide, a glucagon-like protein-1 (GLP-1) mimetic,
cular disease or age greater than 65 years. suppresses glucagon and promotes insulin secretion. The
Tradit ional chemotherapeutic agents, such as doxoru- pancreatic beta cell and alpha cell destruction associated
bicin, are generally ineffective in the management of differ- with chronic pancreatitis precludes this treatment option.
entiated thyroid cancer and would not be indicated for this Postmarketing reports of pancreatitis are also cause for con-
patient. In patients with anaplastic carcinoma of the thyroid. cern for the use of this class of medication in patients with a
however. some studies have demonstrated a possible benefit history of pancreatitis.
with concomitant use of paclitaxe l- based chemotherapy and The sulfonylurea glipizide increases insulin secretion.
external- bea m radiotherapy. This patient has classic pap- The effect would likely be minimal to nonexistent in th is
illary thyroid cancer histology and would not benefit from patient with hyperglycemia resulting from substantial beta
such treatment. cell destruction from chronic pancreatitis.
External-beam radiotherapy is rarely used in patients Metformi n decreases hepatic glucose output by inhibit-
with diffe rentiated thyroid cancer. An exception would be ing gluconeogenesis and increases insulin-mediated glucose
the management of patients with inoperable d isease that uti lization in peripheral tissues. Metformin is a fi rst-line
threatens to cause local extension into vital structures in the agent for initial treatment of type 2 diabetes; however, this
neck such as the trachea, esophagus, or major blood vessels. pat ient has an insulin deficiency from pancreatic beta cell
Because of the extent of disease found at surgery and destruction and should be treated as a patient with type 1
this patient's high risk of recurrence. providing no addi- diabetes.
tional therapy would not be an appropriate next step in
KEY POl NT
management. .
• Hyperglycemia caused by chronic pancreatitis is an
KEY POINT
acquired form of type 1 diabetes mellitus and should
• A patient who has undergone total t hyroidectomy be treated with insulin.
for thyroid cancer and is at high risk lor disease
recurrence should receive adjuvant radioactive Bibliography
iodine therapy. 'vlergener K. Baillie J. Chronic pancreatitis. Lancet. 1997 Nov 8;350(9088) :
1379 85. [P'v!ID: 9365465]

Bibliography
Jonklaas J. Sarlis NJ, Litofsky D. e t al. Outcomes of patients V\ilh diJleren ti
ated thy roid carcinoma following initial therapy. Thyroid. 200 6 Dec:l6(12): Item 64 Answer: A
1229-42. [PM II): 17199433]
Educational Objective: Treat infertility related to
polycystic ovary syndrome.

Item 63 Answer: C The most appropriate treatment is a selective estrogen recep-


tor modulator (SERM) such as clomiphene citrate. SERMs are
Educational Objective: Manage acquired type 1
the established first-line treatment for ovulation induction
diabetes mellitus.
in anovulatory patients with infertility from polycystic ovary
This patient has an acquired form of type 1 diabetes melli- syndrome (PCOS). Typically, therapy is started after menses
tus caused by chronic pancreatitis (pancreoprivic diabetes) . and is given orally for 5 days. Common side effects include
which necessitates the use of insulin for treatment of the vasomotor symptoms and mood changes. Escalating doses
hyperglycemia. Chronic pancreatitis results in permanent of clomiphene are typically prescribed if a patient does not
destruction of the pancreas and may impair both the endo- ovulate on lower doses. More recently, evidence suggests the
crine and exocrine functions of the pancreas. The pancre- effectiveness and possible superiority of aromatase in hibitor
atic exocrine abnormalities arise from loss of the pancreatic therapy (such as with letrozole) in women with PCOS tor
enzymes required for digestion and absorption of food . The ovulation induction. However, this therapy is not currently
pancreatic endocrine abnormalities can present in a similar FDA approved for this indication.
manner as type 1 diabetes with hyperglycemia fro m insuli n A small subset of patients with PCOS may require in
deficiency secondaty to destruction of beta cells. 1herefore vitro fe rtilization, but this therapy is typically explored only

125
Answers and Critiques

after several failed cycles of ovulation induction with clomi- KEY POINT
phene citrate.
• Microprolactinomas in asymptomatic patients do not
In patients with clomiphene resistance. gonadotropin
require treatment; however, surveillance is recom-
therapy would be an appropriate next step; however. caution
mended.
is warranted because higher-order multiple gestation may
result.
Bibliography
A 2012 Cochrane review of the effect of insulin-sensi-
S, Casanueva FF, Hoffman AR, et al. Diagnosis and treatment of
tizing drugs in women with infertility and PCOS included hyperprolactinemia: an Endocrine Society clinical practice guideline J
44 trials, the majority of which involved metformin. Rates Clin Endocrinol Metab. 201 1 Fcb :96(2):273 88. [PMID: 2129699 1] .
of pregnancy were improved w it h mettormin compared
w ith placebo and metformin pl us clomiphene compared
w ith clomiphene alone. but metformin did not change rates Item 66 Answer: B
of live births compared w ith placebo or w ith clomiphene Educational Objective: Manage diabetic neuropathy.
compared to clomiphene alone.
DuJoxetine is a reasonable initial option for this patient's pain-
KEY POINT ful peripheral neuropathy. The typical presentation for distal
• Selective estrogen receptor modulators such as clomi- symmetric polyneuropathy is a bilateral "stocking-glove" dis-
phene citrate are the established first-line treatment tribution. Damage to the small nerve fibers can result in pain,
for ovulation induction in anovulatory patients with numbness. buming, and tingling. It can also in1pair light touch
infertility from polycystic ovary syndrome. and temperature sensation. Damage to the large nerve fibers
leads to abnonnal vibration sensation and proprioception.
or loss of ankle renexes is commonly seen early
Bibliography
Legro RS. Brzyski RG. Diamond MP. et al: Reproducti\·c \ledicine
with diabetic polyneuropathy. Motor weakness can occur as
;\letwork. Letrozole versus clomiphene for infertilil) in the polycystic the polyneuropathy progresses. Several classes of drugs are
ovary synd rome. !\ Engl J Med. Jul 10:371(2):119 29. Erratum in: frequently used for symptomatic pain relief, including the tri-
N Engl J Med. 201·1 Oct 9:317(15):1465. 25006718]
cyclic antidepressants (amitriptyline), other classes of antide-
pressants (duloxetine. venlafaxine), anticonvulsants (pregaba-
lin, gabapentin. valproate), and capsaicin cream. There are few
Item 65 Answer: C head- to-head comparison trials for these classes of dn1gs tor
Educational Objective: Treat a microprolactinoma in distal symmetric polyneuropathy. thus selection must take into
a postmenopausal woman . consideration the potenrial risks and benefits associated with
each drug for an individual patient. DuJoxetine has fewer risks
No therapy is necessary at this time, and the patient should
than amitriptyline for this patient given his cardiac history.
be retested in 12 months. The patient has a microprolac-
Tricyclic antidepressants, such as amitri ptyline, should ·
tinoma, but she is postmenopausal. Luteinizing hormone
be used cautiously in patients w ith known cardiac dis-
and follicle stimulating hormone levels are normally high
ease due to an association between this class of drugs and
in postmenopausal women because of ovarian failure; how-
arrhythmias. heart block. and sudden death. The patient's
ever. her levels are lower than expected. likely because the
history of cardiac disease and a first-degree atrioventricular
elevated prolactin is providing negative feedback. This causes
block may increase his risk of side effects from amitriptyline.
hypogonadism but is not clinically relevant because she is
A nerve conduction study is not routinely required for
already hypogonadal from normal menopause. She has mini-
d iagnosis or management in patients with diabetes w ith a
mal symptoms from menopause and is tolerating it well.
typical presentation of symmetric distal polyneuropathy.
1he prolactinon1a was found incidentally. On MRI. it
Atypical clinical featu res should prompt additional work-up,
has no concerning featu res, and her other pituitary hormone
including electrophysiologic testing.
levels are normal. Although no treatment is necessary for
Vitamin 8 12 deficiency has been associated with long-term
this asymptomatic patient, it is advisable to retest in 6 to
use of metforrnin and can present with peripheral neuropathy.
12 months to make sure that the tumor does not grow.
It is also commonly seen in the setting of megaloblastic anemia.
Dopamine agonists, such as cabergoline. are used to
It is unlikely that vitamin 8 12 deficiency is the cause of this
t reat symptomatic prolactinomas, but it is not necessary in
patient's peripheral neuropathy as he has a classic presentation
this asymptomatic patient.
for symmetric distal polyneuropathy, discontinued mettormin
Radiosurgery is not necessary. It is an option to treat
2 years ago, and has a normal complete blood count.
pituitary tumors that are not amenable to standard surgery
or cannot be fully resected , but it is not indicated for this KEY POINT
asymptomatic patient. • Treatment options for diabetic polyneuropathy include
Transsphenoidal resection of the pituitary tumor is the tricyclic antidepressants, other classes of antidepres-
overly invasive and unnecessary because she is doing well. sants (duloxetine, venlafaxine), anticonvulsants (pregab-
In addition, first-line therapy for symptomatic prolactino-
alin, gabapentin, valproate), and capsaicin cream.
mas are dopamine agon ists, not surgery.

126
Answers and Critiques
---------------------
Bibliography Item 68 Answer: D
American Diabetes Association. (9) Microvascular complications and foot
care. In: Standards of Medical Care in Diabetes 2015. Diabetes Care. 2015
Educational Objective: Diagnose Asherman
Jan:38 Supplt:S58 66. IPMID: 255377!0] syndrome with transvaginal ultrasound.

The most appropriate diagnostic test to perform next is a trans-


Item 67 Answer: D vaginal ultrasound. Th is patient has likely developed Asher-
man syndrome (AS). Because the hormonal evaluation in this
Educational Objective: Evaluate hypoglycemia in a
patient supports an intact hypothalamic-pituitary-ovarian axis.
patient without diabetes mellitus.
a structural abnom1ality such as AS should be suspected. AS
Th is patient should undergo a 72 hour fast w ith hypogly is an uncommon complication of dilatation and curettage,
cemic testing at the time of symptomatic hypoglycem ia. intrauterine device placement, or surgical procedures such as
He had symptomatic hypoglycemia \vith prolonged fasting hysteroscopic myomectomy: it is caused by lack of basal endo-
(>5 hours) O\'ernight that resoh·ed \\ ith glucose adminis- metrium proliferation and formation of adhesions (synechiae).
tration. Since he is not currently hypoglycemic or symp should be considered in any woman with amenor-
tomatic. tasting is necessary in an attempt to recreate the rhea and previous exposure to uterine instrumentation. The
metabolic scenario that induced the h) poglycemia to allO\\ typical presentation is with secondary amenorrhea. at times
definitive diagnosis. Glucose and hypoglycemic stud associated with cyclic pelvic pain, created by distention of
ies. including measurement of insu lin. C peptide. pro the uterine cavity where pockets of functional endometrium
insulin. and hydroxybutyrate levels. should be obtained persist but effitLx of menstrual flow is blocked or slowed by
at the beginning of the fast and then repeated e,·ery 6 adhesion formation. Although some patients with AS may be
hours until the blood glucose le\'el falls below 60 mg dL completely amenorrheic. others may demonstrate hypomen-
(3.3 mmol U. at which time they should be repeated ever) orrhea·and report scant menses compared •vvith the volume of
1 to 2 hours until (I) S)mptomatic hypoglycemia ($45 mg their menstrual flow before the procedure. AS most commonly
dL [2.5 mmol Lll occurs. (2) asymptomatic h)1)0glycemia occurs iJ1 an inflanm1atory settiJ1g such as endometritis or sep-
(:<;55 mg. dL [3.0 mmol L]) occurs w ith pre\ iousl) docu tic abortion. AS may also occur as a result of an overly aggres-
mented Whipple triad. or (3) the 7'2 hour fast concludes. sive curettage. In a patient w ith AS. transvaginal ultrasound will
Whipple triad consists of three components: neuroglycope show a thin endomet rial stripe an d may reveal small pockets of
nia. concurrent hypoglycemia. and resolution of symptoms fluid where menstrual flow has been trapped by neighboring
with correction of hypoglycemia. ad hesions. A functional uterine examination. such as hystero-
Hypoglyc(!lllic studies are not adequately informa salpingogram or saline sonohysterogram. confirms the diag-
ti\·e when the patient is not experiencing symptom nosis. Treatment consists or hysteroscopic resection of lesions.
atic hypoglycemia: therefore. testing now wou ld not be Given this patient's normal gonadotropin levels. a pitll-
appropriate. itary cause for her secondary amenorrhea is unlikely; there- .
Mixed mea l testing invoh·es ingestion of a meal fore, imaging of the pituitary is not warranted at th is time.
containing a mixture of protein. fa t. and carbohydrates Premature ovarian insuf11ciency is not the most likely
intended to raise the plasma blood glucose to assess the diagnosis given this patient's normal estradiol and gonad-
metabolic response to increased glucose. "!his test is appro- o tropin levels: therefore. a peripheral karyotype woulcl be
priate when S) mptomatic hypoglycemia occurs \\ithin 5 expected to be normal and should not be performed.
hours after meal consumption (postprandial hypoglyce Results of a progesti n w ithdrawal test are used to deliJ1-
mia). However. this patient's symptomatic hypoglycemia eate between an estrogen-deficient state (no bleeding) and an
occurred 7 hours after his last meal; I herefore. eYalua estrogen-sufficient state (withdrawal bleeding) . If the patient is
lion for postprandial hypoglycemia with a mixed meal test produciJ1g estrogen, she will have wit hd rawal bleeding within
would not be appropriate. 1 week of completing a course of progesterone. If no w ith-
Oral glucose tolerance testing was pre\ iously used drawal bleeding occurs after the progesterone challenge, then
to evaluate postprandial hypoglycemia. HoweYer. it has the patient has either a low estrogen state and hypothalamic
not demonstrated dfecti\·eness for this purpose and also amenorrhea is the diagnosis. or there is uteri ne outflow block-
would not be an appropriate study for e\aluating fasting age. This patient's history of a previous uterine procedure prior
hypoglycemia. to the onset of amenorrhea and the possibility of denuded
endometrium where synechiae are present will make a nega-
KEY POINT
tive test (no withdrawal bleeding) uninterpretable.
• Symptomatic fasti ng hypoglycem ia is best evaluated
w ith glucose and hypoglycemic studies preceding and KEY POINT
during a 72 hour fast. • The diagnosis of As herman syndrome s hould be con-
sidered in any woman with amenorrhea and previous
Bibliography exposure to uterine instrumentation; the classic pres-
Cryer PE, Axelrod L. Grossman AB. et al. Evaluation and management of entation is with secondary amenorrhea and some-
adult hy poglycem ic disorder s: an Endocrine Society Clinical Practice
Guideline. J Endocrinol 2009 Mar:94(3):709 28. 1PMID: 19088155]
times cyclic pelvic pai n.

127
Answers and Critiques

Bibliography Item 70 Answer: B


Conforti A. Alviggi C. Mollo A. De Placido G. Mugos i\. The management of
/\sherman syndrome: a reviewofliterat ure. Reprod Bioi Endocrinol. 2013
Educational Objective: Treat Graves ophthalmopathy.
Oec 27:11:118. [PMID: 243732091
Methimazole is most appropriate for this patient with Graves
ophthalmopathy (GO). GO may be manifested by lateral gaze
Item 69 Answer: C palsy. scleral injection. periorbital edema, and pressure sen-
sation behind the eyes. Additional manifestations include
Educational Objective: Identify postprandial hyper-
proptosis, lid lag. and. if severe, decreased visual acuity.
glycemia as a cause of elevated hemoglobin A 1 c
Excess deposition of glycosam inoglycans in the retro-orbilal
levels.
space results in increased pressure; the ensuing compression
This patient should measure her postprandial blood glucose of' the muscles and cranial nerves causes the ocular palsy.
level. Given the patient's young age and lack of other major Most patients with GO have mild, nonprogressive symptoms
comorbidities. her hemoglobin A1e goal is less than 6.S'Y., to that do not require specific treatment. The decision to treat
7.0'Yo. Postprandial hyperglycemia often remains undetected depends upon the severity and activity of the disease. For
but still contributes to elevated hemoglobin A1e values. The patients who need treatment. initial therapy should target
effect is more profound when the hemoglobin A1c is close controlling the hyperthyroidism. Methimazole will rapidly
to 7.0%. Adequate meal-time coverage with insulin can be lower circulating thyroid hormone levels, may reduce seru m
determined by measuring postprandial blood glucose levels. thyroid autoantibody titers. and may assist in controlling GO
lfher 1- to 2- hour postprandial blood glucose values are ele- symptoms. Addi tional therapy to control symptoms includes
vated above 180 mg/dL (10.0 mmo]/L). her meal-time insulin use of ocular lubricants and taping of the eyeUds at night
should be increased or the composition of her meals should (if the Lids are unable to completely cover the eye). Because
be altered to decrease her blood glucose. Changing her diet if cigarelle smoking increases the activity of GO and impairs
it is causing postprandial hyperglycemia could eventually lead the response to therapy. smoking cessation is or paramount
to lower insulin requirements. importance.
Checking her postprandial blood glucose values fi rst External-beam radiotherapy is reserved for treatment of
will help identify where the hyperglycemic issue arises that severe GO but typically is employed if symptoms pers ist or
keeps her hemoglobi n Ale above goal. worsen in spite of return to the euthyroid state.
Increasing her glargi ne dose will not adequately affect Because radioactive iodine (RAI) treatment has been
postprandial hyperglycemia and may lead to overnight or associated with (at least transient) worsen ing of GO clue to
fasting ia. an initial increase in circulating antibody levels. its use is
Overnight glucose abnormalities can be identified not recommended in patients with moderate to severe GO.
with the measurement of a 3 A..\1 blood glucose level. Large When RAJ is used. pretreatment with a glucocorticoid to
fluctuations in fasting blood glucose values or consistent mitigate the rise in antibody levels is recommended prior to .
fasting hyperglycemia can be clues to overnight hypogly- RA J therapy.
cemia with subsequent rebound hyperglycemia (Somogyi Total thyroidectomy is typically recommended for long-
effect) or hyperglycemia as a result or rising catecholamines term control of Graves disease in patients with active GO
(dawn phenomenon). The patient reports stable fasting and when medical therapy fails to induce a remission. However.
prepranclial glucose values throughout the day. The most returning the patient to a state of euthyroidism is advisable
likely tin1ing for glucose abnormalities that wou ld affect her before surgery. Surgical decompression is also an option to
hemoglobin A1e value is in the postprandial state. control active GO, particularly if there is compression of the
Sitagliptin is a dipeptidyl peptidase 4 (DPP- 4) inhib- optic nerve.
itor that slows gast ric emptyi ng a nd suppresses glucagon
KEY POINT
secretion. Although it has a modest eft'ect on hemoglobin
A1e lowering. metfo rmin remai ns first line therapy for • In itial treatment of Graves ophthalmopathy is nor-
type 2 diabetes and should be continued as part of her malization of thyroid function.
regimen.
Bibliography
KEY POINT
Phelps PO. Williams K. Thyroid eye disease for the primary care physician.
• Monitoring postprandial blood glucose levels can be Dis Mon. 201-1 Jun:60(6):292 8. [PMID: 2-1906675]

useful to assess prandial insulin coverage in patients


who have type 2 diabetes mellitus with at-goal pre-
prandial readings but with hemoglobin A1e values Item 71 Answer: B
not at goa l. Educational Objective: Diagnose primary hyperaldo-
steronism as a cause of secondary hypertension.
Bibliography The most appropriate diagnostic test to perform next is to mea-
American Diabetes Association. (7) Approaches to glycem ic treatment. In:
sure the plasma aldosterone-plasma renin activity ratio. This
Standards of Medical Care in Diabetes 2015. Diabetes Care. 2015 Jan:38
Suppl l:S4t s.lPMID: 25537707 1 patient has resistant hypettension. defined as blood pressure

128
and Critiques

that remains above goal despite concurrent use of three antihy- adenomatous hyperplasia, and the sonograph ic features of
pertensive agenLS of different classes, one of which is a diuretic. the gland and slow progression of the compressive symptoms
Resistant hypertension may occur in as many as 10% of patients support a diagnosis of a benign lesion . He has evidence of
with hypertension. Although this patient is being treated with compressive symptoms on physical examinalion, with facial
a diuretic, he has significant hypokalemia in the presence of flushing when raising his arms (Pemberton sign). '!11e flush-
treatment with an ACE inh ibitor and potassium supplementa ing indicates venous outflow obstruction by the goiter as
tion. This raises the possibility of primary hyperaldosteronism the vessels course through the thoracic inlet. Additionally,
as a cause or contributing factor of his resistant hypertension. his symptoms of dysphagia and change in voice quality may
Appropriate initial evaluation for this diagnosis is measure- suggest a compressive effect on the esophagus and recurrent
ment of the plasma aldosterone-plasma ren in activity ratio. laryngeal nerves, respectively. Surgical removal of the thyroid
If positive, confirmatory testing is usually accomplished w ith offers the best option for immediate relief of symptoms and
intravenous salt loading. fl udroc01tisone suppression testing. identification of a possible underlying malignancy.
or captopril testi ng. If confirmed, adrenal imaging is indicated External-beam radi otherapy can resu lt in eventual
to determine if hyperaldosteronism is due to a bilateral or uni- shrinkage of the thyroid but will not provide immediate
lateral cause. Mineralocorticoid receptor antagonists (such as relief of his obstructive symptoms, is unlikely to be cura tive.
spironolactone) are indicated for patients with a bilateral cause potentially increases the risk of secondary malignancy, and
of primary hyperaldosteronism and those with a unilateral is associated with significant morbidity.
cause who refuse or are not candidates for surgery. Levothyroxine therapy has been shown to reduce thy-
Dexamethasone suppression testing is used to evalua te roid volume by up to 20 %, bu t the clinical sign ificance of this
for glucocorticoid excess. However, this patient is not taking small change in volume w ith such a large goiter is unli kely to
exogenous glucocorticoids and has no physical examina- improve symptoms. Additionally. levothyroxine therapy may
tion findings (fat redistribution, striae) or laboratory stud- take ri1any months to years to reduce the thyroid volume,
ies (glucose metabolism abnormalities) suggesting Cushing and th is patient needs immed iate relief of his obstructive
syndrome. Therefore, testing for this possibility is not an symptoms.
appropria te next step in diagnosis. Radioactive iodine has been used to shrink multinod-
Measurement of plasma metanephrines and catechol- ular goiters; the volume change is also gradual and optimal
amines is used to evaluate for the possibility of pheochromo- reduct ion averages 40 %. However, radioactive iodine ther-
cytoma, which classically presents with the triad of diapho- apy may be risky in this patient because acute swelling of
resis, headache, and tachycardia, none of which are present the thyroid after iodine uptake can -lead to compromise of
in this patient: ln addition. pheochromocytoma would not the vascular structures at the level of the thoracic inlet and
explain the patient's hypokalemia. fu rther compression of the trachea.
Renal artery Doppler flow studies may be helpful in
KEY POINT
evaluating for renovascular hypertension. However. most
cases of renovascular hypertension occur in patients over • In patients with mu ltinodular goiter w ith compressive
SO years of age and are associated w ith atherosclerotic car- featu res, t hyroidectomy is most likely to provide
diovascular disease or evidence of functional impairment immediate relief of symptoms, decrease the risk
of the kidney, neither of which a re appare nt in this patient. of fatal ailvvay compromise, and possibly identify
underlying malignancy.
KEY POINT
• Patients with suspected primary hyperaldosteronism Bibliography
as a cause of resistant hypertension should be Aslam Rl. Steward D. Surgical ma nagemen t ofrhyroid disease. Otolaryngol
screened with a plasma aldosterone-plasma ren in Clin North Am . 20 10 Apr:43(2):273- 83. IPMID: 20510714 1
activity ratio.
Item 73 Answer: B
Bibliography
Funder JW. Carey RM. Fardella C. et a l. Case detectio n. diagnosis. and treat-
Educational Objective: Evaluate primary infertility
men t of pJticn ts with pri mary aldosteronism: an endocrine society with a hysterosalpingogram.
clinical practice guideline. J Clin Endocrinol Metab. 2008:91(9):3266 81.
[PMID: 18552288] The test most likely to be diagnostic is a hysterosalpingogram
(HSG) to evaluate tubal patency. Primary infertility due to
Item 72 Answer: D a tubal abnormality is common and is best evaluated with
dynamic testing of the female reproductive tract under flu-
Educational Objective: Treat substernal goiter with
oroscopy with an HSG. 'This patient. owing to her history of
compressive symptoms.
pelvic inflammatory disease, is likely to have a distal tubal
This patient should undergo thyroidectomy to treat his occlusion and resulta nt hydrosalpinx. Many women with
obstructive substernal goiter and prevent further airway com- hydrosal pinx have no symptoms; however, symptoms may
promise, which may be rapid and f'atal. The thyroid nodule include pelvic pain (both unilaterally and bilaterally) and
biopsy find ings obtained 5 years ago most likely represent chronic vaginal discharge. Confirmatory evaluation should

129
An swers and Critiques

include diagnostic laparoscopy, but this is not typically per- and pheochromocytoma. Failure to identity and treat a pheo-
formed as first-li ne evaluation given the need for general chromocytoma prior to surgery can result in an intraoperative
anesthesia, intubation , and recovery. Repair of the fallopian hypertensive crisis and, potentially, death.
tubes may be possible with microsurgical techniques; how- While it is important to assess the ris k of metastatic
ever, reocclusion is possible and the risk of subsequent ectopic disease in this high- risk patient. 18-fluoro-deoxyglucose
pregnancy is high. Many women elect to proceed with in vitro positron emission tomography scanning is not the ideal
fertilization in lieu of tubal surgery. imaging modality in patients with MTC, as there is a high
Ovarian reserve assessment and semen analysis are false-negative rate. CT of the lungs and liver is a more effec-
essential when eva luating a couple w ith infertility. Ovarian tive means of identification of distant metastases.
reserve assessment ca n be accomplished with early follicu- While evaluation for hyperparathyroidis m by measur-
lar phase testing of follicle-stimulati ng hormone (FSH) or ing the serum parathyroid hormone level is indicated in this
anti-mUilerian hormone (AMH). Transvaginal ultrasound patient with MEN2A. it is not the most appropriate next step
in the early follicular phase allows For counting of antral in management. Testing for hyperparathyroidis m is recom-
follicles in each ovary, which if present at greater than eight mended prior to surgery because the parathyroid disease
bilaterally support normal ovarian reserve. However, given can be managed simultaneously with the thyroid cancer, but
this patient's history of pelvic inflammatory disease and her wou ld not be an appropriate step before evaluating a patient
husband's history of fathering a child in a previous marriage. with MEN2A for the presence of pheochromocyto ma.
HSG is the d iagnostic test that is most likely to reveal an Total thyroidectomy and lateral neck d issection should
abnormality. not be performed until it is confirmed that this patient does
Semen analysis can be performed after a short w indow not have a coexisting endocrine neoplasm owing to the high
of abstinence on an ejaculated sperm specimen. However. it intraoperative risk associated with untreated pheochromo-
is unlikely to be helpful in this patient's husband. cytoma. If identified, pheochromocytomas should be surgi-
Karyotyping is usually not indicated as part of the initial cally removed prior to thyroidectomy.
evaluation of unexplained female infertility because of the Genetic counseling is a very important component of
low incidence of discovered abnormalities. the treatment plan for all patients with newly diagnosed
Specialized laboratories with andrology services eval- MEN2A as first-degree relatives are at high risk tor also hav-
uate sperm concentration, sperm motility, and sperm mor- ing the RET m utation. With a nearly 100% penetrance for
phology. Ovarian reserve may be evaluated by serum testing development ofMTC in carriers of RET mutations. referral to
(clay 3 Follicle stimulating hormone level, AMH level) or by a team w ith experience in the management of these elisor
transvaginal ultrasound assessment of antral follicle count. ders is critical for timely diagnosis and treatment.
KEY POINT KEY POINT
• Primary infertility due to a tubal abnor.mality is com- • All patients with multiple endocrine neoplasia type
mon , particularly in women with a history of pelvic 2A s hould undergo testing to exclude pheochromocy-
inflammatory disease, and is best evaluated with a toma prior to thyroidectomy; an elevated level of
hysterosalpingogram. plasma fractionated metanephrines should prompt
treatment for pheochromocytoma before addressing
Bibliography the thyroid malignancy.
Maheux- LacroLx S. Boutin A, Moore L. et al. Hysterosalpingosanography for
diagnosing tubal occlusion in subfertile women: a systemaiic review
with meta analysis. Hum Reprod. 20t4 :O,Iay:29 (5):953-63. [P\IID: Bibliography
24578476] American Thyroid Association Guidelines Task Force.20TS Thyroid. IPMm:
258!00471

B c
Item 74 Answer:
Educational Objective: Evaluate newly diagnosed
Item 75 Answer:
Educational Objective: Treat severe hypercalcemia.
Cl
medullary thyroid cancer.
·lhis patient has se\'ere. malignancy related (non parathy
The next most appropriate step in the management of this roid hormone [PTI I] mediated) hypercalcemia w ith associ
patient with medullary thyroid cancer (MTC) and multiple a ted mental status changes and acute kidney injury. I lemo-
endocrine neoplasia type 2A (MEN2A) is to evaluate for the d ialysis is generally reserved for patients w ilh extremely
presence of a pheochromocytoma by measurement of fraction- high serum calcium levels (>18 mg dL [·1.5 m mol L]) asso
ated plasma metanephrines. All patients with a cytologic diag- ciatcd with acute kidney injury that precludes other acute
nosis of MTC should undergo testing of the tumor for genetic treatmenrs because it is able to rapid!) lower serum cal
abnormalities as the initial step in their evaluation as 25% of cium and provide careful management of volume status
patients with MTC will have the inherited form . Identification and electrolytes. Both hemodialysis a nd peritoneal dialysis
of a RET mutation in this patient means that the he has ME 12A. are opt ions. a lthough hemodialysis more rapid!) lowers
an inherited syndrome associated with hyperparathyroidism calcium levels.

1 30
Answe rs and Critiques

rrl Calcitonin is a rapidly acting agent that interferes wit h Continuing the current regimen places the patient at
Ll.l osteoclast funct ion and promotes kidnev excretion of cal increased risk for continued hypoglycemia in the setting of
CONT. ctum.
· H owcver. Its
. ca I<:tum
. Iowenng
. - ,,.eel ts
e . re I attn•
. Iy
hypoglycem ic unawareness.
weak. and it is only able to lower semm calcium k \ds by 1 to Increasing the metformin dose may improve insulin
2 mg d l. (0.2S O.SO mmol L) over 4 to 6 hours. wh ich wou ld sensitivity and decrease hepatic gluconeogenesis, which
not be opt imal loweri ng of this patient's \'ef\' high calcium could improve the hyperglycemia and/or worsen the hypo-
leveL Calcitonin w ill bring her calcium levels down. but not glycemia. An increased mettormin dose does not address the
enough to :JIIeviate her symptoms. serious complication of hypoglycemic unawareness, which
Cinacalce t, a ca lcimi metic agent, is an option lbr lower must be corrected fi rst.
ing calcium f( H· symptomatic patients. bu t it is not indicated Pramlintide could decrease the hemoglobin A1c to
to acutely red uce life threatening hypercalcemia. the patient's goal; however, hypoglycemia is a side effect
In travenous bisphospho nates are used lbr treatment of of pram\intide in conjunction with insulin use. 1his may
malignancy induced hypercalcem ia. with wled ronic acid further increase the risk of hypoglycemia in this scenario.
or pamidronate the most common agents. lloweYer. onset of Pramlintide is an amylin mimetic that decreases glucagon
action is not rapid. and bisphosphonates are cont ra indi<·ated secretion and increases satiety by decreasing gastric emp-
in severe kid ney injury.1herclore. zoledronic acid \\'Ould not tying. Pramlintide should be considered when the intended
be an appropria te next step in treatment. reduction in hemoglobin Ate is modest and the desired
Once the serum calcium has been lowered b) dialysis. effects are a reduction in both weight and postprandial
calcitonin or an intra\·enous bisphosphonate ma) be used hyperglycemia.
to keep the calcium leYels at a manageable le\el. 1he use of
KEY POINT
dialysis in the patient is strictly for resolution of the acute
mental status changes from the elevated calcium le\el. and • Treatment fo r hypoglycemic unawareness is to reduce
longer term management of hypercalcemia is required. the insulin dose and allow permissive hyperglycemia
at all times fo r several weeks to provide the body an
KEY POINT
opportunity to reset the coun terregulatory response
• For patients with serum calcium levels greater than to hypoglycemia to a more appropriate blood glucose
18 mg/dL (4 .5 mmoi!L) v.rith neurologic symptoms or range.
acute kidney injury, hemodialysis is an appropriate
choice to quickly reduce calcium levels. Bibliography
Cryer PE. Axelrod L. Grossman AB. et al. Eva luation and management of
Bibliography adult hy poglycemic d isorders: an Endocrine Society Clinical Practice
Guideline. J Endocrinol 2009 (PMID: 190881SS]
Ros ner MH. D<ltkin AC. Onco-nephrology: the pathophysiology and treat
ment of malignancy-associated hypercalcemia. Clin J .\m Soc :>lcphrol.
2012:7:1722-9. [PMID: 22879438] '
Item 77 Answer: C
Educational Objective : Diagnose elevation of lutein·
Item 76 Answer: D izing hormone related to polycystic ovary syndrome.
Educational Objective: Treat hypoglycemic
The most likely cause of this patient's talsc-positive results
unawareness.
using the urinary luteinizing hormone (LH) kit is polycystic
'lliis patient should decrease his meal-time insuli n and con ovary syndrome (PCOS) . Women with PCOS typically have
tinue metformin. He has hypoglycemic unawareness sec elevated resting LH levels. wh ich may be mistaken on home
ondary to a diminished counterregulatory response that has urinary LH kits for ovulation. The primary clinical man ifes-
developed in the setting of repeated episodes of hypoglyce- tations of PCOS are menstrual irregularity (oligomenorrhea
mia. His hypoglycemia occurs in the postprandial state. thus or amenorrhea). ovulatory dysfunction with resultant infer-
the meal- time insulin should be decreased to allow permis- tility. insulin resistance. and hyperandrogenism. Oligo-ovu-
sive hyperglycemia for 2 to 3 weeks. Permissive hyperglycemia lation or anovulation can result in endometrial hyperplasia
is defined as allowing an increase in blood glucose values to and/or infertility. Hyperandrogenism presents as hirsutism,
the level at which no further episodes of hypoglycemia occur. acne. or androgenic alopecia. Most patients with PCOS also
Using less stringent glycemic goals during this period will have insulin resistance, and studies have shown an increased
provide the body an opportunity to reset the counterregula- incidence of metabolic syndrome. obesity. impaired glucose
tory response to hypoglycemia to a more appropriate blood tolerance, and frank type 2 diabetes mellitus. PCOS remains
glucose range. if possible. Although developing mutual hemo- a diagnosis of exclusion that is made both clinica lly and with
globin A10 goals with the patient is important, hypoglycemia ancillary testing. A patient with PCOS with a positive minary
in th is scenario precludes reaching his goal safely. Once he no LH measurement result may interpret this as a tnte LH surge
longer has hypoglycemia, changes to the meal- lime insulin and expect ovulation to be imminent. Attempts at conception
doses or to meal content and volume can be evaluated to fine- with intercourse or in semination may be focused on this
tune his regimen while avoiding hypoglycem ia. read ing and may therefore be ineffective. A more accurate

131
Answers and Critiques

assessment of LH surge and anticipated ovulation in patients Denosumab is a receptor activator of nuclea r factor KB
with PCOS would be through transvaginal identification of (RANK) ligand inhibitor FDA approved for the treatment of
ovarian folliculogenesis and confirmatory serum assessment osteoporosis in postmenopausal women who are at high risk
of reproductive hormones. of fracture. Since this patient has not failed bisphosphonate
Late-onset (nonclassic) congenital adrenal hyperplasia. therapy (shown a significant decrease in BMD while on
although a common cause of hirsutism and oligo-ovulation, bisphosphonate therapy) nor is she intolerant of the current
is typical ly associated with normal or low LH levels owing to therapy, there is no reason to change her therapy.
the negative feedback of elevated androgens of adrenal origin Teriparatide is appropriate as first li ne therapy for
on the anterior pituitary. patients at high risk for fracture (T-score < - 3.0) or who
Fu nctio nal hypo thalam ic amenorrhea affects 3% of have experienced progressive osteoporotic disease while
women between the ages ofl8 and 40 yea rs and is a diagno- on bisphosphonate therapy. This change would be unnec-
sis of exclusion. Risk factors for this condition include a low essa ry since the patient's BMD has been maintained for the
body weight and fa t percentage, rapid and substantial weight past 5 years.
loss, eating disorders, excessive exercise. severe emotional A drug holiday is indicated for patients who have been
stress, severe nutritional deficiencies, and chronic or acute on bisphosphonate therapy fo r 3 to 5 years. have had no
illness. FSH and LH levels are inappropriately low or normal progression of the disease. and have minimal risk factors
and cannot account for a positive urinary LH test. for additional fractures. Th is patient has multiple risk fac-
Hypothyroidism and elevated serum prolactin levels tors for fractures; therefore. a drug holiday would not be
suppress rather than elevate serum LH levels and would not appropriate.
account for this patient's increased urinary LH measurement.
KEY POINT
KEY POINT • In patients at high risk for osteoporosis, it is appropri-
• Women w ith polycystic ovary syndrome typically ate to continue bisphosphonate therapy alone if ade-
have e levated resting luteinizing hormone (U I) levels, quate bone stability has been achieved.
which may be mistaken on home urinary LH ki ts for
ovula tion. Bibliography
;llatio nal Osteoporosis Foundation (www. my.nof.o rg/ bone-source) .
Clinician's Guide to Prevention a nd Treatment of Osteoporosis.
Bibliography Washington. DC: National Osteoporosis Foundation: 2014.
Roucrdam F.SHRE ASRM-Sponsored PCOS Consensus Workshop Group.
Revised 2003 consensus on diagnoslic crileria and long- term health risks
related to polycystic ovary syndrome. Fenil Ster il. 200 l Ja n:SI (1):19 25. Item 79 Answer: D
[PMID: 14711538)
Educational Objective: Manage primary
hyperaldosteronism.
Item 78 Answer: C
The most appropriate treatment for this patient is spironolac-
Educational Objective: Treat a high-risk patient with
tone. He has primary hyperaldosteronism (PA) due to a bilat-
osteoporosis.
eral adrenal source as evidenced by the lack of lateralization
After counseling about smoking cessation. this patient on adrenal Yein sampling (AVS). Bilateral adrenal hyperplasia
should continue her current alendronate therapy. She has is the most common etiology of PA, accounting for approxi-
documented osteoporosis and is at high risk fo r subsequent mately 60% of cases. and spironolactone is the treatment of
fractures due to multiple risk facto rs. includ ing cu rrent choice. Spironolactone is a mineralocorticoid receptor (MR)
smoki ng and a previous fracture. Her bone mineral density antagonist that can improve blood pressure. normalize serum
(BMD) has been well maintained on an ora l bisphosphonate potassium concentration, and reduce excess ca rdiovascular
for the last several years. The best way to evaluate a dual en risk related to hyperaldosteronism. Eplerenone is a n alterna-
ergy x- ray absorptiometry (DEXA) scan from measurement tive MR antagonist that is less likely to cause gynecomastia
to measurement is to compare the bo ne mineral density in men because of greater MR selectivi ty; however, usc of
readings from year to year, not the T-score. A change in BMD eplerenone fo r this indication is off- label. Antagonists of t he
that is less than about 4% (or the percentage noted by the aldosterone-sensitive sodium channel (a milo ride) can be used
DEXA machine manufacturer) is not considered a statisti- as second- line therapy.
cally significant change. This regimen should be considered Bilateral adrenalectomy is not appropriate for the rou-
successfu I therapy si nee the goal of bisphosphonates is not to tine management of PA, as this would risk primary adrenal
build bone mass but to stabilize bone loss. Since th is patient fai lure. thus necessitating life-long glucocorticoid and min-
has had stable BMD while on alendronate. there is no indi- eralocorticoid therapy.
cation to convert to a more invasive. expensive option at this Dexamethasone. a long-acting synthetic glucocorticoid,
time. It wi ll be important however, to continue to follow her has a role in treating only a small percentage of patients who
tor atypical fractures of the long bone due to her prolonged have glucocorticoid-remed ial hypertension. a very rare auto-
bispbosphonate treatment. If leg pain or an atypical fracture somal dominant condition resulti ng from ectopic expres-
is noted, bisphosphonate therapy should be discontinued. sion of aldosterone synt hase in the cor tisol-producing zona

132
Answers and Critiques

fasciculata. Adm in istration of dexamethasone will suppress There is no indication for pituitary MR I at this time
pituitary adrenocorticotropic hormone (ACTH) secretion in because her hyperprolactinemia is explained by hypothy-
t hese patients and therefore mineralocorticoid production. roidism. It is necessary to make sure that her hyperprolactin-
However. hyperaldosteronism in most patients is indepen- emia normalizes after treatment with levothyroxine.
dent of ACJ'I L secretion, and suppression with exogenous
KEY POINT
glucocorticoid is not an effective therapy.
Left aclrcnalcctomy should not be performed because • In patients with hyperprola ctinemia and hypothy-
the patient has a bilateral cause of PA. The left adrenal roidism. the hypothyroidism should be treated first,
adenoma detected by CT scan is an incidental finding and then the patient should be reevaluated to ensure that
is likely not th e cause of this patient's hyperaldostero nism. the hyperprolactinemia resolves.
Because non secreting adrenal adenomas are co mmon. AVS
is needed in m ost patients with hyperaldosteronis m to Bibliography
determine the source of aldosterone secretion when imag- \ lclmcd S. Casanucva FF. Hoffman i\R. ct al. Diagnosis and 1rea1ment of
hyperprolactinem ia: i\n Endocrine Society clinical pmctice guideline.
ing studi es show an adrenal adenoma to assess its con tri- J Clin Endocrinol 20 11 Feb:96(2):273 2129699 11
bution to excess mineralocorticoid production. AVS s hould
be done at a high volume referral center clue to a high risk
of complications when significant procedural experience
Item 81 Answer: B
is Jacking.
Educational Objective: Diagnose cystic fibrosis as a
KEY POINT cause of congenital bilateral absence of the vas def·
• For patients with primary hyperaldosteronism clue to erens and azoospermia.
bilateral adrenal hyperplasia, m edical therapy wilh a Congeni tal bilateral absence of the vas deferens is a common
m ine ralocorticoid antagonist such as spi ronolactone is cause of obstructive azoospermia and is frequently associated
the treatm en t of choice because of its proven efQcacy with cystic fibrosis (CF). It may also presen t with uni lateral
to lower blood pressure, normalize serum potassium absence of the vas deferens. Many patients are unaware that
concentration, and reduce cardiovascular risk. they have CF because they may have a mild form that causes
only nonspecific symptoms such as ch ronic sinusitis. Partner
Bibliography testing for CF-carrier status should be encouraged to assess
founder JW. Carey RM. f ardella C. et al. Case detection. d i.1gnosis. and treat the likelihood of tran smission to a subsequent generation.
ment of p<lticn u; with primary aldosteronism: an endocrine ;ociet)
clinical pmctice guideline. J Clin Endocrinol 2008 Sep:9:l(9): 3266 Sperm production is often normal in these patients: however.
18552288] the absence of the vas deferens limits any observable sperm
in the ejaculate. Therefore. testicular biopsy is necessary to
retrieve sperm for use in advanced reproductive technjques-
Item 80 Answer: B (ART) such as in vitro fertilization and intracytoplasmic sperm
injection. Utilization of donor sperm is an alternative for cou-
Educational Objective: Manage hyperprolactinemia
ples not interested in ART.
caused by hypothyroidism.
Androgen abuse is common among elite and profes-
The most appropriate treatment for this patient is to begin sional athletes and in young men. Physical examination
levothyroxine. She has primary hypothyroidism with an fi ndings may include excessive muscular bulk, acne, gyneco-
elevated thyroid-st imulating hormone level and a low free mastia. and decreased testicular volume. Low sperm counts
thyrox ine (T 1) level. Her symptoms are consistent with hypo also may be present with exogenous and rogen use. Andro-
thyroidism. Hypothyroidism is a cause of hyperprolacti n gen abuse can result in hypogonadism and infertility, which
emia. When patients present with hyperprolactinemia and occasionally are irreversible. This patient's normal physical
hypothyroidism. the hypothyroidism should be treated and examination. normal testicu lar volume. and absence of the
then the patient s hould be ree\·aluated to ensure that the vas deferens argue against this diagnosis.
hyperprolacti ncmia resolves. The patient's pituita ry gland is Primary hypogonadism is due to testicular failure and
normal. There is no tumor. is defined as a low testosterone level \Nith e levated lutein-
The hypothyroidism should be treated first. and then izing hormone and follicle-s timulating hormone levels.
the patient's prolactin level should be retested. There is no Primary hypogonadism can have congenital or acquired
indication for cabergoline. a dopamine agon ist. at th is point. causes. The most common congenital cause is Klinefelter
C'-abcrgol ine is an appropriate therapy for a patient with syndrome (-n.XX.Y karyotype). The extra sex chromo-
hyperprolactinemia caused by a prolactinoma. ll1is patient's some results in malformation of the seminiferous tubules
hyperprolacti nemia is explained by hypothyroidism. and typically of the Leydig cells. Physical examination is
Sertra line docs not cause hyperprolactinemia or hypo likely to reveal small. firm testes and decreased virilization.
thyroiclism. An tipsychotic agents are a common cause of Additional manifestations include oligospermia and in fer-
hyperprolactinemia. but selective serotonin reuptake inh ib tility. Klinefelter syndrome does not result in obstructive
itors such as scrtraline <1 rc not. azoospermia due to absence of the vas deferens.

133
An swe rs and Critiques

A clinically palpable varicocele typically affects fertility early morning serum cortisol level is unequivocally low, as
by lowering sperm motility through a local heat effect. A in this patient.
scrotal bulge may be noted by the clinician. and the patient Measurement of the plasma ACfH level is used to dif-
may note pain that is worse with the Valsalva maneuver. fe rentiate primary adrenal fa ilure from other causes of low
Azoospermia would not be caused by varicocele alone. cortisol. In primary adrenal failure, the plasma ACfH level is
Y chromosome microdeletions can be associated with typically significantly elevated (200 pg/mL [44 pmoi /L]) and
oligospermia or azoospermia and small testicular volume. would confirm the diagnosis. However, withholding ther-
This chromosomal abnormality is not assoc iated with apy while awaiting diagnostic confi rmation would not be
absence of the vas deferens ::md is therefore not a likely cause appropriate because of the potential life-threa tening nature
of this patient's findings. of pri mary ad renal failure.
Although prednisone is an acceptable agent for glu-
KEY POINT
cocorticoid replacemen t. it has almost pure glucocorticoid
• Congenital bilateral absence of the vas deferens is a activity and would not be an appropriate single agent for
common cause of obstructive azoospermia and is fre- treatment of primary adrenal failure in which replacement
quently associated with cystic fibrosis. of both glucocorticoid and mineralocorticoid is required.
KEY POINT
Bibliography
Stahl PJ. Schlegel P '. Ge netic evaluat ion of the azoospcrmic o r sc,·crcly • Cortisol replacement therapy should be initiated
oligozoospermic male. Curr Opin Obstel Gynecol. 2012 Aug:2-1(-1):221 8. inlmediately in persons with confirmed adrenal insuf-
I P\11 D: 22729088)
ficiency, which is diagnosed by an early morning
serum cortisol level below 3 (82.8 nmoi/L) in the
setting of signs and symptoms of cortisol deficiency.
Item 82 Answer: B
Educational Objective: Manage primary adrenal failure.
Bibliography
Initiation of cortisol replacement therapy with a glucocor- Neary ,'<. Nieman L. Adrenal insufficiency: etiology. diagnosis a nd treatment.
Curr Opin Endocrinol Diabetes Obes. 2010 Jun;17(3):2 17- 23. [PMID:
ticoid (such as hydrocortisone) and mineralocorticoid (such 20375886]
as fludrocortisone) is the most appropriate nex1 step in man-
agement. This patient has symptoms consistent with adre-
nal insufficiency (fatigue. unintentional weight loss. nausea. Item 83 Answer: A
and vomiting); her family history of autoimmune disorders.
Educational Objective: Manage the "honeymoon"
hyperpigmentation noted on physical examination, and
phase of type 1 diabetes mellitus.
hyperkalemia on laboratory testing suggest primary adrenal
fa ilure. Patients with primary adrenal failure frequently have The most appropriate management for this patient's hypo- ·
increased pigmentation over the ex1ensor surfaces and buccal glycemia is to decrease insulin glargine and insulin aspart.
mucosa due to the excessive secretion of melanocyte-stim The glucose toxicity present at the time of diabetic ketoaci-
ulaling hormone. which shares a common precursor with dosis has diminished with an intensive insulin regimen. Her
adrenocorticotropic hormone (ACfH) . Hyperkalemia occurs remaining functional pancreatic beta cells have regained the
due to deficiency of aldosterone. The diagnosis of adrenal ability to produce some insulin in the ··honeymoon" phase,
insufficiency is made by documenting an inappropriately low which explains the hypoglycemia on previously well-toler-
serum cortisol level. An early morning serum cortisol level ated doses of insulin. 1l1e decreased need for insulin w ill not
that is less than 3 (82.8 nmol /L) in the setting of signs be long term as pancreatic beta cells continue to be destroyed
and symptoms of cortisol deficiency is diagnostic of this dis or- over the course of type 1 diabetes. Continuing insulin, even at
der. Treatment should not be withheld while awaiting further low doses. is recommended during the "honeymoon" phase
diagnostic testing since adrenal insufficiency is a potentially in order to preserve beta cell function as long as possible
life threatening condition that may result in hemodynamic by reducing the metabolic stress on these cells. The low-
instability. Treatment of primary adrenal failure requires both dose insulin regimen can consist of a basal and prandial
glucocorticoid and mineralocorticoid replacement.llydrocor insulin combination or a basal insulin regimen. She is expe-
tisone or another long-acting agent with primarily glucocor- riencing symptomatic postprandial and fasting hypoglyce-
ticoid activity should be given along with a mineralocorticoid mia. Decreasing the insulin aspart and insulin glargine doses
agent such as fludrocortisone. would address the prandial and fasting hypoglycemia. while
1l1e synthetic ACfH (cosyntropin) stimulation test is also still providing low-dose insulin to protect the function-
extremely sensitive for detecting either primary or second- ing beta cells. She will require close monitoring of her blood
ary adrenal insufficiency. In patients with nondiagnostic glucose levels to determine when insulin doses should be
basal cortisol values (4-12 [110.4-331.2 nmolJL]). stim- increased as she nears the end of the ··honeymoon" phase. 1l1e
ulation testing with synthetic ACTI! is indicated. A normal ··honeymoon" period may persist for several weeks to months.
response is a peak serum cortisol greater than 20 Meal-time insulin doses are genera lly reduced by 50%
(552 nmoi/L). ACTH stimulation testing is not needed if the when pramlintide is initiated due to the risk of hypoglycemia.

134
Answers and Critiques

"Ihe addition of pramlintide would li kely exacerbate the cur- commonly demonstrate oligoasthenospermia (reduced sperm
rent issue of hypoglycemia even with reduction of meal-lime motility) ancl at times complete azoospermia. Three months
insu lin. of treatment are typically needed prior to return of improved
1l1e use of sliding-scale insulin w ithout basal insu lin semen parameters. Prolactin is secreted by the pituitaty lac-
is d iscou raged. Wh en sliding-scale insulin is used without totroph cells under tonic inhibition by dopa mine. Dopamine
basal insulin , the likelihood of w ide swings from hypergly- agonist therapy can normalize prolactin levels, reverse hypo-
cem ia to hypoglycemia increases. With out a basal insul in gonadism. and shrink tumors by at least 50% in almost 90%
regimen. she may no t consistently receive da ily insulin to of patients. Evaluation of the infertile male w ith abnormal
decrease the me tabolic stress on her functioning pancreatic fi ndings on semen analysis should always include investi-
beta cells. gation of the hypotllalamic-piluilary-testicu lar (HPT) axis.
Discontinuation of both insulin glargine and insulin Disturbances in this <Lxis may result in failure of gonadotropin
aspart· increases metabolic stress on the pa ncreatic beta cells release from the anterior pituitaty and insufficient testosterone
and accelerates the loss of func tional cells producing insulin . production as well as absent or dimin ished spermatogenesis.
As pancreatic beta cell function declines toward the end of Clomiphene citrate is effective only w hen the HPT axis
the .. honeymoon" phase. the risk of diabetic ketoacidosis is intact. which does not apply for this patient. Although
increases w ithout a ny exogenous insulin. the ind ication fo r clomiphene citrate in the infert ile male
populatio n remains controversiaL some clinicians use it
KEY POINT
to increase endogenous follicle-stimula ting hormone and
• Continuing insulin, even at low doses. is recom- lutein izing hormone output from the anterior pituitaty to
mended during the .. honeymoon phase.. of type 1 d ia- support testosterone production by Leyd ig cells.
betes mellitus to reduce metabolic stress on function Sildenafil may improve erectile dysfunction in this
ing beta cells and preserve any residual fu nction for patient. but it w ill not increase endogenous testosterone
as long as possible. levels and will not improve h is hyperprolactinemic state.
Testosterone replacement therapy would be helpful to
Bibliography al leviate the sexual side efl'ects of this patient's hyperpro-
lJcWill DJ·:. I lirsch Ill. Oulpalienl in sulin therapy in type 1 and type 2 d iabc lactinemic state; however. no restoration of spermatogenesis
les m ellitus: scientific review JA:V!A. 2003 :VIay 7:289(17):225-l 6-l. [PM ID:
would occur. and therefore infertility would persist. Neither
1273 1137]
testosterone replacement nor sildena fi l therapy w ill reduce
the size of the patiem ·s prolactinoma.
Item 84 Answer: A
KEY POINT
Educational Objective: Treat hypogonadism second-
ary to hyperprolactinemia in a male patient. • Secondary hypogonadism in a male patient caused by
hyperprolactinemia as a result of a prolactinoma is a
The most appropriate treatment is a dopamin e agonist such as possible cause of erectile dysfu nction and should be
cabergoline. Hyperprolactinemia as a result of a prolactinoma treated with a dopamine agonist such as cabergoline.
is a possible cause of erectile dysfunction and decreased libido
and may be successfully treated w ith a dopamine agonist. In Bibliography
add ition to the sexual dysfunction associated \Nith hypcrpro \VA. Treatment for prolaclino mas and hyperprolactinaemia: a lifeti me
lactinemia. semen parameters are often abnormal: patients approach. Eur J Clin lm·esl. 2011 Mar:-11(:!):33-l -12. [PMID: 20955213]

135
Index

1\otL': Page follmH·d h' f.md t denote figures J lld tJbles. Test Antircsorptive therapy. in Paget disease of bo ne. 71. Q·IO
question!-. :Jrl' indit.:.at t•d b) Q. Ant it h) roid drugs. in Gr<l\Cs disease, --1--1
Apathetic hyperthyroidism. 42
A Ashcrman syn dro me (1\S) . S4. Q68
Abd ominal cr scan. for androgen producing adn:nalturno r. Areno lol. in thyrotoxicosis . .J:l
Q52 Ath erosclerotic cardiovascular disease (ASCVD). 16
Acarbose. for di<lbetes mellit us. St. lOt Autoimmune adrenalitb. and primary adrenal failure. 36
ACE inhibitor. in diabetit nephropalh). 17 ,\utoimmune mediated t) pc I diabetes mellitus. I. :1
Acqu ired type I diabetes. :1 ,\utonomic neuropathy. 18
Acromegal). 27
causes of. 27 8
clinical features and diagnosis in. 27 llariatri<: surge!). 8. Q-19
transsphenoidal surgery for. Q61 aclrenoceptors b lockers. for pheochromocytomas, 33 3.J
trea tm ent of. 27 :.18
ACfH st imulation test. 22 . 23 in thyroid storm treat ment. 49
Adenoma in thyrotoxicosis. 43 . QIO
adrenal. 32 Rigwlllides. for diabetes me llitus. lOt
pituitary. 20 (see u lso Pituitary tumors) Bilattral adrenal hemorrhage. Q.J2
thyroid. toxic. 44 Bisphosphonatcs
Adrenal crisis. 36 in h) percalcemia. 6-1. 6S
Adrenal failure. bilateral adrenal hemorrhage and. Q 12 ir1 osteoporosis. 68 69
Adrenal fat igue. 38 !:lon e mineral den sil\ 66 67. Ql2
Adren al gland. disorders of. 29 .JO decline in. 67
adrenal hormone excess and. •lnd fracture risk. 67
adrenal insu fficiency and . 36 39 increase in. 66
adrena l m asses. :!9 .JO measuremen t of. 67. 681
related anatomy and physiology. 29. 29 1' Bro mocript ine. for pro la<:tinomas. 26
Ad ren al in su fficiency. :l6
during critical illness. :!8 :39 c
primary adrena l fail ure. :36 :38 Cabergoline. QS-1
Adrenal masses. :19 .JO for Cush ing disease:. 28
imaging characteris t ks of. :321 for prolactinomas. 26. Q19
incidentally noted. 39 .JO. 39f. Q50. Q60 C.alcitonin. -12
Ad renal metastases. 39 for osteoporosi;. 69
Adrenal \·ein sampling C\ \ 'S) . 3·1 Calcitriol. 62. 66
Adrenoconical carcinomas (ACCs) . 32. 32t. 39 . .JO Calcium
surgical excision of. Q27 homeostasis and bone physiology.
Adrenocorticotropic hormone (,\Cfl l).l9.19t postsurgica l hypoparat hyroidism and supplem en ta l ion intake,
and cortisol production. 29 Q31
d efi ciency of. 22 2:l. 2:11 Calcium carbonate. in h) 66
Agin g male. androgen dclkkncy in. S7 58 Calcium citrate. in hypocalcemia. 66
Aldosterone. 29 Calcium creatinine clearance ratio. 6-1- 65
Aldosterone producing adrenocortical adenomas. :j.l Canaglifiozin. for diabetes mellitus. lOt
Alcndronalc Ctpsaicin cream. 18
for 69. Q78 C<trdiO\ ascular disease (C\'1)). d i,tbetes mellitus and. 16 17.
for Paget disease of bon e. 71 16t
,\logliplin. for d iabetes mellitus. lOt Catecholamines. 29. 29f
a -adrenocep lors blockers. for p heochromocy toma,. 33 Celiac disease. autoim mune med iated type I diabetes an d. :l
« g lucosicbsc inhib itors. for type 2 diabetes. 4 . St. 101 Cent ra l diabetes insipidus. :.1·1 25
Amen orrhea. 54 SS Central hypert hyroidism. -15
evaluation ol'. 55 Central hypothyroidism . me<tsuremen t of serum free thyroxine (T 1) level
primary. 5,1. Q9 in.Q36
secondary. 5·1 55 Chlorpropamide, for diabete> mellitus. lOt
American Diabetes As;odation (ADA) . on 'creening f(Jr I) pe 2 diabetes. Clomiphene citrate. 57. Q6-1
I. 2t Continuous glucose m oni to ring (CG\1). 7. Q5:J
Amiloride. in primary hy peraldosteronbm. :35 Cont inuous subcutaneous infusion (CSII) therapy, 7 9
Amiodarone induced thyrotoxicosis (AIT). ·16 Conicotrophin releasing hormone (CRl l). 19. 28
type 1. .J6 Cortisol. 29
type 2. -t6 Crit ical illness. adrenal function during. 38 39
Amitrip tyline. in d istal s) mmct ric polyneuropathy. 18 Cushing disease. 28 29
Amylinomimetics. for diabetes mellitus. lOt Cushing ;yndrome (CS). 28. 30 32
Anabolic steroid abuse. in men. 60 causes of. 30. 30t
Androgen abuse. in \\Otm: n. 56 clinical manifestations o f. 30. 30f. 31t
Androgen dl'liciency. in agin g male. 57 58 CT of adrenal glands in. QS
Androgen producing adrenal tumors. 36. QS2 diagnosis of. :lO 32. 31f. Qs. Q.JS
Angiotensin receptor blocker (ARB). in diabetic n ephropathy. 17 endogenous. :30
Anticonvulsants. in distal symmetric polyneuropathy. 18 iatrogenic . :JO. QL3
Antidepressants. in distal symmetric neuropathy. 18 imaging s tudies in. :n. :m
Ant idiuretic h ormone (:\Dl l). 19 and pseudo-Cushing syndrome. 30
deficiency of. 2 1 2S treatment o f. 32
excess of. 28 Cystic fibrosis. and infertility. Q81

137
Index

D Foot care. in diabe tic patient. 18


Dapaglinozin, for diabetes m ellitus, lOt Foot ulcers. in diabetic patient, 18
Denosumab Fracture Risk Assessment Tool (FRAX) calcu lator, 67-68
in hype rcalcemia. 65 Frac tures
in osteoporosis, 69 osteoporosis and. 67
Desmopressin, in central dia betes insipidus. 25 prediction of risk of. 67- 68
Destructive thyroiditis, 44 Framingham Risk Score. 4
Dexamet hasone
in primary ad renal failure. 38t G
in secondary cortisol deficiency. 23 Gaba pen tin. in distal symmetric polyneuropathy. 18
mg dexamethasone suppression test, Q48 Gestational diabetes mellitus. 5
DH EA, in primary adrenal failure. 37. 38t complica tions related to. S
Diabetes mellitus. 1 screening for. S
and chronic kidney disease. QSJ therapy for, 5
classification of, I, 3t Gigantism. 27
insu lin deficiency, 1, 3, 3t Gliclazide. for diabetes mellitus. 10t
insulin resistance. 3- 5. 3t Glimepiride. for diabetes melli tus. lOt
uncommon types. 3t, S Glipizide . for diabe tes mellitus. lOt
complications of Glucagon- like peptide 1 (GLP-1) mim etics. for diabetes mellitus.
acute, 14, !Sf. 1S t 9. lOt
chronic. 16 18, 16t Glucocorticoids
diagnostic cri teria for. L 2t in hypercalcemia. 65
elevated plasma glucose levels in. I in lymphocytic hypophysitis. 20
insulin t herapy in in primary adrenal failure. 37- 38, 381. Q82
basal and prandial insulin. in ho:pitalized patient. Q17 in t hyroid stor m treatment. 49
tim ing issues with. Q38 Glucose m etabolism. disorders of. See also specific disor der
management of. S 6, 6t diabetes mellit us. 1- 11
blood glucose monitoring, 7. 71 hyperglycem ia. 11 12
non pharmacologic approaches. 7-8 hypoglycemia. 12-14
patiem education . 6 7 Glutamic acid decarboxylase (GAD65). I
p harm acologic therapy. 8 II, JOt Glyburide.,for diabetes mellitus. JOt
screening for. I. 2t Glycemic targets. and diabetic complications, QJ4
type 1. 1, 3. 3t (see a lso Type 1 diabetes) Goiter. 41
type 2. 4, St (see a lso Type 2 diabetes mellitus) multinodular. 51- 52
Diabetes self management education (DSME) . 6 simple. 52
Diabetes self- ma nagement support (DS:'v!S) . 6-7 Gonadotropin-producing pituitary adenomas. 28
Diabetic amyotrophy, 18 Gonadotropin releasing hormone (GnRH). 19. 53
Diabe tic complications. and glycemic targets. QJ4 Graves disease. 43-44
Diabetic foot ulcers, 18 Graves ophthalmopathy. 4-1. Q70
Diabetic ketoacidosis (DKA). 3. 14 . lSf. !St. Q24 Growth hormone (GH). 19
Diabetic nephropathy. 16t. l7 deficiency of. 24
Diabetic neuropathy. 16!.17-18. Q66 excess of, 27 (see a lso Acromegaly)
Diabetic retinopathy. 17. Gynecomastia. 60 61
Diet and exercise, for prevention delay of type 2 diabetes. -l, St causes of. 60-61
Dipeptidyl peptidase 4 (DPP 4) inh ibitors, for diabetes mell itus. 9 . JOt examina tion in. 61
Distal symmetric polyneuropathy (DPN). 18
Dopamine agonists (Di\), for prolactinomas. 26 H
Drug-induced thyroid dysfunct ion. 46. 47t Hashimoto thyroiditis. and hypothyroidism. 45
Dual energy x ray absorpliometry (DEXA) scan. 64 . 67. QI2. Q43 Hemochromatosis. diagnosis of. Q30
Duloxetine llemodialysis. in hypercalce mia, 65. Q75
in diabetic polyneuropathy. Q66 Hemoglobin A,, testing. I. 2t. 7. 7t
in distal symme tric polyneuropathy . 18 disadvantages of. I
in patient w ith diabetes melli tus and chronic kidney disease.
E QSl
Epinephrine. 29 Ilirsutism . 55 56
Eplerenone . in primary hyperaldosteronism. 35 Iloncymoon phase. insulin therapy in. 3
Est rogen agonists and antagonists. for osteoporosis, 69 Human chorionic gonadotropin (HCG) testing. SS
Estrogen replacement. in hypergonadotropic hypogonadism. 5-l-55 Hydrocortisone
Euthyroid sick syndrome (ESS). 48. QJ3 after adrenalectomy in pa tient with Cushing syndrome. Q39
Exenalide, for diabetes mellitus, lOt in prim ary adrenal failure. 381. Q82
Exercise in secondary cortisol defi ciency. 23
for diabetes management. 8 25 Hydroxyvilam in D. 61.62
and hypoglycemia, 12. Q35 Hyperandrogenism syndromes. 55 56
Hypercalcemia. 62-66
F causes of, 62. 63t
Familial hyperparathyroidism. and hypercalcemia. 65 clinical features of. 62
Familial hypocalciuric hypercalcemia (FHH). 64-65 diagnosis of. 62. 63f
urine calcium and creatinine levels in. measurement of. Q37 non-parathyroid hormone-mediated, 65. Q22
Fasting hyperglycemia, Q4 parathyroid hormone-mediated. 62-65
Fasting hypoglycem ia , J:3 treatm ent of, 65- 66
Fasting plasma glucose (FPC), L 2t. Q57 Hyperglycemia, management of, II
Female reproductive disorders hospitalized patients with diabetes mellitus, 11 - 12
amenorrhea, 54- SS hospitalized patients without diabe tes mellitus, 12
female infertility, 56-57 Hyperglycemic hyperosmolar syndrom e (HHS) . 11, !Sf. 1St
female reproduction, physiology of. 53. 53f Hypergonadotropic hypogonadism. Klinefelter syndrom e and. Q58
hyperand rogenism syndromes, 55-56 Hyperprolactinemia, 25 27. 25!. Q80 . Q81
Fine needle aspiration (FNA). 42 antipsychotics and. Qll
of thyroid nodules. 51. Si t. Q8 causes of. 25- 26. 251
Fludrocortisone, in primary adrenal failure. 38t. Q44. Q82 clinical feat ures and diagnosis. 26
Fluid resuscitation, in hypercalcem ia. 65 and secondary a meno rrhea, 54
Follicle stimuhl!ing hormone (FSI 1). 19,191,53 therapy for. 26

138
Index

Hyperthyroidism Ketoacidosis. diabetic. 3, 14. !Sf. 15t, Q24


during pregnancy, Ql Ketoconazolc, in Cushing disease, :!8
subclinicaL 45 Klinefelter syndrome, Q58
triiodothyronine (T) level, measurement of. Q21 and primary hypogonadism, 57
Hypocalcemia
clinical features of, 66 L
diagnosis and causes of, 66 Lactic acidosis. 9
hypoparathyroidism and . 66 Laparosco pic adrenalectomy, 35
low m agnesium level and, Q28 Laser photocoagulation. in diabelic retinopathy, 17
treatm ent of, 66 Late autoimmune diabetes in adults (L.<\DA). 3
Hypoglycem ia Late- night (LN) salivary cortisol. 30-32
and altered mental sta tus, Q20 Letrozolc. 57
defini lion of, 1:1 Levolhyroxine
exercise-induced. Q35 for hypothyroidism, 45 . Ql6
in patients with diabcrcs m ellitus, 12-13 hypothyroidism from pituita ry dysfunction and dosage of, Q.56
in patiems without diabetes mellitus. 13 for hypothyroidism in pregnancy. Q4 1
differential diagnosis of, B t Lifestyle modifications. in diabetes m anagement, 4. s
fasting hypoglycemia, 13 Linagliptin, for diabetes mellitus, lOt
postprandial hypoglycemia , 13 Lipase inhibitors. for prevention 'delay of type 2 diabetes. 4, 51
in patient wilhout diabetes mellit us. Q67 Liraglutide. for diabetes mellitus. l Ot
Hypoglycemic unawareness, 18 Lithi um, and hypercalcemia, 65
treatment of. Q76 Loop diuretics, in hypercalcemia . 65
Hypogonadism. 57 59 Lov.·-dose dexamethasone suppressio n test (LOST). 30- 31, 40
androgen defi ciency in aging m ale. 57-58 Low triiodothyronine (T3) syndrom e. 48
desire for fertility and therapy for, Q32 Luteinizing hormone (Ll-1), 19,191, 53. Q77
evaluation of. 58, 59!' Lymphocyt ic hypophysitis. 20
prima ry, 57 Lymphocytic thyroiditis, 40
secondary, 57
testosterone replacemenl therapy for. 58- 59, 60t M
Hypogonadotropic hypogonadism, 54 Male reproduct ive diso rders
GnRH deficiency and, 24 anabolic steroid abuse. 60
hemochromatosis and, Q30 gynecomastia, 60 61
Hypoparathyroidism hypogonadism, 57- 59
and hypocalcemia, 66 male infertility. 60
postsurgical. Q34 male reproduction. physiology of, 57, 57[
Hypopituilarism. 21- 22 Malignancy-associated hypercalcem ia. 65
adrenocorticotropic hormo ne deficiency. 22- 23. 231 Maturity-onset diabetes of young (MODY). 5
causes of, 21, 22t Medical nutrition therapy, for diabetes management. 7- 8
central diabetes insipidus. 24- 25 \1edullary thyroid cancer (MTC) . 53
gonadotropin deficiency, 24 plasma fractionated me tanephrinc levels in. Q74
grow th hormone defi ciency. 24 \leglitinides. for diabetes mellitus. lOt
panhypopit uitarism. 25 \>lenstrual cycle. 53
Sheehan syndwme and. 22 Mental status changes
thyroid-stim ulating hormone deficiency, 23- 24 hypoglycemia and. Q20
llypothalamic amenorrhea. and GnRH deficiency. 24 in myxedema coma. 49
Hypothalamic pituitary ovarian axis. 53 .\oletabolic bone d isease, 66
Hypotherm ia. in myxedema coma, 49 .\oletabolic synd rome, 3
Ilypothyroid, 44 and cardiovascular disease (CVD), 3
llypothyroidism definition of. 3.
causes of. 45 Endocrine Society on screening for. 4
central. Q36 and type 2 diabetes mellitus. 3
evaluation of. 45 10-year CVD risk. calculalion of. 4
1nanagen1en l o f. 45- 46 .\oletformin. for type 2 diabetes. 4, Sl. 9. l Ot. Q29
medication induced, 45 .\olethimazole
in pregnancy. Q 41 in Graves ophthalmopathy, Q70
1-lysterosalpingogram (HSG). for tubal patency. Q73 in t hyrotoxicosis. 43
.\oletoprolol. in subacute thyroidit is. QlO
.\1etyrapone, in Cushing disease. 28
lbandronate, for osreoporosis, 69 Microprolactinomas. in asymptomat ic patients. Q65
Idiopathic type 1 diabetes. 3 Migli tol, for diabetes mellitus. lOt
Incidentally noted adrenal mass. 39-40. 39f. Q50. Q60 \1ineraloconicoid antagonist. in primary hyperaldosteronism, 35
Infertility Mineralocorticoids, in prim ary ad renal failure. 37, 381
fem ale. 56 S7 Mitotane.
m ale. 60 Mixed meal tolerance test. 13
Insulin resistance, 3 Multinodular goiter, 44. 51- 52
gestational diabetes. S Multiple daily injection (MDI) insulin therapy, 7. 8
m elabolic syndrom e. 3- 4, 4t Multiple endocrine neoplasia syndrome (MEN) , 65
obesity and, 3 Multiple endocrine neoplasia syndrome type 2 (ME"2). Q2
type 2 diabetes m ellitus, 4, 5t Myxedema coma. 49
Insulin therapy, for type I diabeles, 8- 9. 8t, Q63. Q83. See a lso Diabetes mellitus
Intra petrosal sinus sampling (IPSS), :!8 N
Iodine deficiency, 41 Nategli nide. for diabetes mellitus. lOt
Iodine drops, in thyroid storm treatment, 49 National Osteoporosis Foundat ion. on osteoporosis eva luation. 67
Ionized calcium level. testing for. Q31 t\erve compression syndromes. 18
t\onthyroidal illness syndrome, 48
J t\orepinephrine, 29
Jod- Basedow phenomenon, 44 Normocalcemic primary hyperparathyroidism, 64

K 0
Kallmann syndrome, and GnR H deficiency. 24 Oral glucose lolerance test (OGTT) . l. 2t
Karyolype testing, Q58 Orlistat. for prevent ion! delay of type 2 diabetes. 5t

139
Index

Osteonecrosis of jaw. 68 Prednisone


Osteopenia. 66 68. See Cllso Osteoporosis in primary adrenal failu re. 38t. Q·l-1
Osteoporosis in cortisol deficiency. 2:3
annual reassessmcm in . 69 Pre?,abalin. in distal symm etric polyneuropathy. IS
BMD testi ng and DEXA scan. 67 68. Q12, Q·13 Prc;gntmcy
bone mineral density loss and, 67, Q25 diabetes in, s
deli ned.67 thyroid function and disease during, -16. Ql
diagnosis of. 67 68 h) pothyroiclism in. Q.J I
hypercortisolism and risk of. Q·H prolactinoma in. 26 27
1\ational Foundation and Endocrine Society and secondary a me norrhetl. 5 I
recommendations. 70 thionamides use during. 48
physiolog) related 10. 66 67 th) roid function in. -16 18 . .JSf
risk assessrnent and screen in?, guidelines. 67. 671, 681 ' isual field testing during. Q3
secondary causes of. 6tl, Q2S Premature ovarian (POl). 54
treatment of. 68 69 Pretibh11111) xedema.
,·ertebral imagjng. 68 Primary adrenal failure. 36 38. Q82
vitamin Dane!. 69-70 causes a nd clin ical featu res. :l6, 37t
Overnight blood glucose m onitoring. Q-1 diagnosis of. 36 :l7
prednisone and nuctrocon isone in. Q.J-1
p treat ment of. :!7 38. :l8t
Page t disease of bone, 70 71. Primary amenorrhea. Q9
clinical manifestation. 70. 71 f Primary h) peralclosteronism (Pi\). 34 36. 351
d iagnosis or. 71 Primar) h) perparathyroidism. 62 64. 64 1. Q2
treat men! for. 71 parathyroidectomy lor. 63. Q17
Pamidrona te. for Paget disease of hone. 71 and vitamin D deficienc). Q·16
Panhypopituitarism. 25 Primary h) pogonadism. 57
ho rmone replacement 1herapy in. Q56 Primary hypothyroidism. and hyperprolactinemia. 26
Paragangliomas. :l2 3-1 Prima ry t hyroid lymphoma. 52, Q-15
Parathyroid carcinoma. 6· 1 Progesterone c hallenge; test, 55
Parathyroidectomy. for primary hype rparathyroidis m, 6:3. Prolactin. t'i
Q17 Prolactinoma. 25
Parathyroid hormone (PTH). 61. 62. See also H) percalcemia in pregnancy. 26 27. Q:l
Paroxetine. in distal symmd ric polyneuropathy. 18 therapy for. 26
Pasireotide. in Cushing d bease. 28 Propr.molol. in thyrotoxicosis. ·13
Patient education, fo r diabetes managem ent. 6 7 Prop) h hiouracil (PTC)
Pegvisomant. 27 fo r h) pcrthyroidism during pregnancy. Ql
Phc noxybcm.amine. for pheochromocytomas. 33. Q55 for thyrotoxicosis . .J3
Pheochromocytomas. 32 34 . 32t. 341. Q55. p,eudo Cushing syndrome. :lo
diagnosis of. 33 P:.eudogynccomastia. 61
m ultiple endocrine neoplasia syndromes and. 33.331 Pseudohypercalcemia. 61. Q31
radiographic localization of. :l:l Pseudoh) pocalcemia. 61
treatment of. 33-3-1
Physiologk in,ulin lhera]Jy. for type I diabetes. 8 9. 8t R
Pioglitazone. for diabete' mdlitus. 51. lOt Radioactive iod ine ablation
Pituitary adenoma . 20. See u/oo Pituitary tumors for t hyrotoxicosis . .JJ
Pituitary apoplexy. 21 22. Q7 for nodules. -14
Piluilar) gland disorders Radioacth c iodine uptake (RAIL). -12. Ql S
anatom) and physiolog) related to.IS-20. 191. 191 Ralo.\ifine. for osteoporosis. 69
hypopiluit;trism. 21 25 Receptor acti\·aror of nuclear factor KB (RAI\K) ligand inhibitors. for
pituitary hormone defi ciency and excess. testing fo r. 19t osteoporosis. 69
pituitary hormone excess. 25 29 Repaglinide. for diabetes mellitus. lOt
pituitary tu mors, 20 21. 20f hypertension. Q71
Pituitary tumors. 20 Risedronatc;
empty sella. 21 for osteoporosis. 69
functional. 25 for Paget disease of bone. 71
incidentall) noted. 20- 21 Rispcridone. and hyperprolactinem ia. Q1 1
macroadenoma. 20. 20f Ro,iglitazone
e ll'e cts or. 21 lor diabetes mellitus. lOt
m icroadenoma, 20 tor prc,•cntion delay of type 2 diabetes. 51
non l'unc tioning, 21
sellar mass, approach to. 20 s
Plasma aldosterone concentration (PAC). 3-1. 351 Sarcoidosis. hypercalcemia in. Q22
Plasma aldosterone plasma renin acti\'ity ratio. Q71 Sa.\.lgliptin. for diabetes mellitus.10t
Plasma free m etanephrines. measurement of. Q50 Second,uy amenorrhea. 5-I 55
Plasma renin acti\'ilv (PRA). 3-1, 3St Secondar)· 57
Polycystic ovary syn'urome (PCOS). 5-I. 55 56, Q6·1 Secondary hypothyroidism. 23- 2·1
elevated resting luteinizing hormone levels in. Q77 Self- monitoring of blood glucose (SMBG). 7
estrogen- progestin oral con traceptive pi lls in, Q26. Q5-1 Serum ferritin level and transfe rrin saturation, mcasuremc;nl of. Q30
Polyuria. hypercalcemia and. 65 Sheehan syndrome. hypopiluitari"n and. 22
Pooled Cohort Equal ion. -1. 16 Sitagliptin. for diabetes mellitus. lOt
Postmenopausal wom en Smoking cessation. for pre\'ention delay oft) pe 2 dia be1es. 5t
fracture risk in. 67 Sodium glucose transporter 2 (SGLT2) inhibitors. for diabetes mellitus, 9, 101
osteoporosis treatment in. 68 69 Somatostatin analogues. in acromegaly. 27
Postpartum thyroiditis. -1-1 Spi ronolactone. in prima ry hyperaldosteronism. 35, Q79
Postprandial hyperglycemia, Q69 Stereotact ic radiosurgery. in acromegaly, 27
in pregnancy. s Subdinical hyperthyroidis m. "IS
Postprandial hypoglycemia. 13 Subd inical hypothyroidism. 16. Q16
Potassium chloride, in diabetic ketoacidosis. Q2.J Sulf'onvlureas
Pramlint ide. for diabetes mellitus, lOt t'or diabc;tes lOt
Prediabetes. I and h) poglycemia. Q20
Prednisolone. in primary adrenal failure. 381 S) ndrom e of'inappropriate ADH secre tion (Sii\DH), 28

140
Index

T Tissue transglutaminase antibody test ing, QIS


Tcriparntidc, for osteoporosis. 69 Tolbutamide. for diabetes mellitus. lOt
Tert iary hyperparat hyroidism. 64 Toxic adeno ma, ,14
fa ilure. See Primary hypogonadism Transsphenoidal pitui tary decompression. Q7
Testosterone repl<Icemenllherapy. 58- 59, 601 Transsphenoidaltumor reseuion
Thiazide diuret ics. a nd hyperca lcemia. 65 in acromegaly. 27
Thi<Izol idinediones in Cushing disease. 28
for diabetes mellitus. lOt Transvaginal ult rasound. for Asherm an syndrome, Q68
for prevenlion/deb y of type 2 diabetes, 4. St Triiodothyronine (T). 41 , Q2t
Thionamides Troglitawne, for prevention/delay of typc 2 d iabetes. St
in thyroid storm lre<Itm enl, 49 T-score. 67
in thyrotoxicosis. 43 Turner syndrome, 54
Thyroglobulin (TgAb), 42 Type 1 diabetes. 1. 3!. Q14. See (1 /so Diabetes mellitus
Thyroid au10antibody measurement, 41 acquired, 3
Thyroid cancer. 5 2 S:l autoimmune-mediated. 1. 3
and adjuvant radioactive iodine I herapy, Q62 idiopathic, 3
incidence of. 52 insulin deficiency and, 1
medullary, s:J 1herapy for. 8 9. St. Q83
staging and prognosis ol', 52 Type 2 diabetes mellitus. 4. Q29. See (1 /so Diabetes mellitus
treatment of. 52 53 diagnosis of. Q57
types of. freq uency of, 52. 52t epidemiology of. 4
Thyroidectomy, fo r substernal goite r. Q72 etiology of. 4
Thyroid emergencies , 48 49 ketosis-prone patients wit h. 4, Q6
Thyroid function, in pregnancy, 46 48, 48f metabolic syndrome and. :l
Thyroid function test. in elderly patient, Q59 obesity and, 4
Thyroid gland prevention delay of. strategies for. 4 . St
anatomy and physiology. 40 4 1 therapy lor. 9 II. lOt
diso rders of
eut hyroid s ick syndrome, 48 u
fun ctional. 42 4 6 Clcers. foot. in diabet ic pat ient. 18
in pregnancy, 46 4S, 4Sf Crine albumin excretion. elevated . 16t. 17
structural , 50 5 2 24 hour urine free cortisol (UI'C) . 30.31
thyroid cancers. 52 5:3 . .52f C.S. Preventive Services Task l'orce (CSPSTF)
I hyroid emergencies, 48 49 B.MD testing and vertebral imaging. recommendations for.
function of. evaluation of, 41 42 68l
hormones of, 41 on screening for ty pe 2 diabetes, 1. 2t
Thyroid hormone replacement therapy, 45
Thyroid hormone therapy. in myxedema coma. 49 v
Thyruidi!is, ·14 Valproate. in distal symm etric polyneuropathy. 18
autoimmune mediated type I diabetes and. 3 Venlafaxine. in dista l symmetric polyneuropathy. 18
destructive, 44 Vertebral fractures. osteoporosis and, 67 69
Hashimoto, 45 Vildagliptin. for diaberes mellitus, lOt •
painful. 44 Viral infect ions, and type I diabetes mellitus, I
pain less. 44 Visual field testing. 21
post partum. 44 during pregnancy. Q3
subacute. QlO Vitamin D. 61-62
Thyroid nodules, 50- 51 hypercalcemia. 62-66
evaluation of. SO 51. 501', QIS hypocalcemia. 66
fine needle aspirat ion of. 51, Sit. QS production of. 62f
history and physical examination. 50 sources of. 61t
incident;llly discovered, 50 Vitamin D, (ergocalciferol), 61
types or. SOt Vitamin D; (cholecalcife rol), 61, Q46
Thyroid peroxidase (TPO). 41 Vitamin D deficiency. 69-70, QIS
Thyroid-stimulating hormo ne (TSH). 19, 19t, -11 Voglibose. for diabetes mellitus. St. lOt
dclkiency of, 23-24 Volume replet ion. lor hypercalcemia. 65
in elderly patient, Q59
Thyroid stimula ting hormone secreting tum ors. 28 w
Thyroid stimula ting im m unoglobulins (lSI) <llltoantibodies. 12 Weight loss. for prevention of type 2 diabetes. 4
Thyroid storm. 48- 49, Q33 Whipple triad . 13
diagnosis of. 49 Wolff-Chaikoff effect, 46
treatment of, 49
Thyrotoxicosis, 42 z
ev;Jluation of. 42- 4:; Zoledronic acid
managem ent of. 43 for hypercalcemia, 65
sym ptoms of, 42--13 for osteoporosis. 69
Thyrotropin receptor (TRA b). 41 for Paget disease of bone. 71
Thyroxine (T,). 41 Z score. 67

141

S-ar putea să vă placă și